Vous êtes sur la page 1sur 591

http://www.aircrafttechtrng.

com

Module 11
Aeroplane Aerodynamics, Structures and Systems

1. As a subsonic aircraft speeds-up, its Centre of


Pressure

 is unaffected
 moves aft
 moves forward

2. Wing spoilers, when used asymmetrically, are associated


with

 ailerons
 rudder
 elevators

3. If an aircraft is yawing to the left, where would you


position the trim tab on the rudder to correct?

 To the left
 To the right
 To the centre

4. If an aircraft is flying with a left wing low, where


would you move the left aileron trim tab to correct?

 Moving the aileron trim tab will not correct the


situation
 Up
 Down

5. When a leading edge flap is fully extended, what is the


slot in the wing for?

 To increase the lift


 To re-energise the boundary layer
 To allow the flap to retract into it when it
retracts
6. With respect to differential aileron control, which of
the following is true?

 The up going Aileron moves through a smaller angle


than the down going aileron
 The down going aileron moves through a smaller angle
than the up going aileron
 The up going and down going ailerons both deflect to
the same angle

7. The aeroplane fin is of symmetrical aerofoil section


and will therefore provide a side-load

 only when the rudder is moved


 if a suitable angle of attack develops due either
yaw or rudder movement
 only if a suitable angle of attack develops due to
yaw

8. An aircraft left wing is flying low. The aileron


trimmer control to the left aileron trim tab in the
cockpit would be

 moved up causing the left aileron to move down


 moved up causing the left aileron to move up
 moved down causing the left aileron to move down

9. The purpose of a slot in a wing is to

 provide housing for the slat


 speed up the airflow and increase lift
 act as venturi, accelerate the air and re-energise
boundary layer

10. Large flap deployment

 causes increased spanwise flow towards tips on wing


upper surface
 causes increased spanwise flow towards tips on wing
lower surface
 has no effect on spanwise flow
Answers for Questions 1 - 10
1) moves aft
Comment/Reference: Assuming that the aircraft is to
remain at constant altitude, it must reduce its angle
of attack as it speeds-up. This alone will move the C
of P rearwards, in accordance with the sub-sonic angle
of attack change theory.
2) ailerons
Comment/Reference: Pallett Automatic Flight Control 4th
Edition Page 51. Jeppesen A&P Technician Airframe
Textbook Page 1-12
3) To the left
Comment/Reference: Automatic flight control, Pallett,
4th Edition Page 43
4) Up
Comment/Reference: Automatic flight control, Pallett,
4th Edition Page 43

5) To re-energise the boundary layer


Comment/Reference: Pallett Automatic Flight Control
2nd Edition Page 50. Jeppesen A&P Technician Airframe
Textbook Page 1-32

6) The down going aileron moves through a smaller angle


than the up going aileron
Comment/Reference: Mechanics of Flight, Kermode, Page
301. AC65-15A pg 41 rh column

7) if a suitable angle of attack develops due either yaw


or rudder movement
Comment/Reference: Rudder deflection or yaw

8) moved up causing the left aileron to move down


Comment/Reference: Down aileron required - which
requires up trim

9) act as venturi, accelerate the air and re-energise


boundary layer
Comment/Reference: A slot is to act as venturi,
accelerate the air and re-energise boundary layer

10) causes increased spanwise flow towards tips on wing


lower surface
Comment/Reference: Flaps increase the pressure
differential between top and bottom surfaces, increase
tip vortices and spanwise flow
Questions 11 – 20

11. Which part of the wing of a swept-wing aircraft stalls


first?

 Tip stalls first


 Root stalls first
 Both stall together

12. During flight, an aircraft is yawing to the right. The


aircraft would have a tendency to fly

 right wing low


 nose up
 left wing low

13. With a drop in ambient temperature, an aircraft service


ceiling will

 not be affected
 rise
 lower

14. Extending a leading edge slat will have what effect on


the angle of attack of a wing?

 Decrease the angle of attack


 Increase the angle of attack
 No effect on angle of attack

15. To ensure that a wing stalls at the root first, stall


wedges are

 installed at the wing trailing edge at the wing root


 installed on the wing leading edge at the wing tip
 installed on the wing leading edge at the wing root
16. With reference to differential aileron control

 drag decreases on the outer wing


 drag increases on the inner wing
 drag increases on the outer wing

17. Dutch role is movement in

 yaw and roll


 yaw and pitch
 pitch and roll

18. If an aircraft is aerodynamically stable

 aircraft returns to trimmed attitude


 C of P moves back
 aircraft becomes too sensitive

19. Ailerons control the aircraft in the

 longitudinal plane
 directional plane
 lateral plane

20. An anti-balance tab is used

 for trimming the aircraft


 to give more feel to the controls
 to relieve stick loads
Answers for Questions 11 – 20

1) Tip stalls first


Comment/Reference: The tip of a swept wing stalls first

2) right wing low


Comment/Reference: The leading wing (left wing) has
increased lift, causing it to rise

3) rise
Comment/Reference: As ambient temperature drops,
density increases and aircraft performance increases

4) Decrease the angle of attack


Comment/Reference:

5) installed on the wing leading edge at the wing root


Comment/Reference:

6) drag increases on the inner wing


Comment/Reference: Automatic Flight Control, Pallett
4th Edition Page 41
A+P Technician Airframe Textbook Page 1-11

7) yaw and roll


Comment/Reference: Avionic Fundamentals Jeppesen page
291

8) aircraft returns to trimmed attitude


Comment/Reference:

9) lateral plane
Comment/Reference: Ailerons control the aircraft 'IN'
the lateral axis, which is 'ABOUT' the longitudinal
axis.

10) to give more feel to the controls


Comment/Reference: Jeppesen A&P Technician Airframe
Textbook 1-29 & http://en.wikipedia.org/wiki/servo_tab
Questions 21 – 30

21. Slats

 act as an air brake


 increase the overall surface area and lift effect of
wing
 keep the boundary layer from separating for longer

22. Due to the change of lift forces resulting from the


extension of flaps in flight

 nose should be raised, increasing AoA


 nose should remain in the same position, maintaining
same AoA
 nose should be lowered, reducing AoA

23. Flight spoilers

 can be used with differential ailerons to reduce


adverse yaw in a turn
 can be deployed on the down going wing in a turn to
increase lift on that wing
 can be used to decrease lift to allow controlled
decent without reduction of airspeed

24. If the aircraft is flying nose heavy, which direction


would you move the elevator trim tab?

 Up to move elevator up
 Down to move elevator up
 Up to move elevator down

25. Wing tip vortices are strongest when

 flying high speed straight and level flight


 flying into a headwind
 flying slowly at high angles of attack
26. An example of a secondary flight control is a

 elevator
 spoiler
 flap

27. A balance tab

 effectively increases the area of the control


surface
 is used to trim the appropriate axis of the aircraft
 assists the pilot to move the controls

28. Which wing increases drag when the ailerons are moved?

 Both wings have an equal increase in drag


 Both wings increase drag but the wing with the down-
going aileron increases more
 Both wings increase drag but the wing with the up-
going aileron increases more

29. Which flap will increase wing area and camber?

 Slot
 Split
 Fowler

30. An automatic slat will lift by itself when the angle of


attack is

 high
 low
 high or lo
Answers for Questions 21 – 30

1) keep the boundary layer from separating for longer


Comment/Reference: Jeppesen A & P technician airframe
textbook page 1-32

2) nose should be lowered, reducing AoA Comment/Reference:


The main purpose of flaps is to increase lift so that
the pilot can lower the nose, increase decent angle and
get a better view of the runway.
http://en.wikipedia.ort/wiki/Flap_(aircraft)

3) can be used to decrease lift to allow controlled decent


without reduction of airspeed

4) Down to move elevator up

5) flying slowly at high angles of attack

6) flap
Comment/Reference: A spoiler is sometimes a 'primary
flight control' when it is used for roll control in
conjunction with the ailerons. On the other hand, a
flap is usually not considered to be a flight control
at-all, rather, a high lift device. So take your pick!

7) assists the pilot to move the controls


Comment/Reference: Jeppesen A & P Technician Textbook
pg 1-29

8) Both wings increase drag but the wing with the down-
going aileron increases more
Comment/Reference: Jeppesen A & P Technician Airframe
Textbook page 1-26 & AC65-15A pg 41, top para’s right
side.

9) Fowler
Comment/Reference: Jeppesen A&P Technician Airframe
Textbook Page 20 & AC65-15A pg 48

10) high
Questions 31 – 40

31. On aircraft fitted with spoilers for lateral control,


roll to the right is caused by

 right spoilers extending, left spoilers remaining


retracted
 left and right spoilers extending
 left spoilers extending, right spoilers remaining
retracted

32. A split flap increases lift by increasing

 the angle of attachment of the lower hinged portion


 the surface area
 the camber of the top surface

33. When the trailing edge flaps are lowered, the aircraft
will

 pitch nose up
 sink
 pitch nose down

34. Dutch roll is

 a type of slow roll


 a combined yawing and rolling motion
 primarily a pitching instability

35. On an aircraft with an all-moving tailplane, pitch up


is caused by

 increasing tailplane incidence


 decreasing tailplane incidence
 up movement of the elevator trim tab
36. A leading edge slat is a device for

 decreasing wing drag


 decreasing the stalling angle of the wing
 increasing the stalling angle of the wing

37. A Krueger flap is

 a leading edge slat which extends forward


 a leading edge flap which hinges forward
 a flap which extends rearwards but does not lower

38. The layer of air over the surface of an aerofoil which


is slower moving, in relation to the rest of the
airflow, is known as

 boundary layer
 none of the above are correct
 camber layer

39. When airflow velocity over an upper cambered surface of


an aerofoil decreases, what takes place?

 Pressure increases, lift increases


 Pressure increases, lift decreases
 Pressure decreases, lift increases

40. What is a controlling factor of turbulence and skin


friction?

 Aspect ratio
 Countersunk rivets used on skin exterior
 Fineness ratio
Answers for Questions 31 – 40

1) right spoilers extending, left spoilers remaining


retracted

2) the angle of attachment of the lower hinged portion


Comment/Reference: Jeppesen A & P Technician Airframe
Textbook page 1-30 & AC65-15A pg 48

3) pitch nose down


Comment/Reference: Due to the centre of pressure moving
aft

4) a combined yawing and rolling motion

5) decreasing tailplane incidence

6) increasing the stalling angle of the wing

7) a leading edge flap which hinges forward


Comment/Reference: Jeppesen A & P Airframe Technician
Textbook page 1-37 figure 1-64

8) boundary layer
Comment/Reference: AC65-15A pg 48

9) Pressure increases, lift decreases

10) Fineness Ratio


Comment/Reference: AC65-15A pg 32 under ‘Shape of the
Airfoil’ third paragraph says Fineness ratio is the
main controlling factor.
Questions 41 – 50

41. Changes in aircraft weight

 will only affect total drag if the lift is kept


constant
 will not affect total drag since it is dependant
only upon speed
 cause corresponding changes in total drag due to the
associated lift change

42. When an aircraft stalls

 lift decreases and drag increases


 lift and drag increase
 lift increases and drag decreases

43. The aircraft stalling speed will

 only change if the MTWA were changed


 increase with an increase in weight
 be unaffected by aircraft weight changes since it is
dependant upon the angle of attack

44. In a bank and turn

 extra lift is not required


 extra lift is required
 extra lift is not required if thrust is increased

45. The angle of attack at which stall occurs

 cannot be varied, it is always constant


 can be varied by using flaps and slats
 depends on the weight of the aircraft
46. The primary function of a flap is

 to alter the lift of an aerofoil


 to trim the aircraft longitudinally
 to alter the position of the centre of gravity

47. The stalling speed of an aircraft

 is increased when it is lighter


 does not change
 is increased when it is heavier

48. A wing flap which has 'dropped' or partially extended


on one wing in flight will lead to

 a pitching moment which would be corrected by used


of the elevators
 a steady rolling tendency which would be corrected
by use of the ailerons
 a fixed banked attitude which would be corrected by
use of the rudder

49. With an increase in the amount of flap deployment, the


stalling angle of a wing

 decreases
 increases
 remains the same

50. Downward displacement of an aileron

 increases the angle at which its wing stalls


 has no effect on its wing stalling angle, it only
affects the stalling speed on that wing
 decreases the angle at which its wing will stall
Answers for Questions 41 – 50

1) cause corresponding changes in total drag due to the


associated lift change

2) lift decreases and drag increases

3) increase with an increase in weight


Comment/Reference:
http://en.wikipedia.org/wiki/Stall_(flight)

4) extra lift is required


Comment/Reference:
http://en.wikipedia.org/wiki/Wing_loading

5) can be varied by using flaps and slats


Comment/Reference:
http://en.wikipedia.org/wiki/Stall_(flight)

6) to alter the lift of an aerofoil


Comment/Reference: Jeppesen A & P Technician Airframe
Textbook page 1-30 & AC65-15A pg 47 ‘High-Lift Devices’

7) is increased when it is heavier


Comment/Reference:
http://en.wikipedia.org/wiki/Stall_(flight)

8) a steady rolling tendency which would be corrected by


use of the ailerons

9) decreases

10) decreases the angle at which its wing will stall


Comment/Reference: An aileron displacement has a
similar effect on stall angle as the deployment of
flaps
Questions 51 – 60

51. Due to the tailplane angle of attack change, the flap-


induced downwash on the tailplane

 will tend to cause an aircraft nose down pitch


 may cause a nose-down or nose-up pitch depending
upon the initial tailplane load
 will tend to cause an aircraft nose-up pitch

52. Due to the change in lift coefficient accompanying


extension of the flaps, to maintain the lift constant
it would be necessary to

 keep the pitch attitude constant


 raise the nose
 lower the nose

53. Which leading edge device improves the laminar flow


over the wing?

 Flap and slat


 Flap
 Slat

54. The tropopause exists at about

 36,000 ft.
 18,000 ft.
 30,000 ft

55. Induced drag curve characteristics of a slender delta


wing are such that there is

 decrease in gradient with wing speed


 no change in gradient with wing speed
 an increase in gradient with wing speed
56. If an aircraft is yawing left, to correct, the trim
tab on the rudder would need to be positioned

 to the centre
 to the right, moving the rudder left
 to the left, moving the rudder right

57. Instability giving roll and yaw

 is dutch roll
 is lateral stability
 is longitudinal stability

58. Vortex generators are fitted to

 move transition point rearwards


 move transition point forwards
 advance the onset of flow separation

59. Leading edge flaps

 decrease stalling angle of the wing


 do not change the stalling angle
 increase stalling angle of the wing

60. Krueger flaps are on

 the trailing edge


 the leading edge
 either the leading or training edge
Answers for Questions 51 – 60

1) will tend to cause an aircraft nose-up pitch


Comment/Reference:
http://en.wikipedia.org/wiki/Downwash

2) lower the nose


Comment/Reference:
http://en.wikipedia.org/wiki/Flap_(aircraft)

3) Slat
Comment/Reference: AC65

4) 36,000 ft.

5) decrease in gradient with wing speed


Comment/Reference:
http://en.wikipedia.org/wiki/Lift-to-drag_ratio

6) to the left, moving the rudder right

7) is Dutch roll

8) move transition point forwards

9) increase stalling angle of the wing


Comment/Reference: http://en.wikipedia.org/wiki/Slats

10) the leading edge


Questions 61 – 70

61. Sweepback will

 decrease lateral stability


 not affect lateral stability
 increase lateral stability

62. A plain flap

 does not increase the wing area on deployment


 forms part of lower trailing edge
 is attached to the leading edge of the wing

63 A split flap, when deployed

 increases drag with little lift coefficient


increase, from intermediate to fully down
 is used only on high speed aircraft
 increases lift without a corresponding increase in
drag

64. Air above Mach 0.7 is

 incompressible
 compressible only when above the speed of sound
 compressible

65. Supersonic air passing through a divergent duct causes


the

 pressure to decrease, velocity to increase


 pressure to increase, velocity to increase
 pressure to increase, velocity to decrease
66. An aircraft flying below the tropopause descends at a
constant True Airspeed. Its Mach. No. will

 not change
 decrease
 increase

67. A nose down change of trim (tuck-under) occurs due to


shock induced

 tip stall on a delta wing aircraft


 root stall on a delta wing aircraft
 tip stall on a straight wing aircraft

68. A symmetrical aerofoil is accelerating through Mach 1


with an angle of attack of 0°. A shock wave will form

 on the upper and lower surface and will move aft


 on the upper and lower surface and will move aft
until the point of maximum camber
 on the upper surface only and move aft

69. At speeds greater than mach 1, airflow in the boundary


layer is

 subsonic
 supersonic
 stationary

70. Before an aircraft reaches critical mach

 the nose pitches down because the CP moves rear


 the aircraft buffets because the CP moves to the
shock wave
 the nose pitches up because the CP moves Forward
Answers for Questions 61 – 70

1) increase lateral stability


Comment/Reference:
http://en.wikipedia.org/wiki/Swept_wing

2) does not increase the wing area on deployment

3) increases drag with little lift coefficient increase,


from intermediate to fully down
Comment/Reference:
http://en.wikipedia.org/wiki/Flap_(aircraft)

4) compressible
Comment/Reference: Aircraft Flight Barnard and
Philpot, 2nd Ed Page123. Mechanics of Flight AC Kermode
10th Ed Page 385

5) pressure to decrease, velocity to increase


Comment/Reference: Mechanics of Flight AC Kermode 10th
Ed Page 340 & AC65-15A pg 59

6) decrease
Comment/Reference: Speed of sound INCREASES with
DECREASING altitude, so mach number will decrease.
Comment/Reference:
http://en.wikipedia.org/wiki/Mach_number

7) root stall on a delta wing aircraft


Comment/Reference: Pallett Automatic Flight Control 2nd
Ed Page 45 & http://en.wikipedia.org/wiki/Mach_tuck

8) on the upper and lower surface and will move aft


Comment/Reference: Shockwaves form on upper and lower
surfaces BEGINNING at point of max curvature, gradually
moving back.

9) subsonic
Comment/Reference: The definition of the boundary layer
is 'airflow ranges from zero to 99% of free flow speed'
However, by same definition, some flow in the boundary
layer is ALWAYS less than Mach 1
http://en.wikipedia.org/wiki/Boundary_layer

10) the nose pitches down because the CP moves rear


Comment/Reference: Mechanics of Flight 10th Ed Barnard
and Philpott Pg 341
Questions 71 – 80

71. On a standard day, at which altitude will the speed of


sound be the greatest?

 20,000 ft
 10,000 ft
 Sea level

72. Which of the following will increase the Critical Mach


Number of an aerofoil?

 Increasing the aspect ratio of the wings


 Decreasing the fineness ratio of the wings
 Using a thin airfoil and sweeping the wings back

73. As an aircraft accelerates through the transonic


region, the centre of pressure tends to

 move rearward
 move forward
 turn into a shock wave

74. Supersonic air going through an incipient shock wave


will decrease its speed and

 increase temperature and decrease density


 decrease temperature and increase density
 increase temperature and increase density

75. On an aircraft travelling at speeds greater than Mach 1


the boundary layer is

 stationary
 supersonic
 thicker than if the aircraft were travelling slower
than Mach 1
76. At speeds above Mach 1, shockwaves will form above and
below the wing

 at the leading edge


 at the trailing edge
 at both the leading edge and the trailing edge

77. A high speed aircraft with MACH warning will

 warn when coming to limits of upper envelope


 warn when Mach 1 has been exceeded
 warn when Mcrit is reached

78. Above the critical mach number, the drag coefficient

 remains the same


 increases
 decreases

79. At high Mach Numbers above Mach 2.2, some aircraft


metals

 such as aluminium, become brittle


 will shrink due to the extreme pressures involved
 lose their strength due to the kinetic heating
effect

80. Mach trim operates

 along the longitudinal axis


 to reduce Dutch roll
 along the lateral axis
Answers for Questions 71 - 80

1) Sea level

2) Using a thin airfoil and sweeping the wings back


Comment/Reference:
http://en.wikipedia.org/wiki/Critical_Mach_number

3) move rearward
Comment/Reference:
http://en.wikipedia.org/wiki/Mach_tuck

4) increase temperature and increase density


Comment/Reference: Mechanics of Flight AC Kermode 10th
Ed Page 326 fig 11.4 & AC65

5) stationary
Comment/Reference: Boundary layer at supersonic speeds
is very thin and considered negligible. Part of it is
stationary. www.wikipedia.org

6) at both the leading edge and the trailing edge


Comment/Reference: AC65-15A pg 59-60
http://en.wikipedia.org/wiki/Shock_wave

7) warn when Mcrit is reached


Comment/Reference: Mach overspeed warning (aka
'henpecker' - cluck, cluck, cluck) is activated at Mcrit
(shown by the barber pole on some ASIs)

8) increases
Comment/Reference:
http://en.wikipedia.org/wiki/Wave_drag

9) lose their strength due to the kinetic heating effect


Comment/Reference:
http://en.wikipedia.org/wiki/Aerodynamic_heating

10) along the longitudinal axis


Comment/Reference:
www.aerospaceweb.org/question/dynamics/q0137.shml
Questions 81 – 90

81. To increase critical mach number

 the wings are swept


 tailerons are fitted
 elevons are fitted

82. An aircraft experiences a large loss of lift and a big


increase in drag in straight and level flight, what
would be the most probable cause?

 Severe head winds


 Atmospheric conditions
 Aircraft reached its critical mach number

83. A Mach Trimmer is a device which

 switches out trim control to prevent damage in the


transonic speed range
 prevents the aircraft from exceeding its critical
Mach No
 automatically compensates for trim changes in the
transonic region

84. Critical Mach No. may be increased by

 sweeping back the wing


 using more powerful engines
 using a higher thickness/chord ratio wing

85. Airflow either side of a normal shock wave is

 sonic upstream and downstream


 subsonic upstream, sonic downstream
 sonic upstream, subsonic downstream
86. Mach Number is defined as

 speed of sound at sea level divided by local speed


of sound
 IAS divided by the local speed of sound
 TAS divided by local speed of sound

87. The reason for sharp leading edged wings on high speed
aircraft is to

 decrease boundary layer


 decrease wave drag
 enable the shockwave to be accurately positioned

88. Critical Mach Number is defined as

 that free-stream Mach Number at which some part of


the airflow over the aircraft becomes sonic
 that number at which the airflow becomes supersonic
 the minimum mach number at which the aircraft can go
supersonic

89. The transonic region is a region of

 mixed airflow
 all subsonic
 all supersonic

90. Immediately downstream of an oblique shockwave is


always

 supersonic
 the same as upstream
 subsonic
Answers for Questions 81 – 90

1) the wings are swept


Comment/Reference:
http://en.wikipedia.org/wiki/Swept_wing

2) Aircraft reached its critical mach number


Comment/Reference:
http://en.wikipedia.org/wiki/Critical_Mach_number

3) automatically compensates for trim changes in the


transonic region
Comment/Reference: Pallet, Auto Flt Control 4th Ed, pg
50 & www.aerospace.org

4) sweeping back the wing


Comment/Reference:
http://en.wikipedia.org/wiki/Swept_wing

5) sonic upstream, subsonic downstream


Comment/Reference: AC65-12A

6) TAS divided by local speed of sound


Comment/Reference: Jeppesen A&P Technician Airframe
Textbook Page 66 & AC65

7) enable the shockwave to be accurately positioned

8) that free-stream Mach Number at which some part of the


airflow over the aircraft becomes sonic
Comment/Reference:
http://en.wikipedia.org/wiki/Critical_mach

9) mixed airflow
Comment/Reference: AC65-12A

10) supersonic
Comment/Reference: AC65-12A
Questions 91 – 100

91. Wave drag

 increases in the supersonic region


 increases in the transonic region
 increases at the low speed stall

92. For increased Mcrit

 decrease sweepback
 decrease true airspeed
 decrease thickness/chord ratio

93. Symptoms of shock stall are

 buffet, loss of control, and instability


 decrease in speed, buffet and movement of the centre
of pressure
 compressibility effects, buffet and loss of control

94. Sweepback increases Mcrit by

 decreasing the amount of airflow over the lowest


point on the aerofoil section
 decreasing the amount of airflow over the highest
point on the aerofoil section
 increasing the amount of airflow over the highest
point on the aerofoil section

95. Mach number is

 the ratio of the aircraft's TAS to the speed of


sound at sea level
 the ratio of the aircraft's TAS to the speed of
sound at the same atmospheric conditions
 the ratio of the aircraft's IAS to the speed of
sound at the same atmospheric conditions
96. The critical Mach number is

 the Mach No. when a shock wave forms at the leading


edge
 the Mach No. when the aircraft reaches the speed of
sound
 the aircraft Mach. No. when the airflow reaches the
speed of sound at some point on the aircraft

97. Above the Critical Mach No. the drag coefficient will

 start to decrease
 remain the same
 start to increase

98. A wing of low thickness/chord ratio, the Critical Mach


No. will be

 higher than a wing of high thickness/chord ratio


 the same as a wing of high thickness/chord ratio
 lower than a wing of high thickness/chord ratio

99. An aeroplane flying above the Critical Mach No. will


usually experience

 a nose down pitch


 a nose up pitch
 an oscillation in pitch

100. Tuck-under can be counteracted by

 aileron reversal
 trim tabs
 mach trim
Answers for Questions 91 – 100

1) increases in the transonic region


Comment/Reference:
http://en.wikipedia.org/wiki/Wave_drag

2) decrease thickness/chord ratio


Comment/Reference:
http://selair.selkirk.bc.ca/aerodynamics1/High-
Speed/Page2e.html

3) buffet, loss of control, and instability


http://aerodyn.org/HighSpeed/shock-stall.html

4) decreasing the amount of airflow over the highest point


on the aerofoil section
Comment/Reference: see question 2 link

5) the ratio of the aircraft's TAS to the speed of sound


at the same atmospheric conditions
Comment/Reference: Flight Instruments and Automatic
Flight Control, David Harris Page 19
http://en.wikipedip.ort/wiki/Mach_number

6) the aircraft Mach. No. when the airflow reaches the


speed of sound at some point on the aircraft
Comment/Reference:
http://en.wikipedia.org/wiki/Critical_mach

7) start to increase
Comment/Reference:
http://en.wikipedia.org/wiki/Wave-Drag

8) higher than a wing of high thickness/chord ratio


Comment/Reference: see question 2 link

9) a nose down pitch


Comment/Reference:
http://selair.selkirk.bc.ca/aerodynamics1/High-
Speed/Page6.html

10) mach trim


Comment/Reference: see question 9 link
Questions 101 – 110

101. What causes tuck-under?

 Shock stall
 Aileron reversal
 Flap-back effect

102. When does a shock stall occur?

 When the aircraft forward speed is above Mach One


 At the critical Mach number of the aeroplane
 When the aircraft reaches speed of sound in a dive

103. With an increase in altitude under I.S.A. conditions,


the temperature in the troposphere

 increases
 remains constant
 decreases

104. Air either side of an oblique shockwave is generally

 supersonic
 sonic
 subsoni

105. Downstream of a normal shock wave

 pressure and temperature decrease


 pressure decreases temperature increases
 pressure and temperature increase
106. Speed of sound varies with

 temperature
 pressure
 altitude

107. Immediately downstream of a normal shockwave, air is


always

 the same as upstream


 subsonic
 supersonic

108. Increased sweepback

 improves tip stall characteristics


 decreases stability
 raises Mcrit

109. Aerodynamic heating

 increases as a function of airspeed


 decreases with altitude
 increases with skin friction

110. To overcome ineffective control surface problems in the


transonic region

 hydraulic powered elevators may be used


 an all moving tailplane may be used
 Frise ailerons may be used
Answers for Questions 101 – 110

1) Shock stall
Comment/Reference:
http://en.wikipedia.org/wiki/Mach_tuck

2) At the critical Mach number of the aeroplane


Comment/Reference:
http://aerodyn.org/HighSpeed/shock-stall.html

3) decreases
Comment/Reference:
http://en.wikipedia.org/wiki/Troposphere

4) supersonic
Comment/Reference: AC65-12A

5) pressure and temperature increase


Comment/Reference: AC65-12A

6) temperature
Comment/Reference: AC65-12A

7) subsonic
Comment/Reference: AC65-12A

8) raises Mcrit
Comment/Reference:
http://selair.selkirk.bc.ca/aerodynamich1/High-
Speed/Page2e.html

9) increases as a function of airspeed


Comment/Reference:
http://en.wikipedia.org/wiki/Aerodynamic_heating

10) an all moving tailplane may be used


Comment/Reference:
http://en.wikipedia.org/wiki/Tailplane#Control
Questions 111 – 120

111. An aircraft flying below the tropopause descends at a


constant True Airspeed, its Mach. No. will

 remain the same


 decrease
 increase

112. To counter the effect of a shift of centre of pressure


as an aircraft flies through the transonic region, fuel
is pumped

 backwards
 sideways
 forwards

113. To make flying controls more effective at high speed

 area rule is used


 wing fences are used
 vortex generators are used

114. An aircraft flying above the tropopause descends at a


constant True Airspeed, its Mach. No. will

 remain the same


 increase
 decrease

115. The velocity of sound with an increase in altitude will

 increase
 decrease
 remain constant
116. Mach number equals the ratio of

 sonic speed to indicated airspeed


 altitude to airspeed
 true airspeed to local sonic speed

117. Tuck-under is caused by

 root stall on a swept wing aircraft


 tip stall on a straight wing aircraft
 tip stall on a swept wing aircraft

118. The purpose of sweepback on an aerofoil is to

 decrease drag
 decrease Mcrit
 increase Mcrit

119. As the airspeed over a cambered wing is increased, a


shock wave will appear initially

 at the trailing edge


 at the leading edge
 near the point of maximum curvature

120. In the transonic speed range

 the position of the wing centre of pressure remains


constant
 the centre of pressure initially moves forward, then
back
 the centre of pressure movement may become
oscillatory
Answers for Questions 111 – 120

1) decrease
Comment/Reference:
http://selair.selkirk.bc.ca/aerodynamics1/High-
Speed/Page2.html

2) backwards
Comment/reference: Pallet, Auto Flight Cont pg 51 &
http://selair.selkirk.bc.ca/aerodynamics1/High-
Speed/Page6.html

3) vortex generators are used


Comment/Reference: AC65-12A

4) remain the same


Comment/Reference: see question 1 link

5) decrease
Comment/Reference: Flight Instruments and Automatic
Flight Control Systems, David Harris Page 19 & see
question 1 link

6) true airspeed to local sonic speed


Comment/Reference: Flight Instruments and Automatic
Flight Control Systems, David Harris Page 19 & see
question 1 link

7) root stall on a swept wing aircraft


Comment/Reference:
http://en.wikipedia.org/wiki/Mack_Tuck

8) increase Mcrit
Comment/Reference:
http://selair.selkirk.bc.ca/aerodynamics1/High-
Speed/Page2e.html

9) near the point of maximum curvature


Comment/Reference:
http://selair.selkirk.bc.ca/aerodynamics1/High-
Speed/Page2b.html

10) the centre of pressure movement may become oscillatory


Comment/Reference: see question 7 link
Questions 121 – 130

121. Zone 320 under the ATA system is

 horizontal stabiliser
 central fuselage
 vertical stabilizer

122. When doing a bonding check the maximum resistance


between component and earth is

 1/50 ohms
 50 milliohms
 0.005ohms

123. The fuel drain of a combustion heater must be protected


from

 lightning strike
 ice build-up
 accidental damage

124. Engine mounts provide for

 lateral movement
 torsional movement
 longitudinal movement

125. Dyna engine mounts

 prevent all vibration from engine to airframe


 reduce vibration from engine to airframe
 are constructed from plastic
126. How is damage to the nose cone of an aircraft
prevented during a lightning strike?

 Special paint
 Bonding Strip
 Earthing Strap

127. Tension is the stress of

 twisting
 crush or compression
 elongating or stretch

128. A Fuselage Station is a

 longitudinal point on the aircraft fuselage


 lateral point on aircraft fuse
 lateral point on aircraft wing

129. Composite materials are bonded by

 aluminium wire
 special paint
 copper wire

130. ATA Zone 100 is

 upper fuselage
 left-hand wing
 lower fuselage
Answers for Questions 121 – 130

1) vertical stabiliser
Comment/Reference: Aircraft Maintenance and Repair
Kroes/Watkins/Delp Page 22

2) 50 milliohms
Comment/Reference: CAIPs EEL/1-6 3.8

3) ice build-up
Comment/Reference: CS 29.859 (i) (2)

4) longitudinal movement
Comment/Reference: Jeppesen Helicopter Maintenance Page
269

5) reduce vibration from engine to airframe


Comment/Reference:

6) Bonding Strip
Comment/Reference: Aircraft Electricity and Avionics
5th Ed Eismin Page 343 &
www.lightningdiversion.com/Strips.htm

7) elongating or stretch
Comment/Reference: AC65-15A pg 2. Tension is the stress
of elongation or stretch.

8) longitudinal point on the aircraft fuselage


Comment/Reference: AC65-15A pg 6. Fuselage Stations are
longitudinal measurements on the fuselage

9) special paint
Comment/Reference: CAAIPs Leaflet 9-1 3.4.4

10) lower fuselage


Comment/Reference: ATA Zone 100 is lower fuselage
(below floor) ATA chapter 06-20 (A319)
Questions 131 – 140

131. The bonding lead to a remote aircraft component must be

 22 AWG
 0.5 in wide
 0.25 in wide and 22 AWG

132. What force is an I-Beam subjected to?

 Tension
 Bending
 Shear

133. A radome is protected from static electricity build-up


by

 special conductive grease


 conductive paint
 bonding strips

134. Precise points are located on an aircraft by a system


of

 longitudinal, vertical and lateral lines


 frame stations, vertical lines and lateral lines
 frame stations, water lines and buttock lines

135. Which of the following is an example of a fail safe


structure?

 Spar
 Longeron
 Stringer
136. Damage tolerant design

 is applied only to secondary structure


 allows for certain damage to the structure to go un-
repaired between scheduled maintenance
 allows for damage to structure by distributing loads
to other structure

137. In the ATA 100 zonal system the passenger entry door
will have a designation of

 600
 800
 400

138. Which area of the aircraft is subject to hoop stress?

 Control surfaces
 Pressure cabin
 Wings

139. Shear stress is described as

 pulling forces
 compressing forces
 slip away under the action of forces

140. The ground cable must be

 single strand copper wire 0.5 in. cross sectional


area
 single strand 18 AWG
 copper stranded 0.5 in. cross sectional area
Answers for Questions 131 – 140

1) 0.5 in wide
Comment/Reference: CAAIPs Leaflet 9-1 Para. 3.3.1 a)i)

2) Bending
Comment/Reference: An I beam is subject to Bending,
although different parts of it are subject to tension
(upper boom) and shear (the web)

3) conductive paint
Comment/Reference: CAIPs RL/2-5 Para 3.5

4) frame stations, water lines and buttock lines


Comment/Reference: AC65-15A pg 6 & CAIPs AL/7-2 para 6

5) Stringer
Comment/Reference: Answer chosen due to a stringer's
duplicity.

6) allows for certain damage to the structure to go un-


repaired between scheduled maintenance

7) 800
Comment/Reference: Checkout most modern aircraft
Maintenance Manuals for zonal locations

8) Pressure cabin
Comment/Reference: CAIPs AL/7-2 2.7

9) slip away under the action of forces


Comment/Reference: AC65-15A pg 2. The keyword is
'slip', but it is a terrible definition of shear
stress.

10) single strand copper wire 0.5 in. cross sectional area
Comment/Reference: CAIPs EEL/1-6 3.3.1 a (i)
Questions 141 – 150

141. Where on the aircraft is FS245, RWS45?

 245 inches from the nose of the aircraft and 45


inches from the centreline of the right wing
 245 inches from the datum line of the aircraft and
45 inches from the centreline of the right wing
 245 inches from the nose of the aircraft and 45
inches from the tip of the right wing

142. How is the radome protected from lightning strike?

 Special paint
 Diverter strips
 Special grease on the hinges

143. If you short the two prongs with the single prong of a
bonding tester together, what would the gauge read?

 Zero
 Centre scale
 Full scale deflection

144. A condition after which a permanent deformation of a


material is caused, is known as

 shear
 strain
 bending

145. Semi-monocoque construction

 utilizes the safe-life design concept


 offers good damage resistance
 is used only for the fuselage
146. Most radio aerials are

 bonded
 not bonded
 insulated from the fuselage

147. Secondary bonding is usually provided with

 stranded copper 0.25 inch


 18 AWG
 single strand 0.25 inch

148. Water Lines (WLs) are measured points on a

 horizontal line
 wing line
 vertical line

149. The various parts of the aircraft airframe are


maintained at the same potential by

 static wicks
 earthing
 bonding

150. The cross sectional area of a secondary conductor must


be not less than

 22 SWG x 0.25
 22 SWG x 0.5
 18 SWG for a single wire
Answers for Questions 141 – 150

1) 245 inches from the datum line of the aircraft and 45


inches from the centreline of the right wing
Comment/Reference: CAIPs AL/7-2 fig 15

2) Diverter strips
Comment/Reference: CAIPs RL/2-5 3.4.2

3) Zero
Comment/Reference: CAP 562 leaflet 9-1 para 3.10.2 b) &
CAIPs EEL/1-6 3.10.2 B

4) strain
Comment/Reference: Strain is 'best' of the answers.
Strain is 'deformation' but does not necessarily cause
a permanent deformation.

5) offers good damage resistance


Comment/Reference: Jepessen A & P Technician Airframe
textbook page 1-3

6) bonded
Comment/Reference: Jeppesen A&P Technician Airframe
Textbook 12-56

7) 18 AWG
Comment/Reference: CAAIPs Leaflet 9-1 3.3.1 (a) (ii)

8) vertical line
Comment/Reference: AL/7.2 Page 6.2 Fig 15

9) bonding
Comment/Reference:
http://en.wikipedia.org/wiki/Electrical_bonding

10) 18 SWG for a single wire


Comment/Reference: CAAIPs Leaflet 9-1 3.3.1 (ii)
Questions 151 – 160

151. What governs the ultimate fatigue life of an aircraft?

 Landings
 Flying Hours
 Pressure Cycles

152. The main forces on an aircraft structure are

 tension, compression, torsion and strain


 tension, compression, torsion and shear
 tension, compression, twisting and shear

153. The life of the structure is counted by

 flying hours
 landings
 pressurization cycle

154. What are buttock lines?

 Horizontal measurement lines


 Measurements from the centre line
 Vertical measurement lines

155. Aircraft fibreglass panels are protected against


lighting strikes, partially by

 bonding
 non-conductive paint
 conductive pain
156. A member taking a compression load is called a

 beam
 cable
 strut

157. Stringers are used in which of the following types of


aircraft fuselage construction?

 Semi-monocoque
 Monocoque
 Truss type

158. Wing stations are measured

 inboard from the wing upper surface


 outboard from the fuselage centreline
 outboard from the wing root

159. What load is a tie rod designed to accept?

 Bending
 Tensile
 Torsion

160. If a colour is used to identify primary structure, it


will be

 yellow
 red
 green
Answers for Questions 151 – 160

1) Flying Hours
Comment/Reference:
http://en.wikipedia.org/wiki/Air_safety#Metal_fatigue

2) tension, compression, torsion and shear


Comment/Reference: AC65-12A

3) pressurization cycle
Comment/Reference:

4) Measurements from the centre line


Comment/Reference: AC65-15A pg 6 & CAIPs AL/7.2 Page 17
Para 6.2

5) conductive paint
Comment/Reference:

6) strut
Comment/Reference:
http://en.wikipedia.org/wiki/Strut

7) Semi-monocoque
Comment/Reference: AC65-15A pg 3

8) outboard from the fuselage centreline


Comment/Reference:
www.aerospaceweb.org/question/design/q0289.shml

9) Tensile
Comment/Reference:
http://en.wikipedia.org/wiki/Tie_rod

10) red
Comment/Reference:
Questions 161 – 170

161. Which of the following is primary structure?

 Skin
 Frame
 Stringer

162. Fuselage station numbers are measured from the front of


the aircraft in

 feet and inches


 inches
 feet

163. Which parts of the aircraft are classified secondary


structures?

 These parts of the airframe are highly stressed and


if damaged may cause failure of the aircraft and
loss of life
 These parts of the airframe are highly stressed but
if damaged will not cause failure of the aircraft
 These are lightly stressed parts such as fairings,
wheel shields and minor component brackets etc.

164. Structure with built-in redundancy is called

 safe life
 failsafe
 double safe

165. Stress

 is the load per unit area acting on a material


 is the deformation of a material caused by applied
load
 is the property of a material to resist fracture
166. A piece of structure which must be replaced at a
specified number of cycles, flying hours or years,
regardless of its physical condition is what type of
item?

 Condition monitored
 Safe-life
 Fail-safe

167. If you are unable to identify a structure


'classification' as either Primary or Secondary, what
action should you adopt?

 Upgrade it to primary
 Grade it as secondary
 Paint it red and stamp it as 'tertiary'

168. Safe-life is

 the maximum number of flying hours that should


elapse before a major structural failure occurs
 the minimum number of flying hours that should
elapse before a major structural failure occurs
 the sharing of loads between adjacent members

169. Bending stresses are a combination of

 torsional and compression stresses


 tension and shear stresses
 tension and compression stresses

170. The Airworthiness Notice that refers to structural


surveys is

 Notice 79
 Notice 89
 Notice 65
Answers for Questions 161 – 170

1) Skin
Comment/Reference:

2) inches
Comment/Reference: AC65-12A

3) These parts of the airframe are highly stressed but if


damaged will not cause failure of the aircraft
Comment/Reference:

4) failsafe
Comment/Reference:

5) is the load per unit area acting on a material


Comment/Reference: AL/7-2 2.2

6) Safe-life
Comment/Reference:

7) Upgrade it to primary
Comment/Reference:

8) the minimum number of flying hours that should elapse


before a major structural failure occurs
Comment/Reference: Safe life maybe determined by using
a similar structure which is tested to establish the
minimum number of events which should elapse before a
major structural failure occurs

9) tension and compression stresses


Comment/Reference: AC65-15A pg 2 fig 1-1

10) Notice 89
Comment/Reference: These AWNs are now transferred to
CAP747
Questions 171 – 180

171. Structural survey inspections are normally called up by


the

 operator
 manufacturer
 maintenance engineer

172. Where are wing stations measured from?

 Zone Line (ZL)


 Butt Line (BL)
 Water Line (WL)

173. Where is Zone 323?

 Between rear spar and trailing edge


 Between front and rear spar
 Tip of vertical stabilizer

174. To prevent a system being affected by high current


flows after a lightning strike to a composite aircraft,
electricity is discharged through

 a sprayed coat of conductive paint


 a sprayed coat of non-conductive paint
 electrically connected primary conductors

175. A structural member intended to resist compression is a

 tie
 strut
 web
176. An aircraft structure, having multiple load paths, is
known as a

 fail-safe design
 monocoque design
 safe-life design

177. Wrinkling of the skin on the upper surface of the


fuselage indicates

 shedding
 sagging
 hogging

178. If a redundant structure fails it becomes

 fatigued
 failsafe
 safe-life

179. A redundant structure is

 a failsafe structure
 on-condition structure
 a safe-life structure

180. The measurement of the fuselage perpendicular to


horizontal plane measured in inches from bottom of the
fuselage is

 fuselage station
 butt line
 water line
Answers for Questions 171 – 180

1) manufacturer
Comment/Reference: AWN 89

2) Butt Line (BL)


Comment/Reference: Obscure, but the fuselage
centreline is 'technically' a buttock line of sorts.
www.aerospaceweb.org

3) Between front and rear spar


Comment/Reference: Airbus A320 AMM 06-20-00-00 shows
zone 323 as between the front and rear spar of the
vertical stab. Zone 324 is the tip of the vertical
stab.

4) electrically connected primary conductors


Comment/Reference:

5) strut
Comment/Reference:
http://en.wikipedia.org/wiki/Strut

6) fail-safe design
Comment/Reference:
http://en.wikipedia.org/wiki/Fail_safe

7) sagging
Comment/Reference:
http://en.wikipedia.org/wiki/Hogging_and_sagging

8) safe-life
Comment/Reference: See Dan Air Boeing 707 (Zambia)
crash report Para 2.3.1
http://www.geocities.com/CapCanaveral/9575/770514-0.htm

9) a failsafe structure
Comment/Reference: AC65-15A pg 9

10) water line


Comment/Reference: AC65-15A pg 6
Questions 181 – 190

181. What are the four stresses to be considered when


building an aircraft?

 Compression, Torsion, Tension, Shear


 Compression, Tension, Torsion, Stress
 Compression, Torsion, Stress, Strain

182. A structure that has a high designed reserve strength


would be classified as

 secondary
 primary
 tertiary

183. An example of primary stress is

 shear
 tension
 bending

184. What is the water line?

 The zero datum from which all lateral locations are


measured
 A line below which redux bonding can not be used
 The datum from which vertical locations refer

185. Lateral stations have station zero at the

 nose
 centre line
 left wing tip
186. Airworthiness requirements for large aircraft are found
in

 ANO 25
 JAR 25
 CS 25

187. An Anthropomorphic Test Dummy (ATD) is strapped into a


large aircraft forward facing seat. It is put through a
series of crash tests. This is to

 to measure the amount of force applied to the


abdomen of the ATD to ensure it is not above 236 kg.
 test the aircraft structure and seating mount points
for structural integrity.
 to determine whether or not the ATD\'s head comes
into contact with any structure or seat, and if so
to measure the force applied to the head in line
with a specific Head Injury Criterion (HIC)

188. Diffusion bonding and superplastic forming provides

 high strength and high stiffness


 high stiffness and high ductility
 high strength and high ductility

189. Hi-lok fastener installation requires that

 the fastener is accurately torqued


 an interference fit hole is drilled
 the shank is lubricated

190. What kind of seal is used on firewall bulkheads?

 Fire-proof grommets
 Soft rubber
 None is required
Answers for Questions 181 – 190

1) Compression, Torsion, Tension, Shear


Comment/Reference: AC65-15A pg 1

2) primary
Comment/Reference:

3) tension
Comment/Reference:

4) The datum from which vertical locations refer


Comment/Reference: AC65-15A pg 6

5) centre line
Comment/Reference:

6) CS 25
Comment/Reference: JAR 25 is replaced by EASA
Certification Specification CS 25

7) to determine whether or not the ATD\'s head comes into


contact with any structure or seat, and if so to
measure the force applied to the head in line with a
specific Head Injury Criterion (HIC)
Comment/Reference: CS 25.562 (c) para 5

8) high stiffness and high ductility


Comment/Reference:

9) an interference fit hole is drilled


Comment/Reference: Hiloks are pre-lubricated. Hole is
drilled 0.0015 inch interference. Hi-lok/Hi-tique
installation guide can be downloaded from Tutorial
Support Section.

10) Fire-proof grommets


Comment/Reference:
Questions 191 – 200

191. To remove a rivet you should

 drill half way through


 drill it all the way through
 drill to the depth of the head

192. The two stages in a good adhesive bond are

 spreading and setting


 wetting and Setting
 wetting and gripping

193. Prior to aluminium alloy bonding, we use

 alkaline etch
 acid etch
 solvent etch

194. The purpose of a primer is to

 provide shiny surface for the topcoat


 provide flexible surface for the top coat
 help bonding of the topcoat

195. In semi-monocoque construction, compression loads are


taken by

 bulkheads
 stringers
 frames
196. Most large transport aircraft skins are

 5056
 7075
 2024

197. Which of the following statements is correct, in


relation to PLI washers used in critical bolted joints?

 PLI washers can only be used with self locking nuts


and the washers should be used once
 PLI washers can be used more than once, providing
they are used in critical bolted joints
 PLI washers can be affected by thread or nut
friction or by lubrication

198. What opposes buckling in a semi-monocoque structure?

 Bulkheads
 Frames
 Stringers

199. In a monocoque structure, which component carries the


majority of the loads?

 Stringers
 Skin
 Longerons

200. Which anti-corrosive treatment is found on alloy


steels?

 Zinc plating
 Nickel plating
 Cadmium plating
Answers for Questions 191 – 200

1) drill to the depth of the head


Comment/Reference: CAAIPs Leaflet 6-4 3.7.1

2) wetting and Setting


Comment/Reference: A module 7 questions. The two
processes in adhesive bonding is 'wetting and setting'

3) acid etch
Comment/Reference: Phosphoric acid and chromic acid
wash

4) help bonding of the topcoat


Comment/Reference: CAIPs BL/6-20 PAra.2.1

5) stringers
Comment/Reference: CAIP AL/7-2 para 2.6 (last sentence)

6) 2024
Comment/Reference: Jeppesen A&P Technician Airframe
Textbook 2-8

7) PLI washers can only be used with self locking nuts and
the washers should be used once
Comment/Reference: CAIPs AL/7-8 Para 4.5.4
CAAIPs Leaflet 6-7 para 4.5.3 & 4.5.4

8) Stringers
Comment/Reference: CAIPs AL/7.2 para 3.3

9) Skin
Comment/Reference: A&P Mechanic Handbook Page 25-28

10) Cadmium plating


Comment/Reference: CAIPs BL/7-2
Questions 201 – 210

201. What material can be chromated as a protection against


corrosion?

 Aluminium alloys
 Ferrous alloys
 Magnesium alloys

202. When carrying out a symmetry check on a large aircraft,


what method of measurement is normally used?

 Longitudinal alignment method


 Lateral alignment method
 Steel tape and spring balance

203. Paint remover substances

 are not damaging to any aircraft parts


 are damaging to some aircraft parts
 should only be used once

204. To remove a rivet

 chisel off the rivet head, and remove the shank with
a metal punch
 drill the head with a drill bit slightly smaller
than the rivet shank, chisel off the rivet head, and
remove the shank with a metal punch
 drill the head with a drill bit the same size as the
rivet shank, chisel off the rivet head, and remove
the shank with a metal punch

205. Battery trays are

 absorbent to soak up electrolyte


 metal with PVC coating and anti corrosive paint
 metal for earthing purposes
206. The primary purpose of sealant in use in pressurized
aircraft is

 to seal the cabin


 to prevent corrosion
 to provide external streamlining

207. When installing a 'Hi-lock' bolt, it is necessary to

 lubricate the collar


 lubricate the shank and threads
 simply fit the bolt as they are pre-lubricated

208. Sealant or leveling compound is installed during


structure repair:

 according to SB instructions
 according to separate manufacturer's documentations
such as BAC
 according to AMM and SRM chapter 51

209. A bonded waffle doubler, as well as acting as a skin


strengthener, also acts as a

 tear stopper
 shear tie
 jury strut

210. What are the faying surfaces of a repair?

 Edges of repair metal


 Material under repair
 Middle of repair
Answers for Questions 201 – 210

1) Magnesium alloys
Comment/Reference: CAIPs BL/7-3

2) Steel tape and spring balance


Comment/Reference: CAP 562 Leaflet 6-5 para 3.3.5 a) &
CAIPs AL/7-12 3.3.5 i

3) are damaging to some aircraft parts


Comment/Reference: CAIPs BL/6-20 11.1

4) drill the head with a drill bit the same size as the
rivet shank, chisel off the rivet head, and remove the
shank with a metal punch
Comment/Reference: CAAIPs Leaflet 6-4 3.7.1 says 'drill
equal in diameter than that of the rivet'.

5) metal with PVC coating and anti corrosive paint


Comment/Reference: Aircraft Electrical Systems 3rd ed
Pallett Page 24

6) to seal the cabin


Comment/Reference: CAIP AL/7-2 para. 3.7 & AC65-9A pg
169 &
www.tpub.com/content/aviation/14022/css/14022_156.htm

7) simply fit the bolt as they are prelubricated


Comment/Reference:

8) according to AMM and SRM chapter 51


Comment/Reference: Chapter 51 is General Practices

9) tear stopper
Comment/Reference: CAIPs AL/7-2 (crack stopper band
fig.4)

10) Material under repair


Comment/Reference: Open to interpretation. We have gone
for 'material under repair' as the surfaces joined
together.
Questions 211 – 220

211. Dissimilar metal fusion bonding is best for

 high strength high toughness


 low strength high toughness
 high strength high stiffness

212. The ideal conditions for paint spraying an aircraft are

 15°C to 25°C and humidity below 75%


 20°C to 30°C and humidity below 70%
 15°C to 25°C and humidity above 60%

213. What are the types of true bonded joints?

 Cemented and specific


 Mechanical and cemented
 Mechanical and specific

214. Why is a joggle joint used?

 To provide a smooth contour to surface


 To provide a flush fit
 To add strength

215. When both sides of a structural repair are not easily


reached, which type of fastener would you use?

 Blind rivet
 Pop rivet
 Hi lock bolt
216. With regard to extraneous spilt fluids

 they can be ignored, they provide extra protection


 they should only be cleaned up if they are on the
external surface of the aircraft
 they may be harmful to the aircraft structure

217. Skin panels may be strengthened by

 struts
 cleats
 stringers

218. What is the normal form of construction of a spar?

 Two webs separated by a boom


 Two booms separated by a web
 One boom mounted under a web

219. A crack stopper is fitted

 before a crack starts, to prevent its initiation


 after a crack starts, to slow its rate of
propagation
 before a crack starts, to slow its rate of
propagation

220. Which of the following should be accomplished before


jacking an aircraft?

 Install critical stress panels or plates


 Remove all optional equipment
 Determine the fuel tanks are empty
Answers for Questions 211 – 220

1) high strength high stiffness


Comment/Reference:

2) 15°C to 25°C and humidity below 75%


Comment/Reference: BL/6-20 5 CAAIPs Leaflet 2-7 para 5

3) Mechanical and specific


Comment/Reference:

4) To provide a flush fit


Comment/Reference: AC65-15A pg 162, the joggle does
not provide the 'contour'.

5) Blind rivet
Comment/Reference: AC65-9A pg 155. Pop rivet is not a
blind fastener because the broken stem must be
retrieved

6) they may be harmful to the aircraft structure


Comment/Reference:

7) stringers
Comment/Reference:

8) Two booms separated by a web


Comment/Reference:

9) before a crack starts, to slow its rate of propagation


Comment/Reference:

10) Install critical stress panels or plates


Comment/Reference: AC65-9A pg 525
Questions 221 – 230

221. Synthetic resins are made from nylon, vinyl and

 cellulose
 asphalt
 acrylics

222. One of the advantages of the semi-monocoque


construction is that it

 shares the loads


 is easier to manufacture
 takes all the loads in the skin

223. In a fully monocoque fuselage, all the loads are


carried by the

 skin
 longerons
 frames

224. Intercostals are

 vertical struts joining the upper and lower wings of


a biplane
 compression ribs in cantilever wings
 longitudinal fuselage members attached at each end
to adjacent frames

225. Joints that are designed to stop the propagation of


cracks are known as

 failsafe joints
 secondary joints
 crack limiting joints
226. If an aircraft has alkaline batteries, the battery
compartment drain pipes will be

 plastic
 aluminium alloy
 stainless steel

227. An overweight landing is one in which the aircraft has

 an excessive fuel load on take-off


 too much kinetic energy on landing
 a missed placed centre of gravity on landing

228. Symmetry checks should be carried out

 on the ramp with the aircraft on its wheels


 in the hangar with the aircraft on its wheels
 in the hanger with the aircraft on jacks

229. The purpose of a wash primer and primer is to

 provide a flexible surface for top-coat


 provide more aerodynamic finish for top-coat
 help bonding for top-coat

230. Dents are generally not permitted in a tubular member

 if the major axis of the dent is parallel to the


tube axis irrespective of the location
 if they are located in the end thirds of the length
of the member
 if they are located in the middle third of the
length of the member
Answers for Questions 221 – 230

1) acrylics
Comment/Reference:

2) shares the loads


Comment/Reference: AC65-15A pg 3

3) skin
Comment/Reference: AC65-15A pg 3

4) longitudinal fuselage members attached at each end to


adjacent frames
Comment/Reference: See a Picture in B737 SRM chap 53

5) failsafe joints
Comment/Reference:

6) stainless steel
Comment/Reference:

7) too much kinetic energy on landing


Comment/Reference:

8) in the hanger with the aircraft on jacks


Comment/Reference: CAAIPs Leaflet 6-5 para 2

9) help bonding for top-coat


Comment/Reference: Jeppesen A&P Airframe Textbook. page
6-8

10) if they are located in the middle third of the length


of the member
Comment/Reference: AC43
Questions 231 – 240

231. When an item is cocooned, the visual indicator will


indicate

 toxic gases
 temperature
 humidity

232. The maximum permissible bow in a steel tube is

 1 in 400
 1 in 600
 1 in 200

233. Buckling in a semi-monocoque structure is prevented by

 longerons
 bulkheads
 stringers

234. Which loads do longerons resist?

 Bending, compression and tensile


 Bending, compression, tensile and torsion
 Torsional only

235. Blowout panels are fitted to floors of pressurised


aircraft to

 provide access for pressurising the aircraft


 provide access for pressurising the aircraft
 provide venting should a differential pressure build
up across the floor
236. Acrylic windows must be able to flex in-situ because
they have

 smaller coefficient of expansion than surrounding


structure
 larger coefficient of expansion than surrounding
structure
 slight shrinkage due to age

237. A cargo bay must have signs stating

 no hazardous cargo
 maximum dimensions
 maximum loadings

238. Zone D Cargo compartment windows

 must have blinds pulled down


 are made from fire retardant Perspex
 must be blanked off

239. A fire zone in which a crew member could enter to fight


a fire is

 class B
 class A
 class E

240. Windscreen delamination is

 bubbling of the glass due to overheat


 windscreen peeling away from the airframe
 separation of the glass from the vinyl interlayer
Answers for Questions 231 – 240

1) humidity
Comment/Reference:

2) 1 in 600
Comment/Reference: CAAIPs Leaflet 6-4 p13

3) stringers
Comment/Reference:

4) Bending, compression and tensile


Comment/Reference: AC65-15A pg 3-4

5) provide venting should a differential pressure build up


across the floor
Comment/Reference: Example DC10 Cargo door disaster.
Cargo compartment depressurised, cabin pressure caused
floor to collapse. Blowout panels made compulsory
thereafter

6) larger coefficient of expansion than surrounding


structure
Comment/Reference: Acrylic has a thermal expansion
approximately 8x that of aluminium alloy

7) maximum loadings
Comment/Reference: Usually, pallet size is fixed, but
the weight is critical.

8) must be blanked off


Comment/Reference: Transport Category Aircraft Systems
9-3

9) class B
Comment/Reference: Transport Category Aircraft Systems
Page 9-3 and CS 25.857 (b)

10) separation of the glass from the vinyl interlayer


Comment/Reference: CAIPs AL/7-10 Para 5.1.1
Questions 241 – 250

241. What is the ideal storage temperature for windscreen


panels?

 30°C to 50°C
 50°C to 70°C
 50°F to 70°F

242. What is the maximum temperature of warm water which is


used to clean dirty aircraft acrylic panels?

 40°F
 104°F
 125°F

243. How are passenger door pressure loads transmitted to


the fuselage structure?

 By the pressure seal


 By door stops
 Through the latching mechanism

244. Passenger cabin windows are constructed with two panes


and a seal between them. Under normal flight conditions
which pane(s) take the differential pressure loads?

 Inner
 Outer
 Both

245. How are passenger seats secured to the floor?

 By both front and back legs


 By the front legs while the back attachment provides
anti rattle.
 By the back legs while the front attachment provides
anti rattle
246. A material commonly used for pressure bulkhead sealing
is

 aluminium
 PTFE
 soft rubber

247. When acrylic and sheet metal is joined together,


allowance should be made for

 expansion of the acrylic due to heat


 galvanic corrosion of the metal
 expansion of the metal due to heat

248. Which of the following is Perspex resistant to?

 Acetone
 MEK
 Kerosene

249. In a pressurised aircraft cabin window, the


differential pressure can be taken by

 the inner pane only


 both main panes
 the scratch panel

250. The purpose of the vinyl interlayer in window


construction is

 to improve on brittle behaviour


 to enable inclusion of heater film
 to increase static strength
Answers for Questions 241 – 250

1) 50°F to 70°F
Comment/Reference: CAP 562 leaflet 1-8 para 3.21.3 a)

2) 104°F
Comment/Reference: CAIPs AL/7-4 Para 6.1 says 40°C,
which is 104°F

3) By door stops
Comment/Reference:

4) Outer
Comment/Reference: The inner panel has a small hole in
it to let the pressure through to the outer panel

5) By the back legs while the front attachment provides


anti rattle
Comment/Reference: The back legs are secured as they
have the upward load during a sudden stop.

6) soft rubber
Comment/Reference: CAIPs AL/7-2 para 3.9

7) expansion of the acrylic due to heat


Comment/Reference: AL/7-4 para 4

8) Kerosene
Comment/Reference: AL/7-4 2.3

9) both main panes


Comment/Reference:

10) to improve on brittle behaviour


Comment/Reference: CAIPs AL/7-10 3.1.1
Questions 251 – 260

251. On passenger aircraft of two or more zones;

 the seat backs fold forward at over wing emergency


exits
 there should be at least one exit per zone
 the emergency exits are to be clearly signed

252. Class D cargo bay windows require

 the blind down for flight


 fire proof coatings
 double acrylic panels

253. Cargo bay panels are normally constructed from

 monolithic Nomex
 Kevlar
 Fibreglass and resin

254. The fatigue life of a pressure hull is based on

 the number of negative differential pressure


applications
 the total number of pressurizations
 the number of explosive decompressions

255. In pressurized aircraft, undercarriage bays

 are unpressurised
 are pressurized
 may be pressurized or unpressurised
256. The main longitudinal members in a fuselage are called

 frames
 longerons
 spars

257. A partition within the aircraft's structure is called a

 cleat
 bulkhead
 frame

258. When inspecting stowage doors, special attention should


be given to the

 light microswitches
 trim
 catches

259. Seat track cutouts provide for various seat spacings,


usually in

 half inch increments


 one inch increments
 one foot increments

260. Wall mounted flight attendant seats are fixed to

 rails
 tracks
 stanchions
Answers for Questions 251 – 260

1) there should be at least one exit per zone


Comment/Reference: CS 25.783, but does not talk about
Zonal exits

2) fire proof coatings


Comment/Reference: CS 25.857 (d)

3) Fibreglass and resin


Comment/Reference: AWN 80 Para.2.3 and 4.1.1

4) the total number of pressurizations


Comment/Reference: CAIPs AL/3-23

5) are unpressurised
Comment/Reference:

6) longerons
Comment/Reference: AC65-15A pg 4

7) bulkhead
Comment/Reference:

8) catches
Comment/Reference:

9) one inch increments


Comment/Reference:

10) stanchions
Comment/Reference:
Questions 261 – 270

261. Windshields are heated to

 demist, ant-ice and maintain strength and toughness


 prevent misting and icing only
 maintain strength and toughness only

262. The internal doors of an aircraft

 carry pressure loads


 never carry pressure loads
 only carry pressure loads when the aircraft is at
cruise altitude

263. A pressure door warning light illuminates only when the


door is

 not fully opened


 not locked
 locked

264. If an electrically operated cargo door lifting actuator


fails, it may be operated

 by alternate actuators
 manually by winding
 be electrical ground power

265. After installation of an acrylic window panel,


protective paper and adhesive remaining on the panel
may be removed by use of soap and water and

 Cellulose thinners
 MEK
 Methylated spirits
266. If heat is applied to the vinyl interlayer of an
aircraft windscreen, its ability to withstand impact
will

 increase
 not change
 decrease

267. When fitting a transparent plastic panel (acrylic) to a


metal frame, provision must be made for the relative
movement of the panel and the frame for the reason of

 the lower coefficient of expansion of the


transparent panel when compared with the frame
 the known slight shrinkage of the panel due to
ageing
 the higher coefficient of expansion of the
transparent panel when compared with the frame

268. Perspex is resistant to

 Paraffin
 Acetone
 C.T.C.

269. Moisture drain valves in a pressurised fuselage

 are open when the cabin is depressurised


 are open when the cabin is pressurised
 are open all the time to ensure the bilge areas are
well drained

270. Wide body transport aircraft have cabin floor vents

 to ensure adequate ventilation of the underfloor


areas
 to equalise pressures above and below the floor
after the loss of a cargo compartment door
 to provide access to class A cargo compartments in
the event of fire
Answers for Questions 261 – 270

1) demist, ant-ice and maintain strength and toughness


Comment/Reference:

2) never carry pressure loads


Comment/Reference:

3) not locked
Comment/Reference:

4) manually by winding
Comment/Reference:

5) Methylated spirits
Comment/Reference:

6) increase
Comment/Reference:

7) the higher coefficient of expansion of the transparent


panel when compared with the frame
Comment/Reference:

8) Paraffin
Comment/Reference: CAIPS AL/7-4 2.3

9) are open when the cabin is depressurised


Comment/Reference: CAIPs AL/7-2 3.6 and fig 9

10) to equalise pressures above and below the floor after


the loss of a cargo compartment door
Comment/Reference:
Questions 271 – 280

271. On an inspection, you are aware of vinyl bubbling. What


action is required?

 Replace the defective window


 Only replace the window if the 'bubbling' affects
vision
 No action is required with this defect

272. Passenger windows in jet transport aircraft normally


have

 three structural panes


 two structural panes
 one structural pane

273. A plug type window is fitted

 from the inside


 from either side
 from the outside

274. Air is used to seal the cabin door in pressure cabins.


The source of this air is usually

 cabin pressurised air


 from storage bottles
 ambient pressurised air

275. Drain valves are normally closed by

 spring pressure
 rubber strips
 cabin air pressure
276. Drain holes in unpressurised areas are

 always open
 always closed
 either open or closed

277. A cargo compartment in which there is a separate


approved smoke or fire detector system to give warning
and means to shut off the ventilating airflow to the
compartment is a

 Class 'C' compartment


 Class 'B' compartment
 Class 'E' compartment

278. Aircraft frames are constructed of

 'I' section
 'U' section
 'Z' section

279. The life of a fuselage depends on

 the pressurisation cycles excluding maximum


differential
 the pressurisation cycles not including ground
cycles
 all pressurisation cycles

280. Passenger cabin seat rails form part of the

 longerons
 stringers
 frames
Answers for Questions 271 – 280

1) Only replace the window if the 'bubbling' affects


vision
Comment/Reference: AL/7-10 5.1.6

2) two structural panes


Comment/Reference: B737-3/4/500 AMM 56-21-00 pg 1 para
A)

3) from the inside


Comment/Reference:

4) cabin pressurised air


Comment/Reference:

5) cabin air pressure


Comment/Reference: AL/7-2 3.6

6) always open
Comment/Reference: AL/7-2 3.6

7) Class 'E' compartment


Comment/Reference: CS 25.857 & Transport Category
Aircraft Systems, Jeppesen, 9-3

8) 'Z' section
Comment/Reference:

9) all pressurisation cycles


Comment/Reference:

10) longerons
Comment/Reference:
Questions 281 – 290

281. A false rib is usually used to

 support and give aerodynamic shape to the wing


leading edge
 allow the fitment of aileron control tubes through a
wing
 lose a torsion box structure

282. A spar web will take loads in

 bending
 shear
 tension

283. Wing bending and shear loads are taken by

 spar cap
 main spar
 skin

284. An aspect ratio of 8 could mean

 span 64 ft., mean chord 8 ft.


 mean chord 64 ft., span 8 ft.
 span squared 64 ft., chord 8 ft.

285. A cantilever wing is a

 top wing of a biplane


 swept-back wing
 usual airliner wing
286. On a mono-spar wing, what gives the wing its profile
contour?

 Milled stringers
 Ribs
 The position of the spars

287. A wing's leading edge would have provisions and


linkages for slats and

 leading edge flaps


 trailing edge flaps
 slots

288. The mid-spar is fitted in large aircraft to

 support fitting the engine mount & landing gear


mount
 provide redundant design
 assist the main spar with operational loads

289. The main undercarriage is attached to the

 rear main spar


 front main spar
 aircraft structure

290. The final coat of sealing in a integral fuel tank is


called

 brush coat
 fillet
 interfay
Answers for Questions 281 – 290

1) support and give aerodynamic shape to the wing leading


edge
Comment/Reference:

2) shear
Comment/Reference: The spar as a whole, takes bending
and shear. The bending is taken by the booms (as
compression-top and tension - bottom) and the web takes
the shear. (Sometimes called a 'shear' web).

3) main spar
Comment/Reference: The main spar takes the wing bending
and shear loads

4) span 64 ft., mean chord 8 ft.


Comment/Reference: Aspect Ratio = span/mean chord

5) usual airliner wing


Comment/Reference: AC65-15A pg 13 & CAIPs AL/7-2 fig 2

6) Ribs
Comment/Reference: AL/7-2 para 4.2

7) leading edge flaps


Comment/Reference: Slots are fixed features - no
linkages, or are produced when the slats open.

8) assist the main spar with operational loads


Comment/Reference:

9) aircraft structure
Comment/Reference: The only wholly correct answer

10) brush coat


Comment/Reference: B737 ANN 28-11-00 page 811, the 3
coats of sealant for repair fuel tanks leak are termed
as fillet, injection and prepack. Fillet seal is the
first coat.
Questions 291 – 300

291. The principle load bearing members of the wing are

 spars
 struts
 ribs

292. One purpose of a rib is to

 support the bending loads on a fuselage


 form the main lateral member in an aerofoil
 maintain the correct contour of an aerofoils
covering

293. What is a cantilever wing?

 One that has no external supporting struts


 One that folds for access to limited space
 One that has external supporting struts

294. A spar is tapered from root to tip because

 centre of lift occurs close to the root


 bending moment is greatest at the root
 shear forces are greatest at the root

295. A spar web is

 an area between two spar caps


 a member between the spar and wing/fuselage
connection
 a rib/spar joint
296. A leading edge slat is attached to the

 front spar
 slat track
 wing upper skin

297. The web of an 'I' beam takes mainly which type of load?

 Bending
 Tension
 Shear

298. The term 'empennage' incorporates

 elevators, stabiliser, rudder


 rudder, ailerons, spoilers
 elevators, stabiliser, ailerons

299. The four main structural items making up a horizontal


stabiliser are

 spar, rib, longerons, skin panels


 spar, rib, bulkheads, skin panels
 spar, rib, stringers, skin panels

300. To correct for nose heaviness on an aircraft fitted


with a variable incidence tailplane, the incidence of
the tailplane would be

 decreased, which is done by lowering the leading


edge
 decreased, which is done by lowering the trailing
edge
 increased, which is done by lowering the leading
edge
Answers for Questions 291 – 300

1) spars
Comment/Reference: AC65-15A pg 8

2) maintain the correct contour of an aerofoils covering


Comment/Reference: AC65-15A pg 10

3) One that has no external supporting struts


Comment/Reference: AC65-15A pg 13

4) bending moment is greatest at the root


Comment/Reference: AL/7-2 para 4.1

5) an area between two spar caps


Comment/Reference:

6) slat track
Comment/Reference: B737 AMM 27-81-00 page 4 fig 1 sheet
3 say each slat is atttached with 2 main and 2 aux
tracks & www.b737.org.uk/flightcontrols.htm

7) Shear
Comment/Reference: A 'web' always takes shear loads

8) elevators, stabiliser, rudder


Comment/Reference:

9) spar, rib, stringers, skin panels


Comment/Reference:

10) decreased, which is done by lowering the leading edge


Comment/Reference:
Questions 301 – 310

301. An upward elevator deflection on the reverse camber


tailplane

 will decrease tailplane download


 may increase or decrease download depending upon the
aircraft C of G position
 will increase tailplane download

302. Variable incidence tailplanes

 always move slowly


 move rapidly when trimming the aircraft during the
landing approach and slowly at all other times
 move rapidly when trimming the aircraft during climb
out and landing approach and slowly during cruise

303. The direction of travel of an electrically operated


variable incidence tailplane is determined by

 direction of rotation of the electric motor


 solenoid operated clutches
 a gearbox

304. On an aircraft with a variable incidence trimming


tailplane, the tailplane incidence changes

 if the control column is moved back or forward


 automatically if the elevator moves
 if the trim wheel is turned back or forward

305. Construction such as horn balance and inset hinge


balance installed on control surface assembly

 is meant to trim CG of control surfaces


 has same effect of the balance tab
 serves as a 'servo' system of balance
306. The fin helps to give

 longitudinal stability about the normal axis


 directional stability about the longitudinal axis
 directional stability about the normal axis

307. Which of the following are primary control surfaces?

 Roll spoilers, elevators, tabs


 Elevators, ailerons, rudder
 Elevators, roll spoilers, tabs

308. Aerodynamic balance

 will cause CP to move forward of hinge and cause


overbalance
 will cause CP to move towards the trailing edge and
cause instability
 will reduce aerodynamic loading

309. Flutter can be reduced by using

 servo tabs
 a horn balance
 mass balancing

310. An elevator provides control about the

 lateral axis
 longitudinal axis
 horizontal stabilizer
Answers for Questions 301 – 310

1) will increase tailplane download


Comment/Reference:

2) move rapidly when trimming the aircraft during climb


out and landing approach and slowly during cruise
Comment/Reference: B737-400, AMM 27-41-00 PAGE 5 para
1.c, trim speed is depend on the flap position. Trim
rate with flaps retracted is 1/3 the trim rate with
flaps extended

3) solenoid operated clutches


Comment/Reference:

4) if the trim wheel is turned back or forward


Comment/Reference:

5) has same effect of the balance tab


Comment/Reference: A&P Technician Airframe Textbook.
Jeppesen 1-24 fig 1-59, balance tab 1-23 -para 3

6) directional stability about the normal axis


Comment/Reference: Jeppesen A&P Technician Airframe
Textbook 1-22

7) Elevators, ailerons, rudder


Comment/Reference:

8) will reduce aerodynamic loading


Comment/Reference:

9) mass balancing
Comment/Reference: CAP 562 leaflet 11-22 appx 27-1 para
2

10) lateral axis


Comment/Reference:
Questions 311 – 320

311. The outboard ailerons on some large aircraft

 are isolated at low speeds


 are isolated at high speeds
 are isolated to improve sensitivity

312. An excess of aerodynamic balance would move the control


surface centre of pressure

 rearwards, resulting in loss of assistance


 forwards, resulting in an unstable overbalance
 rearwards, resulting in too much assistance

313. A flying control mass balance weight

 keeps the control surface C of G as close to the


trailing edge as possible
 tends to move the control surface C of G close to
the hinge line
 ensures that the C of G always acts to aid the pilot
thus relieving control column load

314. What is attached to the rear of the vertical


stabilizer?

 Aileron
 Rudder
 Elevator

315. The method employed to mass balance control surfaces is


to

 attach weights forward of the hinge line


 fit bias strips to the trailing edge of the surfaces
 allow the leading edge of the surface to project
into the airflow
316. Control surface flutter may be caused by

 excessive play in trim tab attachments


 incorrect angular movement of trim tabs
 high static friction in trim tab control tabs

317. A 'frise' aileron is incorporated to

 provide aerodynamic balancing so assisting the pilot


to move the control
 ensure aileron control is retained at high angles of
attack
 equalize aileron drag in a turn

318. Aerodynamic balance of a control surface may be


achieved

 by weights added to the control surface aft of the


hinge line
 by a horn at the extremity of the surface forward of
the hinge line
 by a trimming strip at the trailing edge of the
surface

319. A control surface is provided with aerodynamic


balancing to

 assist the pilot in moving the control


 increase stability
 decrease the drag when the control is deflected

320. The extension to the rudder (shaded portion shown on


the diagram), is provided to

 make the pilot aware of the aerodynamic forces


encountered when moving the control
 prevent control surface flutter
 provide aerodynamic assistance for the pilot when
moving the rudder
Answers for Questions 311 – 320

1) are isolated at high speeds


Comment/Reference:

2) forwards, resulting in an unstable overbalance


Comment/Reference:)

3) tends to move the control surface C of G close to the


hinge line
Comment/Reference: CAP 562 leaflet 11-22 Appx 27-1 para
2

4) Rudder
Comment/Reference:

5) attach weights forward of the hinge line


Comment/Reference: CAP 562 leaflet 11-22 Appx 27-1 para
2

6) excessive play in trim tab attachments


Comment/Reference:

7) equalize aileron drag in a turn


Comment/Reference:

8) by a horn at the extremity of the surface forward of


the hinge line
Comment/Reference: Jeppesen A&P Technician Airframe
Textbook Page 24 Para 7

9) assist the pilot in moving the control


Comment/Reference: Jeppesen A&P Technician Airframe
Textbook Page 24 Para 7

10) provide aerodynamic assistance for the pilot when


moving the rudder
Comment/Reference: Jeppesen A&P Technician Airframe
Textbook Page 25 Para 7
Questions 321 – 330
321. The balance tab is an auxiliary surface fitted to a
main control surface

 operating automatically to assist the pilot in moving


the controls
 operated independently at which point in the length of
cable the tensiometer is applied
 operating automatically to provide 'feel' to the
controls

322. Aerodynamic balancing of flight controls is achieved by

 placing a weight ahead of the hinge point


 placing a weight in the leading edge of the control
surface
 providing a portion of the control surface ahead of the
hinge point

323. Aerodynamic balance is used to

 reduce the control load to zero


 make the flying controls easier to move
 prevent flutter of the flying controls

324. A horn balance is

 a projection of the outer edge of the control surface


forward of the hinge line
 a rod projecting upward from the main control surface
to which the control cables are attached
 a rod projecting forward from the control surface with
a weight on the end

325. A control surface is mass balanced by

 the attachment of weights acting on the hinge line


 fitting a balance tab
 the attachment of weights acting forward of the hinge
line
326. The purpose of anti-balance tabs is to

 give more feel to the control column


 relieve stick loads
 trim the aircraft

327. A flying control mass balance weight

 tends to move the control surface C of G forward of the


hinge line
 tends to move the control surface C of G close to the
hinge line
 keeps the control surface C of G as close to the
trailing edge as possible

328. An elevator controls the aircraft motion in

 yaw
 roll
 pitch

329. Jet engines are usually mounted by

 aluminium castings
 welded steel tubing
 forged mounts and bolted to aircraft forged structure

330. A pylon structural member supports the

 centre section
 engine
 empennage
Answers for Questions 321 – 330

1) operating automatically to assist the pilot in moving


the controls
Comment/Reference: AC65-15A pg 46

2) providing a portion of the control surface ahead of the


hinge point
Comment/Reference: AL/3-24 para 4.2

3) make the flying controls easier to move


Comment/Reference:

4) a projection of the outer edge of the control surface


forward of the hinge line
Comment/Reference: Jeppesen A&P Technician Airframe
Textbook Page 16

5) the attachment of weights acting forward of the hinge


line
Comment/Reference: CAP 562 leaflet 11-22 Appx 27-1 para
2

6) give more feel to the control column


Comment/Reference:

7) tends to move the control surface C of G close to the


hinge line
Comment/Reference: CAP 562 leaflet 11-22 appx 27-1

8) pitch
Comment/Reference: AC65-15A pg 44 ‘Lateral Axis’

9) forged mounts and bolted to aircraft forged structure


Comment/Reference:

10) engine
Comment/Reference:
Questions 331 – 340

331. Wing mounted podded engines and integral fuel tanks

 provide increased safety if the undercarriage collapses


on landing
 reduce tailplane download
 provide wing bending relief

332. A spill valve opens

 to control the air from the cabin to outside


 to prevent an excessive pressure difference
 to control the flow to the cabin

333. Which of the following can be used on the ground?

 Turbo brake
 Turbo compressor
 Turbo fan

334. What is the minimum cabin air mass flow?

 10 lbs per minute per person


 0.5 lbs per minute per person
 1 lbs per minute per person

335. The function of an air mass flow control valve is to

 maintain a reasonably constant air mass flow into the


cabin at all altitudes
 control the airflow out of the cabin
 ensure that system differential pressure is not
exceeded
336. Air supplied for pressurisation and conditioning is

 hottest from an engine compressor bleed


 the same for both the above sources
 hottest from a compressor driven by an engine gear box

337. The mass flow delivery from engine driven blowers is


controlled by

 spill valves
 engine speed variations
 automatic control devices

338. The function of spill valves is to control

 air supply to the cabin


 the rate of pressurisation
 cabin pressure differential

339. The purpose of a 'Spill' valve in a cabin air supply


system is

 to relieve the pressure in the air supply ducting to


atmosphere
 to give a heating effect
 to spill overboard excess air delivered at S.L. and
lower altitudes

340. The function of the mass airflow control valve is to

 ensure that constant airflow out of the cabin is


dictated by cabin altitude
 maintain a reasonable mass flow of air into the cabin
irrespective of aircraft altitude
 ensure system operating pressure is not exceeded
Answers for Questions 331 – 340

1) provide wing bending relief


Comment/Reference:

2) to control the flow to the cabin


Comment/Reference: CAIPs AL/3-23

3) Turbo fan
Comment/Reference: Used on air conditioning systems to
supply air on ground.

4) 0.5 lbs per minute per person


Comment/Reference: AMC CS 25.831 or the old BCAR
Section D

5) maintain a reasonably constant air mass flow into the


cabin at all altitudes
Comment/Reference:

6) hottest from an engine compressor bleed


Comment/Reference: AL/3-24 2.3/2.5

7) spill valves
Comment/Reference: AL/3-24 Fig. 3

8) air supply to the cabin


Comment/Reference: CAIPs AL/3-24 fig 3

9) to spill overboard excess air delivered at S.L. and


lower altitudes
Comment/Reference: CAIPs AL/3-24 fig 3

10) maintain a reasonable mass flow of air into the cabin


irrespective of aircraft altitude
Comment/Reference:
Questions 341 – 350

341. In a Bleed Air air-conditioning system, the warm air


supply is provided by

 the compressor of the gas turbine engine


 the engine exhaust heat
 the gas turbine exhaust

342. In most pressurization systems, the amount of


compressed air delivered to the cabin is

 reasonably constant irrespective of altitude


 variable, depending on the amount selected by the cabin
rate of change selector
 constant at any particular altitude but varies for
different altitudes

343. The purpose of a mass flow controller is to

 allow pilot to select the desired cabin altitude


 ensure that a constant mass of air is delivered to
cabin at all times
 ensure the cabin altitude remains constant during
cruise at all altitudes

344. How much air is required for the Flight Deck?

 10 cubic feet/minute
 Whatever the captain sets
 10 lbs/minute

345. In large air conditioning systems, the main fan is


activated to

 ensure conditioned air is distributed and maintain


positive duct pressure
 ensure conditioned air is distributed
 ensure positive duct pressure is maintained
346. If a reduction in pack outlet temp is demanded, the
temperature of the air at the outlet of the ACM
compressor will

 rise
 remain the same
 fall

347. A large aircraft air conditioning system's cabin


temperature control

 all zone temperatures are controlled from one master


switch
 involves modulating the pack valve
 is selectable for each zone individually from the
flight station

348. What is the maximum allowable concentration (by volume)


of carbon monoxide in occupied aircraft compartment?

 0.02% (200 p.p.m)


 0.0005% (50 p.p.m)
 0.01% (100 p.p.m)

349. A refrigerant is used in which of the following?

 Air cycle machine


 Vapour cycle
 Pneumatic pump

350. A water separator is located

 downstream of heat exchanger


 downstream of turbine
 upstream of the turbine
Answers for Questions 341 – 350

1) the compressor of the gas turbine engine


Comment/Reference:

2) reasonably constant irrespective of altitude


Comment/Reference:

3) ensure that a constant mass of air is delivered to


cabin at all times
Comment/Reference:

4) 10 cubic feet/minute
Comment/Reference: CS 25.831 (a)

5) ensure conditioned air is distributed and maintain


positive duct pressure
Comment/Reference:

6) remain the same


Comment/Reference: Pack outlet temperature is
determined by bypassing engine bleed air around the ACM
(trim air). The ACM is unaffected.

7) is selectable for each zone individually from the


flight station
Comment/Reference: AMM 767 21-61-00

8) 0.0005% (50 p.p.m)


Comment/Reference: CAP 562 leaflet 5-3 para 1.2

9) Vapour cycle
Comment/Reference: AC65-15A pg 576 & Jeppesen A&P
Technician Airframe Textbook Page 14-34

10) downstream of turbine


Comment/Reference: AC65-15A pg 573 or Jeppesen A&P
Technician Airframe Textbook Page 16-30 or CAIPs AL/3-
24 4.2.3 fig 3
Questions 351 – 360

351. The velocity of air from the cabin ventilation system


should not exceed

 20 feet per second


 40 feet per second
 120 feet per second

352. In typical vapour cycle system, the sub-cooler

 delivers extra cooling effect when the aircraft is


on ground
 cools the vapour further to prevent slugging
 is a heat exchanger to superheat the vapour

353. In an air conditioning system, heat is added to air by

 restricting compressor inlet


 restricting compressor outlet
 restricting duct outlets

354. Air conditioning systems

 increase and decrease the temperature of air


 decrease the temperature of air
 increase the temperature of air

355. An air cycle machine turbine

 drives compressor to decrease temperature


 drives compressor to increase temperature
 drives compressor to pressurise aircraft
356. Air exiting the compressor side of an ACM

 will have increased pressure and temperature


 will have decreased pressure and temperature
 no change in temperature or pressure as it is a
centrifugal compressor

357. Where is the water trap located in a bootstrap


compressor?

 At the inlet of the compressor


 At the outlet of the compressor
 At the inlet of the turbine

358. Where is the silencer located in a 'blower' air


conditioning system?

 At the inlet to the cabin


 At the outlet of the blower
 At the inlet of the blower

359. When does a 'blower' air conditioning system produce


the most air?

 At high altitudes
 At low altitudes
 It is not affected by altitude

360. In an air conditioning system, before distribution, the


air goes through the

 TCV and mixer valve


 TCV
 flow control valve
Answers for Questions 351 – 360
1) 120 feet per second
Comment/Reference: BCAR Section D6
2) cools the vapour further to prevent slugging
Comment/Reference: AC65-15A pg 578 & A&P Technician
Airframe Textbook page 14-40
3) restricting compressor inlet
Comment/Reference: Heat is added by varying the amount
of compressor inlet air that bypasses the heat
exchanger (Trim Air).
4) decrease the temperature of air
Comment/Reference: AC65-15A pg 558 & CAIPs AL/3-24 Para
2.2.
The source of air is usually the engine bleed, which is
always far too hot to feed directly into the cabin, so
the air conditioning system always cools, even if the
'trim air' bypasses a little more hot air around the
ACM/refrigeration unit, the end result is still cooler
than the input.
5) drives compressor to increase temperature
Comment/Reference: The compressor is to INCREASE the
temperature to increase the rate at which heat energy
can be extracted.
6) will have increased pressure and temperature
Comment/Reference: AC65-15A pg 571/2 mentions pre-
cooled air is compressed by action of compressor
impeller and diffuser assembly. Hence increase in
pressure. Pressure laws says; Temperature is
proportional to Pressure.
7) At the inlet of the turbine
Comment/Reference: AC65-15A pg 572 fig 14-32 or CAIPs
shows it at the outlet of the turbine (not one of the
answers). B757 has an additional water trap at the
inlet of the turbine
8) At the outlet of the blower
Comment/Reference: AL/3-24 Figure 3

9) It is not affected by altitude


Comment/Reference: AL/3-24 Figure 3 (effect of the
Spill Valve)

10) TCV
Comment/Reference: b737-400 AMM ref 21-00-00 para 5b
mentions TCV is located before distribution for pack
temperature control for amount of hot air bypassing the
Air Cycle Machine. B737-300 name the same valve as
mixer valve
Questions 361 – 370

361. In a cabin air recirculation system, recirculated air


and fresh air are supplied in the proportions

 40% of fresh air, 60% of recirculated air


 50% of fresh air, 50% of recirculated air
 60% of fresh air, 40% of recirculated air

362. An air-to-air heat exchanger is provided to

 reduce the air supply temperature


 increase the air supply temperature
 provide an emergency ram air supply

363. Temperature control of cabin air is achieved by

 varying the ambient airflow to the heat exchanger


 regulating the amount of air by-passing the cooling
system
 controlling the water vapour in the supply

364. In pressurized aircraft, temperature conditioning is


mainly achieved by

 varying cabin pressure


 adding heat to the pressurising air
 extracting heat from the pressurising air

365. If pressurisation air supplies come from an engine


compressor, an internal oil leak will

 contaminate the air


 not contaminate the air
 not affect the issue as it is automatically detected
and vented overboard
366. A cold air unit produces a drop in temperature by

 driving the unit's compressor


 reducing pressure
 reducing pressure and driving the unit's compressor

367. Heating for pressure cabins is obtained from

 driving the units compressor


 only by adding heat electrically to the air supplies
 air supply heated by the pressurising process

368. A cold air unit allows for cabin temperatures to be

 lower than ambient air temperature despite


compression heating
 a little more than ambient air temperature
 same as ambient temperature, despite compression
heating

369. A cold air unit produces a drop in temperature by

 expanding hot air across a turbine which is driving


a compressor
 air supply to the cabin
 directing compressed air into a heat exchanger

370. When a 'muff' or air ducting is built around the engine


exhaust system and air is directed around inside the
muffler, this is an

 combustion heater
 thermal heater
 exhaust heater
Answers for Questions 361 – 370

1) 60% of fresh air, 40% of recirculated air


Comment/Reference:

2) reduce the air supply temperature


Comment/Reference: AL/3-24 4.2

3) regulating the amount of air by-passing the cooling


system
Comment/Reference: CAIPs AL/3-24 Fig 3

4) extracting heat from the pressurising air


Comment/Reference: CAIPS AL/3-24 4

5) contaminate the air


Comment/Reference:

6) reducing pressure and driving the unit's compressor


Comment/Reference: AC65-15A pg 572 rh column 3rd para
or CAIPs AL/3-24 4.2.2

7) air supply heated by the pressurising process


Comment/Reference: CAIPs AL/3-24 4

8) a little more than ambient air temperature


Comment/Reference:

9) expanding hot air across a turbine which is driving a


compressor
Comment/Reference: AC65-15A pg 572 rh column 3rd para
or CAIPs AL/3-24 4.2

10) exhaust heater


Comment/Reference: AC65-15A pg 561 or
CAIPs AL/3-24 3.3
Questions 371 – 380

371. In a turbo fan cold air system, the heat exchanger


cooling air is

 bled from cabin air supply duct


 air bled directly from engine or through blower
 ram air from ambient conditions

372. Conditioned air is

 moisture removed
 temperature and pressure adjusted
 oxygen added

373. Which of the following sometimes inhibits an air


conditioning pack?

 Flap position switches


 Undercarriage switches
 Throttle switches

374. When carrying out a ground pressure test, you should


use

 ground trolley and clean air


 a G.P.U. and A.P.U. combination
 the aircraft engines because you can test the whole
system

375. The ventilation air in the aircraft cabin must have a


minimum humidity of

 60 percent
 30 percent
 20 percent
376. In a Boot-Strap Air Conditioning supply system the
source of compressed air is from

 ram air at the wing leading edge


 gas turbine intake ram air
 gas turbine compressor bleed air

377. The temperature within the cabin of the aircraft is


normally maintained at

 18ºC to 24ºC
 20ºC to 24ºC
 12ºC to 18ºC

378. In a 'bootstrap' cooling system the supply of air is


first

 passed across an expansion turbine, then compressed


and passed through a heat exchanger
 compressed then passed through a heat exchanger and
across an expansion turbine
 passes across an expansion turbine, then directly to
the heat exchanger

379. The humidity within a passenger cabin should

 be between 30% and 70%


 not be less than 60%
 not be greater than 40%

380. One of the principles of cooling employed in an air


cycle system is

 conversion of heat energy to mechanical energy in


the C.A.U.
 by compression of ambient air across a turbine
 by surface heat exchange in the C.A.U.
Answers for Questions 371 – 380
1) ram air from ambient conditions
Comment/Reference: Cooling air for heat exchanger is
taken from the ambient ram air which is controled by
varying opening of the ram air exhaust louvers. Ref
B737-400 AMM 21-00-00 page 8
2) temperature and pressure adjusted
Comment/Reference:

3) Throttle switches
Comment/Reference:

4) the aircraft engines because you can test the whole


system
Comment/Reference: The ground pressure test is
typically known as confidence check of airplane ability
to maintain cabin pressure in flight on single pack
operation for boeing term. The test involve both left
and right engine. Ref AMM 21-00-05 page 201.

5) 30 percent
Comment/Reference:

6) gas turbine compressor bleed air


Comment/Reference: B737 has the config where the air-
conditioning air supply is taken from the pneumatic
system which draw air from the 5th or 9th compressor
stage of the gas turbine engine. Ref AMM 21-00-00 pg 8

7) 18ºC to 24ºC
Comment/Reference: B737-400 AMM ref 21-61-00 page 11,
the selectable range of temperature is between 18 and
29 deg C. For alternate operation when all selectors to
OFF, the Left Pack operate at 24deg C and Right Pack at
18deg C.

8) compressed then passed through a heat exchanger and


across an expansion turbine
Comment/Reference: CAIP AL/3-24 figure 2

9) be between 30% and 70%


Comment/Reference:

10) conversion of heat energy to mechanical energy in the


C.A.U.
Comment/Reference: A&P Technician Airframe Textbook
Page 770 Fig 16-34 & AC65-15A pg 565
Questions 381 – 390
381. Before filling a vapour cycle cooling system

 pre-heat the system to 100°F


 flush the system with a solvent
 apply suction to remove air and moisture

382. The rate of flow of air from the punkha louvers should
not be less than

 25 ft/min.
 200 ft/min.
 300 ft/min.

383. An aircraft cabin is air conditioned and pressurized in


order to

 ensure that the air density within the cabin is


maintained at a lower figure than outside the cabin
in order to prevent moisture precipitation during
rapid decompression
 maintain human efficiency and comfort during flights
at high altitudes
 ensure that the pressure within the fuselage is
always less than the ambient pressure, thus
increasing the fatigue life of the fuselage

384. In the case of a vapour cycle cooling, system heat is


removed from the charge air by

 reducing the pressure of a vapour


 changing a liquid into a vapour
 changing a vapour into a liquid

385. Subsequent to passing through the primary heat


exchanger, the supply air in a turbo-fan cold air
system flows to the

 fan
 inter cooler or secondary heat exchanger
 turbine
386. The heat exchanger in a turbo-fan system is cooled by

 engine bleed air or blower air


 air bled from the main cabin supply duct
 ambient ram air

387. The effective temperature of a cabin is given by

 temperature only
 temperature and humidity
 temperature, humidity, thermal inertia and heat load

388. When the refrigerant dissipates heat in a vapour cycle


system

 the liquid sublimates


 the vapour converts to a liquid
 the liquid converts to a vapour

389. International markings for air conditioning pipelines


are

 rectangles
 dots
 triangles

390. A cabin humidifier is operated

 at high altitudes
 on the ground
 at low altitudes
1) apply suction to remove air and moisture
Comment/Reference: CAIP AL/3-24 para 9.7(b) says,
'before charging a newly installed system, or
recharging a system which has been partly disconnected,
all air should be evacuated IAW MM.'

2) 25 ft/min.
Comment/Reference: BCAR Section D

3) maintain human efficiency and comfort during flights at


high altitudes
Comment/Reference: CAIP AL/3-23 para 2.1 says 1) to
maintain a safe cabin altitude relative to aircraft
altitude. 2) to provide a comfortable environment when
flying at higher altitudes

4) changing a liquid into a vapour


Comment/Reference: AC65-15A pg 576 & AL/3-24 para
13.3.2

5) fan
Comment/Reference: 'Fan' is sometimes used to describe
the compressor. CAIP AL/3-24 figure 2

6) ambient ram air


Comment/Reference:

7) temperature and humidity


Comment/Reference:

8) the vapour converts to a liquid


Comment/Reference: AC65-15A pg 576

9) dots
Comment/Reference:

10) at high altitudes


Comment/Reference:
Questions 391 – 400

391. A turbo fan CAU used for air cycle cooling will

 increase the pressure but decrease the temperature


 not affect the charge air pressure
 decrease pressure and temperature of the charge air

392. Pressurisation system operation may be inhibited by

 flap microswitches
 air/ground microswitches
 throttle microswitches

393. As an aircraft descends from cruising altitude


(34,000ft), the cabin altitude must

 decrease
 increase
 stay the same

394. The signal line between the controller and discharge


valve is leaking. This will cause

 it will not effect on cabin pressure


 the cabin pressure to decrease
 the cabin pressure to increase

395. The basic system of cabin pressurisation is to arrange


a constant

 inlet and vary the outlet


 inlet and outlet
 outlet and vary the inlet
396. The purpose of the differential capsule in a pressure
controller is to control

 the rate of pressurisation


 cabin differential pressure
 cabin air flow

397. With a pressurised aircraft at maximum differential


pressure and a cabin pressure increase occurs, the
differential capsule in the pressure controller will

 let pressurisation to be switched off until leaks


cause a drop in pressure
 let all pressurising air to be spilled overboard
 have a constant mass flow

398. A comfortable rate of cabin altitude climb for


passengers is

 500 ft. per min.


 100 ft. per min.
 300 ft. per min.

399. Before carrying out a ground pressure check,

 set altimeter to QNH


 check all pitot and static lines are fitted
 turn on all instruments

400. On touch-down of aircraft

 the cabin pressure will be zero


 the outflow valve will be shut
 the outflow valve will be fully open
Answers for Questions 391 – 400

1) decrease pressure and temperature of the charge air


Comment/Reference: AC65-15A pg 571/2

2) throttle microswitches
Comment/Reference: AMM 767 21-31-00

3) decrease
Comment/Reference: Jeppesen A&P Technician Airframe
Textbook Page 14-22. CAIPs AL/3-23

4) the cabin pressure to increase


Comment/Reference: The discharge valve is opened by
vacuum in the signal line. If line leaks , valve will
close, and cabin pressure will increase. Jeppesen A&P
Airframe Technician Textbook Page 14-22

5) inlet and vary the outlet


Comment/Reference: Jeppesen A&P technician Airframe
Textbook Page 14-20.

6) cabin differential pressure


Comment/Reference: Jeppesen A&P technician Airframe
Textbook Page 14-22. CAIPs AL/3-23

7) let all pressurising air to be spilled overboard


Comment/Reference: CAIPs AL/3-23 para 4.3.13

8) 500 ft. per min.


Comment/Reference: CAIPs AL/3-23

9) check all pitot and static lines are fitted


Comment/Reference: CAIPs AL/3-23 Para 12.2.5

10) the outflow valve will be fully open


Comment/Reference: B737 Pressurization system senses
the air-ground sw and the toggle sw FLT-GRD in the
cockpit control. Before touch down, at GRD position,
controller commands signal to close the discharge valve
to 200ft below landing field elevation. Upon touch
down, controller drives discharge fully open. Purpose
to prevent rapid pressure bump
Questions 401 – 410

401. When pressurising the aircraft on the ground for test


purposes, internal doors, cupboards etc. must be

 removed
 all open
 all closed

402. Prior to conducting a ground pressurisation test, it is


necessary to

 disconnect the emergency pressure relief valve


 set QFE
 reset/disable the pressure controller

403. In the flightdeck of an unpressurised aircraft, there


is a gauge that shows

 cabin pressure altitude


 aircraft altitude
 cabin differential pressure

404. Cabin differential pressure is the pressure difference


between

 the pressure inside the aircraft and the ambient air


pressure
 8,000ft and standard barometric pressure
 sea level air pressure and indicated dynamic
pressure in the cabin

405. If the pressure controller is set to 0 ft.

 cabin remains at sea level untill maximum


differential
 cabin will not pressurise
 maximum differential is reached immediately after
take-off
406. During a pressurisation check at maximum differential,
if the engines are shut-down

 aircraft remains pressurised until the dump valve


opens
 cabin ROC indicator gives indication of cabin seal
efficiency
 outflow valve opens immediately

407. Cabin pressure is maintained by

 controlling the amount of air discharged from the


cabin
 controlling the output of the compressor
 controlling the supply of air to the cabin

408. Control of rate of change of cabin pressure is

 equally important in descent and ascent


 most important in descent
 most important in ascent

409. Cabin rate of climb is shown by

 warning lights
 a special instrument
 a double scale on the aircraft

410. During normal pressurized climb following take-off

 the differential pressure is constant


 the cabin R.O.C. is more than ambient R.O.C.
 the cabin R.O.C. is less than ambient R.O.C.
1) all open
Comment/Reference: AL/3-23 12.3.4

2) reset/disable the pressure controller


Comment/Reference: The pressurisation system must be
controlled manually. Setting QFE is for the automatic
control of the valves.

3) aircraft altitude
Comment/Reference: Jeppesen A&P Technician Airframe
Textbook Page 14-23

4) the pressure inside the aircraft and the ambient air


pressure
Comment/Reference:

5) cabin will not pressurise


Comment/Reference:

6) cabin ROC indicator gives indication of cabin seal


efficiency
Comment/Reference:

7) controlling the amount of air discharged from the cabin


Comment/Reference:

8) most important in descent


Comment/Reference: CAIPs AL/3-23

9) a double scale on the aircraft


Comment/Reference: CAIPs AL/3-23 Fig 3. Ascent scale
and decent scale

10) the cabin R.O.C. is less than ambient R.O.C.


Comment/Reference: CAIPs AL/3-23 fig 2
Questions 411 – 420

411. Cabin differential pressure is the difference between

 cabin pressure and ambient pressure


 8,000 ft and sea level
 I.S.A. conditions and aircraft altitude

412. Cabin altitude in pressurized flight is the

 altitude corresponding to cabin pressure


irrespective of the altitude for the aircraft
 altitude at which cabin altitude equals outside air
pressure
 pressure altitude of the cabin as corrected to mean
sea level I.S.A. conditions

413. When an aircraft has reached max. diff. and is at


constant level (altitude) the system allows for

 constant mass flow


 all pressurising air to be spilled overboard
 pressurisation to be switched off until leaks cause
a drop in pressure

414. The control of ventilating and pressurising air


released to atmosphere is achieved by a

 pressure controller/dump valve combination


 pressure controller/discharge valve combination
 discharge valve/inwards relief valve combination

415. Rate of change of cabin pressure is most noticeably


shown on a

 cabin V.S.I.
 cabin altimeter
 cabin pressure gauge
416. Pressurisation control ensures that

 at operational altitude the cabin altitude is below


10,000 ft
 pressurisation does not start before aircraft is
above 8,000 ft.
 the cabin is always maintained at sea level

417. 'Pitot' and 'Static' lines during a cabin pressure test


should be

 disconnected
 connected
 cross connected

418. If the cabin pressure fails to reach its maximum cabin


pressure differential the

 discharge valve should be adjusted


 outward relief valve is inoperative
 pressure controller should be adjusted

419. When the aircraft has reached its maximum cabin


pressure differential the

 discharge valve closes


 mass flow ceases through the cabin
 discharge valve opens

420. Would you operate the flying controls during a cabin


pressure ground test?

 Occasionally
 No
 Yes
1) cabin pressure and ambient pressure
Comment/Reference: AC65-15A pg 541/2 or
CAIPs AL/3-23 fig 2

2) altitude corresponding to cabin pressure irrespective


of the altitude for the aircraft
Comment/Reference: CAIPs AL/3-23 fig 2

3) constant mass flow


Comment/Reference:

4) pressure controller/discharge valve combination


Comment/Reference: CAIPs AL/3-23 9

5) cabin V.S.I.
Comment/Reference: CAIPs AL/3-23 fig 3

6) at operational altitude the cabin altitude is below


10,000 ft
Comment/Reference: CAIPs AL/3-23

7) connected
Comment/Reference: CAIPs AL/3-23 12.2.5

8) pressure controller should be adjusted


Comment/Reference: CAIPs AL/3-23 Para 10.2.2 iv

9) discharge valve opens


Comment/Reference:

10) Yes
Comment/Reference: AL/3-23 12.3.9
Questions 421 – 430

421. Which component must be isolated when carrying out a


ground cabin pressure test?

 Pressure regulator controller


 Pressure relief valve
 Pressure discharge valve

422. A cabin pressure air leak from the front of the


fuselage is

 most desirable because it increases the air-flow


 most undesirable because of the drag created
 not effective in any way

423. The principle of cabin pressurisation is

 whilst the aircraft climbs to altitude, the cabin


climbs to a lower altitude
 cabin altitude climbs eventually to that of the
aircraft
 cabin altitude will always maintain a constant
differential to that of aircraft altitude

424. When the cabin differential pressure has reached the


required value and the height is maintained

 constant mass airflow is permitted through the cabin


 all pressurized air is spilled to atmosphere
 the pressure system ceases to function until the
cabin pressure is reduced

425. Cabin pressure controller maintains a pre-set cabin


altitude by

 regulating the mass flow into the cabin


 regulating the position of the outflow valve
 regulating the position of the inward relief valve
426. If cabin height is set lower than airfield height when
the aircraft is on the ground with squat switches
overridden, then the outflow valve will normally

 not operate
 open
 remain closed

427. The cabin altitude is

 the difference between cabin pressure and


atmospheric pressure
 the equivalent height of the aircraft above sea
level
 the actual height of the aircraft above sea level

428. The cabin differential pressure is

 the equivalent height of the aircraft above sea


level
 the difference between cabin pressure and
atmospheric pressure
 the actual height of the aircraft above sea level

429. The outflow of air from the cabin is regulated by

 the vent valves


 the outflow valves
 the dump control valves

430. When air is pressurized, the oxygen content

 decreases
 remains constant
 increases
Answers for Questions 421 – 430

1) Pressure discharge valve


Comment/Reference: AL/3-23 12.3.10

2) most undesirable because of the drag created


Comment/Reference: CAP 562 leaflet 6-6 para 3.2

3) whilst the aircraft climbs to altitude, the cabin


climbs to a lower altitude
Comment/Reference:

4) constant mass airflow is permitted through the cabin


Comment/Reference:

5) regulating the position of the outflow valve


Comment/Reference:

6) remain closed
Comment/Reference:

7) the equivalent height of the aircraft above sea level


Comment/Reference:

8) the difference between cabin pressure and atmospheric


pressure
Comment/Reference:

9) the outflow valves


Comment/Reference: CAIP AL/3-23 para 5.1

10) increases
Comment/Reference: AC65-15A pg 539; Oxygen content
increases as mass per unit volume, but NOT as a
percentage of the other gases in air. Your
interpretation of the may differ.
Questions 431 – 440
431. If an altitude of 8000 feet is selected on the cabin
pressure controller and provided maximum cabin pressure
differential is not exceeded

 8000 feet cabin conditions will be maintained at


aircraft altitudes above 8000 feet
 8000 feet cabin conditions will be maintained at all
aircraft altitudes from sea level
 sea level cabin conditions will be maintained to
8000 feet aircraft altitude

432. If cabin pressure is increasing, the cabin rate of


change indicator will show

 a rate of descent
 zero, provided the rate of change is within the
normally accepted limits
 a rate of climb

433. During a normal climb from aerodrome level with the


pressurization system 'ON'

 the cabin differential pressure is maintained


constant
 the pressurization system does not control pressure
until 10,000 ft is reached
 the atmospheric pressure decreases more quickly than
the cabin pressure

434. Rate of change of cabin pressure is

 selected by the pilot and controlled by the pressure


controller
 selected by the pilot and controlled by the spill
valve
 automatic
435. An aircraft has a maximum differential pressure of 8.5
PSI at cruising altitude. If the ambient pressure is
2.9 PSI, the pressure inside the cabin at cruising
altitude would be

 5.6 PSI
 8.5 PSI
 11.4 PSI
436. Cabin differential is determined only by

 the selected cabin height


 the height at which the aircraft is flying and by
the selected cabin height
 the height at which the aircraft is flying

437. If the pilot selected a cabin height of 8000 ft. whilst


taxiing and activated the pressurization system, the
cabin pressure would

 decrease to a pressure equivalent to about 500 ft.


 decrease to a pressure equivalent to 8000 ft.
 remain at ground level pressure

438. A silencer is installed in a pressurization system to


reduce

 engine noise coming through the ventilators


 the noise from the blowers and/or compressors in the
system
 the noise from the high speed of airflow within the
system

439. To avoid discomfort, the rate of change of pressure


should be low, particularly

 when cabin pressure is decreasing


 during descent
 during ascent

440. If an aircraft is operating at 40,000 ft. the


pressurization ensures that

 sea level pressure is maintained in the cabin


 the cabin pressure is progressively increased until
the operational height is reached
 the cabin pressure is equivalent to an altitude of
less than 10,000 ft.
1) 8000 feet cabin conditions will be maintained at
aircraft altitudes above 8000 feet
Comment/Reference:

2) a rate of descent
Comment/Reference:

3) the atmospheric pressure decreases more quickly than


the cabin pressure
Comment/Reference: AL/3-23 Fig 2 Pg 3

4) selected by the pilot and controlled by the pressure


controller
Comment/Reference:

5) 11.4 PSI
Comment/Reference:)

6) the height at which the aircraft is flying and by the


selected cabin height
Comment/Reference:)

7) remain at ground level pressure


Comment/Reference: AL3/23 Fig 2

8) the noise from the blowers and/or compressors in the


system
Comment/Reference: AL/3-24 fig 3 and para 2.5

9) during descent
Comment/Reference:

10) the cabin pressure is equivalent to an altitude of less


than 10,000 ft.
Comment/Reference:
Questions 441 – 450

441. The pressure controller activates

 the spill valve


 the blower or compressor
 the cabin discharge valve

442. When cruising near the operational ceiling, the flight


altitude set on the pressurization control panel may be
500 ft. more than the actual flight altitude so as to
prevent

 safety valve operation


 pressure controller hunting
 inward relief valve operation

443. The cabin rate of climb is shown

 as being inside or outside limits by green and red


lights, a gauge being used
 by a differential scale on the aircraft rate of
climb indicator
 on a specific indicator

444. A water separator would be installed in a


pressurization system to

 extract surplus water from the charge air


 extract water from the cabin air before it is
discharged to atmosphere
 collect any rain accompanying the ram air

445. Cabin pressure controller maintains a particular cabin


altitude by control of

 cabin mass air flow


 outflow valve position
 inward relief valve position
446. Ditching control is used for

 deploying life rafts


 closing all valves and inlets
 rapid aircraft depressurization

447. A cabin altitude is protected against reaching an


altitude of 13,000 ft. by

 bellows in the outflow valve


 cabin over pressure relief valve
 altitude sensor

448. If the cabin altitude increases above the normal


maximum

 an inward relief valve opens


 compressor delivery is automatically boosted
 a warning light comes on in the cockpit

449. On an aircraft employing a heater system on the air


conditioning system, after an overheat, how is the
heater reset?

 After cooling below 300°C it auto resets


 On ground only by engineer
 After it cools the pilot resets

450. Ditching control is used to

 maintain cabin pressure at sea level


 close the outflow valves
 achieve rapid depressurization
Answers for Questions 441 – 450

1) the cabin discharge valve


Comment/Reference: A&P Technician Airframe Textbook
Chapter 14-20 lh column last para.

2) pressure controller hunting


Comment/Reference:

3) on a specific indicator
Comment/Reference:

4) extract surplus water from the charge air


Comment/Reference: AC65-15A pg 573

5) outflow valve position


Comment/Reference:

6) closing all valves and inlets


Comment/Reference:

7) altitude sensor
Comment/Reference: CAIP AL/3-23 para 7

8) a warning light comes on in the cockpit


Comment/Reference: CAIPs AL/3-23 para 4.4.4

9) On ground only by engineer


Comment/Reference: A&P Technician Airframe pg 14-30
under Safety Features

10) close the outflow valves


Comment/Reference: CAIPs AL/3-2 fig 5 item Y, CAP 562
Leaflet 5-2
Questions 451 – 460

451. Inward vent valves are fitted to

 increase ventilation
 limit negative differentials
 limit positive differentials

452. Inward vent valves will operate when

 depressurising after descent


 aircraft altitude exceeds cabin altitude
 cabin altitude exceeds aircraft altitude

453. Inwards relief valves usually open at a negative


differential pressure not exceeding

 0.5 p.s.i.
 1.2 p.s.i.
 0.16 p.s.i.

454. Failure of the normal maximum pressure differential


control is allowed for by fitting

 inwards relief valves


 airport altitude selectors
 safety relief valve

455. To what position is the inward relief valve spring


loaded?

 Open
 Closed
 Both position
456. Inward relief valves are interconnected in pressurized
aircraft

 to achieve maximum pressure differential


 to allow controlled pressure during descent
 to relieve cabin pressure and allow outside pressure
to be greater

457. A safety valve will normally relieve at

 higher differential pressure than the discharge


valve
 negative differential pressure
 lower differential pressure than the discharge valve

458. A negative differential pressure is prevented by

 a blow off valve


 an inward relief valve
 a spill valve

459. The inward relief valve is usually set to operate at a


cabin differential of

 -0.5 PSI
 +9.25 PSI
 +0.5 PSI

460. An inward relief valve will operate

 after an aircraft has landed, to restore ground


level conditions is the cabin
 when climbing with pressurization 'OFF'
 when cabin pressure is lower than ambient pressure
Answers for Questions 451 – 460

1) limit negative differentials


Comment/Reference: AC65-15A pg 554/5
CAIPs AL/3-23 6.2

2) cabin altitude exceeds aircraft altitude


Comment/Reference: AC65-15A pg 554/5
CAIPs AL/3-23 6.2

3) 0.5 p.s.i.
Comment/Reference:

4) safety relief valve


Comment/Reference: CAIPs AL/3-23 6.1

5) Closed
Comment/Reference:

6) to allow controlled pressure during descent


Comment/Reference: CAIPs AL/3-23 Para 5

7) higher differential pressure than the discharge valve


Comment/Reference:

8) an inward relief valve


Comment/Reference:)

9) -0.5 PSI
Comment/Reference:

10) when cabin pressure is lower than ambient pressure


Comment/Reference: AL/3-23 6.2
Questions 461 – 470

461. An inward relief valve is installed in a pressurization


system to ensure that the pressure hull of an aircraft
is not subjected to

 too high an internal pressure


 a high negative differential pressure
 forces which would cause the aircraft to explode

462. Failure of the normal maximum differential pressure


control is catered for by

 cabin safety relief valves


 inwards relief valve
 spill valves

463. Which of these barometric instruments uses a restrictor


to compute its output?

 VSI
 ASI
 Machmeter

464. Slat asymmetry may be monitored by using

 spring actuators
 torque sensors
 position pick-offs

465. A standby compass has a pair of magnets for adjustment.


They are initially placed

 45 degrees to each other


 90 degrees to each other
 parallel with each other
466. A gyro spinning on a vertical axis will detect the
aircraft's

 roll and pitch


 yaw only
 yaw and pitch

467. The rigging of a non magnetic proximity sensor requires

 ensuring the target contacts the sensor


 checking the target clearance in the NEAR position
against reference figures
 checking the target clearance in the FAR position
against reference figures

468. How do you detect and locate a leak in a pitot-static


system?

 Isolate the component and perform a systematic


check, starting from the instrument
 Pressurize the system and use the soap bubble
detection method
 Disconnect each line in turn and perform a pressure
leak test

469. A +/. symbol on a compass system indicates

 failure of the compass system


 compass card and directional gyro are being
synchronised
 magnetic deviation and compass card are in alignment

470. An altimeter in a pressurised aircraft at altitude has


a case leak. The altimeter will

 under read
 not be affected
 over read
1) a high negative differential pressure
Comment/Reference: AL/3-23 6.2

2) cabin safety relief valves


Comment/Reference:

3) VSI
Comment/Reference:

4) position pick-offs
Comment/Reference: AMM 767 27-88-00

5) parallel with each other


Comment/Reference:

6) roll and pitch


Comment/Reference:

7) checking the target clearance in the NEAR position


against reference figures
Comment/Reference:

8) Pressurize the system and use the soap bubble detection


method
Comment/Reference:

9) compass card and directional gyro are being


synchronised
Comment/Reference: Aircraft Instruments and Integrated
Systems Pallett Page 197

10) under read


Comment/Reference:
Questions 471 – 480

471. The earth's magnetic dip affects a standby compass

 above its pivot only


 below its pivot only
 equally above and below its pivot

472. An electrical gyro has usually a speed of

 10,000 RPM
 20,000 RPM
 5,000 RPM

473. A flux valve is used to

 drive the HSI


 align the standby compass with the earth's magnetic
field
 align the directional gyro with the earth's magnetic
field

474. The decision height light will illuminated when

 the decision height is selected


 the aircraft's indicated altitude is above decision
height
 the aircraft's indicated altitude is below decision
height

475. In a Central Air Data Computer (CADC) system, Mach


information is derived from

 indicated air speed and total air temperature


 indicated airspeed and altitude
 pitot and static pressures
476. What instrument connects to pitot pressure?

 Both the airspeed indicator and the vertical speed


indicator
 The airspeed indicator
 The vertical speed indicator

477. In a Central Air Data Computer (ADC), the rate of climb


of an aircraft is produced in the

 Indicated Airspeed Module


 Mach Module
 Altitude Module

478. In a Central Air Data Computer, altitude is produced


from

 pitot air pressure


 pitot air pressure and total air temperature
 static air pressure

479. In a Central Air Data Computer (ADC), Indicated Air


Speed (IAS) is produced from

 pitot pressure only


 pitot and static air pressure
 static pressure only

480. The Tx and Rx rotors of a control synchro are 90


degrees shifted. The output is

 zero
 low
 high
Answers for Questions 471 – 480

1) below its pivot only


Comment/Reference:

2) 20,000 RPM
Comment/Reference: Aircraft Instruments and Integrated
Systems, Pallett Page 113

3) align the directional gyro with the earth's magnetic


field
Comment/Reference: B737-200 AMM 34-21-0 pg 1

4) the aircraft's indicated altitude is below decision


height
Comment/Reference: Decision Height comes on at Decision
Height and remains on as aircraft descends to landing.

5) indicated airspeed and altitude


Comment/Reference:

6) The airspeed indicator


Comment/Reference:

7) Altitude Module
Comment/Reference:

8) static air pressure


Comment/Reference:

9) pitot and static air pressure


Comment/Reference:

10) zero
Comment/Reference:
Questions 481 – 490

481. In a Central Air Data Computer (CADC) system, True Air


Speed (TAS) is derived from the Mach information and

 Total Air Temperature


 Static Air Pressure
 Pitot Air Pressure

482. Track altitude in a radio altimeter system begins at

 10 000 ft radio alt.


 2 500 ft radio alt.
 1 000 ft radio alt.

483. When testing using a thermocouple test set

 a 28 VDC supply is required


 a serviceable battery is required
 no power is required

484. What is the maximum radio altimeter track altitude?

 10,000 ft
 500 ft
 2,500 ft

485. A Compass swing would be required after replacing a

 compass Amplifier
 H.S.I.
 compass sensing unit
486. Where is the attitude director gyro stator situated?

 Around the rotor


 Inside the rotor
 Underneath the rotor

487. What is the error in the lubber line called?

 Index error
 Coefficiant P
 Coefficiant B

488. An ECAM system is tested under the following


conditions:-

 Aircraft in the air with both engines running


 Aircraft on the ground with one engine running
 Aircraft on the ground with parking brake set/on

489. Millibar is used to express what kind of pressure?

 Pitot pressure
 Barometric pressure
 Ambient pressure

490. The maximum deviation on a standby magnetic compass


must not be more than

 3°
 5°
 1°
Answers for Questions 481 – 490

1) Total Air Temperature


Comment/Reference:

2) 2 500 ft radio alt.


Comment/Reference: Jeppesen Avionics Fundamentals Page
223 & Electricity & Electronics 5th ed Eisman pg 323/4

3) a serviceable battery is required


Comment/Reference: AC65-12A Page 494
Battery operation or 115V AC input

4) 2,500 ft
Comment/Reference: Aircraft Electricity & Electronics,
5th Ed Eismin p323-324

5) compass sensing unit


Comment/Reference:

6) Inside the rotor


Comment/Reference: Aircraft Instruments and Integrated
Systems, Pallett Page 112

7) Index error
Comment/Reference: The Lubber line is the vertical
white line painted on the front of the compass. If
there is an error in that, there is an error in the
compass mounting position. CAP 562 Leaflet 8-2 para 3
a)

8) Aircraft on the ground with parking brake set/on


Comment/Reference:
Pallett Aircraft Instruments and Integrated Systems
Page 391

9) Ambient pressure
Comment/Reference:
http://en.wikipedia.org/wiki/Millibar

10) 3°
Comment/Reference: CAP 562 leaflet 8-2 para 14.8
Questions 491 – 500

491. Compass base classification Class 2 is periodically re-


surveyed every

 2 years
 5 years
 1 year

492. The aircraft heading is 270°. The magnetic compass


deviation is -1°. The pilot should fly

 269°
 271°
 270°

493. Which instrument is most likely to damage if you have a


rapid drop in pressure, when carrying out a pitot-
static leak check?

 Air speed indicator


 vertical speed indicator
 Altimeter

494. The operation of the Angle of Attack indicator is used


to detect airflow direction relative to

 the angle of attack of the aircraft


 the longitudinal axis of the aircraft
 the pitch angle of the aircraft

495. The runway heading is

 QDM
 QFU
 QDR
496. Apparent drift of a vertical gyro is a function of

 cos of latitude
 sin of latitude
 tan of latitude

497. A machmeter works

 always
 above 10,000 ft
 always except on the ground

498. Radio marker information is displayed on

 EICAS
 HSI
 ADI

499. Angle of Attack alarm is sounding too close to stall.


Rectification is to

 move probe down


 move probe up
 move probe laterally

500. Where is alpha angle used?

 IRS
 Angle of attack
 Accelerometer
Answers for Questions 491 – 500

1) 2 years
Comment/Reference: CAP 562 leaflet 8-1 para 4 c) ii)

2) 271°
Comment/Reference: CAP 562 leaflet 8-2 table 1 pg 18

3) Altimeter
Comment/Reference: Altimeter is most sensitive to rapid
pressure changes

4) the longitudinal axis of the aircraft


Comment/Reference:

5) QFU
Comment/Referene:
www2.tky.3web.ne.jp/~jahfa/kokuningen/Q.html

6) cos of latitude
Comment/Reference:
Pallett Aircraft Instruments and Integrated Systems
Page 103

7) always
Comment/Reference:
Pallett Aircraft Instruments and Integrated Systems
Page 45

8) ADI
Comment/Reference:
Aircraft Electricity and Avionics (5th Edition) Eismin
Page 358

9) move probe up
Comment/Reference:
Move probe up, closer to the Leading Edge stagnation
point, so it operates sooner

10) Angle of attack


Comment/Reference:
Aircraft Instrument and Integrated Systems - Pallett
Page 73.
Questions 501 – 510

501. Where is the spinup/rundown brake on a gyro instrument?

 Outer gimbal
 Rotating vane
 Inner gimbal

502. On replacing a pre-indexed flux valve you would

 fit the serviceable pre indexed flux valve into the


same position as the unserviceable pre-indexed flux
valve was removed from
 carry out a check swing after fitment
 align the aircraft onto it’s A coefficient so that
no error is induced

503. In Airways flying, what is the barometric scale of the


altimeter set to?

 QNH
 QFE
 1013.25

504. Vibration monitoring signals are sent

 via a signal conditioner to the gauge


 via a half-wave rectifier to the gauge
 direct to the gauge

505. The vacuum system gauge reads 5 inches of mercury. This


is from

 zero and positive


 zero and minus
 ambient and minus
506. A direct reading Bourdon gauge has a restriction in
the inlet. This is to

 prevent FOD ingestion


 dampen sudden pressure changes
 allow for calibration

507. The hot junction of thermocouple is

 aft of combustion chamber


 in the instrument
 in the combustion chamber

508. When a rad. alt. reaches 2,500 ft. what happens to the
display?

 Rad. alt. goes out of view


 Rad. alt. flag in view
 Error warning in view

509. Pitot tubes are heated

 by compressed bleed air


 electrically
 by kinetic heating

510. The suction gauge reads 5 inches of mercury. This is

 above ambient pressure


 above zero pressure
 below ambient pressure
Answers for Questions 501 – 510

1) Inner gimbal
Comment/Reference: Look in B737-200 AMM 34-21-00 pg 1
para 3.B.

2) fit the serviceable pre indexed flux valve into the


same position as the unserviceable pre-indexed flux
valve was removed from
Comment/Reference:

3) 1013.25
Comment/Reference: 1013.25 mb or QNE. A & P Technician
Airframe Textbook Chap XIII page 592

4) via a signal conditioner to the gauge


Comment/Reference: Jeppesen Aircraft Instruments and
Avionics Page 90

5) ambient and minus


Comment/Reference: The 5 inches Hg refers to the
'suction' from ambient.

6) dampen sudden pressure changes


Comment/Reference: The restriction is to damp out
surges in pressure

7) aft of combustion chamber


Comment/Reference:
The hot junction is the sensor, aft of the combustion
chamber

8) Rad. alt. goes out of view


Comment/Reference: Jeppesen Aircraft Radio Systems Page
195

9) electrically
Comment/Reference: CAIPs AL/10-1 Para 3.2

10) below ambient pressure


Comment/Reference: The 5 inches Hg refers to the
'suction' from ambient.
Questions 511 – 520

511. What are the primary colours for use in CRT displays?

 Yellow, cyan, magenta


 Red, blue, green
 Red, blue, yellow

512. Alpha vane signal could be fed to _______when close to


stall.

 flap position
 thrust levers
 fast/slow switch

513. When performing maintenance operations on an aircraft


equipped with RVSM system, and a quick release
disconnect connection is disturbed

 a full test of the system should be carried out


 the allowances for the system should be halved
 a full test of the system should be carried out only
if the aircraft manufacturer recommends to do so

514. For aircraft certificated after 1997 and with RVSM, the
maximum tolerance for the system would be

 +/- 500 feet for the system overall


 +/- 200 feet plus +/- 50 feet for instrument error
 +/- 300 feet plus +/- 50 feet for instrument errors

515. The HSI provides information on

 VOR, ILS, plan, attitude


 VOR, plan, map, ILS, radar
 VOR, ILS, map, radar, attitude
516. The sensing element of the flux valve

 aligns itself to the new heading of the aircraft


after it has stabilised
 remains in the same position attached to the
aircraft structure
 aligns itself to the new heading as the aircraft
turns

517. In a compass swing:


North error -2 degrees, South error - 2 degrees.
The coefficient C is

 0 degrees
 -2 degrees
 +2 degrees

518. When aligning an aircraft for a compass swing, the


maximum allowable error is

 5 degrees
 1 degrees
 3 degrees

519. The pitot head is fitted on the aircraft. The alignment


of pitot head is carried out with

 spirit level
 micrometer
 an inclinometer

520. What kind of gyro is a rate gyro?

 Displacement
 Tied down
 Space
Answers for Questions 511 – 520

1) Red, blue, green


Comment/Reference: Aircraft Instruments and Integrated
Systems, Pallett Page 290

2) thrust levers
Comment/Reference:

3) a full test of the system should be carried out only if


the aircraft manufacturer recommends to do so
Comment/Reference: TGL No.6 Para 10.4 e

4) +/- 200 feet plus +/- 50 feet for instrument error


Comment/Reference:
Download TGL No. Para.8.3

5) VOR, plan, map, ILS, radar


Comment/Reference: Attitude is on the EADI

6) remains in the same position attached to the aircraft


structure
Comment/Reference: Jeppesen - Avionics Fundamentals
Page 106-110 & also read B737-200 AMM 34-21-00 pg 5
para 9.C.

7) 0 degrees
Comment/Reference: CAP 562 Leaflet 8-2 table 1
(-2)-(-2)/2 = 0

8) 5 degrees
Comment/Reference: CAP 562 leaflet 8-2 para 12 Note

9) an inclinometer
Comment/Reference:

10) Tied down


Comment/Reference: AL/10-2 A rate gyro has only 2 axis
of freedom. Aircraft Instruments & Integrated Systems
Pallett page 129
Questions 521 – 530

521. A radio altimeter system can be self tested

 both on the ground only and in the air


 in the air only
 on the ground only

522. The apparent wander for directional gyros is

 maximum at the pole


 compensated by applying a constant torque
 dependant on longitude

523. The Directional Gyro is checked every 15 minutes for

 toppling
 drift
 erection

524. Random drift of a gyro is caused by

 gyro friction and unbalance


 aircraft turning with an error in roll
 error in roll when aircraft is turning

525. Standby Compass adjusting magnets (Flinders Bars) exert


the most amount of influence

 when 90 degrees apart


 when 45 degrees apart
 when parallel to each other
526. With engine static and engine blanks fitted, EPR gauge
reads just above 1.

 This is normal
 Gauge requires re-calibration
 Transmitter is unserviceable

527. Coefficient A is adjusted

 at 270 degrees
 at 360 degrees
 on any heading

528. With an aircraft which has more than one compass system

 both are adjusted on each heading


 master adjusted, slave adjusted, each having a
corrected compass card
 master and slave adjusted, correction card for
master only

529. A DC electrical cable must be positioned how far away


from a compass?

 24 inches
 So as to give no more than 2 degrees deflection of
compass
 20 inches

530. Which pitot probe provides information to the captains


instruments?

 Lower
 Both
 Upper
1) on the ground only
Comment/Reference: B757 training notes

2) compensated by applying a constant torque


Comment/Reference:

3) drift
Comment/Reference: Aircraft Instruments and Integrated
Systems Pallett Page 192

4) gyro friction and unbalance


Comment/Reference: Automatic Flight Control Pallett and
Coyle Page 104

5) when 90 degrees apart


Comment/Reference: Angle between flinders bars
determines their correcting influence.

6) Gauge requires re-calibration


Comment/Reference:

7) on any heading
Comment/Reference: AL/10-5 9.2.7

8) master adjusted, slave adjusted, each having a


corrected compass card
Comment/Reference: Al/10-5 9.2 Note.

9) 24 inches
Comment/Reference: CAP 562 Leaflet 8-2 para 6.1.2

10) Upper
Comment/Reference: http://www.b737.org.uk/probes.htm
Questions 531 – 540

531. On the CWP, what does amber indicate?

 Cautionary info
 Warning
 Present status

532. Which instrument shows Decision Height?

 ADI
 HSI
 ECAM

533. Where does the HSI receive GND speed information from?

 INS
 Pitot static probes
 EICAS

534. When power is switched off, the gimbal brake

 restricts inner gimbal


 stops outer gimbal
 restricts outer gimbal

535. On an EADI the command bars show the

 required flight path compared with horizon


 required flight path compared to planned flight path
 required flight path compared with aircraft position
536. The airdata computer inputs to

 altimeter, FMC, secondary radar


 cabin rate sensors, mach-meters, ASI, altimeter
 mach meter, standby altimeter, ASI

537. What deviation is indicated by 2 dots in a VOR system?

 6°
 10°
 2.5°

538. How may the basic principle of the radio altimeter be


described?

 As a measure of the time between a RF pulse


transmission and the reception of its echo from the
ground directly below the aircraft
 As a series of radio pulses to the ground their
frequency depending on the expansion or contraction
of an evacuated capsule and the deflection of an E
and I bar transducer
 As a comparison of radio altitude against a
barometric altitude referenced to ISA sea level
(1013.25mb)

539. In a compass system, what senses the horizontal


component of the earths magnetic field and where is it
normally fitted?

 A precession device mounted on the yaw axis of the


aircraft
 Flux detectors fitted in the wing tips
 Directional gyro mounted on the roll axis of the
aircraft

540. An RMI has inputs from VOR and

 a remote compass input


 an azimuth gyro
 no other sources
Answers for Questions 531 – 540

1) Cautionary info
Comment/Reference:

2) ADI
Comment/Reference: Avionics Fundamentals Page 185

3) INS
Comment/Reference: Avionics Fundamentals Page 128

4) restricts inner gimbal


Comment/Reference: Used for gimbal lock prevention.
Energised off.

5) required flight path compared with aircraft position


Comment/Reference: Aircraft Instruments and Integrated
Systems Pallett Page 212/213

6) altimeter, FMC, secondary radar


Comment/Reference: FMC for nav. Secondary Radar is ATC
transponder encoder.

7) 10°
Comment/Reference: www.allstar.fiu.edu/aero/FltDirS.htm
VOR is 5° per dot. ILS is 2 1/2° per dot

8) As a measure of the time between a RF pulse


transmission and the reception of its echo from the
ground directly below the aircraft
Comment/Reference:

9) Flux detectors fitted in the wing tips


Comment/Reference: Aircraft Instruments and Integrated
Systems Pallett Page 182

10) an azimuth gyro


Comment/Reference: Aircraft Instruments and Integrated
Systems Pallett Page 194
Questions 541 – 550

541. In a modern HSI, the displays are

 direction and attitude


 course and direction
 course and attitude

542. At what height does the rising runway appear?

 200 ft.
 500 ft.
 300 ft

543. An H on the EHSI indicates

 DME hold
 ILS approach
 VOR hold

544. Compared to air driven gyros, the electric gyro runs

 faster
 slower
 same speed

545. A pneumatic indicator takes its indications from

 reservoir
 compressor outlet
 compressor inlet
546. What is apparent drift due to?

 Errors when aircraft banking


 Earths rotation
 Gyro pivot friction

547. An aircraft airspeed indicator has

 pitot to the capsule


 pitot to the capsule and static to the out side of
the capsule
 static to the capsule

548. Above 2500 ft. the rad. alt.

 pointer is hidden behind a mask with off flag out of


view
 pointer goes to zero to show system is being
monitored
 continues to indicate but with a warning flag

549. How does a machmeter work?

 True airspeed and speed of sound


 Indicated airspeed / temperature
 True airspeed / indicated airspeed

550. When changing a pitot static instrument with quick


release couplings

 a leak check is always required


 a leak check is only required if stated by
manufacturer
 a leak check is not required
Answers for Questions 541 – 550

1) course and direction


Comment/Reference: Aircraft Instruments and Integrated
Systems Pallett page 303

2) 200 ft.
Comment/Reference: Aircraft Radio Systems Powell Page
199 &
www.artietheairplane.com/flat_panel/pfd_attitude_direct
or_indicator.htm

3) DME hold
Comment/Reference: Aircraft Electricity & Electronics
5th Ed Eismin Page 358

4) faster
Comment/Reference:

5) reservoir
Comment/Reference:
External website...

6) Earths rotation
Comment/Reference: Aircraft Instruments and Integrated
Systems Pallett Page 102

7) pitot to the capsule and static to the out side of the


capsule
Comment/Reference: AC65-15A pg 476 figure 12-17

8) pointer is hidden behind a mask with off flag out of


view
Comment/Reference: Avionic Systems: Operation and
Maintenance Page 189

9) True airspeed and speed of sound


Comment/Reference: Kermode Mechanics of Flight 10th
edition p339, and Flight Instruments and Automatic
Flight Control, Harris Page 20 & AC65-15A pg 480

10) a leak check is only required if stated by manufacturer


Comment/Reference: TGL 6 (RVSM) para 10.4 e
Questions 551 – 560

551. The flux detector element

 gives heading with respect to magnetic north


 changes it position after the aircraft heading is
changed
 changes heading with the heading of the aircraft

552. If rate feedback in a flight director goes open


circuit, position indication will

 be sluggish
 oscillate
 go hard over

553. The needle of a resolver is connected to

 two coils and a permanent magnet


 two coils only
 two coils and an electromagnet

554. If the compass fluid has bubbles at low altitude

 this has no influence on compass readings


 this is due to excessive high cabin altitude flying
 this is due to insufficient de-aeration

555. A remote compass compensator unit is replaced. Which of


the following is correct?

 The swing can be performed at a later date


 A compass swing must be performed
 No swing is required if the new heading is within 5
degrees of the old
556. A compass has a residual deviation of +1 degree. To
steer a true heading of 180 degrees the pilot must
steer

 179 degrees
 180 degrees
 181 degrees

557. How is a leading edge flap position indicated in the


cockpit?

 Servomotor
 A measuring device
 Torque synchro

558. In a Machmeter, what type of compensation is there?

 Compensation is not required


 Hair spring
 Square-Law compensation

559. What effect on the rate of precession will a change of


gyro rotor speed have?

 No effect
 Decrease the rotor speed, decrease the rate of
precession
 Increase the rotor speed, decrease the rate of
precession

560. A gyroscopic body has its rigidity increased by

 decreasing the mass of the rotor


 increasing the rotor speed
 decreasing the rotor speed
Answers for Questions 551 – 560

1) gives heading with respect to magnetic north


Comment/Reference: B737-200 AMM 34-21-00

2) oscillate
Comment/Reference:

3) two coils only


Comment/Reference:)

4) this is due to excessive high cabin altitude flying


Comment/Reference: The bubble can be caused by using
the compass at higher elevations and/or exposure to
cold temperatures. The bubble forms as the liquid in
the vial expands or contracts at a rate faster than the
rigid plastic vial, which forms a vacuum or a 'bubble'.

5) A compass swing must be performed


Comment/Reference: AL/10-5 10 & CAP 562

6) 179 degrees
Comment/Reference: CAP 562 leaflet 8-2 table 1
columns 7 and 8

7) Torque synchro
Comment/Reference:

8) Square-Law compensation
Comment/Reference: Aircraft Instruments and Integrated
Systems - Pallett, page 45

9) Increase the rotor speed, decrease the rate of


precession
Comment/Reference: Flight Instruments and Automatic
Flight Control Systems, David Harris Page 33

10) increasing the rotor speed


Comment/Reference: Flight Instruments and Automatic
Flight Control Systems, David Harris Page 33
Questions 561 – 570
561. If a constant torque is applied to a gyroscope, the
rate of precession

 is unaffected by changes in rotor speed


 increases with a higher rotor speed
 increases with a lower rotor speed

562. Random drift of a gyro is caused by

 unbalance and bearing friction in the gyro


 aircraft turning with an error in roll
 rotation of the earth

563. A pendulous vane type erection system fitted to a gyro


horizon works on the principle of

 increased reaction of the air from a fully open port


 increased reaction of the air from a bisected port
 decreased reaction of the air from a fully open port

564. During the normal straight and level flight, the gyro
of an electrical artificial horizon is kept erect in
pitch by a

 torque motor fitted between the outer gimbal ring


and the case controlled by a mercury switch in the
fore and aft axis
 torque motor fitted between the outer gimbal ring
and the case controlled by a mercury switch in the
athwartships axis
 mercury switch in the fore and aft axis controlling
a torque motor fitted between the inner and outer
gimbal rings

565. The normal erection supply to the mercury switches of


the artificial horizon is disconnected

 for the first 40 seconds after initially switching


'ON'
 when the fast erection button is pressed
 during turns
566. The pitch/bank erection system is used in an electrical
gyro horizon to

 to prevent the pitch switch giving a false


indication due to centrifugal effects during a turn
 give full erection control to the roll switch during
a turn
 give full erection control to the pitch switch
during a turn

567. The fast erection push on an electrical gyro horizon


must not be used for a set period after switching on
because

 excessive hunting will take place


 overheating of the gyro rotor windings will occur
 the normal erection switch contact will burn out

568. Apparent drift of a directional gyro is due to

 unbalance of the gimbals


 bearing friction
 the effect of the earth's rotation

569. The erection system on a directional gyroscope has

 a switch on the outer gimbal controlling a motor on


the inner gimbal
 a switch on the inner gimbal controlling a motor on
the outer gimbal
 a switch on the inner gimbal controlling a motor on
the inner gimbal

570. In a rate gyro used to detect movements about a


vertical axis, the amount the gimbal ring moves would
be increased if the

 rotor speed decreases


 spring tension was increased
 angle through which the gyro moves in azimuth
increases
Answers for Questions 561 – 570

1) increases with a lower rotor speed


Comment/Reference: Flight Instruments and Automatic
Flight Control Systems, David Harris Page 33

2) unbalance and bearing friction in the gyro


Comment/Reference: Flight Instruments and Automatic
Flight Control Systems, David Harris Page 37 (Real
Drift)

3) increased reaction of the air from a fully open port


Comment/Reference: Flight Instruments and Automatic
Flight Control Systems, David Harris Page 49

4) torque motor fitted between the outer gimbal ring and


the case controlled by a mercury switch in the fore and
aft axis
Comment/Reference: Flight Instruments and Automatic
Flight Control Systems, David Harris Page 51 (see page
48 for 'athwartship).

5) during turns
Comment/Reference: Flight Instruments and Automatic
Flight Control Systems, David Harris Page 51 bottom

6) to prevent the pitch switch giving a false indication


due to centrifugal effects during a turn
Comment/Reference: Flight Instruments and Automatic
Flight Control Systems, David Harris Page 51

7) excessive hunting will take place


Comment/Reference: AL/10-2 9.3.2 a)

8) the effect of the earth's rotation


Comment/Reference: AL/3-23 12.3.2

9) a switch on the inner gimbal controlling a motor on the


outer gimbal
Comment/Reference: Flight Instruments & Automatic
Flight Control Systems, David Harris Page 46/7

10) rotor speed decreases


Comment/Reference: Rigidity!
Questions 571 – 580

571. The Turn and Slip indicator employs

 an azimuth gyro
 a rate gyro
 a vertical gyro

572. In a Turn and Slip indicator, the effect of increasing


the rotor speed would be

 it would have no effect


 it would under read
 it would over read

573. How is the information on a directional gyro outer


gimbal taken off?

 By a switch on the outer gimbal


 By a flux take-off device
 By a switch on the inner gimbal

574. Why is a directional gyro inner gimbal restricted to


+/- 85º?

 To prevent outer gimbal rotating


 To ensure outer gimbal erection system works
correctly
 To prevent gyro going into gimbal lock

575. The speed of the rotor in a Turn and Slip indicator is


approximately

 4,200 rpm
 22,500 rpm
 2,400 rpm
576. In a directional gyro, the inner/outer gimbal is
corrected to

 15 cosine latitude
 15 sine latitude
 15 sine longitude

577. The electrolyte switches used in gyro systems rely upon

 change in inductance to operate


 change in resistance to operate
 change in voltage applied to operate

578. Precession of a gyro depends on

 angular velocity of the rotor only


 moment of inertia of the rotor only
 moment of inertia AND angular velocity of the rotor

579. Angular momentum of a gyro rotor depends on

 moment of inertia and angular velocity of the rotor


 moment of inertia of the rotor
 angular velocity of the rotor

580. Air driven gyros, compared to electric gyros, generally


rotate

 slower
 faster
 the same speed
Answers for Questions 571 – 580

1) a rate gyro
Comment/Reference: Flight Instruments and Automatic
Flight Control, David Harris Page 54

2) it would over read


Comment/Reference: Aircraft Instruments and Integrated
Systems Pallett page 131

3) By a flux take-off device


Comment/Reference: Flight Instruments and Automatic
Flight Control, David Harris Page 77/78

4) To prevent gyro going into gimbal lock


Comment/Reference:

5) 4,200 rpm
Comment/Reference: Flight Instruments and Automatic
Flight Control, David Harris Page 53

6) 15 sine latitude
Comment/Reference: Flight Instruments and Automatic
Flight Control, David Harris Page 37/8

7) change in resistance to operate


Comment/Reference: Flight Instruments and Automatic
Flight Control, David Harris Page 50/51 & Automatic
Flight Control 4th ed Pallett & Coyle

8) moment of inertia AND angular velocity of the rotor


Comment/Reference: Flight Instruments and Automatic
Flight Control, David Harris Page 53

9) moment of inertia and angular velocity of the rotor


Comment/Reference: Flight Instruments and Automatic
Flight Control, David Harris Page 33

10) slower
Comment/Reference: Flight Instruments and Automatic
Flight Control, David Harris Page 41
Questions 581 – 590

581. Gyro rigidity is proportional to

 mass and speed


 mass, speed and radius of mass from spin axis
 mass, and radius of mass from spin axis

582. Gyro precessional force is

 inversely proportional to the applied force


 directly proportional to applied force
 proportional to the square of the applied force

583. The speed of a vacuum driven gyro horizon rotor is


approximately

 22,000 rpm
 4,200 rpm
 15,000 rpm

584. A V.S.I. is connected to

 static pressure
 pitot pressure
 vacuum

585. With a V.S.I. pointer at position 1 on the upper half


of the scale it indicates

 100 ft/minute rate of climb


 1,000 ft/minute rate of descent
 1,000 ft/minute rate of climb
586. After disconnecting the supply to an electrically
operated gyro instrument it is recommended that, to
allow the gyro rotor to stop, before moving the
instrument

 three minutes should elapse


 fifteen minutes should elapse
 seven minutes should elapse

587. The Port and Starboard static vents on an aircraft are


inter-connected to

 reduce compressibility error


 minimize errors caused by leaks in the system
 cancel errors caused in the static system when the
aircraft yaws

588. Which of the following would cause a displacement gyro


to topple?

 Running gyro at low speed


 Running gyro at high speed
 Inverting the gyro

589. In level flight, a V.S.I. will indicate

 vertically up
 vertically down
 horizontal to left

590. On descent, the pressure in a V.S.I. capsule

 is the same as case pressure


 lags the case pressure
 leads the case pressure
Answers for Questions 581 – 590

1) mass, speed and radius of mass from spin axis


Comment/Reference: Flight Instruments and Automatic
Flight Control, David Harris Page 33

2) directly proportional to applied force


Comment/Reference: Flight Instruments and Automatic
Flight Control, David Harris Page 33

3) 15,000 rpm
Comment/Reference: Flight Instruments and Automatic
Flight Control, David Harris Page 46

4) static pressure
Comment/Reference: AC65-15A pg 478 ‘Rate-of-Climb
Indicator’ & Flight Instruments and Automatic Flight
Control, David Harris Page 22/23

5) 1,000 ft/minute rate of climb


Comment/Reference: AC65-15A pg 478 & Flight Instruments
and Automatic Flight Control, David Harris Page 24

6) fifteen minutes should elapse


Comment/Reference: CAIPs AL/10.2 8.1 c)

7) cancel errors caused in the static system when the


aircraft yaws
Comment/Reference:

8) Running gyro at low speed


Comment/Reference:

9) horizontal to left
Comment/Reference: AC65-15A pg 478 & Flight Instruments
and Automatic Flight Control, David Harris Page 22

10) leads the case pressure


Comment/Reference: AC65-15A pg 478/479 & Flight
Instruments and Automatic Flight Control, David Harris
Page 22
Questions 591 – 600

591. A small constant leak in the case of V.S.I. fitted in a


pressurized aircraft would, during level flight, cause
the instrument to indicate

 a rate of descent
 zero
 a rate of climb

592. When a force is applied to a horizontal gyro, the


precession of the rotor will continue until

 as long as the force is applied


 plane of rotation is in line with the plane of the
applied force
 plane of rotation is in line with the precessional
force

593. An artificial horizon has

 the inner gimbal pivoted vertically


 the inner gimbal pivoted longitudinally
 the inner gimbal pivoted laterally

594. A pressure of one atmosphere is equal to

 14.7 PSI
 100 millibar
 1 inch Hg

595. In the directional gyro

 the outer gimbal is pivoted vertically


 the outer gimbal is pivoted longitudinally
 the outer gimbal is pivoted laterally
596. The millibar is a unit of

 barometric pressure
 atmospheric temperature
 pressure altitude

597. In the Turn and Slip indicator

 the spin axis is longitudinal


 the spin axis is vertical
 the spin axis is lateral

598. In an altimeter, the

 capsule is evacuated and sealed


 inside of the capsule is connected to static
pressure
 capsule and case are connected via a calibrated
choke

599. The units on the calibrated scale of a V.S.I. are


expressed in

 knots (kts)
 miles per hour (mph)
 hundreds of feet per minute

600. A standby air supply for gyro operation could be


obtained from

 a tapping from the induction manifold


 a pitot head
 a venture
Answers for Questions 591 – 600

1) a rate of climb
Comment/Reference: Flight Instruments and Automatic
Flight Control, David Harris Page 22

2) plane of rotation is in line with the plane of the


applied force
Comment/Reference: Flight Instruments and Automatic
Flight Control, David Harris Page 33

3) the inner gimbal pivoted laterally


Comment/Reference: Flight Instruments and Automatic
Flight Control, David Harris Page 47

4) 14.7 PSI
Comment/Reference: AC65-15A pg 540

5) the outer gimbal is pivoted vertically


Comment/Reference: Flight Instruments and Automatic
Flight Control, David Harris Page 42

6) barometric pressure
Comment/Reference: AC65-15A pg 540

7) the spin axis is lateral


Comment/Reference: AC65-15A pg 482 & Flight Instruments
and Automatic Flight Control, David Harris Page 53

8) capsule is evacuated and sealed


Comment/Reference: AC65-15A pg 476 & Flight Instruments
and Automatic Flight Control, David Harris Page 6

9) hundreds of feet per minute


Comment/Reference: AC65-15A pg 478. (Light aircraft
only - large aircraft are 1000s ft/min)

10) a venturi
Comment/Reference: Flight Instruments and Automatic
Flight Control, David Harris Page 40
Questions 601 – 610

601. A rate two turn is

 180 degrees per minute


 90 degrees per minute
 360 degrees per minute

602. The rate of turn information from a turn coordinator


(compared to a Turn and Slip) is

 more accurate
 more instantaneous
 less accurate

603. Temperature correction in a sensitive altimeter


mechanism is provided by a

 U-spring acting on the capsule


 bi-metal U-spring acting on the capsule
 balance weight

604. The capsule in a vertical speed indicator will be


expanded when the aircraft is

 climbing
 in level flight
 descending

605. After replacing an instrument of the pitot-static


group, it is necessary to

 calibrate the instrument concerned


 carry out a leak test on the appropriate system(s)
 blow through the lines with a clean low pressure air
supply
606. If the pitot and static pipe lines were cross
connected at the instrument panel connection,
application of pressure to the pitot head would cause
the

 altimeter reading to decrease, vertical speed


indicator to indicate descent
 altimeter reading to decrease, vertical speed
indicator to indicate climb
 altimeter reading to increase, vertical speed
indicator to indicate descent

607. A constant force applied to the inner gimbal of a


vertical gyro would cause

 a continual precession in roll


 a pitch error and gyro topple
 a roll error and gyro topple

608. A rate of turn indicator dial marked '2 minutes'


refers to a

 rate 3 turn
 rate 1 turn
 rate 2 turn

609. A pitot or static leak check is carried out

 only when an instrument is changed


 whenever the pitot or static systems are disturbed
 only when a leak is suspected

610. The temperature of boiling water at standard pressure


on the Fahrenheit and Centigrade scale is

 180 deg. and 100 deg. respectively


 100 deg. and 32 deg. respectively
 212 deg. and 100 deg. Respectively
1) 360 degrees per minute
Comment/Reference: http://en.wikipedia.org/wiki/ROT
Flight Instruments and Automatic Flight Control, David
Harris Page 54, Aircraft Instruments and Integrated
Systems Page 131.

2) more accurate
Comment/Reference: Flight Instruments and Automatic
Flight Control, David Harris Page 55

3) bi-metal U-spring acting on the capsule


Comment/Reference: Flight Instruments and Automatic
Flight Control, David Harris Page 8

4) descending
Comment/Reference: AC65-15A pg 478 & Flight Instruments
and Automatic Flight Control, David Harris Page 22/23

5) carry out a leak test on the appropriate system(s)


Comment/Reference: AL/10-1 15.5

6) altimeter reading to decrease, vertical speed


indicator to indicate descent
Comment/Reference:

7) a continual precession in roll


Comment/Reference:

8) rate 1 turn
Comment/Reference: http://en.wikipedia.org/wiki/ROT
Flight Instruments and Automatic Flight Control, David
Harris Page 54

9) whenever the pitot or static systems are disturbed


Comment/Reference: AL/10-1 15.5

10) 212 deg. and 100 deg. respectively


Comment/Reference:
Questions 611 – 620

611. A temperature of 59°F is equivalent to

 15°C
 14.69°C
 32°C

612. Aircraft heading (HDG) is

 the angle between True North and the desired track


 the angle between True North and the actual track
 the angle between True North and the longitudinal
axis of the aircraft

613. Wind angle is the direction of the wind measured

 in degrees from the aircraft's heading


 in degrees from True North
 in degrees from the desired track

614. Limit stops are fitted in an artificial horizon to

 limit the outer gimbal movement


 reduce gimbal nutation
 prevent gimbal lock

615. The electrical output from a remote gyro to an


attitude director indicator is

 by a torque synchro
 by a differential synchro
 by a control synchro
616. Agonic lines link places of

 equal variation
 different variation
 zero variation

617. Position error is caused by

 mechanical imperfections in an instrument


 pitot head position
 instrument location in the instrument panel

618. To convert degrees Centigrade to degrees Kelvin

 use the formula (deg C x 9/5) + 32


 add 273 degrees
 add 112 degrees

619. Damping on a RATE GYRO can be either

 eddy current, variable spring or moving iron


 viscous, eddy current or air dash pot
 viscous, eddy current or variable spring
Answers for Questions 611 – 620

1) 15°C
Comment/Reference: AC65-9A pg 29

2) the angle between True North and the longitudinal axis


of the aircraft
Comment/Reference: Flight Instruments and Automatic
Flight Control, David Harris Page 87

3) in degrees from True North


Comment/Reference:

4) prevent gimbal lock


Comment/Reference:

5) by a control synchro
Comment/Reference: Avionics Fundamentals Page 93 Fig
5-20

6) zero variation
Comment/Reference:
https://ntc.cap.af.mil/ops/DOT/school/L23CockpitFam/mag
neticcompass.cfm

7) pitot head position


Comment/Reference: Flight Instruments and Automatic
Flight Control Systems, David Harris Page 17

8) add 273 degrees


Comment/Reference: AC65-9A pg 226

9) viscous, eddy current or air dash pot


Comment/Reference:
Questions 621 – 630

621. Electrical driven gyros are

 rotated slower than air driven gyros


 rotated at the same speed as air driven gyros
 rotated faster than air driven gyros

622. In a ratiometer temperature indicating circuit, there


is a break in the circuit to the bulb. This will give

 no scale deflection
 full scale deflection
 mid scale deflection

623. With the gyro at normal running speed, a torque


applied to the inner gimbal ring of a vertical gyro
will cause the

 inner ring to move


 outer ring to move
 outer and inner ring to move

624. When an aircraft is descending, the pressure in the


altimeter case

 will not affect the aneroid capsule


 will cause the aneroid capsule to contract
 will cause the aneroid capsule to expand

625. The supply of Desynn indicating system

 is alternating current at 400 c/s


 is direct current
 is alternating current at 50 c/s
626. A sensitive altimeter reading 100 ft. when the
millibar scale is set to the atmospheric pressure at
airfield level (QFE)

 indicates that the airfield is 100 ft. above sea


level
 indicates that the aircraft is in a region of high
pressure and the reading must be corrected to I.S.A.
standards
 indicates that the instrument is unserviceable

627. After using a pitot-static test set the pressure in


the aircraft static system should be released to the
atmosphere by

 removing the static connector from its static vent


after waiting for a period of three minutes
 venting the static system via an internal bleed in
the test set
 removing the static connector from its static vent

628. An altimeter is operated

 by the pitot system


 by the vacuum system
 by the static system

629. What is the purpose of the bimetallic strip in the


altimeter?

 Compensates for change in density


 Corrects for capsule elasticity
 Compensates for non-linear tension in the hairspring

630. Which axis does the directional gyro spin on?

 Both vertical and horizontal


 Vertical
 Horizontal
Answers for Questions 621 – 630

1) rotated faster than air driven gyros


Comment/Reference: Flight Instruments and Automatic
Flight Control Systems, David Harris Page 41

2) full scale deflection


Comment/Reference: Think of it this way - resistance
increases with temperature, and that drives it toward
fsd. If the bulb circuit is broken, the resistance
will be infinite

3) outer ring to move


Comment/Reference:

4) will cause the aneroid capsule to contract


Comment/Reference: Flight Instruments and Automatic
Flight Control Systems, David Harris Page 5

5) is direct current
Comment/Reference: AC65-15A pg 483

6) indicates that the instrument is unserviceable


Comment/Reference: Flight Instruments and Automatic
Flight Control Systems, David Harris Page 9

7) venting the static system via an internal bleed in the


test set
Comment/Reference:

8) by the static system


Comment/Reference: AC65-15A pg 476

9) Corrects for capsule elasticity


Comment/Reference: Flight Instruments and Automatic
Flight Control Systems, David Harris Page 5

10) Horizontal
Comment/Reference: Flight Instruments and Automatic
Flight Control Systems, David Harris Page 8, and page
41/42
Questions 631 – 640

631. The capsule in an altimeter responds to

 absolute pressure
 gauge pressure
 differential pressure

632. The aeroplane monitor on the artificial horizon is


fitted to the

 inner gimbal
 instrument case
 rotor

633. True airspeed in an Air Data Computer is a function of

 airspeed and altitude


 mach number and temperature
 airspeed and temperature

634. The advantage of an instantaneous V.S.I. over a


conventional one is

 it has an accelerometer which prevents the lag of a


conventional one
 it does not require pitot/static pressure
 it does not require warming up

635. For a particular I.A.S. as the density decreases with


altitude, the T.A.S.

 remains the same


 decreases
 increases
636. The type of gyro used in a gyro compass is a

 rate gyro
 vertical gyro
 directional gyro

637. A desiccant used in the storage of instruments

 is anti-freeze oil
 is silica-gel
 is sodium-bicarbonate

638. What effect on the rate of precession will a change of


gyro rotor speed have?

 Increase the rotor speed, decrease the rate of


precession
 No effect
 Decrease the rotor speed, decrease the rate of
precession

639. Bourdon Tubes have

 oval cross section


 circular cross section
 toroidal cross section

640. In a Bourdon tube

 one end is sealed and the other end open to


atmosphere
 both ends sealed
 one end is sealed and the other end open to the
pressure source
1) absolute pressure
Comment/Reference: Flight Instruments and Automatic
Flight Control Systems, David Harris Page 36

2) instrument case
Comment/Reference: Flight Instruments and Automatic
Flight Control Systems, David Harris Page 47

3) mach number and temperature


Comment/Reference:
Aircraft Instruments and Integrated Systems Pallett
Page 272

4): it has an accelerometer which prevents the lag of a


conventional one
Comment/Reference:
Flight Instruments and Automatic Flight Control
Systems, David Harris Page 27

5) increases
Comment/Reference:
Flight Instruments and Automatic Flight Control
Systems, David Harris Page 24

6) directional gyro
Comment/Reference:

7) is silica-gel

8) Increase the rotor speed, decrease the rate of


precession

9) oval cross section


Comment/Reference: AC65-15A pg 471 fig 12-3 &
Flight Instruments and Automatic Flight Control
Systems David Harris Page 182 Figure 7.1

10) one end is sealed and the other end open to the
pressure source
Comment/Reference: AC 65-15A pg 471 Fig 12-3 &
Flight Instruments and Automatic Flight Control
Systems David Harris Page 182
Questions 641 – 650

641. Pressure gauge calibrators (Dead Weight Testers) use


the

 Charle's Law
 Brahm's press principle
 Boyle's Law

642. The fluid suitable for use in a Dead Weight Tester is

 kerosene
 castor-oil
 anti-freeze oil

643. The distance readout on an HSI is

 dialed in by the pilot


 from the aircraft ATC system
 from the aircraft DME system

644. A gyroscope having one plane of freedom at right


angles to the plane of rotation, and its gimbal
restrained either electrically or by a spring, is
known as

 a tied gyro
 a rate gyro
 an earth gyro

645. Pressure may be expressed in

 force per unit area


 force per unit volume
 weight or mass
646. What is the purpose of the hair spring in a Bourdon
tube pressure gauge?

 To return the pointer to zero


 To reduce 'backlash'
 To act a controlling force

647. The ADIs attitude information is normally obtained


from the aircraft's

 directional gyros
 attitude rate gyros
 vertical gyros

648. One dot VOR deviation represents

 2 miles
 5°
 1¼°

649. An instrument used for measuring negative pressures

 cannot be of the Bourdon tube type


 has the Bourdon tube reversed within the case
 has anti-clockwise pointer movement if Bourdon tube
operated

650. An absolute pressure gauge measures

 pressures extremely accurately


 the applied pressure referred to atmospheric
pressure
 the applied pressure referred to a perfect vacuum
1) Brahm's press principle
Comment/Reference:

2) castor-oil
Comment/Reference:

3) from the aircraft DME system

4) a rate gyro
Comment/Reference: www.tpub.com/neets/book15/63e.htm
Flight Instruments and Automatic Flight Control
Systems, David Harris Page 52

5) force per unit area


Comment/Reference: AC65-9A pg 226

6) To reduce 'backlash'
Comment/Reference:

7) vertical gyros

8) 5°

9) has anti-clockwise pointer movement if Bourdon tube


operated
Comment/Reference:
Flight Instruments and Automatic Flight Control Systems
David Harris Page 182.
http://en.wikipedia.org/wiki/Manometer#Bourdon
The Bourdon tube is a coil, as the gauge pressure
increases the tube will tend to uncoil, while a reduced
gauge pressure will cause the tube to coil more
tightly.

10) the applied pressure referred to a perfect vacuum


Comment/Reference:
Questions 651 – 660

651. Gauge pressure as indicated on a direct reading


Bourdon Tube pressure gauge is equal to

 atmospheric pressure minus absolute pressure


 absolute pressure plus atmospheric pressure
 absolute pressure minus atmospheric pressure

652. The to/from indicator on an HSI informs the pilot of


which direction he is tracking relative to

 an ADF station
 a VOR station
 an ILS station

653. A pressure gauge, such as a hydraulic brake pressure


gauge, indicates 1,000 p.s.i. In terms of absolute
pressure, this represents

 1,014.7 p.s.i.
 985.3 p.s.i.
 1,000 p.s.i.

654. To fill a Dead Weight Tester

 screw out hand wheel and fill reservoir


 remove platform and fill cylinder
 screw in hand wheel and fill reservoir

655. With an aircraft on the ground and QNH set on the


millibar scale of the altimeter, the altimeter will
read

 the airfield height


 zero
 off scale
656. When checking a sensitive altimeter on a pre-flight
inspection

 the standard sea level barometric pressure is always


set on the millibar scale
 the ambient air pressure is set on the millibar
scale
 the ambient air pressure corrected for temperature
is set on the millibar scale

657. The HSI compass card is positioned by the

 aircraft ADF system


 heading select knob
 compass system

658. The applied pressure to an A.S.I. varies with the

 square root of the speed


 square of the speed
 cube root of the speed

659. The supply to an A.S.I.

 are pitot and static pressure


 is static pressure only
 is pitot pressure only

660. The capsule for an A.S.I. will be expanding when the


aircraft is

 decelerating
 accelerating
 climbing
Answers for Questions 651 – 660

1) absolute pressure minus atmospheric pressure


Comment/Reference:

2) a VOR station
Comment/Reference:

3) 1,014.7 p.s.i.
Comment/Reference:

4) screw out hand wheel and fill reservoir


Comment/Reference:

5) the airfield height


Comment/Reference: Flight Instruments and Automatic
Flight Control Systems, David Harris Page 9

6) the ambient air pressure is set on the millibar scale


Comment/Reference: QFE is set and it should read
airfield height

7) compass system
Comment/Reference:

8) square of the speed


Comment/Reference: Flight Instruments and Automatic
Flight Control Systems, David Harris Page 15

9) are pitot and static pressure


Comment/Reference: AC65-15A pg 476 Flight Instruments
and Automatic Flight Control Systems, David Harris Page
13

10) accelerating
Comment/Reference: AC65-15A pg 476 & Flight Instruments
and Automatic Flight Control Systems, David Harris Page
13
661. The deflection of the ADI command bars when flying a
localiser approach is proportional to

 the difference between the amplitudes on the two


modulations
 the sum of the amplitude of the two modulations
 the difference in amplitude of the two r.f. carriers

662. If an aircraft is flying straight and level in still


air, airspeed will be

 equal to the ground speed


 less than the ground speed
 greater than the ground speed

663. If an aircraft flying in still air at 400 knots,


encounters a head wind of 50 knots, its ground speed
is

 450 knots
 350 knots
 400 knots

664. To provide a linear scale on an A.S.I., a

 ranging bar and screws are fitted


 10 to 1 gearing is used
 bi-metal corrector is employed

665. A machmeter is an instrument which indicates the speed


of

 sound relative to the aircraft's altitude


 the aircraft relative to the local sonic speed
 the aircraft relative to the speed of sound at
ground level
666. The moving element of a ratiometer has

 one coil
 two coils
 three coils

667. When carrying out a pressure leak test on an


altimeter, you are checking for leaks in the

 capsule stack
 instrument case
 pressure chamber

668. If an altimeter millibar scale was set to 1013.25 and


the barometric pressure at that time was 1020, the
altimeter should read

 zero feet
 below zero feet (negative altitude)
 positive altitude

669. An aircraft flying towards a VOR station shows


indications of 120° and 'TO'. After passing over the
station, on the same course, the indications will be

 300° and FROM


 120° and FROM
 300° and TO

670. The command bars in a flight director system indicate

 the actual path with respect to required path


 true horizon
 the required path with respect to actual path
Answers for Questions 661 – 670

1) the difference between the amplitudes on the two


modulations
Comment/Reference:

2) equal to the ground speed


Comment/Reference:

3) 350 knots

4) ranging bar and screws are fitted


Comment/Reference: Aircraft Instruments and Integrated
systems Pallett Page 44 fig 2.18 and text below

5) the aircraft relative to the local sonic speed


Comment/Reference: AC65-15A pg 480 & Flight Instruments
and Automatic light Control Systems, David Harris Page
19, and aircraft Instruments and Integrated Systems
Pallett age 45

6) two coils
Comment/Reference: Flight Instruments and Automatic
light Control systems Page 186 Fig 7.4 and para below

7) instrument case
Comment/Reference: Jepessen A & P Technician Textbook
p11-9

8) below zero feet (negative altitude)


Comment/Reference: Assuming aircraft is on the ground

9) 120° and FROM


Comment/Reference:

10) the required path with respect to actual path


Comment/Reference:
Questions 671 – 680

671. When changing a windscreen panel which has a standby


magnetic compass located in the vicinity

 precautions should be taken that the bonding tag is


attached to the correct attachment bolt
 precautions must be taken to ensure that the
attachment bolts are of the specified type
 The attachment bolts should be tightened in an anti-
clockwise direction around the window

672. On a conventional RMI the angle between the compass


datum and the radio pointer arrow is

 the relative bearing


 the complimentary bearing
 the magnetic bearing

673. A compass is made aperiodic by

 locking
 using fluid
 tying it to the case

674. Isogonal lines link places of

 different variation
 equal variation
 zero variation

675. In an artificial horizon Pendulosity Error is caused


by

 displacement of erection control device


 bottom heaviness of inner gimbals
 bottom lightness of inner gimbals
676. If True Airspeed is 470 knots, what is the Equivalent
Air speed?

 278 knots
 550 knots
 662 knots

677. If the Airspeed Indicator reading is 300 Knots, what


is the Calibrated Airspeed?

 296 knots
 293 knots
 304 knots

678. In an Artificial Horizon, Erection Error is caused by

 bottom heaviness of inner gimbals


 bottom lightness of inner gimbals
 displacement of erection control device

679. A Vertical Speed indicator Metering Unit Consist of

 an orifice
 a capillary
 both an orifice and capillary

680. Flux Valve senses angle of horizontal component with


respect to the aircraft's

 both lateral and longitudinal axis


 longitudinal axis
 lateral axis
Answers for Questions 671 – 680

1) precautions must be taken to ensure that the


attachment bolts are of the specified type
Comment/Reference: CAP 562 Leaflet 6-8 para 4.7

2) the relative bearing

3) using fluid
Comment/Reference:

4) equal variation
Comment/Reference: Aircraft Instruments and Integrated
Systems Pallett Page 81

5) bottom heaviness of inner gimbals


Comment/Reference: Flight Instruments and Automatic
Flight Control Systems, David Harris Page 49
(acceleration error)

6) 278 knots
Comment/Reference: True airspeed is always higher than
EAS (or IAS) at any altitude above Sea Level.

7) 304 knots
Comment/Reference: Flight Instruments and Automatic
Flight Control Systems, David Harris Page 14

8) displacement of erection control device


Comment/Reference: Aircraft Instruments & Systems.
Pallett pg 122

9) both an orifice and a capillary


Comment/Reference:

10) longitudinal axis


Comment/Reference: Flight Instruments and Automatic
Flight Control Systems, David Harris Page 64/65 & B737-
200 AMM 34-21-00
Questions 681 – 690

681. The manual VOR input is for

 the course deviation bar


 the radio magnetic indicator
 the ADI

682. After correction of the north-south heading reading on


a compass swing, the resultant correction is known as

 magnetic heading
 residual deviation
 correct heading

683. Apparent drift on directional gyro is corrected by

 mercury switch on outer ring


 an adjustment nut on inner ring
 series of balance holes drilled in gyro rotor

684. If a micro adjuster is replaced in a compass system,


you would

 carry out a new compass swing


 set it to zero datum
 set it up the same as the one removed

685. In a compass swing:


North error -2 degrees, South error + 2 degrees.

The coefficient C is

 -2 degrees
 0 degrees
 +2 degrees
686. Purpose of Altitude Alerting is to warn the pilot of

 approach to or deviation from selected altitude


 selection of altitude
 altitude information

687. Aircraft certified before 1997 with RVSM, maximum


tolerance for the system is

 +/- 500ft system tolerance


 +/- 300ft system, +/-50ft for instrument error
 +/- 200ft system, +/-50ft for instrument error

688. Machmeters work on

 pitot
 pitot and static
 static

689. An aircraft with Mach warning will warn

 when Mcrit is reached


 when envelope limit is reached
 when Mach 1 is exceeded

690. An HSI provides what information?

 VOR, map, attitude, ILS


 VOR, ILS, plan, attitude
 VOR, plan, ILS, map, radar
Answers for Questions 681 – 690

1) the course deviation bar

2) residual deviation

3) an adjustment nut on inner ring


Comment/Reference: Flight Instruments and Automatic
Flight Control Systems Page 45

4) carry out a new compass swing


Comment/Reference:

5) -2 degrees
Comment/Reference: This is not he same as a previous, &
CAP 562 Leaflet 8-2 para 14.1 Table 1

6) approach to or deviation from selected altitude


Comment/Reference:
www.domingoaereo.hpg.ig.com.br/boeing727/manual/warning
s.htm

7) +/- 300ft system, +/-50ft for instrument error


Comment/Reference: TGL No.6 Para.8.3
www.ecacnav.com/RVSM

8) pitot and static


Comment/Reference: AC65-15A pg 480

9) when Mcrit is reached


Comment/Reference:

10) VOR, plan, ILS, map, radar


Comment/Reference:
Questions 691 – 700

691. Compressibility error in a pitot head is caused by

 compression of air in the tube at high speed


 misalignment of pitot head
 blockage in the pitot tube

692. The earth's magnetic field is

 horizontal across the earth


 vertical at the poles, horizontal at the magnetic
equator
 vertical across the earth

693. There is an air bubble in the compass:

 It is due to high altitude


 It is required, to compensate for expansion of the
fluid
 The fluid is not aerated properly

694. A flux detector output is a

 A.C. voltage at twice the frequency of the


excitation voltage
 A.C. voltage at the same frequency as the excitation
voltage
 rectified D.C. voltage

695. Primary Radar is

 reflected signal from aircraft's own radar


 interrogation of aircraft's transponder
 for ground use only
696. The general function of the ATC is to

 reply with an identification signal after being


interrogated by a ground station
 interrogate other aircraft which reply with an
identification signal
 interrogate the ground station which replies with an
identification

697. ACARS is primarily used for communicating with

 air traffic control


 maintenance base
 other aircraft

698. SELCAL is used to

 alert ground stations


 monitor ground signals
 alert other aircraft

699. HF communication frequency is in the range

 108 - 118 MHz


 23 - 30 kHz
 3-30 MHz

700. When an autopilot is Fail Passive,

 the autoland can continue


 the aircraft reverts to fail operational autopilot
 the pilot must resume control and land the aircraft
Answers for Questions 691 – 700
1) compression of air in the tube at high speed
Comment/Reference: Flight Instruments and Automatic
Flight Control Systems - David Harris P17 ASI Errors
(3)

2) vertical at the poles, horizontal at the magnetic


equator
Comment/Reference:

3) It is due to high altitude


Comment/Reference: The bubble can be caused by using
the compass at higher elevations and/or exposure to
cold temperatures. The bubble forms as the liquid in
the vial expands or contracts at a rate faster than the
rigid plastic vial, which forms a vacuum or a 'bubble'.
www.brunton.com/faq.php?id=53&faq_cat_selected=Compass

4) A.C. voltage at twice the frequency of the excitation


voltage
Comment/Reference: Aircraft Instruments and Integrated
Systems Pallett Page 189

5) reflected signal from aircraft's own radar


Comment/Reference: ATC is ground use, but the Wxr and
TCAS are also 'primary radar'

6) reply with an identification signal after being


interrogated by a ground station
Comment/Reference:

7) maintenance base
Comment/Reference:
8) monitor ground signals
Comment/Reference: B737-3/4/500 AMM 23-00-00 pg 1 para
2 D) & Aircraft Electricity & Electronics 5th Ed Eisman
pg 297

9) 3-30 MHz
Comment/Reference: AC65-15A pg 524
http://en.wikipedia.org/wiki/High_Frequency

10) the pilot must resume control and land the aircraft
Comment/Reference: Pallett Automatic Flight Control
pg 211
Questions 701 – 710

701. The fundamental theory of operation of a ranging radar


is

 time taken to receive returns


 percentage of emitted signal returned
 frequency of returns

702. With Control Wheel Steering (CWS), if the flight path


is disturbed by the pilot, the course will be
corrected by the

 flight director input


 autopilot panel
 normal flight control

703. An HF coupling is used to match the

 receiver
 aerial
 boom-set (headphones and mic)

704. On an aircraft's navigation display, distance


measuring equipment gives

 slant range to beacon


 ground range to beacon
 height and range to beacon

705. On a modern aircraft flight instrument display system,


radio deviation is shown on the

 EHSI & EADI


 EADI
 EHSI
706. On a flight management system, the database is updated
every

 1 Month
 3 Months
 2 Months

707. What is the purpose of the autopilot?

 To enable the aircraft to land in bad weather


 To relieve the flight crew of their duties during
long haul flights
 To reduce the flight crew's workload

708. An ADF system includes

 a loop antenna
 a sense antenna
 both a sense and loop antenna

709. Autopilot servo brake is energized

 at the same time as the clutch


 to actuate on
 to actuate off

710. Which category are hand mikes considered essential?

 Heavy passenger aircraft


 Aerial work aircraft
 Light aircraft
Answers for Questions 701 – 710

1) time taken to receive returns


Comment/Reference:

2) normal flight control


Comment/Reference:

3) aerial
Comment/Reference: AC65-15A pg 524 & Aircraft Radio
Systems Powell Page 30

4) slant range to beacon


Comment/Reference: Jeppesen A & P Technician Airframe
Textbook page 12-17 & Aircraft Electricity &
Electronics 5th Ed Eisman pg 310

5) EHSI & EADI


Comment/Reference: Radio deviation is VOR or ILS. The
deviation indicator is duplicated. It is on the EHSI
and EADI

6) 1 Month
Comment/Reference:

7) To reduce the flight crew's workload


Comment/Reference: AC65-15A pg 511

8) both a sense and loop antenna


Comment/Reference: AC65-15A pg 529 & Aircraft
Electricity & Electronics 5th Ed Eisman pg 303

9) to actuate off
Comment/Reference: Pallett Aircraft Electrical Systems
3rd Ed Page 142

10) Light aircraft


Comment/Reference: ANO Schedule 4 Scale N and Article
47, prohibit them on Transport Category aircraft.
However, no direct reference to them being 'essential'
on light aircraft is found.
Questions 711 – 720

711. Emergency frequency is

 121.5 MHz
 123.5 MHz
 125.5 MHz

712. 121.5 MHz is what frequency?

 VHF
 ILS
 VOR

713. 112.1 MHz is what frequency?

 ILS
 VHF
 VOR

714. On a fibreglass aerial, what paint should be used?

 Polyurethane
 Cellulose only
 Not cellulose

715. IRS obtains basic position information by

 integrating twice the accelerometer output


 differentiating the accelerometer output
 integrating the accelerometer output
716. When painting a neoprene coated radio antenna

 use cellulose paint


 do not use cellulose paint
 use any paint

717. To minimize cross cable interference

 run radiating and affected cables as separate


bundles
 run cables as single bundle
 place filter in power supply

718. How many axis does the aircraft autopilot control?

 Four
 Three
 Two

719. What does 7700 indicate on an ATC control panel?

 Hijack
 Emergency
 Radio Failure

720. The number of VHF frequency channels is

 760
 360
 720
Answers for Questions 711 – 720

1) 121.5 MHz
Comment/Reference: JAR OPS 1.820 or 1.850. Jeppesen A&P
Technician Airframe Textbook Page 12-25 & AC15-15A pg
535 ‘test equipment’

2) VHF
Comment/Reference: 118-137 MHz is VHF frequency.
Jeppesen A&P Technician Airframe Textbook Page 12-13
AC65-15A pg 520

3) VOR
Comment/Reference: 112-118 MHz is VOR frequency. Less
than 112 MHz odd decimals are ILS. Jeppesen A&P
Technician Airframe Textbook Page 12-14

4) Not cellulose
Comment/Reference: CAIPs RL/2-2 8.1.2

5) integrating twice the accelerometer output


Comment/Reference:

6) do not use cellulose paint


Comment/Reference: CAIPs RL/2-2 8.1.2

7) run radiating and affected cables as separate bundles


Comment/Reference:

8) Three
Comment/Reference: Pallett Automatic Flight Control 2nd
Edition Page 81. Jeppesen A&P Technician Airframe
Textbook Page 12-36

9) Emergency
Comment/Reference:
http://en.wikipedia.org/wiki/7700#Emergency_codes

10) 720
Comment/Reference: 720 channels at 25kHz spacing, do a
search
Questions 721 – 730

721. The autopilot is instructed by the

 FMC
 Flight Director
 GPS

722. ILS marker beacon lights are

 blue, amber, white


 green, blue, amber
 blue, white, green

723. 111.1 MHz is

 a HF frequency
 a VOR frequency
 an ILS frequency

724. An autopilot PFCU servo brake is

 energised off
 energised at the same time as the clutch
 energised on

725. The aviation distress frequency is

 121.5 kHZ
 121.5 MHz
 122.5 MHz
726. A radar altimeter in track mode is effective to

 2500 ft.
 100 ft.
 2000 ft.

727. 112.1 MHz is

 an ILS frequency
 an ADF frequency
 a VOR frequency

728. What does a Decca Navigation system operate on?

 Very High Frequency


 Low Frequency
 High Frequency

729. Which of the following has a hyperbolic curve?

 Loran C
 DME
 VOR

730. A GPS satellite will come into view

 10° above the horizon with respect to the viewer


 15° above the horizon with respect to the viewer
 20° above the horizon with respect to the viewer
Answers for Questions 721 – 730

1) FMC
Comment/Reference:

2) blue, amber, white


Comment/Reference: Aircraft Electricity and Avionics
5th Ed Eismin Page 312. Jeppesen A&P Technician
Airframe Textbook Page 12-24

3) an ILS frequency
Comment/Reference: Jeppesen Aircraft Radio Systems
Page 69 shows this as a localiser frequency. Jeppesen
A&P Technician Airframe Textbook Page 12-24,
Automatic Flight Control 4th ed pg 181 says band 108.0
– 112.1 as Localiser frequencys

4) energised off
Comment/Reference: Aircraft Electrical Systems,
Pallett Page 142.

5) 121.5 MHz
Comment/Reference: AC65-15A pg 533 & JAR OPS 1.820 or
1.850 c. Jeppesen A&P Technician Airframe Textbook Page
12-25

6) 2500 ft.
Comment/Reference: Aircraft Electricity & Electronics.
Eismin Page 323 - 324

7) a VOR frequency
Comment/Reference: 112 - 118 MHz are VOR frequencies.
Jeppesen A&P Technician Airframe Textbook Page 12-14

8) Low Frequency
Comment/Reference: Decca navigation is low frequency.

9) Loran C
Comment/Reference: Loran C is a type of LF Navigation.
Jeppesen Avionic Fundamentals Page 153. Also Aircraft
Radio Systems by James Powell Page 101

10) 15° above the horizon with respect to the viewer


Comment/Reference: 'Elevation mask' is 15 degrees
www.gpscontrol.com/php/support/tutorial/accuracy.php
Questions 731 – 740

731. Restrictions to the use of hand held microphones apply


to

 private aircraft
 transport category aircraft only
 aerial work and transport category aircraft

732. The purpose of the clutch in an autothrottle servo is

 to allow the pilot to override


 to protect the servo motor in the event of
inadvertent runaway
 to limit the range of control movement

733. Track mode of an RA is operational

 from 1.0 to 100 feet


 from 0 to 2,500 feet
 above 10,000 feet

734. How many aerials are there in a TCAS system?

 1
 3
 2

735. Wavelength of X band radar is

 3 cm
 5 cm
 10 m
736. Precipitation static is caused by

 lightning strikes
 skin to air particle collisions
 HF radiation

737. HF aerials have weak points designed at

 the front end


 both ends
 the back end

738. What is the reply frequency of an aircraft


transponder?

 1000 MHz
 1030 MHz
 1090 MHz

739. CAT 2 RVR limit is

 1200 ft
 1000 ft
 10,000 ft

740. With autopilot engaged, which control surface is


inhibited?

 THS
 Ailerons
 Elevators
Answers for Questions 731 – 740

1) transport category aircraft only


Comment/Reference: ANO Schedule 4 Scale N

2) to allow the pilot to override


Comment/Reference: Pallett Automatic Flight Control
Pg 289

3) from 0 to 2,500 feet


Comment/Reference: The radio altimeter is operational
from 0 - 2500 ft Jeppesen Avionic Fundamentals Page
223

4) 2
Comment/Reference: Avionic Systems: Operation and
Maintenance page 160

5) 3 cm
Comment/Reference: Introduction to Avionics Dale
Cundy Page 82 &
www.everythingweather.com/weather-radar/bands.shtml
Xband Radars operate on a wavelength of 2.5-4 cm and
a frequency of 8-12 GHz

6) skin to air particle collisions


Comment/Reference: Aircraft Electricity and
Electronics - Eismin page 211& CAP 562 Leaflet 9-12
para 3 a)

7) the back end


Comment/Reference: CAIPs RL/2-2 para 2.2.4

8) 1090 MHz
Comment/Reference: Avionic Fundamentals page 211 &
Aircraft Electricity & Electronics Eisman pg 322

9) 1200 ft
Comment/Reference: CAT II ILS Runway Visual Range
(RVR) is 'not less than 1200ft'. Ref: Avionic
Fundamentals page 199 & Automatic Flight Control 4th
Ed Pallett pg 279 400m = 1312 ft

10) THS
Comment/Reference: A&P Airframe Technician Textbook
Pg 12-35
Questions 741 – 750

741. When flaps are lowered, the automatic trim system will

 angle of incidence remains the same


 decrease the angle of incidence of the THS
 increase the angle of incidence of the THS

742. In autopilot, the control column

 does not move


 moves in pitch and roll
 moves in pitch

743. A ˜hyperbolic’ system is

 VOR
 ILS
 LORAN C

744. When is ‘autothrottle’disengaged?

 On landing
 After thrust reverser has deployed beyond 90% so
that TO/GA can be selected in case of emergency
 On selection of thrust reverse

745. In aircraft with an autopilot and an auto trim, a


pitch command input will cause

 column to move but trim system not to move


 column to move and trim system to move
 column will not move and trim system will move
746. In regard to the aircraft transponder, what is the
pulse frequency?

 Amount of times reply signal is sent per second


 Number of pulses per signal
 Amount of times interrogating signal is sent per
second

747. CAT 2 RVR limit is

 800 m
 200 m
 400 m

748. How many programs can a FMC store?

 Two. Both active


 Two. One active and one standby
 One current

749. EPR and speed for autothrottle is activated at

 take off
 approach
 cruise

750. How does an IRS calculate velocity?

 Integration of accelerometers
 Differentiation of laser gyro
 Double integration of accelerometers
 Answers for Questions 741 – 750

1) decrease the angle of incidence of the THS


Comment/Reference:

2) moves in pitch and roll


Comment/Reference: A&P Airframe Technician Textbook
Pg 12-47 (Parallel system). This is assuming it is a
non-fly-by-wire aircraft.

3) LORAN C
Comment/Reference: Loran C is a type of Omega
Navigation. Jeppesen Avionic Fundamentals Page 153

4) On selection of thrust reverse


Comment/Reference: Pallett Automatic Flight Control
Page 286

5) column to move and trim system to move


Comment/Reference: A&P Technician Airframe Textbook
12-47 - This is assuming it is a non-fly-by-wire
aircraft.

6) Amount of times reply signal is sent per second


Comment/Reference:

7) 400 m
Comment/Reference: CAT II ILS Runway Visual Range
RVR) is 'not less than 1200ft'. 1200 ft = 400m Ref:
Avionic Fundamentals page 199 & Automatic Flight
Control 4th ed Pallett pg 279

8) Two. One active and one standby


Comment/Reference: Pallett Aircraft Instruments and
Integrated Systems Page 399 on. Boeing 757 chapter 34-
61-00 page 201.

9) take off
Comment/Reference: Pallett Automatic Flight Control
4th Edition Page 286

10) Integration of accelerometers


Comment/Reference: Pallett Automatic Flight Control
4th Edition Page 191
Questions 751 – 760

751. In an autopilot, what controls pitch mode?

 Localizer
 Glideslope
 VOR

752. Glideslope controls autopilot in

 roll
 yaw
 pitch

753. Static dischargers help eliminate radio interference


by dissipating static electricity into the atmosphere
at

 low current levels


 high voltage levels
 all voltage levels

754. What is B-RNAV?

 Indicates bearing and airspeed until next active


waypoint
 Indicates true airspeed
 Ability to store 6 waypoints

755. An Automatic Flight Control System receives inputs


from the following ground based transmitters

 RA, ADF, ILS


 DME, ILS, ADF
 VOR, ILS
756. What is the wavelength of C band radar?

 3 cm
 17 m
 7 cm

757. What is primary radar?

 Radar that sends out pulse and receives reflected


pulse
 Radar that gives height and position
 Land based

758. What is ILS marker beacon frequency?

 75 MHz
 50 MHz
 100 MHz

759. TCAS is selected

 automatically
 by a switch, by pilot on selector panel
 not available in cruise

760. The manual VOR input is for

 course deviation bar


 glideslope
 RMI
Answers for Questions 751 – 760

1) Glideslope
Comment/Reference: Pallett Automatic Flight Control 4th
Edition Page 187

2) pitch
Comment/Reference: Pallett Automatic Flight Control 4th
Edition Page 187

3) low current levels


Comment/Reference: Reference found in an older
version of A&P Technician Airframe Textbook Page 700
(Not in latest edition). Quote 'If the aircraft is
equipped with static dischargers, the static discharge
occurs at lower current and more frequently'.

4) Indicates bearing and airspeed until next active


waypoint
Comment/Reference:

5) VOR, ILS
Comment/Reference: An AFCS uses VOR and ILS (RA is
not ground based) Aircraft Electricity & Electronics
5th Ed Eisman pg 364

6) 7 cm
Comment/Reference: C Band is 4 - 8 cm. Most aircraft
systems use about 5.6cm. Ref Boeing and Jeppesen A&P
Technician Airframe Textbook Page 12-27 &
www.everythingweather.com/weather-radar/bands.shtml

7) Radar that sends out pulse and receives reflected


pulse
Comment/Reference: www.bbc.co.uk/dna/h2g2/A588684

8) 75 MHz
Comment/Reference: AC65-15A pg 527 & Automatic Flight
Control. Pallett, 4th Edition Page 183/4

9) by a switch, by pilot on selector panel


Comment/Reference:

10) course deviation bar


Comment/Reference: B737 MM Chapter 34-31-02
Questions 761 – 770

761. The mach trim is initiated by

 an electric motor
 the autopilot motor
 a PCU

762. What is the colour of the middle marker beacon?

 White
 Blue
 Amber

763. A GPS system is formed from

 receiver, processing unit, interactive console


 satellites, processing unit, display unit
 space, control, user

764. For aircraft with dual CMCs, when only one CMC is
available, it

 must be connected to the left side


 can be connected to either side
 must be connected to the right side

765. Laser gyros are

 aligned to the magnetic north


 aligned to the aircraft structure
 aligned to the true north
766. Laser gyros

 have rotational parts


 do not have rotational parts
 have movable parts

767. A radio coupled approach is

 glideslope first, followed by localiser


 localiser first, followed by glideslope
 in any order

768. Aileron signal is fed to the rudder channel

 for yaw damping compensation


 for turn command back-up
 for turn coordination

769. The rate of G/S warning in GPWS

 changes with radio altitude


 does not change
 changes with barometric altitude

770. What frequency are VOR and ILS?

 UHF
 VHF
 HF
Answers for Questions 761 – 770

1) the autopilot motor


Comment/Reference: Automatic Flight Control, Pallett,
Page 231

2) Amber
Comment/Reference: AC65-15A pg 527 & Jeppesen -
Avionics Fundamentals, Page 219 fig 14-2 & Aircraft
Electricity & Electronics 5th Ed Eisman pg 312

3) space, control, user


Comment/Reference: Aircraft Electricity & Electronics
5th Ed Eisman pg 319

4) must be connected to the left side


Comment/Reference: Aircraft Electricity & Electronics
5th Ed Eismin Chap 13 page 271

5) aligned to the aircraft structure


Comment/Reference: Jeppesen Avionics Fundamentals
Page 99

6) do not have rotational parts


Comment/Reference:
http://en.wikipedia.org/wiki/Ring_laser_gyroscope &
Jeppesen Avionics Fundamentals Page 99

7) localiser first, followed by glideslope


Comment/Reference: Automatic Flight Control Pallett
Page 184-185

8) for turn coordination


Comment/Reference: Automatic Flight Control. Pallett.
4th Ed Page 121/122

9) changes with radio altitude


Comment/Reference: B737-3/4/500 AMM 34-42-00 pg 18
Avionic Fundamentals page 263

10) VHF
Comment/Reference: AC65-15A pg 525 & Aircraft
Instruments and Integrated Systems Page 426
Questions 771 – 780

771. A radio frequency of 16 MHz would be used for

 marker beacons
 HF communications
 weather radar

772. An aircraft lands in autoland. After touch down and


thrust reversers are deployed. What happens to
autothrottle?

 Advances throttles
 Automatically switches off
 Stays armed for go around in an emergency

773. Pilot's instinctive cut-out buttons are positioned

 on right side of control wheel


 on side of control wheel furthest from throttles
 on left side of control wheel

774. ILS and VOR operate in which range

 VHF
 UHF
 HF

775. FMC secondary flight-plan is selected

 by calendar date monthly


 in the air by the pilot
 on the ground by the pilot
776. GPWS operating in mode 1 and 2 when close to ground
will give

 amber warning
 red caption and aural 'pull up, undercarriage,
flaps, throttle'
 red caption and aural 'whoop whoop pull up'

777. The components of an ILS are

 a localizer and a glide slope


 a localizer, a glide slope and the marker beacons
 a localizer and the marker beacons

778. In ILS, the glideslope provides

 vertical steering
 lateral steering
 distance checks

779. If the 90 Hz tone modulation in a localizer receiver


predominates, the deviation indicator will show

 fly right
 fly left
 the flag

780. Autopilot will operate above what altitude?

 750 ft.
 1000 ft.
 500 ft.
Answers for Questions 771 – 780

1) HF communications
Comment/Reference: Jepessen A & P Technician Textbook
p12-3 fig 12-2 shows HF COMM as 2-30 MHz

2) Automatically switches off


Comment/Reference: Automatic Flight Control Pallett
and Coyle Page 286

3) on side of control wheel furthest from throttles


Comment/Reference: JAR 25.1329 (d)

4) VHF
Comment/Reference: AC65-15A pg 525 & Jeppesen A&P
Technician Airframe Textbook 12-1

5) in the air by the pilot


Comment/Reference: FMC has a primary (active) and a
secondary (alternative) stored flight plan.

6) red caption and aural 'whoop whoop pull up'


Comment/Reference: B737-3/4/500 AMM 34-42-00

7) a localizer, a glide slope and the marker beacons


Comment/Reference: AC65-15A pg 525 & Aircraft Radio
Systems by James Powell page 69

8) vertical steering
Comment/Reference: Aircraft Radio Systems by James
Powell page 69 & Automatic Flight Control 4th ed
Pallett pg 181

9) fly right
Comment/Reference: Automatic Flight Control 4th Ed
Pallett pg 182 & Aircraft Radio Systems by James Powell
page 73

10) 500 ft.


Comment/Reference:
Questions 781 – 790

781. A radio coupled autopilot in pitch uses

 glideslope
 ADF
 VOR

782. EPR and thrust modes in autothrottle are the only modes
that can be selected in

 approach
 take off
 cruise

783. An inertial navigation unit uses pin programming for

 aircraft type
 magnetic orientation
 location

784. How can it be verified if FMC update is correct?

 BITE
 FMC via CDU
 Dataplate on the FMC

785. GPWS will show a fault if the following fails:

 Pressure altimeter
 Air speed indicator
 Radio altimeter
786. INS has mercury switches on

 outer gimbal
 inner gimbal
 all gimbals

787. If the runway picture in the EADI moves down during an


ILS approach, the aircraft must fly

 down
 nowhere - this is normal on an ILS approach
 up

788. What modulation is used for the beams of a localiser in


an ILS?

 90 Hz below the glide path, 150 Hz above the glide


path
 150 Hz right of runway centerline, 90 Hz left of
runway centreline
 150 Hz left of runway centerline, 90 Hz right of
runway centerline

789. In what frequency range does the automatic direction


finding (ADF) system operate?

 190 - 1759 KHz


 108.00 - 117.95 MHz
 1025 - 1150 KHz

790. How does an aircraft distinguish its own Distance


Measuring Equipment reply from those for other
aircraft?

 By changing at random the time delay between the


pulse pairs of the interrogation signal
 By using an alternate frequency
 By modulation of an audio tone
Answers for Questions 781 – 790

1) glideslope
Comment/Reference:

2) take off
Comment/Reference: Automatic Flight Control 4th Ed page
286

3) location
Comment/Reference: Pin programming for IRU`s and INU`s
are carried out on the aircraft rack mounted side. The
purpose being to tell the installed unit which position
it is serving within the aircraft i.e left, ctr or
right side! The unit is universal for all

4) FMC via CDU


Comment/Reference:

5) Radio altimeter
Comment/Reference: Of the three, GPWS has only
connection to the RA. It does use barometric vertical
speed however.

6) all gimbals
Comment/Reference: Aircraft Instruments and Integrated
Systems Pallett Page 116

7) down
Comment/Reference:

8) 150 Hz right of runway centerline, 90 Hz left of runway


centreline
Comment/Reference: Automatic Flight Control 4th Ed
Pallett pg 182 & Aircraft Radio Systems Powell Page 70

9) 190 - 1759 KHz


Comment/Reference: AC65-15A pg 529 & Aircraft
Electricity & Electronics 5th Ed Eisman pg 303 &
Jeppesen A&P Technician Airframe Textbook Page 12-13

10) By changing at random the time delay between the pulse


pairs of the interrogation signal
Comment/Reference: Aircraft Electricity & Electronics
Eisman 5th Ed pg 310 & Aircraft Radio Systems Powell
Page 106
Questions 791 – 800
791. From where is bearing information received for display
on the digital-distance radio-magnetic indicator
(DDRMI)?

 From ADF only


 From VOR and ADF systems
 From VOR only

792. Which frequency is used to achieve line of sight radio


communication?

 HF
 VHF
 VHF and UHF

793. Why is an aerial tuning unit used in a high frequency


(HF) communication system?

 To select the transmission/reception frequency in


the HF band
 To electrically lengthen or shorten the aerial for
optimum matching of impedance
 To extend or retract the aerial and so vary its
physical length

794. What would be the purpose of an input from an inertial


reference system being connected to a weather radar?

 To provide stabilisation for the radar antenna


 To place the weather radar target in azimuth and
distance for the display
 To ensure that there is no radar transmission with
aircraft on ground

795. Which systems provide envelope modulation information


for a Ground Proximity Warning System (GPWS)

 Rudder/ailerons
 Autothrottle
 Flaps/undercarriage
796. The Time Reference Scanning Beam (TRSB) corresponds to
ILS localiser and glidepath. How does it operate?

 It operates in the same manner as ILS but utilises


higher frequencies
 It operates in conjunction with DME at lower
frequencies than ILS
 Azimuth and elevation transmitters produce a narrow
beam which is rapidly scanned TO and FRO or UP and
DOWN

797. The FMS navigation database is updated

 daily
 after a B or C check has been completed
 every 28 days

798. In an IRS system you would expect to find

 three strap down accelerometers


 an azimuth gyro system
 ring laser gyros

799. In an INS system the accelerometer is a mass

 a mass suspended in free air


 suspended between two springs in a tube
 a remotely mounted mass on the airframe

800. In a modern aircraft the ACARS system is used primarily


for

 as part of the passenger telephone system


 communications between the aircraft and base
 reporting defects on the aircraft automatically
Answers for Questions 791 – 800

1) From VOR and ADF systems


Comment/Reference: Aircraft Instruments & Integrated
Systems Pallett Page 197

2) VHF and UHF


Comment/Reference:
www.tpub.com/content/avaition/14014/css/14014_166.htm

3) To electrically lengthen or shorten the aerial for


optimum matching of impedance
Comment/Reference:

4) To provide stabilisation for the radar antenna


Comment/Reference: www.artietheairplane.com/radar.htm

5) Flaps/undercarriage

6) Azimuth and elevation transmitters produce a narrow


beam which is rapidly scanned TO and FRO or UP and DOWN
Comment/Reference: Aircraft Electricity & Electronics
5th Ed Eisman pg 313 & Aircraft Radio Systems Powell
Page 224

7) every 28 days
Comment/Reference:

8) ring laser gyros


Comment/Reference: Assuming they mean a strapdown
system.

9) suspended between two springs in a tube


Comment/Reference: Aircraft Instruments & Integrated
Systems Pallett Page 256. (Note: the mass is suspended
in fluid, and is not remote)

10) communications between the aircraft and base


Comment/Reference:
Questions 801 – 810

801. What are the main areas of the autopilot?

 Error, correction, follow up, demand


 Error, correction, follow up, command
 Error, correction , demand, resolved

802. VHF frequency is

 108 - 136 MHz


 108 - 112 MHz
 108 - 118 MHz

803. The call system for the captain will have the audio
signal of a

 hi tone chime
 horn
 two tone chime

804. Channel 3 on a CVR records

 captain
 flightdeck
 first officer

805. DME works on the frequency of

 UHF
 HF
 VHF
806. A radial is referenced

 on a compass
 to a VOR
 from a beacon

807. ADF works by using

 both loop and sense aerial


 sense aerial
 loop aerial

808. How is the next database on the FMC activated?

 Automatically by due date


 Manually, on the ground
 Manually in the air

809. What GPWS mode gives a 'Whoop Whoop, Pull-up' command?

 Mode 2
 Mode 6
 Mode 3

810. A transponder that is compatible for use with a TCAS


system would be

 Mode S
 Mode A
 Mode C
Answers for Questions 801 – 810

1) Error, correction, follow up, command


Comment/Reference:

2) 108 - 136 MHz


Comment/Reference: AC65-15A pg 522 & Aircraft
Electricity & Electronics Eismin Page 295

3) hi tone chime
Comment/Reference: B737

4) flightdeck
Comment/Reference: CS 25.1457 para c) (3)

5) UHF
Comment/Reference: AC65-15A pg 528 & Aircraft
Electricity & Electronics Eismin Page 310 & 278

6) from a beacon
Comment/Reference: radial is FROM a VOR beacon

7) both loop and sense aerial


Comment/Reference: AC65-15A pg 529 & Aircraft
Electricity & Electronics 5th Ed Eisman pg 303

8) Manually, on the ground


Comment/Reference:

9) Mode 2
Comment/Reference: B737 AMM 34-42-00 pg 10 & Transport
Category Aircraft Systems Page 9.19 &
www.boeing-727.com/Data/systems/infogpws.html

10) Mode S
Comment/Reference: Introduction to TCAS version 7 US
Department of Transport FAA Page 17 (Target
Surveillance) & Aircraft Electricity & Electronics 5th
Ed Eisman pg 321
Questions 811 – 820

811. GPS

 uses 21 satellites equally spaced around 7 orbits


 uses 18 satellites equally spaced around 6 orbits
 uses 24 satellites equally spaced around 6 orbits

812. Random precession of the inner gimbal ring is detected


by placing mercury switches

 on both gimbal rings


 on outer gimbal ring
 on inner gimbal ring

813. The IRS laser gyro is a

 displacement gyro
 rate gyro
 displaced gyro

814. What manoeuvre does TCAS II advise the pilot to make?

 either RA or TA
 RA
 TA

815. What are the shapes of traffic shown on a TCAS display?

 White squares, red diamonds and amber circles


 White diamonds, red squares and amber circles
 White circles, red diamonds and amber squares
816. The laser ring gyro

 has a stabilized platform


 does not have gimbal and rotating parts
 does not have gimbal

817. 3 autopilot computers are considered

 Fail Operable
 Fail resistant
 Fail Passive

818. In autopilot with THS in motion, the

 Auto Trim is inhibited


 Mach trim is inhibited
 elevator is inhibited

819. How can a pilot over-ride the auto-throttle?

 It is not possible
 By deselecting auto-throttle first
 Manually through a clutch on the throttle levers

820. Where is the autothrottle disconnect switch?

 Within reach of the Captain


 Within reach of both pilots
 Within reach of the First Officer
Answers for Questions 811 – 820

1) uses 24 satellites equally spaced around 6 orbits


Comment/Reference:

2) on inner gimbal ring


Comment/Reference:

3) rate gyro
Comment/Reference: Rate integrated gyro. Aircraft
Electricity and Electronics 5th Ed. Eismin Page 373

4) RA
Comment/Reference: B737-2/4/500 AMM 34-45-00 pg 1 para
1 c) & Introduction to TCAS version 7 US Department of
Transport FAA Page 27 (Threat Detection)

5) White diamonds, red squares and amber circles


Comment/Reference: B737-3/4/500 AMM 34-45-00 pg 9 fig 3
& Introduction to TCAS version 7 US Department of
Transport FAA Page 27 (Threat Detection)

6) does not have gimbal and rotating parts


Comment/Reference: Aircraft Electricity and
Electronics 5th Ed. Eismin Page 373

7) Fail Operable

8) Auto Trim is inhibited

9) Manually through a clutch on the throttle levers


Comment/Reference:

10) Within reach of both pilots


Comment/Reference:
Questions 821 – 830

821. Decca navigation uses

 VHF
 LF
 HF

822. There are two FMS installed on the aircraft. If one FMS
fails during flight

 the whole FMS system is unserviceable until the


pilot switches over to standby
 it has no effect, because the second FMS was in the
stand-by mode, now it is active.
 the failed FMS has a blank screen

823. Which of the following has priority over TCAS warnings?

 Stall warning
 Resolution Advisories
 Gear position warning

824. The ILS marker beacon operates at a frequency of

 50 MHz
 100 MHz
 75 MHz

825. The manual input for the VOR course corrector is


related to

 the CDI offset bar


 the ILS system
 the RMI
826. A flat plate antenna is a

 a Doppler antenna
 parabolic antenna
 a series of slots and wave guides

827. To obtain an accurate GPS fix, the GPS receiver must be


in line of sight of

 3 satellites
 4 satellites
 6 satellites

828. Which of the following systems is inhibited when a test


is performed of the rad. alt. system?

 TCAS
 GPWS
 altitude alert

829. The GPS satellite system consists of

 21 satellites and 3 standby satellites


 20 satellites and 5 standby satellite
 24 satellites and 1 standby satellites

830. A Mode C transponder gives the following info:

 Altitude and interrogation


 Interrogation
 Altitude
Answers for Questions 821 -830

1) LF
Comment/Reference:
http://en.wikipedia.org/wiki/Decca_Navigator_System

2) the failed FMS has a blank screen


Comment/Reference: Aircraft Instruments and Integrated
Systems Pallett page 406

3) Stall warning

4) 75 MHz
Comment/Reference: AC65-15A pg 527 & Automatic Flight
Control 4th ed Pallet pg 183 & Aircraft Electricity and
Electronics Eismin page 311

5) the CDI offset bar

6) a series of slots and wave guides


Comment/Reference: Aircraft Electricity & Electronics
Eismin page 342

7) 4 satellites
Comment/Reference: Aircraft Electricity & Electronics
Eismin Pages 319. 4 satellites are required to provide
height information as well as position.

8) GPWS

9) 21 satellites and 3 standby satellites


Comment/Reference: Aircraft Electricity & Electronics
Eismin Pages 318 & 319

10) Altitude
Comment/Reference: Aircraft Electricity & Electronics
5th Ed Eisman pg 321 & Avionics Fundamentals Page 211
Questions 831 – 840

831. The autothrottle system at touchdown will

 go to idle
 apply reverse thrust
 go to idle and disconnect

832. When will the decision height aural warning sound?

 At decision height
 After decision height
 Before decision height

833. FMCS Pin Programming is allowed

 under CAA Rules


 for the database of aircraft landing altitudes
 to compensate for FMC position on the aircraft

834. Before the aeroplane is moved from the loading pier,


the pilot must

 insert the latitude and longitude of the pier into


the INS
 set the altitude to be fed into the INS
 insert the latitude and longitude of the first
waypoint into the INS

835. A 'strap-down' inertial navigation system has

 accelerometers on a stable platform and gyros fixed


to the airframe
 accelerometers fixed to the airframe and gyros on a
stable platform
 accelerometers and gyros fixed to the airframe
836. A force re-balance accelerometer in an IN system has
the acceleration force balanced by a

 linear force
 constant force
 non-linear force

837. An accelerometer in an IN system must be able to detect


accelerations down to

 10-6 g
 10-2 g
 10-3 g

838. A laser gyro output is

 directly proportional to frequency addition


 directly proportional to angular turning rate
 inversely proportional to angular turning rate

839. In an IN system, Coriolis effect is the result of

 the effect of the earth's rotation on a stable


platform
 gyro wander
 platform misalignment

840. The Inertial Navigation System computes distance from


acceleration by

 a single integration
 two successive integrations
 a differential followed by an integration
Answers for Questions 831 – 840

1) go to idle and disconnect


Comment/Reference: Pallett Automatic Flight Control
Page 286

2) At decision height

3) to compensate for FMC position on the aircraft

4) Insert the latitude and longitude of the pier into the


INS
Comment/Reference: Flight Instruments and Automatic
Flight Control Systems, David Harris Page 82

5) accelerometers and gyros fixed to the airframe


Comment/Reference: Flight Instruments and Automatic
Flight Control Systems, David Harris Page 90

6) linear force
Comment/Reference: Flight Instruments and Automatic
Flight Control Systems, David Harris Page 33

7) 10-6 g

8) directly proportional to angular turning rate


Comment/Reference: Aircraft Electricity & Electronics
5th Ed Eisman pg 374

9) the effect of the earth's rotation on a stable platform


Comment/Reference: Automatic Flight Control 4th Ed
Pallett pg 58 & Flight Instruments and Automatic Flight
Control, David Harris Page 85

10) two successive integrations


Comment/Reference: Flight Instruments and Automatic
Flight Control, David Harris Page 81
Questions 841 – 850

841. Coriolis effect is corrected for by

 adding a correction term to the accelerometer


outputs
 re-aligning the stable platform
 torquing the gyros

842. In an IN system, the output of the accelerometer is


linear because of a

 linear spring
 force balance system
 pendulous suspension

843. The three accelerometers on a stable platform are


mounted

 orthogonally
 120 degrees apart
 parallel to each other

844. In an IN system, the purpose of the stable platform is


to

 stop the gyros from toppling


 prevent unwanted acceleration affecting the
accelerometers
 provide attitude reference

845. The type of gyro generally used in an IN system is a

 rate gyro
 rate integrating gyro
 displacement gyro
846. Earth rate is approximately

 5 degrees per hour


 84 degrees per hour
 15 degrees per hour

847. In a gimbal system, the stable platform is the

 azimuth gimbal
 roll gimbal
 pitch gimbal

848. To prevent gimbal lock on the stable platform it is


normal to use

 four gimbals
 three gimbals
 a pitch gimbal

849. When the inertial platform is torqued to perform like a


Schuler pendulum

 the platform remains fixed with respect to the local


vertical
 the platform rotates with respect to the aircraft
 the platform oscillates with respect to the local
vertical

850. A Schuler pendulum has a period of oscillation of

 8.4 minutes
 84.4 seconds
 84.4 minutes
Answers for Questions 841 – 850

1) adding a correction term to the accelerometer outputs


Comment/Reference: Flight Instruments and Automatic
Flight Control, David Harris Page

2) force balance system


Comment/Reference: Flight Instruments and Automatic
Flight Control, David Harris Page 76

3) orthogonally

4) prevent unwanted acceleration affecting the


accelerometers
Comment/Reference: Flight Instruments and Automatic
Flight Control, David Harris Page 77

5) rate integrating gyro


Comment/Reference: Flight Instruments and Automatic
Flight Control, David Harris Page 77

6) 15 degrees per hour


Comment/Reference: Flight Instruments and Automatic
Flight Control, David Harris Page 37

7) azimuth gimbal
Comment/Reference: Flight Instruments and Automatic
Flight Control, David Harris Page 77/78

8) four gimbals
Comment/Reference:
http://en.wikipedia.org/wiki/Gimbal_lock

9) the platform oscillates with respect to the local


vertical
Comment/Reference: Flight Instruments and Automatic
Flight Control, David Harris Page 86

10) 84.4 minutes


Comment/Reference:
http://en.wikipedia.org/wiki/Schuler_tuning &
Flight Instruments and Automatic Flight Control, David
Harris Page 86
Questions 851 – 860

851. An IN system requires data from the

 air data computer


 satellites
 Doppler system

852. When in manual mode, the C.D.U. alert lamp of the IN


system will flash

 thirty seconds before a track change is required


 two minutes before the next waypoint
 when an error is detected

853. XTK (cross track) is the

 perpendicular distance from the desired track


 angle in degrees that aircraft track is left or
right of desired track
 actual track across the earth's surface

854. The output of an INS can be fed to

 vertical speed indicators


 altimeters
 attitude indicators

855. The three accelerometers on a strapdown platform are


mounted

 90º to each other


 parallel to each other
 120º apart
856. A basic I.N.S. platform has

 3 axis accelerometer
 3 accelerometers and 2 gyros (pitch and roll)
 2 accelerometers and 3 gyros

857. Using I.N.S. an aircraft flies

 great circle arc


 course directed by ground station
 rhumb line

858. What must be entered in to the I.N.S. before the


aircraft moves?

 Waypoints
 E.T.A.
 Present position

859. Selection of the INS Mode Selector Unit (MSU) to ATT


REF is made

 to feed information to the Captain and 1st Officers


ADI displays
 when navigation information is lost
 when attitude information is lost

860. For the INS, the Battery Unit provides

 both when airborne and on the ground


 standby power when airborne, switched by weight-off
switches in the undercarriage
 standby power only when on the ground, to maintain
the alignment phase
Answers for Questions 851 – 860

1) air data computer


Comment/Reference: B737-3/4/500 AMM 34-28-00 pg 1 para
1 D)

2) two minutes before the next waypoint


Comment/Reference: Flight Instruments and Automatic
Flight Control, David Harris Page 89

3) perpendicular distance from the desired track


Comment/Reference: Automatic Flight Control 4th ed
Pallett & Coyle pg 193 & Flight Instruments and
Automatic Flight Control, David Harris Page 88

4) attitude indicators
Comment/Reference: Flight Instruments and Automatic
Flight Control, David Harris Page 85

5) 90º to each other

6) 2 accelerometers and 3 gyros


Comment/Reference: Flight Instruments and Automatic
Flight Control, David Harris Page 78. 1 accelerometer
referenced north and the other to east, see AC65-15A pg
531

7) great circle arc


Comment/Reference: Flight Instruments and Automatic
Flight Control, David Harris Page 81

8) Present position

9) when navigation information is lost


Comment/Reference: Flight Instruments and Automatic
Flight Control, David Harris Page 85

10) both when airborne and on the ground


Comment/Reference: Flight Instruments and Automatic
Flight Control, David Harris Page 85
Questions 861 – 870

861. Control Display Unit (CDU) selection of TKE displays

 distance perpendicular from the selected track


 difference in degrees that the aircraft track is to
the right or left of the desired track
 difference in degrees from True North in a clockwise
direction from the desired track

862. The Earth Rate Compensation is computed from

 the earth's rotational rate (15°/hour) times the


sine of the longitude
 the earth's rotational rate (15°/hour) times the
cosine of the latitude
 the earth's rotational rate (15°/hour) times the
sine of the latitude

863. INS wind speed is calculated from

 the vectorial addition of IAS and TAS


 the vectorial addition of IAS and GS
 the vectorial addition of TAS and GS

864. Centripetal error compensation is achieved by

 an additional signal is added to the N/S


accelerometer to cancel the centripetal error
 allowing the platform to oscillate at a fixed rate
 the platform is torqued to align the N/S
accelerometer along its insensitive axis

865. Transport Rate compensation is achieved by

 an additional signal added to the N/S accelerometer


output depending on heading
 allowing the platform to oscillate at a fixed rate
 the platform being torqued by a computed torquing
signal
866. If the battery fails on the ground (INS System)

 an amber light appears on the MSU and a horn sounds


 a red warning light appears on the MSU and a horn
sounds
 a red light appears on the CDU and a horn sounds

867. An IRS alignment

 takes 10 minutes and present position can be entered


any time during the alignment
 takes 10 minutes and the previous flight shut down
present position is used for the alignment
 takes 10 minutes and present position must be
entered before alignment

868. For an IRS system to pass the 'Alignment System


Performance Test' the

 latitude entered must be within given limits of the


latitude computed by IRU
 entered present latitude and longitude must agree
with the latitude and longitude at the last power
down
 the No. 1 and No.2 must both have the same latitude
and longitude present position entered

869. A laser gyro dither mechanism ensures that

 that the two contra-rotating beams each operate at


different frequencies
 the contra-rotating beams are synchronised together
 optical 'backscatter' does not cause the contra-
rotating beams to lock together

870. The localiser deviation signal for the flight director


comes from the

 flight director computer


 VHF nav system
 VHF comm system
Answers for Questions 861 – 870

1) difference in degrees that the aircraft track is to the


right or left of the desired track
Comment/Reference: Automatic Flight Control 4th ed
Pallett & Coyle pg 193 & Flight Instruments and
Automatic Flight Control, David Harris Page 88

2) the earth's rotational rate (15°/hour) times the sine


of the latitude

3) the vectorial addition of TAS and GS

4) an additional signal is added to the N/S accelerometer


to cancel the centripetal error

5) the platform being torqued by a computed torquing


signal

6) a red warning light appears on the MSU and a horn


sounds
Comment/Reference:
Flight Instruments and Automatic Flight Control
Systems, David Harris Page 85

7) takes 10 minutes and present position can be entered


any time during the alignment

8) latitude entered must be within given limits of the


latitude computed by IRU

9) optical 'backscatter' does not cause the contra-


rotating beams to lock together
Comment/Reference: Flight Instruments and Automatic
Flight Control Systems, David Harris Page 47

10) VHF nav system


Questions 871 – 880

871. The heading error signal used in the heading select


mode

 is the difference between the desired course and the


actual course
 comes direct from the compass system
 is the difference between the desired heading and
the actual heading

872. The crab angle of the aircraft during VOR or LOC modes
is displayed by the

 difference between the course arrow and aircraft


heading
 difference between the selected heading and aircraft
heading
 selected course counter

873. VOR left-right deviation signals come from the

 DME system
 VLF nav system
 VHF nav set

874. Above the glideslope, the ILS glideslope signal


modulation is

 150 Hz
 90 Hz
 90 KHZ

875. The localiser modulation signal to the left of the


localiser centre line, as seen from the localiser
transmitter, is

 90 KHz
 90 Hz
 150 Hz
876. The correct sense demand generated for a selected
heading 180°, when the aircraft heading is 150° is

 turn left
 turn right
 straight ahead

877. When an aircraft is flying along the extended centre


line of the runway it is in the

 equi-signal sector
 150 Hz modulation sector
 90 Hz modulation sector

878. The localiser system offers approach guidance to the


runway in terms of

 the horizontal plane


 distance to touch down
 the vertical plane

879. The glideslope transmitter is located

 adjacent to the touch-down point of the runway


 at the end opposite to the approach end of the
runway
 at the approach end of the runway

880. The glideslope and localiser frequencies

 have to be selected separately


 are fixed and common to all runways therefore
frequency selection is not necessary
 are paired and one frequency selector suffices for
both
Answers for Questions 871 – 880

1) is the difference between the desired heading and the


actual heading

2) difference between the course arrow and aircraft


heading

3) VHF nav set

4) 90 Hz
Comment/Reference: Automatic Flight Control, Pallett,
pg 182, Fig.6.6 & Aircraft Radio Systems Powell, pg 72,
top of L/H column

5) 150 Hz
Comment/Reference: Automatic Flight Control 4th ed
Pallett & Coyle pg 182. It says 'as seen from the
localiser transmitter'

6) turn right

7) equi-signal sector

8) the horizontal plane

9) adjacent to the touch-down point of the runway


Comment/Reference: Automatic Flight Control, Pallett
pg 182 & Aircraft Radio Systems, Powell, page 72, Fig.
5.5

10) are paired and one frequency selector suffices for both
Comment/Reference: Aircraft Radio Systems, Powell, page
71 and Avionics Fundamentals, Jeppesen, page 206.
Each airport has a separate ILS freq.
Questions 881 - 890

881. The glideslope system offers approach guidance to


runways in terms of

 the vertical plane


 distance to touchdown
 the horizontal plane

882. The glideslope equipment operates in the

 UHF band
 VHF band
 HF band

883. The localiser equipment operates in the

 HF band
 VHF band
 UHF band

884. The aircraft equipment determines the bearing of a


ground station by comparing

 the phase of two 30 Hz modulations


 the phase of one 30 Hz modulation with that of a
9960 Hz modulation
 the amplitude of two 30 Hz modulations

885. The number of different radials provided by a ground


station is

 360
 180 per quadrant, i.e. 720 in 360°
 infinite
886. Which of the following frequencies is allocated to VOR?

 127.2 MHz
 114.3 MHz
 103.9 MHz

887. Aerial masts may be damaged by

 killfrost anti-icing fluid


 water
 Skydrol hydraulic fluids

888. Most radio aerial masts are

 bonded
 not bonded
 insulated from the fuselage

889. When an aircraft is heading due north (magnetic)


towards a VOR station the reference and variable
signals will be

 in phase
 180° out of phase
 270° out of phase

890. The middle marker modulation is keyed with

 alternate dots and dashes


 dashes
 dots
Answers for Questions 881 – 890

1) the vertical plane


Comment/Reference: Automatic Flight Control Pallett
Page 182

2) UHF band
Comment/Reference: AC65-15A pg 526 & Automatic Flight
Control Pallett Page 181

3) VHF band
Comment/Reference: Automatic Flight Control Pallett
Page 181

4) the phase of two 30 Hz modulations


Comment/Reference: Aircraft Radio Systems, Powell Pages
59 and 60 and Radio Aids, R.B.Underdown and David
Cockburn Page 72

5) infinite
Comment/Reference: Automatic Flight Control 4th Ed
Pallett pg 188

6) 114.3 MHz
Comment/Reference: Aircraft Radio Systems, Powell
Pages 58. All frequencies between 112.00 and 117.95 MHz
(High Power VORs) and all odd frequencies between
108.00 and 111.95 MHz (Terminal VORs)

7) Skydrol hydraulic fluids

8) bonded

9) 180° out of phase


Comment/Reference: Automatic Flight Control 4th Ed
Pallett pg 188. Aircraft is due South of the station

10) alternate dots and dashes


Comment/Reference: AC65-15A pg 527
Questions 891 – 900

891. The modulation of the outer marker is

 3000 Hz
 1300 Hz
 400 Hz

892. The approximate distance of the middle marker from the


runway threshold is

 3500 ft.
 3 miles
 7 miles

893. Marker information is usually provided to the pilot

 aurally
 visually
 both visually and aurally

894. An over station sensor (OSS) detects

 radio deviation signals proportional to distance


from a localiser transmitter
 the rapid rate of the VOR signal over the cone of
confusion
 radio deviation signals proportional to distance
from a VOR transmitter

895. Incompatible Flight Director modes are

 altitude hold and ILS


 VOR and glidepath
 VRU and compass
896. The VOR system comprises

 reference phase signal


 variable and reference phase signals
 variable phase signal

897. The most sensitive system between ILS and VOR is

 VOR
 they both have the same sensitivity
 ILS

898. If an aircraft is flying on a heading of 000 away from


a VOR station, the TO/FROM indicator would show

 to
 from
 no indication

899. How does the flight director computer differentiate


between VOR and ILS frequencies?

 Trigger pulse from ground station


 Frequency discriminator in receiver
 Discriminator on control panel

900. The glideslope transmitter operates on

 the VHF band


 the UHF band
 frequencies of 108 to 118 MHZ
Answers for 891 – 900

1) 400 Hz
Comment/Reference: AC65-15A pg 527 & Aircraft
Electricity & Electronics Eismin Pages 311-312

2) 3500 ft.
Comment/Reference: AC65-15A pg 527

3) both visually and aurally


Comment/Reference: AC65-15A pg 527 & Aircraft
Electricity & Electronics Eismin Page 311

4) the rapid rate of the VOR signal over the cone of


confusion

5) altitude hold and ILS

6) variable and reference phase signals


Comment/Reference: Aircraft Electricity & Electronics
5th Ed Eismin Pages 306 and 307

7) ILS

8) from

9) Frequency discriminator in receiver

10) the UHF band


Comment/Reference: AC65-15A pg 526 & Aircraft
Electricity & Electronics 5th Ed Eismin Page 308
Questions 901 – 910

901. The pilots instinctive autopilot disengage button is on


the

 side of the controls away from the throttles


 right of the control column
 left of the control column

902. If the autopilot automatically disconnects in the


autoland mode, the audible warning

 is switched off by the instinctive cut-out button


 switches off after a time interval
 can only be switched off by re-engaging the
autopilot

903. A category 3B aircraft using fail operational automatic


landing equipment will have

 a decision height of 50ft


 a decision height depending on RVR
 no decision height

904. For an aircraft to be certified for automatic landing,


an autothrottle system is

 dependent on the operation of the aircraft at slow


speeds
 a matter of choice for the operator
 mandatory

905. With autothrottle engaged, the application of reverse


thrust will

 drive the throttles to the minimum thrust position


 disconnect the autothrottle
 drive the throttles to the reverse thrust position
906. During ATC transponder operation, side lobe suppression
acts to

 supply altitude readout


 mute coms transmission during transponder operation
 mute the DME operation during transmit phase

907. During operation of a twin HF radio system transceiver

 #1 HF system operation is inhibited during #2


operation
 #1 HF system can transmit but not receive
 both systems can be operated simultaneously

908. L band DME transmits on a frequency of

 2210 MHz
 4133 MHz
 1090 MHz

909. Function of ADF & VOR and DME in navigation system with
reference to aircraft and beacon is;

 None of above
 the first provides distance between aircraft and
beacon and latter provides bearing line from
aircraft to beacon
 the first provides bearing line from aircraft to
beacon and latter provides distance between aircraft
and beacon

910. Which two frequencies are paired?

 Localizer and DME


 DME and Glideslope
 Glideslope and localizer
Answers for Questions 900 – 910

1) side of the controls away from the throttles


Comment/Reference: Flight Instruments and Automatic
Flight Control Systems, David Harris Page 133

2) is switched off by the instinctive cut-out button


Comment/Reference: JAR AWO Para 153

3) no decision height
Comment/Reference: Automatic Flight Control Pallett
Page 279

4) dependent on the operation of the aircraft at slow


speeds
Comment/Reference: CS-AWO Para 306 b

5) disconnect the autothrottle

6) mute the DME operation during transmit phase

7) #1 HF system operation is inhibited during #2 operation

8) 1090 MHz

9) the first provides bearing line from aircraft to beacon


and latter provides distance between aircraft and
beacon
Comment/Reference: AC65-15A pg 528/529

10) Glideslope and localizer


Comment/Reference: Jeppesen A & P Technician Airframe
Textbook page 12-24
Questions 911 – 920

911. Localizer beam width is the angle where the two edges
of beam are apart at the runway threshold by

 7 ft
 700 ft
 70 ft

912. What happens if frequency decreases without altering


the physical length of aerial?

 The aerial becomes inductively capacitive


 The aerial becomes capacitively reactive
 The aerial becomes inductively reactive

913. What happens if frequency increases without altering


the physical length of aerial?

 The aerial becomes inductively reactive


 The aerial becomes inductively capacitive
 The aerial becomes capacitively reactive

914. Aerials provide optimum output at one particular


frequency, when its load is purely

 resistive
 capacitive
 inductive

915. The torque pre-set in an autopilot system is

 to allow it to be overridden at a certain force


 to give control surface feel
 to stop the motor overheating
916. Most aerials are

 bonded
 made from non-conductive material
 not bonded

917. The Middle Marker beacon is what colour?

 Amber
 White
 Blue

918. Laser gyros have

 no rotating parts
 a spinning rotor
 moving parts

919. Laser gyros are aligned to

 true north
 magnetic north
 aircraft structure

920. Triplex autopilot is

 fail soft
 fail passive
 fail operational
Answers for Questions 911 – 920

1) 700 ft
Comment/Reference: Avionics Fundamentals Page 200

2) The aerial becomes capacitively reactive

3) The aerial becomes inductively reactive

4) resistive

5) to allow it to be overridden at a certain force

6) bonded

7) Amber
Comment/Reference: AC65-15A pg 527

8) moving parts
Comment/Reference: A laser gyro has moving parts
(dither motor vibrates).

9) aircraft structure

10) fail operational


Comment/Reference: Automatic Flight Control 4th Ed
Pallett Page 282
Questions 921 – 930

921. Flight management control system (FMCS) utilises

 ILS, VOR, ADF


 VOR, ADF, DME
 ILS, DME, ADC,

922. ACARS is

 a way of reporting defects to maintenance base in


flight
 a navigation system
 a satellite communication system

923. Where is an ATC transponder mode 'A' selected ON?

 The airspeed indicator


 The ATC control panel
 The altimeter

924. How many data bases are required in an FMS system?

 2
 1
 3 (one is used for redundancy)

925. What channel of the autopilot does the glideslope


control?

 Pitch
 Yaw
 Roll
926. What is secondary radar?

 Reflected radar bounce from an aircraft


 A backup radar on an airfield
 Signal returned from a transponder

927. In an autothrottle system, when is EPR or thrust mode


used?

 Approach
 Cruise
 Take-off

928. What is Mode 1 & 2 of GPWS used for?

 Excessive descent rate, unsafe terrain clearance


 Excessive descent rate, excessive terrain closure
rate
 Excessive terrain closure rate, altitude loss after
take-off

929. For radio communication over a distance of over 250


miles we use

 HF
 VHF
 VLF

930. Autopilot, when on approach to landing, how many axis


are used?

 4
 2
 3
Answers for Questions 921 – 930

1) ILS, DME, ADC,


Comment/Reference: FMCS does not normally utilise ADF

2) a way of reporting defects to maintenance base in


flight
Comment/Reference: Aircraft Electricity & Electronics
5th Ed Eismin Page 250

3) The ATC control panel


Comment:/Reference: AC65-15A pg 530

4) 2

5) Pitch

6) Signal returned from a transponder

7) Take-off
Comment/Reference: E.H.J.Pallet 3rd edition page 284
3rd paragraph. OF WHAT BOOK?

8) Excessive descent rate, excessive terrain closure rate


Comment/Reference: B737-3/4/500 AMM 34-42-00 pg 1 para
1.A. (1) & (2) & Flight Instruments & AFCS, 6th Ed
David Harris. P158/159

9) HF
Comment/Reference: AC65-15A pg 524

10) 3
Comment/Reference: AC65-15A pg 511
Questions 931 – 940

931. A lead-acid battery is considered to be fully charged


when the

 SG and voltage remain constant for specified period


 cells begin to gas freely
 SG reaches 1.180

932. A diode across a contactor

 reduces contact arcing


 reduces contact bounce
 enables it to be connected to either an AC or DC
circuit

933. The preferred method of battery charging a Ni-Cad


battery is constant

 voltage
 current
 power

934. A battery in situ on an aircraft

 may charged at an excessive rate if initial battery


voltage is low
 charges at a fairly constant rate because generator
voltage remains constant
 may charge slowly if initial battery voltage is low

935. To parallel an alternating current generator with one


or more other generators, phase A must be

 in-phase with other generators and CBA only


 in-phase with other generators and ABC
 90 degrees out-of-phase and ABC
936. On an unparalleled twin generator alternating current
system, the phase of the second generator to be brought
on line
 must be in-phase prior to paralleling
 is unimportant
 is important if the first generator fails

937. On the direct current short shunt generator circuit


shown, output voltage at the load is 200V. What is the
voltage across the shunt winding?

 206V
 200V
 212V

938. In a constant volume hydraulic EDP, the short shaft


which engages with the engine gearbox would have

 2 female splined portions separated by a waisted


section
 an inner female splined portion and an outer male
splined portion
 2 male spline portions seperated by a waisted
section designed to shear should the pump seize

939 On a Direct Current circuit, Current Limiters

 limit generator field excitation


 limit current to field circuit
 prevent overheating by limiting current to load

940. On a direct current circuit, current limiters

 allow excessive current to the load circuit


 prevent excessive current to the load circuit
 prevent excessive current to the field circuit
Answers for Questions 931 – 940

1) SG and voltage remain constant for specified period


Comment/Reference: EEL/1-1 4.4.8 and Pallett Aircraft
Electrical Systems 3rd Ed Page 23

2) reduces contact arcing


Comment/Reference:
www.homepages.which.net/~paul.hills/SolenoidsBody.html
Para 7.1

3) current
Comment/Reference: CAP 562 Leaflet 9-7 Appx 1 para
4.5.1 & CAIPs EEL/1-5 Para 4.1

4) may charged at an excessive rate if initial battery


voltage is low
Comment/Reference: Read AC65-9A pgs 307-315

5) in-phase with other generators and ABC


Comment/Reference: AC generators are paralleled for
phase, voltage, frequency, load and phase rotation.

6) is unimportant
Comment/Reference: The Question says it is unparalleled

7) 200V
Comment/Reference: The 10 ohm shunt, 0.06 ohm and the
0.04 ohm are the field windings. The generator output
is split between the 0.06 ohm and the 100 ohm load,
but only by 0.06/100 x 200V = 0.12V (across the 0.06
ohm). So the voltage across the shunt winding is
closer to 200V than the other two answers

8) 2 male spline portions seperated by a wasted section


designed to shear should the pump seize
Comment/Reference: AC65-15A pg 320

9) prevent overheating by limiting current to load


Comment/Reference: Aircraft Electrical Systems 3rd Ed
Pallett pg 112

10) prevent excessive current to the load circuit


Comment/Reference: Aircraft Electrical Systems 3rd Ed
Pallett pg 112
Questions 941 – 950

941. When a current transformer is disconnected from its


load

 a resistor should be placed across the terminals


 the terminals should be shorted together
 the terminals should be left disconnected

942. On a direct current twin generator system, the


paralleling relay contacts close when the oncoming
generators voltage is

 above bus voltage


 below bus voltage
 the same as bus voltage

943. Where is a commutator found?

 A DC generator
 A DC alternator
 An AC generator

944. IDG oil is used for

 internal hydraulic functions only


 cooling, lubrication and internal hydraulic
functions
 cooling and lubrication only

945. A silver oxide cell has a voltage of

 1.8 V
 1.2 V
 1.55 V
946. Lithium cells

 are very efficient


 are not rechargeable
 have an unlimited re-charge cycle capacity

947. On a split bus electrical system, the bus tie breakers


are

 always open in flight


 always closed in flight
 only closed in an emergency

948. Parallel electrical system bus tie breakers are

 always closed in flight


 closed in flight and open on ground
 closed on ground and open in flight

949. When a current transformer is disconnected, what should


be done?

 Resistor placed across terminals


 Left open circuit
 Terminals shorted

950. What does the differential sensing coil sense?

 Volts
 Current
 Power
Answers for Questions 941 – 950

1) the terminals should be shorted together


Comment/Reference: Aircraft Electrical Systems 3rd Ed
Pallett pg 61 (to of rh side)

2) above bus voltage

3) DC generator
Comment/Reference: AC65-9A pg 392

4) cooling, lubrication and internal hydraulic functions

5) 1.55 V

6) are not rechargeable


Comment/reference:
http://en.wikipedia.org/wiki/lithium_batery

7) only closed in an emergency

8) always closed in flight

9) Terminals shorted
Comment/Reference: Pallett Aircraft Electrical Systems
3rd Edition Page 61

10) Current
Questions 951 -960

951. Where does the GCU gets its power?

 RAT
 PMG
 Battery

952. Galley loads are wired in

 parallel so load shedding will lower current


consumption
 series
 either series or parallel depending on the design

953. In a merz-price protection system of a single phase,


if, when a fault occurs, only one relay contactor
opens,

 only the relay on the side which has the fault will
energise
 then it is probable that one of the coils has failed
open circuit
 then it is probable that one of the relays has
failed short circuit

954. A fuse-type current limiter

 limits current flow to a load


 opens a circuit after a time/current condition has
been exceeded
 can be used as a radio suppressor

955. When a load is shed from a busbar, the

 current consumption from the bar decreases


 busbar voltage decreases
 busbar voltage increases
956. In unparalleled AC generation systems, the phase
rotation of one generator in relation to the others

 is unimportant
 must be BCA
 must be synchronised prior to paralleling

957. In a multi-generator system, the generator which takes


more than its reactive power share is protected by

 over-current and under-current protection circuits


 over-excitation and under-excitation protection
circuits
 over-voltage and under-voltage protection circuits

958. In a pulsed width modulating type voltage regulator,


the generator output voltage is increased by

 decreasing the mark to space ratio


 increasing the voltage sensed by the voltage
regulator
 increasing the mark to space ratio

959. Sharing of electrical loads by paralleled DC generators


is controlled by

 equalising circuits which control the speed of the


generators
 equalising circuits which control the field
excitation of the generators
 automatic load shedding

960. Excessive commutator sparking can be caused by

 rotating field diode failure


 brushes positioned on MNA
 weak spring tension
Answers for Questions 951 – 960
1) PMG
Comment/Reference: Jeppesen A&P Technician Airframe
Textbook Page 7-24. PMG is main supply for GCU, battery
is back up power for GCU
2) parallel so load shedding will lower current
consumption
Comment/Reference: All loads on an aircraft are wired
in parallel
3) only the relay on the side which has the fault will
energise
Comment/Reference: Aircraft Electrical Systems 3rd Ed
Pallett pg 122&www.transformerworld.co.uk/diffprot.htm

4) opens a circuit after a time/current condition has been


exceeded
Comment/Reference: Pallett Aircraft Electrical Systems
3rd Ed Pg 112, or, Aircraft Electricity & Electronics,
Eismin Page 99. However, another type of current
limiter holds the current but does and open the
circuit. Aircraft Electricity & Electronics Eismin Page
202.
5) current consumption from the bar decreases
Comment/Reference: Aircraft Electricity & Electronics,
Eismin, Page 241. Load shedding reduces current
consumption

6) is unimportant
Comment/Reference: Phase relationship is unimportant in
an unparalleled system

7) over-voltage and under-voltage protection circuits


Comment/Reference: Pallett Aircraft Electrical Systems
3rd Ed Page 49

8) increasing the mark to space ratio


Comment/Reference:

9) equalising circuits which control the field excitation


of the generators
Comment/Reference: Changing the speed would also change
the frequency and thus the phase relationship.
Generators are always controlled by adjusting the field
strength
10 weak spring tension
Comment/Reference: Weak spring tension of the brushes
can cause excessive sparking
Questions 961 – 970

961. Before checking the SG of a battery recently removed


from an aircraft

 allow to stabilise for one hour


 take the temperature of the electrolyte
 carry out a capacity test

962. On an AC external power plug, the interlock circuit is


controlled by pins

 E and F
 A and B
 C and D

963. Over-voltage protection circuits are activated

 dependent on the magnitude of the overvoltage


 after a fixed time delay
 before the over voltage limit is exceeded

964. A diode connected across a coil will

 prevent back EMF and allow faster switching


 prevent the coil being fitted the wrong way
 suppress arcing when relay is opening and closing

965. Which fault does not always trip the GCR?

 Over-frequency and under-frequency


 Under-frequency and under-volts
 Over-frequency and over-volts
966. Differential protection in a generating system

 uses the volts coil to trip the GCR


 detects voltage difference between source and load
 detects current difference between source and load

967. Open phase sensing on a current transformer

 is detected using any phase


 is detected on generator neutral circuit
 is detected using all phases

968. If the battery is switched off in flight, the

 captain's instruments will be powered from the


standby bus
 battery is disconnected from bus
 generator voltage falls to zero

969. To flash a generator field the

 generator rotates but is not on line


 generator must be stationary
 generator is on line and producing a voltage less
than the flashing voltage

970. Voltage is controlled by

 the swashplate
 the constant speed drive
 the voltage regulator
Answers for Questions 961 -970

1) take the temperature of the electrolyte


Comment/Reference: CAIPs EEL/1-1

2) E and F
Comment/Reference: Pallett Aircraft Electrical Systems
3rd Ed Page 72

3 dependent on the magnitude of the overvoltage


Comment/Reference: Pallett Aircraft Electrical Systems
3rd Ed Page 119

4) prevent back EMF and allow faster switching


Comment/Reference:
http://homepages.which.net/~paul.hills/Solenoids/Soleno
idsBody.html Para 7.1

5) Over-frequency and under-frequency


Comment/Reference: under and over voltage will both
trip the relay. On some aircraft, under frequency will
NOT trip the relay, so c is the 'least incorrect'

6) detects current difference between source and load


Comment/Reference: Pallett Aircraft Electrical Systems
3rd Ed Page 121

7) is detected on generator neutral circuit


Comment/Reference: Pallett Aircraft Electrical Systems
3rd Ed Page 121, & B737 Manual

8) battery is disconnected from bus


Comment/Reference: Pallett Aircraft Electrical Systems
3rd Ed Page 4

9) generator must be stationary


Comment/Reference: Pallett Aircraft Electrical Systems
3rd Ed Page 4

10) the voltage regulator


Comment/Reference: The voltage is controlled by the
voltage regulator.
Questions 971 – 980

971. Distilled water should be added to a ni-cad battery

 when fully charged


 only in the battery workshop
 at any time

972. Overflowing electrolyte in a ni-cad battery indicates

 low electrolyte temperature


 excessive charging current
 not enough charging current

973. During a normal engine shutdown, generator tripping is


initiated by

 underfrequency
 overcurrent
 undervoltage

974. What would you use to neutralise ni-cad battery


spillage?

 Distilled water
 Boric acid
 Bicarbonate of soda

975. Before operating generators in parallel, their voltages


should be matched to prevent

 overloading the generator drive and shearing of


splines
 large circulating currents developing
 voltage spikes of sufficient magnitude to trigger
GCU overvoltage circuits
976. Consumer loads are connected to a busbar

 in series so when loads are shed voltage is reduced


 in series so when loads are shed current is reduced
 in parallel so when loads are shed current is
reduced

977. A current transformer connected on the neutral star


point of a generator is for

 negative sequence protection


 differential current protection
 overload protection

978. Differential protection as applied to AC generation


systems compares

 ambient pressure to cabin pressure


 generator voltage to busbar voltage
 generator current to busbar current

979. In an AC generation system, the frequency and phase


rotation

 must be synchronised prior to paralleling


 must be out of phase prior to paralleling
 is of no consequence after paralleling

980. Diodes are placed across a contactor to

 ensure smooth contactor operation


 prevent contact bounce
 speed up operation by reducing back EMF
Answers for Questions 971 – 980

1) only in the battery workshop


Comment/Reference: CAIP EEL/1-3 para 4

2) excessive charging current


Comment/Reference: Jeppesen A&P Airframe Technician
Page 7-31, 7-32

3) undervoltage
Comment/Reference: Pallett Aircraft Electrical Systems
3rd Ed Page 120

4) Boric acid
Comment/Reference: CAP 562 Leaflet 9-2 para 5.2.1 &
5.4.2 & AC65-9A pg 314

5) overloading the generator drive and shearing of splines


Comment/Reference: Pallett Aircraft Electrical Systems
3rd Ed Page 52.

6) in parallel so when loads are shed current is reduced


Comment/Reference: All loads are connected in parallel
with each other

7 differential current protection


Comment/Reference: Pallett Aircraft Electrical Systems
3rd Ed Page 121

8) generator current to busbar current


Comment/Reference: Pallett Aircraft Electrical Systems
3rd Ed Page 121

9) must be synchronised prior to paralleling


Comment/Reference: Pallett Aircraft Electrical Systems
3rd Ed Page 48

10) speed up operation by reducing back EMF


Comment/Reference:
http://homepages.which.net/~paul.hills/SolenoidsBody.ht
ml
Questions 981 – 990
981. Placing the battery master switch to off during flight
will

 disconnect all power to the main busbar


 shut down the APU
 isolate the battery from the main busbar

982. Whilst operating paralleled AC constant frequency


generators, it is important to maintain generator
outputs at the same voltage

 to prevent a circulating current of considerable


magnitude developing between two or more generators
 to prevent a circulating current developing between
two or more TRUs
 to prevent CSD shock loading

983. A voltage regulator installed in series with a


generator field

 uses a voltage coil in series with generator output


 uses a current coil in parallel with generator
output
 uses a voltage coil in parallel with generator
output

984. CSD warning lights on the flight deck normally indicate

 high oil pressure, low oil temperature


 low oil pressure, low oil temperature
 low oil pressure, high oil temperature

985. The output frequency of a hydraulically driven standby


generator is controlled by

 an IDG
 a CSD
 an hydraulically actuated swash plate
986. An AC generator used with a CSD

 needs a voltage controller to maintain voltage with


increasing load
 does not need a voltage controller because voltage
is kept constant
 does not need a voltage controller because current
is kept constant

987. Power to a GCU is supplied

 by the generator output only


 initially by a 'field tickling' supply and then PMG
output
 by the battery bus or rectified generator output

988. Trip signals for a GCR are

 overfrequency and undervoltage


 underfrequency and overcurrent
 differential protection and undervoltage

989. Abnormal CSD operation is monitored on the flight deck


by observing indications of

 High oil temperature / low oil quantity


 high oil temperature / low oil pressure
 low oil quantity / low oil temperature

990. Wires in hot temperature areas should be bound with

 Teflon
 Nomex
 PVC
Answers for Questions 981 – 990

1) isolate the battery from the main busbar


Comment/Reference: The Battery master switch simply
connects the battery to the main busbar, allowing it to
be charged from the generator(s).

2) to prevent a circulating current of considerable


magnitude developing between two or more generators
Comment/Reference:

3) uses a voltage coil in parallel with generator output


Comment/Reference: Aircraft Electricity and
Electronics, Eismin page 199-200

4) low oil pressure, high oil temperature


Comment/Reference: Jeppesen A&P Airframe Technician
Textbook, page 7-50

5) an hydraulically actuated swash plate

6) needs a voltage controller to maintain voltage with


increasing load

7 initially by a 'field tickling' supply and then PMG


output
Comment/Reference: Jeppesen A&P Technician Airframe
Textbook Page 7-24. PMG is main supply for GCU, battery
is back up power for GCU

8) differential protection and undervoltage


Comment/Reference: B737-3/4/500 AMM 24-21-00 para 5.b.
pg 7

9) high oil temperature / low oil pressure


Comment/Reference: Jeppesen A&P Airframe Technician
Textbook , page 7-50 & B737-3/4/500 AMM 21-11-00 pg 1
para 1.B.

10) Teflon
Comment/Reference: This is probably referring to the
Teflon tape referred to in Aircraft Electricity
Electronics by Eismin 5th Ed Page 69. Also AC43
Para.11.96 z and Appendix 1 (Teflon is Dupont trade
name for PTFE)
Questions 991 – 1000

991. A heat gun should be operated at

 100°C below the specified


 the specified temperature
 100°C above the specified

992. High voltages on a generator in a parallel system


should be prevented because

 it will cause high circulating current in the bus /


other generators
 it will trip the GCR
 it will damage the drive shaft

993. Over-voltage condition trips the generator. The time


taken to trip the GCU depends on

 closing of fuel and hydraulic valves


 time delay circuit
 amplitude of voltage

994. Speed of an HMG is controlled by a

 swashplate
 CSD unit
 ID unit

995. A permanent magnet should have

 low retentivity, high coercivity


 high retentivity, low coercivity
 low retentivity, low coercivity
996. What will happen if the aircraft battery becomes
overheated?

 The hot battery bus will be disconnected from the


battery
 The battery charger will switch to trickle charge
mode
 The power supply to the battery charger will be
interrupted

997. When the temperature increases in a lead acid battery,


the SG will

 decrease
 increase
 remain the same

998. On testing a generator it is found to require


adjusting, where would you adjust it?

 In the cockpit on the flight engineers panel


 On the GCU
 By means of a remote trimmer

999. Control of hydraulically powered emergency electrical


generator frequency is

 by angle of swash plate


 by CSU
 by IDG

1000.When a ni-cad battery is fully charged

 the electrolyte level is higher level than normal


 the electrolyte level stays the same
 the electrolyte level is lower than normal
Answers for Questions 991 – 1000

1) the specified temperature


Comment/Reference: A heat gun should be operated at the
specified temperature, or 100°C (approx) above the
heatshrink temperature.

2) it will trip the GCR

3) amplitude of voltage
Comment/Reference: Inverse time delay - c could also be
considered correct. Aircraft Electricity & Electronics,
Eismin, page 203

4) swashplate
Comment/Reference: Speed of an HMG is controlled by a
swashplate

5) high retentivity, low coercivity


Comment/Reference: A permanent magnet should have a low
coercivity (so it is easy to magnetise) and high
retentivity (so it is difficult to demagnetise)

6) The power supply to the battery charger will be


interrupted
Comment/Reference: B737 AMM 24-31-00 para 4.D. 2) on pg
3. & Pallett Aircraft Electrical Systems 3rd Ed Figure
1.32 for charger disconnect system & pg 29

7) decrease
Comment/Reference: Since volume increases, but mass
remains constant, with temperature, density decreases.

8) On the GCU

9) by angle of swash plate


Comment/Reference: Control of an HPG is via a
swashplate

10) the electrolyte level is higher level than normal


Comment/Reference: AC65-9A pg 314 & Jeppesen A&P
Technician Airframe Textbook 7-32
Questions 1001 – 1010

1001.Electrical load on aircraft is controlled by

 a GCU
 an IRS
 a BPCU

1002.A 4:1 step-up transformer receives 120VAC in its


primary and has 1600 Ohms resistance in its secondary.
What is the current drawn from the source?

 3 A
 1.2 A
 0.3 A

1003.After engine start using a Ni-Cad battery, the pilot


observes on the battery charger indicator, an initial
high current draw followed by a rapid decrease in
current

 no cause for concern


 there is a problem with the charging circuit and he
must shut-down the engines
 the battery is faulty

1004.Topping up a Ni-Cad battery in situ

 is only allowed in the shop


 is permitted
 is not allowed

1005.In a lead-acid battery after numerous checks, one cell


has low reading.

 It is defective
 It must be replaced
 It needs topping up
1006.A short range passenger aircraft has complete
electrical failure. What time duration would the
battery be expected to power essential equipment?

 30 minutes
 60 minutes
 Forever

1007.An external power plug has two short pins. These are
used for

 the interlock circuit


 ground handling bus
 a guide for correct alignment of the plug in the
socket

1008.The purpose of a synchronising bus bar is

 to monitor total current load


 to enable two propeller systems to be synchronised
 to allow monitoring and trimming from the flight
deck

1009.To confirm the state of charge of a ni-cad battery

 subject the battery to load and check the voltage


 subject the battery to load, check voltage and check
 the SG of each cell
 measure the discharge time

1010.Circulating currents are associated with

 AC generators in parallel
 AC and DC generators in parallel
 DC generators in parallel
Answers for Questions 1001 – 1010

1) a BPCU
Comment/Reference: B767 MM 42-41-00 quote: 'The BPCU
shares status information with the GCU's. The BPCU
controls all electrical buses and controls load
shedding.'

2) 1.2 A
Comment/Reference: Find current in secondary I = V/R =
480/1600 = 3/10A. Next find current in Primary = 4 x
3/10 = 12/10 = 1.2A

3) no cause for concern


Comment/Reference: This is normal

4) is not allowed
Comment/Reference: CAIPs EEL/1-3 Para 6.4.

5) It must be replaced

6) 30 minutes
Comment/Reference: CAP 747 GR no 4 para 2.3

7) the interlock circuit


Comment/Reference: B737-3/4/500 AMM 24-41-00 para 1.B.
(2) & Aircraft Instruments and Integrated Systems,
Pallett Page 290

8) to allow monitoring and trimming from the flight deck


Comment/Reference: Aircraft Electrical Systems Pallett
Page 51 and 52

9) measure the discharge time


Comment/Reference: CAIPs EEL/1-3 4.5.6 and 4.7

10) AC generators in parallel


Questions 1011 – 1020

1011.Generator brush bedding

 should not be done on the aircraft


 should only be done if 50% of the width and 80% of
the length is already being done
 can be done on the aircraft

1012.What is the amperage at point 'X' to the nearest amp?

 102 Amps
 101 Amps
 100 Amps

1013.Pitot tubes are heated by

 DC
 AC or DC
 AC

1014.Paralleling is used for

 DC electrical generators only


 AC and DC electrical generators
 AC electrical generators only

1015.Battery charging procedures can be found in ATA

 Chapter 24 Section 21
 Chapter 24 Section 31
 Chapter 31 Section 21
1016.In a carbon pile regulator, the resistive element is

 in series with the field and changes resistance with


changing length
 in series with the field and changes resistance with
surface area contact
 in parallel with the field and changes resistance
with changing length

1017.Maximum battery temperature on charging before


protection circuit starts is

 144 degrees C
 144 degrees F
 71 degrees F

1018.If an aircraft has no battery charger, the battery is


charged by

 constant voltage
 constant current until a predetermined limit when it
automatically switches to constant voltage
 constant current

1019.The output of a carbon pile regulator with no


variation of loading is

 fluctuating
 pulse width modulating
 stationary

1020.In a transistor voltage regulator, the voltage output


is controlled by

 transformers and transistors


 diodes and transformers
 zeners and transistors
Answers for Questions 1010 – 1020

1) should not be done on the aircraft


Comment/Reference: CAIPs EEL/1-4 4.3.6 (Does not say
that it cannot be done on the aircraft - but would be
difficult)

2) 101 Amps
Comment/Reference: 100 volts across far right load,
therefore 100 volts also across 100ohm load
(parallel circuit theory)
I = V/R = 100/100 = 1A through 100 ohm load.
(Ohms Law)
Total current at 'X' is 100 + 1 = 101 Amps.
(Kirkchoff's Law)

3) AC or DC
Comment/Reference: Aircraft Instruments and Integrated
Systems, Pallett 3rd Ed Page 32.

4) AC and DC electrical generators


Comment/Reference: Aircraft Electrical Systems 3rd Ed
Pallet pg 16 for DC & 47 for AC

5) Chapter 24 Section 31

6) in series with the field and changes resistance with


surface area contact
Comment/Reference: Aircraft Electrical Systems 3rd Ed
Pallet pg 12 & CAIPs EEL/1-2 3.1

7) 144°F
Comment/Reference: B737-3/4/500 AMM 24-31-00 para 4.E.
(4) on pg 5

8) constant voltage

9) stationary
Comment/Reference: Aircraft Electrical Systems 3rd Ed
Pallett pg 12 & CAIPs EEL/1-2 3.1.1

10) zeners and transistors


Comment/Reference: Aircraft Electrical Systems 3rd Ed
Pallett pg 14 & CAIPs EEL/1-2 3.3.1
Questions 1021 – 1030

1021.A paralleled relay for a DC system is energised and


connected by

 voltage coil
 voltage and current coil
 current coil

1022.Increasing the real load primarily

 decreases output voltage


 decreases frequency
 increases output voltage and increases frequency

1023.Inductive reactive load causes

 no effect on torque but produces extra heat


dissipated
 increase in torque only
 increase in torque and increase in heat dissipated

1024.The power factor of an AC generator identifies the


proportion of

 reactive power from the generator that does work


 real power from the generator that does work
 apparent power from the generator that does work

1025.Differential protection in an AC system protects


against

 short circuits
 line-line, line-line-line, line-earth faults
 a reverse current flowing from the battery
1026.When resetting the CSD on the ground, the engine
should be

 stationary
 rotating at idle
 rotating at Nsync

1027.One of the main purposes of a CSD is to

 prevent engine overload


 maintain constant load on the generator
 enable generators to be paralleled

1028.In an undervolt condition in an AC generator system,


the most likely consequence is

 activation of the time delay circuit


 deactivation of the field regulatory TRs
 energise the bus tie relay

1029.If voltage and frequency of the generator drop to zero


in flight, it would be an indication that the

 phase sequence detection circuit has operated


 CSD driveshaft had sheared
 bus tie interlock is inoperative

1030.Assuming all systems are operating normally, as


aircraft electrical load increases, generator output
voltage will

 decrease and amperage output increases


 increases and amperage output increases
 remain constant and amperage output increases
Answers for Questions 1021 – 1030

1) voltage and current coil


Comment/Reference: Pallett Aircraft Electrical Systems
Page 16, 17, 18

2) decreases frequency

3) no effect on torque but produces extra heat dissipated

4) real power from the generator that does work

5) short circuits
Comment/Reference: Aircraft Electrical Systems 3rd Ed
Pallett Page 121

6) stationary
Comment/Reference: B737-3/4/500 AMM 24-11-00 Para 8.B
on page 10

7) enable generators to be paralleled


Comment/Reference: Pallett Aircraft Electrical Systems
3rd Ed Page 37

8) activation of the time delay circuit

9) CSD driveshaft had sheared

10) remain constant and amperage output increases


Questions 1030 – 1040

1031.When installing multiple batteries on aircraft, they


are connected in

 series
 parallel
 either parallel or series and switched between as an
option

1032.What is the small pin on the DC ground power


connector?

 Ground or earth
 Positive to battery relay
 Positive to external power relay

1033.How is RAT driven?

 Airstream
 Compressor
 Fan

1034.How is an aircraft battery rated?

 Ampere hours
 Watts
 Joules

1035.A reduction in field strength of a DC shunt motor


driving a constant load will result in

 increased torque and reduced speed


 reduced torque and increased speed
 increased torque and speed
1036.What are the 2 short prongs on an AC plug for?

 To help plug location/fit


 Using AC current to pull on interlock circuit
 To prevent arcing during plug connection /
disconnection

1037.A vibration contact voltage regulator consists of a


shunt winding and a

 current regulation coil in series


 voltage regulating coil in parallel
 voltage regulating coil in series

1038.What is the purpose of diode in a power GCR?

 To speed up the operation


 To allow flow in the event of failure of the relay
 To prevent back-flow of current in the EMF coil

1039.Eddy currents are

 caused by resistance of the coil


 caused by heating effect of the coil
 circulating currents

1040.The small pins in the power connector plug are

 negative
 neutral
 positive
Answers for Questions 1031 – 1040

1) either parallel or series and switched between as an


option
Comment/Reference: Aircraft Electrical Systems. Pallett
Page 23/26/27

2) Positive to external power relay


Comment/Reference: Aircraft Electrical Systems 3rd Ed
Pallett Page 70

3) Airstream
Comment/Reference: Ram Air Turbine

4) Ampere hours
Comment/Reference: Aircrfat Electrical Systems 3rd Ed
Pallett Page 21

5) increased torque and speed


Comment/Reference: Aircraft Electrical Systems 3rd Ed
Pallett Page 138 (reduces Back EMF)

6) To prevent arcing during plug connection /


disconnection
Comment/Reference: The short prongs are for DC only.
Aircraft Electrical System, Pallett pg 72

7) current regulation coil in series


Comment/Reference: Aircraft Electrical Systems. Pallett
Page 11

8) To prevent back-flow of current in the EMF coil


Comment/Reference: The diode is to prevent the high
current flow in the coil due to induction as the relay
switches off

9) circulating currents
Comment/Reference:

10) positive
Comment/Reference: Aircraft Electrical Systems 3rd Ed
Pallett Page 70
Questions 1041 – 1050

1041.When an overvolt is present on an AC generator system,


the tripping circuit

 trips at a set level


 trips after a time delay
 trips after a time delay inversely proportional to
overvolt amplitude

1042.In an AC distribution system, the purpose of the


synchronising bus-bar is to

 supply essential services


 provide a means of monitoring the load
 provide interconnection between the generator load
busbars

1043.Vibrating contact voltage regulator sensing coil is

 in series with the field windings


 in parallel with the field windings
 across the generator output

1044.A current transformer connected across the neutral of


a 3 phase generator is for

 differential protection
 phase sequence monitoring
 load monitoring

1045.Zener diodes are for

 voltage regulation
 voltage stabilisation
 rectification
1046.AC generator output indications are measured in

 KW
 KVA & KW
 KW & KVAR

1047.Load shedding allows

 more current to the busbar


 more voltage to the busbar
 less current to the busbar

1048.How may the output of a transistorised voltage


regulator be described?

 Slightly variable DC
 Variable AC
 Pulse width modulated DC

1049.In the event of complete AC generator failure in an AC


split bus-bar generation system, how is power
maintained to the essential AC bus-bar?

 By power fed from a hot battery bus-bar only


 By power fed from a battery supplied static inverter
 By power fed from static inverter only

1050.Which components within a generator control unit


connects the output of the field power supply to the
generator exciter?

 Generator line contactor


 Generator control relay
 Generator power pilot relay
Answers for Questions 1041 – 1050
1) trips after a time delay inversely proportional to
overvolt amplitude
Comment/Reference: Aircraft Electrical Systems Pallett
Page 119/120
2) provide interconnection between the generator load
busbars
Comment/Reference: Aircraft Electrical Systems Pallett
Page 81
3) in parallel with the field windings
Comment/Reference: EEL/1-2 Page 5, Aircraft Electrical
Systems Page 11.
4) differential protection
Comment/Reference: Aircraft Electrical Systems Pallett
Page 121
5) voltage regulation
Comment/Reference: Although Zener diodes can be used
(back-to-back) to protect equipment from voltage spikes
(voltage stabilisation) they are more commonly
associated with voltage regulation.

6) KVA & KW
Comment/Reference: An AC generator is rated in KVA and
KW (or KVA and power factor, from which KW can be
calculated). However, flightdeck indications are KVAR
and KW.

7) less current to the busbar


Comment/Reference: Load shedding is carried out to
reduce current (voltage is constant).

8) Pulse width modulated DC


Comment/Reference: A voltage regulator has two outputs,
one to the generator field (pulse width modulated on a
transistorised regulator) and one to the busbar
(constant voltage but variable current depending on
load). The answer here depends upon which output they
are referring.

9) By power fed from a battery supplied static inverter


Comment/Reference:
Aircraft Electrical Systems Pallett Page 78 Fig. 5.2

10) Generator control relay


Comment/Reference: B737-3/4/500 AMM 24-21-00 para 5.A.
pg 5. The GCR is the only one which is (sometimes)
'within' the GCU.
Questions 1051 – 1060

1051.In what units are static inverters rated?

 kW
 kVAR
 kVA

1052.In a generator system, a stability winding is used

 in series with the field to prevent oscillations


 to prevent voltage overshoot
 to control output current

1053.On engine shutdown, what prevents GCR being tripped?

 Under-current
 Under-volt
 Under-frequency

1054.How is real and reactive load sharing achieved in an


AC parallel generator system?

 Real load by control of the constant speed drives,


reactive load by adjustment of generator fields
 Real load by control of generator fields, Reactive
loads by adjustment of the constant speed drives
 Real and reactive loads by control of the constant
speed drives only. Output voltage is controlled by
adjustment of generator fields

1055.Other than when there is a fault in an aircraft


battery or its charging system, when is the charger
switched off automatically?

 When there is a failure of one alternator in a


split-bus bar system
 When the battery is used for excessively high
discharge currents such as in starting
 When the alternators are fully loaded
1056.The trimmer resister in a carbon pile volts regulator
is

 in parallel with the generator volts coil


 in series with the generator field
 in series with the generator volts coil

1057.When a field relay trips the generator off-line, it


can be reset

 on the ground only


 after the fault has been cleared
 by cycling the generator switch

1058.In a paralleled AC generator system both A phases must


be in synchronization and

 CBA must equal CBA


 CBA must equal ABC
 phase rotation does not matter

1059.When loads are shed from a busbar automatically

 bus bar current rises


 bus bar voltage rises
 bus bar current decreases

1060.What controls output from the generator?

 BPCU
 ELCU
 GCU
Answers for Questions 1051 – 1060

1) kVA
Comment/Reference: VA or KVA, just like an AC generator

2) to prevent voltage overshoot


Comment/Reference: EEL/1-2 3.4.5 (d)

3) Under-frequency
Comment/Reference: Aircraft Electrical Systems Pallett
page 120

4) Real load by control of the constant speed drives,


reactive load by adjustment of generator fields
Comment/Reference: Aircraft Electrical Systems Pallett
page 48 and 49

5) When the battery is used for excessively high discharge


currents such as in starting

6) in series with the generator volts coil


Comment/Reference: EEL/1-2 Fig 1

7) after the fault has been cleared


Comment/Reference: Boeing opps manual states that you
are allowed to attempt to reset the field after
checking all parameters are now normal, as it may have
been a transient fault B747 classic 10.20.02

8) CBA must equal CBA


Comment/Reference:

9) bus bar current decreases

10) GCU
Comment/Reference: Aircraft Electricity & Electronics
5th Ed, Eismin, has full descriptions of all 3 units.
B737-3/4/500 AMM 24-21-00 para 6. pg 8
Questions 1060 – 1070

1061.Specific Gravity in a lead acid battery should give an


indication of

 electrolyte level
 electrolyte temperature
 charge

1062.During normal engine shut-down, the generator

 control relay is tripped only


 breaker is tripped only
 breaker and control relay are both tripped

1063.The generator warning light will come on when

 voltage is below battery voltage


 voltage is too high
 voltage is above battery voltage

1064.When replacing a current transformer

 it can be fitted any way round


 it can only be fitted one way round
 voltage/current selection must be made prior to
fitting

1065.The economy coil on a relay

 reduces current required to hold closed


 reduces current required to close
 makes it cheaper to make
1066.What is the function of the generator control relay?

 Control output voltage


 Bring the generator on-line
 Connect the generator to the busbar

1067.The current consumed by a DC starter motor will

 remain constant
 decrease as the engine speed increases
 increase as the engine speed increases

1068.A neutralising agent for sulphuric acid is

 citric acid
 borax powder
 sodium bicarbonate

1069.In a split bus power distribution system

 power is split between 115V AC and 28V DC


 generators are paralleled on connection to the tie
bus
 each generator supplies its own bus and distribution
system in normal operation

1070.Flashing a generator field

 increases resistance
 restores magnetism
 decreases resistance
Answers for Questions 1060 – 1070

1) charge

2) breaker is tripped only


Comment/Reference: B737-3/4/500 AMM 24-21-00 para
11.H.pg 21

3) voltage is below battery voltage

4) it can only be fitted one way round


Comment/Reference: Aircraft Electrical Systems Pallett
3rd Ed page 61

5) reduces current required to hold closed


Comment/Reference:

6) Bring the generator on-line


Comment/Reference: B737-3/4/500 AMM 24-21-00 para 5.A.
on pg 5

7) decrease as the engine speed increases


Comment/Reference: Decreases due to the back EMF as
engine speed increases

8) sodium bicarbonate
Comment/Reference: Jeppesen A & P Technician Airframe
Textbook p 7-30 refers

9) each generator supplies its own bus and distribution


system in normal operation
Comment/Reference:

10) restores magnetism


Comment/Reference: Aircraft Electricity & Electronics
5th Ed Eisman pg 206
Questions 1071 – 1080

1071.When checking SG of electrolyte of lead acid battery,


what should be taken into account?

 Electrolyte temperature
 Battery charge
 Ambient temperature

1072.How do you check condition of Ni-Cad battery in situ?

 By voltmeter connected to the battery terminal


 By voltmeter in the cockpit when battery is off load
 By voltmeter in the cockpit, when battery is on load

1073.The reactive load circuit in a multi-engined AC


generator system

 modifies generator field excitation voltage


 modifies generator field excitation current
 modifies generator drive speed

1074.A vibrating type voltage regulator uses a volts coil

 in parallel with the generator output


 in series with the generator output
 in series with the current coil

1075.The purpose of the economy contacts in a relay is to

 prevent contact pitting and burning


 reduce the current required to hold the contacts
closed
 close the auxiliary contacts before the main
contacts
1076.A vibrating contact voltage regulator has

 a volts coil in parallel with the points


 a resister in parallel with the points
 a resistor in series with the points

1077.Brush bedding-in on a starter generator can be carried


out

 during flight
 when 80 percent of the brush area has been
previously bedded in, on the bench
 only off the aircraft

1078.What would be the cause of a low electrolyte level in


a ni-cad battery?

 Evaporation
 Over charging current
 Undercharging current

1079.An AC generator connected to a CSD requires

 a separate voltage regulator


 no voltage regulator as the CSD will control the
 voltage
 no voltage regulator as the CSD will control
generator frequency

1080.If electrolyte from a lead-acid battery is spilled in


the battery compartment, which procedure should be
followed?

 Apply sodium bicarbonate solution to the affected


are followed by a water rinse
 Rinse the affected area thoroughly with clean water
 Apply boric acid solution to the affected area
followed by a water rinse
Answers for Questions 1071 – 1080

1) Electrolyte temperature
Comment/Reference:
EEL/1-1 4.3.4

2) By voltmeter in the cockpit, when battery is on load


Comment/Reference:
Aircraft Electrical Systems, Pallett Page 23

3) modifies generator field excitation current


Comment/Reference: Aircraft Electrical Systems, Pallett
Page 49

4) in parallel with the generator output


Comment/Reference: Aircraft Electrical Systems 3rd Ed
Pallett pg 13 Fig 1.14 & EEL/1-2 3.2.1

5) reduce the current required to hold the contacts closed

6) a resister in parallel with the points


Comment/Reference: Aircraft Electrical Systems 3rd Ed
Pallett pg 13 Fig 1.14 & EEL/1-2 Figure 3

7) only off the aircraft

8) Over charging current


Comment/Reference: CAIPs EEL/1.3 Page 3 Para 3.4.1

9) a separate voltage regulator

10) Apply sodium bicarbonate solution to the affected are


followed by a water rinse
Comment/Reference: CAP 562 leaflet 9-2 para 5.2.1 &
5.4.1 & AC65-9A pg 311 btm rh side.
Questions 1081 – 1090
1081.Which statement regarding the hydrometer reading of a
lead-acid storage battery electrolyte is true?

 The hydrometer reading does not require a


temperature correction if the electrolyte
temperature is 60°F
 A specific gravity correction should be subtracted
from the hydrometer reading if the electrolyte
temperature is above 20°C
 The hydrometer reading will give a true indication
of the capacity of the battery regardless of the
electrolyte temperature

1082.An increase in the speed of a DC generator will cause


the voltage to rise with the following resultant action
in the voltage regulator:-

 The volts coil to increase its magnetic field thus


decreasing the field excitation
 The volts coil to exert more pull thus increasing
the current in the generator field
 The volts coil to exert less pull thus increasing
the resistance of the carbon pile

1083.The equalizing coils on voltage regulators will

 have current passing through them from the higher


loaded generator to the lower
 be connected to each other when the generators are
paralleled
 have current passing through them only when the
generators are not paralleled

1084.A fully charged lead-acid battery will not freeze


until extremely low temperatures are reached because

 the acid is in the plates, thereby increasing the


specific gravity of the solution
 most of the acid is in the solution
 increased internal resistance generates sufficient
heat to prevent freezing
1085.What determines the amount of current which will flow
through a battery while it is being charged by a
constant voltage source?

 The total plate area of the battery


 The ampere-hour capacity of the battery
 The state-of-charge of the battery

1086.The method used to rapidly charge a nickel-cadmium


battery utilizes

 constant current and constant voltage


 constant voltage and varying current
 constant current and varying voltage

1087.The presence of small amounts of potassium carbonate


deposits on the top of nickel-cadmium battery cells
that have been in service for time is an indication of

 normal operation
 excessive plate sulphation
 excessive gassing

1088.The servicing and charging of nickel-cadmium and lead-


acid batteries together in the same service area is
likely to result in

 contamination of both types of batteries


 increased explosion and/or fire hazard
 normal battery service life

1089.If the current drawn from No.1 generator of a twin


generator DC system is less than that drawn from No.2
generator, the current in the No.2 generator equalizing
coil will flow

 in the same direction as the current flow in the


voltage coil, reducing the output of No.2 generator
 in the same direction as the current flow in the
voltage coil, increasing the output of No.2
generator
 in the opposite direction to the current flow in the
voltage coil, reducing the output of No.2 generator
1090.On a C.S. driven generator, operation of the
Disconnect Solenoid will disconnect the generator drive
and

 it will automatically reset itself when the fault is


rectified
 it can only be reset when all loads are switched off
 resetting can only be carried out on the ground
Answers for Questions 1081 – 1090

1) The hydrometer reading does not require a temperature


correction if the electrolyte temperature is 60°F
Comment/Reference: AC65-9A pg 311 & CAIPS EEL/1-1 4.3.4
(American Text such as Jeppesen A & P Technician
Airframe Textbook Page 7-27 specify 80°F as the
standard, but CAIPs specifies 60°F as standard)

2) The volts coil to increase its magnetic field thus


decreasing the field excitation
Comment/Reference: EEL/1-2 3.1.1 & AC65-9A pg 398

3) be connected to each other when the generators are


paralleled
Comment/Reference: Aircraft Electricity & Electronics
Eismin 5th Ed page 200 & Aircraft Electrical Systems
Pallett Pages 15 and 16

4) most of the acid is in the solution


Comment/Reference: CAIPs EEL/1-1 4.6.2

5) The state-of-charge of the battery


Comment/Reference:

6) constant voltage and varying current


Comment/Reference: Aircraft Electricity & Electronics
Eismin Page 52. CAIPs EEL/1-3 4.5.1

7) excessive gassing
Comment/Reference: Aircraft Electrical Systems Page 23
and Aircraft Electricity & Electronics Eismin Page 50 &
AC65-9A pg 314 para 3 & CAIPs EEL/1-3 4.2 (h)

8) contamination of both types of batteries


Comment/Reference: CAIPs EEL/2-1 Page 1 Para 2.11

9) in the same direction as the current flow in the


voltage coil, reducing the output of No.2 generator
Comment/Reference: Aircraft Electrical Systems 3rd Ed
Pallett pg 18

10) resetting can only be carried out on the ground


Comment/Reference: Jeppesen A & P Technician Airframe
Textbook p7-25 & B737-3/4/500 AMM 24-11-00 para 8.B. pg
10
Questions 1090 – 1100

1091.In a constant frequency AC system, real load sharing


is achieved by regulating the

 generator speed
 generator drive torque
 voltage regulator

1092.A constant speed drive unit has a warning light


indication

 for when it is disconnected from the generator


 for low oil pressure
 for low speed

1093.The electrolyte of a nickel-cadmium battery is the


lowest when the battery is

 under load condition


 being charged
 in a discharged condition

1094.Nickel-cadmium batteries which are stored for a long


period of time will show a low fluid level because the

 fluid evaporates through vents


 electrolyte becomes absorbed in the plates
 fluid level was not periodically replenished

1095.The purpose of 'field flashing' is to

 change the polarity of residual magnetism in a field


winding
 test the insulation resistance of a field winding
with a 1000V supply
 measure the residual magnetism in a field winding 24
hours after the field supply is removed
1096.The purpose of an inverter is to convert

 AC to a higher frequency AC
 AC to DC
 DC to AC

1097.How can the state-of-charge of a nickel-cadmium


battery be determined?

 By measuring the specific gravity of the electrolyte


 By measuring discharge
 By the level of the electrolyte

1098.What may result if water is added to a nickel-cadmium


battery when it is not fully charged?

 Excessive spewing will occur during the charging


cycle
 No adverse results since water may be added anytime
 The cell temperature will run too low for proper
output

1099.In nickel-cadmium batteries, a rise in cell


temperature

 causes a decrease in internal resistance


 increases cell voltage
 causes an increase in internal resistance

1100.When a charging current is applied to a nickel-cadmium


battery, the cells emit gas only

 when the electrolyte level is low


 toward the end of the charging cycle
 if they are defective
Answers for Questions 1091 – 1100

1) generator drive speed


Comment/Reference: Aircraft Electrical Systems,
Pallett 3rd Ed Page 48-49

2) for low oil pressure

3) in a discharged condition
Comment/Reference: AC65-9A pg 314 & EEL/1-3 3.4.2

4) electrolyte becomes absorbed in the plates


Comment/Reference: Jeppesen A&P Technician Airframe
Textbook page 7-32 and 7-33.

5) change the polarity of residual magnetism in a field


winding
Comment/Reference: Aircraft Electrical Systems, Pallett
3rd Ed Page 4 & Aircraft Electricity & Electronics 5th
Ed Eisman pg 206

6) DC to AC
Comment/Reference: AC65-9A pg 435 & Aircraft Electrical
Systems 3rd Ed Pallett pg 66

7) By measuring discharge
Comment/Reference: CAIPs EEL/1-3 4.5.6 &
HTTP://web.telia.com/~u85920178/begin/batt-00.htm

8) Excessive spewing will occur during the charging cycle


Comment/Reference: CAIPs EEL/1-3 4.3

9) causes a decrease in internal resistance


Comment/Reference: 0CAIPs EEL/1-3 4.5.8 (a)

10) toward the end of the charging cycle


Comment/Reference: CAIPs EEL/1-3 4.5.5
Questions 1101 – 1110

1101.The equalising coil in the carbon pile regulator

 is connected in series with the carbon pile


 becomes inoperative when the undervolt relay
energises
 modifies the effect of the voltage sensing coil

1102.In a constant frequency AC system, reactive load


sharing is achieved by regulating the

 generator drive torque


 generator speed
 voltage regulator

1103.Under-frequency in an AC supply would cause

 overspeeding of AC motors
 overvoltage of capacitive devices
 overheating of inductive devices

1104.The electrolyte in a lead-acid battery contains

 sulphuric acid
 nitric acid
 hydrochloric acid

1105.In the procedure to be followed after spillage of


battery acid, neutralizing may be carried out

 by washing with distilled water


 with a dilute solution of sodium bicarbonate
 by applying a coating of petroleum jelly
1106.Frequency (Hz) is the number of cycles per

 revolution
 second
 minute

1107.The three voltages of a three phase generator are

 180 degrees apart


 90 degrees apart
 120 degrees apart

1108.To restore the level of electrolyte which has been


lost due to normal gassing during battery charging

 electrolyte of the same specific gravity as that


contained in the battery must be added
 sulphuric acid must be added
 distilled water must be added

1109.A 12 volt lead-acid battery

 has 6 cells
 has 12 cells
 has 24 cells

1110.The active component of an A.C. circuit is 4 amps and


the reactive is 3 amps. Ammeter reading will be

 5 amps
 7 amps
 3.4 amps
Answers for Questions 1101 – 1110

1) modifies the effect of the voltage sensing coil


Comment/Reference: AC65-9A pg 399-400

2) voltage regulator
Comment/Reference: Pallett Aircraft Electrical Systems
Page 48

3) overheating of inductive devices


Comment/Reference: Inductive devices will pass more
current if it is lower frequency

4) sulphuric acid
Comment/Reference: Pallett Aircraft Electrical Systems
3rd Ed Page 22, Table 1.

5) with a dilute solution of sodium bicarbonate


Comment/Reference:

6) second
Comment/Reference: Aircraft Electricity & Electronics
5th Ed Eisman pg 84 & Aircraft Electrical Systems 3rd Ed
pg 32

7) 120 degrees apart


Comment/Reference: AC65-9A pg 413 & Aircraft
Electricity & Electronics 5th Ed Eisman pg 84 &
Aircraft Electrical Systems 3rd Ed pg 34

8) distilled water must be added


Comment/Reference: AC65-9A pg 314 & CAP 562 Leaflet 9-7
para 4.3.1

9) has 6 cells
Comment/Reference: AC65-9A pg 310

10) 5 amps
Comment/Reference: Total current is the vector sum of
active (real) and reactive currents. 3, 4, 5 triangle
in this case
Questions 1111 – 1120
1111.In a delta connected generator

 phase volts equals 1.73 line volts


 line volts equals phase volts
 line volts equals 1.73 phase volts

1112.The only practical method of maintaining a constant


voltage output from an aircraft generator under varying
conditions of speed and load is to vary the

 speed at which the armature rotates


 strength of the magnetic field
 number of conductors in the armature

1113.As the generator load is increased (within its rated


capacity), the voltage will

 remain constant and the amperage output will


increase
 remain constant and the amperage output will
decrease
 decrease and the amperage output will increase

1114.When checking the specific gravity of the electrolyte


in a battery

 each cell should be checked as the readings may


differ
 one cell only need be checked as the remainder will
read the same
 only the end cells need be checked as they contain
the terminals

1115.A lead-acid battery is considered to be fully charged


when the

 S.G. and voltage remain constant for specified


period
 cells begin to gas freely
 S.G. reaches 1.180
1116.Which aircraft circuit would be most likely to use
frequency wild 200V AC?

 Hydraulic pump
 Windscreen heating
 Standby compass

1117.To prevent corrosion at the terminals of a lead-acid


battery

 petroleum jelly may be applied to the connections


 copper connectors are used
 the connectors may be painted

1118.Before taking S.G. readings of a lead-acid battery


recently removed from an aircraft

 the battery must be charged at the 10 hour rate for


one hour
 the electrolyte temperature must be noted
 a period of 1 hour should have elapsed to allow the
S.G. to stabilize

1119.The output of a DC generator is controlled by varying


the

 armature current
 field current
 engine speed

1120.A generator rated at 30 kVA power factor 0.8 has a


maximum continuous power output of

 24 kW
 30 kW
 37.5 kW
Answers for Questions 1111 – 1120

1) line volts equals phase volts


Comment/Reference: Aircraft Electrical Systems 3rd Ed
Pallet pg 35

2) strength of the magnetic field


Comment/Reference:

3) decrease and the amperage output will increase


Comment/Reference: Assuming no voltage regulator is
considered.

4) each cell should be checked as the readings may differ


Comment/Reference:

5) S.G. and voltage remain constant for specified period


Comment/Reference: AC65-9A pg 310 & CAIPs EEL/1-1 4.7

6) Windscreen heating
Comment/Reference: Aircraft Electrical Systems 3rd ed
Pallett pg 36. Any heating or lighting circuit can be
frequency wild

7) petroleum jelly may be applied to the connections


Comment/Reference: Do a search in CAP 562. CAIPs EEL/1-
1 4.2 (vi)

8) the electrolyte temperature must be noted


Comment/Reference: AC65-9A pg 311

9) field current

10) 24 kW
Comment/Reference: PF = TP/AP 0.8 = TP/30 TP = 24
Questions 1121 – 1130
1121.The specific gravity of the electrolyte in a lead-acid
battery

 remains substantially constant regardless of the


state of charge and is not therefore a guide to the
state of charge
 remains constant with changes in the state of charge
but is a useful guide to the amount of acid
contained in the electrolyte
 changes with the state of charge and is therefore a
measure of the state of charge
1122.A battery which is assumed to be 100% efficient and to
have a capacity of 60 ampere-hours at the 10 hour rate
will deliver

 60 amps for 10 hours


 6 amps for 10 hours
 60 amps for 10 hours or 6 amps for 10 hours
depending upon the rate of demand

1123.Two 12V 25 ampere batteries connected in series will


produce

 12V with 50 ampere hour capacity


 12V with 25 ampere hour capacity
 24V with 25 ampere hour capacity

1124.As an installed battery becomes fully charged by the


aircraft generator

 the battery contactor isolates the battery from the


generator
 the generator voltage decreases to supply the
steadily decreasing current
 the battery voltage nears its nominal level so the
charging current decreases

1125.On a DC shunt generator without a voltage regulator,


if the load increases the generator output voltage will

 remain constant
 increase
 decrease
1126.For battery charging, the electrical supply connected
to the battery must be

 DC
 AC at 50 c.p.s.
 AC at 400 c.p.s.

1127.When using A.C. power on the bus bar you require 28V
D.C. for battery charging

 an A.C. inverter would be required


 a D.C. transformer and rectifier would be required
 an A.C. transformer and rectifier would be required

1128.Two similar 12V batteries connected in parallel will


produce

 12V e.m.f. with twice the capacity of each battery


 24V e.m.f. with twice the capacity of each battery
 24V e.m.f. with the same capacity as each battery

1129.If a nicad battery overheats, it is an indication of

 the generator is not connected to the bus bar


 thermal runaway
 the generator voltage regulator setting is too low

1130.When checking a nicad battery in situ for


serviceability

 the electrical circuits must be isolated before


installing shorting strips
 a load is applied to the battery and the voltmeter
reading noted
 a hydrometer must be used
Answers for Questions 1121 – 1130

1) changes with the state of charge and is therefore a


measure of the state of charge
Comment/Reference: AC65-9A pg 311

2) 6 amps for 10 hours


Comment/Reference: AC65-9A pg 310 & Aircraft
Electricity & Electronics 5th ed Eisman pg 44

3) 24V with 25 ampere hour capacity


Comment/Reference:

4) the battery voltage nears its nominal level so the


charging current decreases
Comment/Reference:

5) decrease
Comment/Reference:

6) DC
Comment/Reference: It’s a Battery

7) an A.C. transformer and rectifier would be required


Comment/Reference: Your going AC to DC

8) 12V e.m.f. with twice the capacity of each battery


Comment/Reference:

9) thermal runaway
Comment/Reference: CAP 562 leaflet 9-7 para 4.5.8

10) a load is applied to the battery and the voltmeter


reading noted
Questions 1131 – 1140

1131.The output from an AC generator is taken from

 connections on the stator


 slip rings via commutator
 slip rings

1132.Load sharing on a DC generation system is achieved by

 the load differences causing torque signals to vary


the engine speed
 circulating currents affecting the carbon pile for
voltage trimming
 tachogenerators varying the generators outputs as
the engine speed varies

1133.If a nickel cadmium aircraft battery is not required


for immediate service, it should be stored

 in a totally dry condition


 in the fully discharged condition
 in the fully charged condition

1134.The correct way to determine the state of charge of a


nickel-cadmium battery is

 by checking the specific gravity of each cell


 by checking the voltage of each cell under load
 by a complete discharge and a measured recharge

1135.A DC generator will be connected to the busbar after


the RCCO contacts close due to

 increasing generator voltage


 increasing battery voltage
 a differential voltage of 0.5V battery voltage
higher than generator
1136.Spillage of a lead acid electrolyte is detected by

 indicating paper that turns red


 litmus paper that turns black
 indicating paper that turns blue

1137.To restore electrolyte after spillage

 add distilled water only


 add a mixture of acid and water of the same s.g. as
that in the battery
 add acid only

1138.When a DC generator fails, the pilot will receive the


following indications:

 An audio bell and red PFWL


 Rising voltage and current discharge with a red PFWL
 Zero current on the ammeter and a red PFWL

1139.The purpose of an inverter is to convert

 AC to DC
 DC to AC
 AC to a higher frequency AC

1140.With a vibrating-contact type voltage regulator the


adjustment of generator voltage is carried out by

 adjustment of voltage coil resistance


 alteration of the field resistance
 alteration of contact spring tension
Answers for Questions 1131 – 1140

1) connections on the stator


Comment/Reference:
www.tpub.com/content/doe/h1011v3/css/h1011v3_102.htm

2) circulating currents affecting the carbon pile for


voltage trimming

3) in the fully discharged condition


Comment/Reference: CAP 562 Leaflet 9-7 para 8 & CAIPs
EEL/1-3 8

4) by a complete discharge and a measured recharge


Comment/Reference: CAP 562 Leaflet 9-7 para para 4.5.6
& para 4.7 & CAIPs EEL/1-3 4.5.6

5) increasing generator voltage


Comment/Reference: Aircraft Instruments & Integrated
Systems Pallett pg 201

6) indicating paper that turns red


Comment/Reference: Acid turns litmus paper red
http://en.wikipedia.org/wiki/PH#Measurement

7) add a mixture of acid and water of the same s.g. as


that in the battery
Comment/Reference: Jeppson A & P Airframe Technician pg
7-28. But AC65-9A pg 311 & 314 for both types of
batteries it says distilled water only.

8) Zero current on the ammeter and a red PFWL

9) DC to AC
Comment/Reference: AC65-9A pg 435 & Aircraft Electrical
Systems Pallett 3rd Ed Page 66

10) alteration of contact spring tension


Comment/Reference: Aircraft Electrical Systems, 3rd Ed
Pallett Page 12, & Aircraft Electricity & Electronics
5th Ed Eismin Page 199
Questions 1141 – 1150

1141.Battery SG is measured using a

 hygrometer
 hydrometer
 hypsometer

1142.A battery for use on aircraft must have a capacity of

 90%
 50%
 80%

1143.When connecting aircraft batteries, the last lead to


connect is the

 positive earth
 negative
 positive

1144.Polarization in a cell is

 the establishing of a potential across the cell


 the fact that hydrogen bubbles form on the -ve plate
 the fact that as a battery discharges, the -ve
plate's potential approaches that of the +ve plate's

1145.Battery capacity is checked by

 discharging while maintaining the voltage constant


by varying a resistor in the circuit and checking
the time to be discharged
 discharging at a constant current and checking the
time taken to reach the discharged state
 measuring the terminal voltage whilst on load and
multiplying by the current drawn
1146.The frequency of an AC generator is dependant on the

 field strength and the speed of the generator


 number of pairs of poles and the speed of the rotor
 number of conductors and the field strength

1147.To adjust the voltage output of an AC generator,


whilst maintaining constant frequency, it is necessary
to

 alter the reactance of the stator winding circuit


 alter the driving speed
 alter the field current

1148.In a paralleled AC generation system load sharing is


for

 reactive load only


 real and reactive load
 real load only

1149.Single phase components in a 3 phase system may be


connected between

 any phase and earth or between any two phases


 the phase and earth only
 phases A and B only

1150.A frequency wild supply would be suitable for

 instruments and navigation systems


 3-phase torque motors
 deicing loads
Answers for Questions 1141 – 1150

1) hydrometer
Comment/Reference: AC65-9A pg 311 & CAP 562 Leaflet 9-2
para 8.2 a) & http://en.wikipedia.org/wiki/Hydrometer

2) 80%
Comment/Reference: CAP 562 leaflet 9-6 para 7.2

3) negative
Comment/Reference: Aircraft Electricity & Electronics
5th Ed Eismin Page 43

4) the fact that hydrogen bubbles form on the -ve plate


Comment/Reference:

5) discharging at a constant current and checking the time


taken to reach the discharged state
Comment/Reference: Pallett Aircraft Electrical Systems
Page 21 & CAP 562 Leaflet 9-13 para 5 d)

6) number of pairs of poles and the speed of the rotor


Comment/Reference:

7) alter the field current


Comment/Reference: Pallett Aircraft Electrical Systems
Page 46

8) real and reactive load


Comment/Reference:

9) any phase and earth or between any two phases


Comment/Reference:

10) deicing loads


Comment/Reference: Aircraft Electrical Systems 3rd ed
Pallett pg 36
Questions 1151 – 1160

1151.Three 12V, 40Ah accumulators in series would give

 36V 40Ah
 36V 120Ah
 12V 120Ah

1152.Three 12V, 40Ah accumulators in parallel would give

 36V 120Ah
 12V 40Ah
 12V 120Ah

1153.An inverter can be an AC generator driven by

 either an AC or DC motor
 an AC motor
 a DC motor

1154.When mixing electrolyte for lead acid batteries

 add acid to water


 the method of mixing is unimportant
 add water to acid

1155.The RPM of a 6 pole, 400 Hz alternator is

 6000
 4000
 8000
1156.The ratio between apparent power and true power is the

 power factor
 power rating
 efficiency

1157.Apparent power is measured in

 Kilovolts
 Volt Amps
 Watts

1158.A Ni-Cad battery consists of 19, 1.2 volt cells. What


will be the nominal battery output?

 22V
 20V
 24V

1159.The preferred method of battery charging a Ni-Cad


battery is constant

 speed
 current
 voltage

1160.When operating two AC generators unparalleled, the


phase of each

 must be synchronized
 must be in opposition to one another
 is unimportant
Answers for Questions 1151 – 1160

1) 36V 40Ah

2) 12V 120Ah

3) a DC motor
Comment/Reference: Aircraft Electrical Systems 3rd Ed
Pallett pg 66

4) add acid to water

5) 8000
Comment/Reference: AC65-9A pg 417 & Aircraft Electrical
Systems 3rd Ed Pallett pg 32. 400/3 x 60

6) power factor
Comment/Reference: Aircraft Electrical Systems 3rd Ed
Pallett pg 3

7) Volt Amps
Comment/Reference: Aircraft Electrical Systems 3rd Ed
Pallett pg 35

8) 22V
Comment/Reference: 19 X 1.2 = 22

9) current
Comment/Reference: CAP 562 Leaflet 9-7 para 4.5.1

10) is unimportant
Comment/Reference: It says 'UNparalleled'
Questions 1161 – 1170

1161.When the GPU is connected to the aircraft

 it is paralleled with the aircraft generator


 it supplies essential services
 it is not paralleled with the aircraft generator

1162.A lead-acid battery with 12 cells connected in series


(no-load voltage = 2.1 volts per cell) furnishes 10
amperes to a load of 2 ohms resistance. The internal
resistance of the battery in this instance is

 2.52 ohms
 5.0 ohms
 0.52 ohms

1163.The rating of an aircraft alternator is 40 KVA at PF


0.8 lagging.

 The maximum load permitted is 32KW


 The maximum load under all conditions is 40KVA.
 The maximum load under all conditions is 40KW.

1164.An I.D.G (integrated drive generator)

 does not require a CSDU


 incorporates the alternator and CSD as one unit
 has separate CSDU

1165.A P.M.G (Permanent Magnet Generator) is a small built


in generator which

 only rotates during starting


 only rotates after starting
 provides initial excitation
1166.To parallel an alternator with another alternator the
alternators must have the same

 frequency, RPM and phase relationship


 frequency, voltage and phase relationship
 frequency, RPM and speed

1167.Speed of an alternator may be controlled by

 keeping engine rpm constant


 varying the field current
 CSDU (constant speed drive unit)

1168.'Frequency wild' alternators supply

 resistive circuits
 capacitive circuits
 inductive circuits

1169.The real load of an alternator is directly related to

 output shaft torque


 output shaft rpm
 output voltage

1170.How is initial voltage build-up ensured in a brushless


generating system, before full output is available from
the voltage regulator?

 A switch must be operated by the crew to provide


excitation form the dc busbar to bring generator on
line
 Excitation form the DC busbars is automatically
provided during the start sequence
 Permanent magnets are mounted between the exciter
field poles
Answers for Questions 1161 – 1170

1) it is not paralleled with the aircraft generator


Comment/Reference:

2) 0.52 ohms
Comment/Reference: OCV = 2.1 x 12 = 25.2 CCV = 10 x 2 =
20. Internal voltage drop = 5.2 V. Internal R=V/I =
5.2/10 = 0.52

3) The maximum load permitted is 32KW


Comment/Reference: PF = TP/AP 0.8 = TP/40 AP = 40 x 0.8
= 32 KW

4) incorporates the alternator and CSD as one unit


Comment/Reference: Aircraft Electrical Systems 3rd Ed
Pallett pg 43

5) provides initial excitation


Comment/Reference:

6) frequency, voltage and phase relationship


Comment/Reference:

7) CSDU (constant speed drive unit)


Comment/Reference:

8) resistive circuits
Comment/Reference: Aircraft Electrical Systems 3rd ed
Pallett pg 36

9) output shaft torque


Comment/Reference: Aircraft Electrical Systems 3rd
Pallett pg 48

10) Permanent magnets are mounted between the exciter field


poles
Comment/Reference:
Questions 1171 – 1180
1171.Insulation resistance of electrical machines is

 not affected by change of temperature of machine


 normally higher when the machine is hot
 normally lower when the machine is hot

1172.The purpose of a thermistor in a brushless AC


generator is to

 keep the exciter field resistance nearly constant at


varying temperatures
 compensate for temperature changes in the main AC
stator windings
 keep the resistance of the rectifiers nearly
constant at varying temperatures

1173.The term 'random paralleling' means the incoming


alternator's output voltage frequency and phase
relationship is

 the same
 not the same
 the outputs are constant

1174.One lamp is connected across corresponding phases of


two alternators. They will be synchronised when

 the lamp is maximum brightness


 the lamp is extinguished
 the lamp is either at maximum brightness or is
extinguished

1175.Automatic synchronising means that the incoming


alternator's voltage and frequency

 cannot be checked
 can be synchronised using the two lamps provided at
the system's panel
 an be checked at the system's electrical panel by
selecting the relevant alternator
1176.The danger of random paralleling is the possibility of

 disconnection of the other alternators


 ground power disconnection
 crash engagement

1177.What is monitored in a constant speed drive?

 Low oil press and CSD speed


 Low oil temperature and low oil press
 Hi oil temperature and low oil press

1178.The CSD or IDG speed governor setting

 is adjusted with the engine running


 is adjusted with the engine stationary
 is not allowed to be adjusted

1179.An external ground power supply can be used for

 only the ground handling busbar services


 all normal services required by flight and ground
crew
 only the emergency busbar services

1180.The APU generator can normally be

 connected in parallel with the aircraft's main


generators
 used during flight as a back up supply if the main
aircraft supply fails
 connected in parallel with the ground power supply
Answers for Questions 1171 – 1180

1) normally higher when the machine is hot


Comment/Reference:

2) keep the exciter field resistance nearly constant at


varying temperatures
Comment/Reference: Aircraft Electrical Systems 3rd Ed
Pallett pg 45 and diagram.

3) not the same


Comment/Reference:

4) the lamp is extinguished


Comment/Reference:

5) can be checked at the system's electrical panel by


selecting the relevant alternator
Comment/Reference:

6) crash engagement
Comment/Reference:

7) Hi oil temperature and low oil press


Comment/Reference:

8) is adjusted with the engine stationary


Comment/Reference:

9) all normal services required by flight and ground crew


Comment/Reference:

10) used during flight as a back up supply if the main


aircraft supply fails
Comment/Reference:
Questions 1181 – 1190

1181.The impedance on a run of co-axial cable is

 proportional to the length of cable


 inversely proportional to the length of cable
 not affected by length of cable

1182.Galley loads are supplied from the

 battery bus bar


 main bus bar
 ground service bus

1183.A ni-cad battery is stored in the

 fully discharged condition


 fully charged condition
 dry condition

1184.A 6-diode Transformer Rectifier Unit (TRU) changes

 single phase AC to direct current


 direct current to AC
 three phase AC to direct current

1185.A frequency wild AC generator is used for

 fluorescent lights
 deicing loads
 instruments
1186.What should you do if the electrolyte is found to be
low in one cell of a lead acid battery?

 Replace the defective cell


 Top-up the individual cell with distilled water
 Cell is defective, replace the battery

1187.What is the purpose of the diode on a contactor?

 Prevent bounce
 To prevent current going the wrong way
 Prevent spikes

1188.Cargo and baggage compartments in which a fire is


completely confined come under

 Class B
 Class D
 Class C

1189.Aircraft emergency escape slides are usually


pressurised with

 helium
 oxygen
 CO2

1190.Vertical projected separation between seat rows must


be

 3 inches
 5 inches
 4 inches
Answers for Questions 1181 – 1190

1) not affected by length of cable


Comment/Reference:
www.epanorama.net/documents/wiring/cable_impedance.html

2) main bus bar


Comment/Reference:

3) fully discharged condition


Comment/Reference: CAP 562 Leaflet 9-7 para 8

4) three phase AC to direct current


Comment/Reference: Aircraft Electrical Systems Pallett
Pages 65-67

5) deicing loads
Comment/Reference: Aircraft Electrical Systems 3rd ed
Pallett pg 36

6) Top-up the individual cell with distilled


waterComment/Reference:

7) Prevent spikes
Comment/Reference: It is known as a 'flywheel diode'

8) Class D
Comment/Reference: CS 25.857 or the old JAR 25.857

9) CO2
Comment/Reference: All inflatable emergency equipment
are inflated with CO2
10) 3 inches
Comment/Reference: CAP 747 GR No.2 para 4.3
Questions 1191 – 1200

1191.Life jackets are inflated with cylinders of

 Nitrogen
 CO2
 Air

1192.The minimum passenger aisle width for an aircraft with


a seating capacity of 20 or more is

 dependant upon aircraft type


 24 inches
 15 inches

1193.When testing emergency slide bottles

 any pin could be checked by connecting directly to


ground
 all three pins must be shorted together and grounded
to the cartridge body
 one pin must be grounded to the cartridge body at
one time

1194.Where would you find information regarding galley


equipment?

 BCARs
 ANO
 AWN 99

1195.How often are life jackets inspected?

 In accordance with the manufacturers specification


 In accordance with the AMM
 Every 12 months
1196.Toilets must have a smoke detection

 light in the cockpit


 connected to a pump to pump the smoke out
 aural and visual warning in the main cabin

1197.Where would you find information on life-raft


deployment?

 JARs
 Cabin safety on-board card
 Cabin operations manual

1198.How do you clean seat belts?

 With warm water with low alkalinity soap


 With white spirit
 With MEK

1199.After a successful ditching, the liferafts deployment


will

 be briefed by the cabin crew


 be rapid and obvious with easily and readily
displayed instructions adjacent to the liferafts
 have detailed instructions in a booklet in a seat
pocket

1200.Type and Quantity of emergency equipment to be carried


is specified in which document?

 CAAIPs Leaflet 5-7


 Airworthiness Notice 79
 Schedule 4 of the ANO
Answers for Questions 1191 – 1200

1) CO2
Comment/Reference: CAIPs AL/3-12. 4 b All inflatable
safety equipment use CO2

2) 15 inches
Comment/Reference: CS 25.815

3) all three pins must be shorted together and grounded to


the cartridge body
Comment/Reference: Testing of the cartridges is the
same procedure as testing fire bottle cartridges. CAIPs
EEL/1-7 3.6.3 on.

4) AWN 99
Comment/Reference: Now it’s in CAP 747 GR No. 22

5) In accordance with the manufacturers specification


Comment/Reference: CAP 562 Leaflet 5-2 para 4.1

6) light in the cockpit


Comment/Reference: CS 25.854

7) Cabin operations manual


Comment/Reference:

8) With warm water with low alkalinity soap


Comment/Reference:

9) be rapid and obvious with easily and readily displayed


instructions adjacent to the liferafts
Comment/Reference: CS 25.1561 d)

10) Schedule 4 of the ANO


Comment/Reference:
Questions 1201 – 1210

1201.Where the aircraft passenger seating capacity exceeds


149 but less than 200 then:

 2 megaphones are required


 1 megaphone is required
 3 megaphones are required

1202.Megaphones must be stowed so they can withstand an


inertia force of

 15g
 9g
 12g

1203.The cylinder in a life jacket is inflated using

 CO2
 nitrogen
 compressed air

1204.Information on Galley equipment can be found in

 CAAIPS
 BCAR Section L
 Airworthiness Notice 99

1205.Where is information regarding flame resistant


furnishing materials?

 CAP 747 Mandatory Requirements for Airworthiness


 CAAIPs
 AWN 58
1206.What is the EASA regulation of the minimum separation
between a seat and another seat or structure?

 10 inches
 7 inches
 There is no EASA guideline

1207.What is the minimum distance between a seat cushion


and the seat in-front of it?

 10 inches
 5 inches
 7 inches

1208.What is the minimum separation between a seat back


cushion and another seat or fixed structure?

 28 inches
 26 inches
 24 inches

1209.Cargo nets are manufactured from

 hessian
 polypropylene
 nylon

1210.A galley trolley will be designed to

 not exceed the floor limits more than 3 times in one


hour
 not exceed the floor loading limits
 does not matter as the trolleys move
Answers for Questions 1201 – 1210

1) 2 megaphones are required


Comment/Reference: JAR Ops 1.810

2) 9g
Comment/Reference: CS 25.561 & CS 25.1421

3) CO2
Comment/Reference: AL/3-12 2

4) Airworthiness Notice 99
Comment/Reference: Now in CAP 747 Appx 1 GR No. 22

5) CAP 747 Mandatory Requirements for Airworthiness


Comment/Reference: CAP 747 Appx 1 GR No. 13

6) 7 inches
Comment/Reference: CAP 747 Appx 1 Tabel 1 and Appx 1 GR
No.2 para 4.2

7) 7 inches
Comment/Reference: CAP 747 Appx 1 GR No. 2 para 4.2

8) 26 inches
Comment/Reference: CAP 747 Appx 1 GR No. 2 para 4.1

9) nylon
Comment/Reference: Cargo nets are made from nylon or
polyester (not polypropylene).

10) not exceed the floor loading limits


Comment/Reference: CAP 747 Appx 1 GR No. 22 para 5.6
Questions 1211 – 1220

1211.When loading a cargo aircraft

 the manufacturer specifies the maximum floor loads


 the weight limits of the aircraft could be exceeded
under careful monitored circumstances
 the CofG limits of the aircraft could be exceeded under
careful monitored circumstances

1212.Galley equipment which does not need electrical power


is called

 uncontrolled equipment
 controlled equipment
 minimum equipment

1213.Galley trolleys must have placards indicating

 Must be stowed for take-off, landing and turbulence.


 maximum weight, position, flight number
 location, position, flight number

1214.PTV type entertainment equipment

 is installed with seat electronic unit and multiplexing


techniques
 is not the part of maintenance schedule
 has no capacity for digital information processing and
transmitting

1215.Hot fluids in an aircraft galley must have closed


container lids at temperatures above

 55°C
 45°C
 65°C
1216.With regard to forward/rearward passenger seat
installation

 you can fit seats facing forward or rearward


 you can only fit seats facing forward
 you can only fit seats as stated in the Declaration of
Design and Performance (DPP)

1217.Attendant call is achieved by pressing a button on the


passenger service unit (PSU). To reset the light

 a reset switch on the attendants panel is operated


 the button is pushed again
 the button is pulled out

1218.On large aeroplanes, containerised cargo is held down


by

 rope
 side cart guides and fore and aft latches
 netting

1219.A galley catering trolley with electrics is classed as

 an uncontrolled item
 a controlled item
 neither, as it is removable

1220.The minimum vertical projected separation between seat


rows is

 5 inches
 3 inches
 7 inches
Answers for Questions 1211 – 1220

1) the manufacturer specifies the maximum floor loads

2) uncontrolled equipment
Comment/Reference: CAP 747 Appx 1 GR No. 22 para 4.2

3) Must be stowed for take-off, landing and turbulence


Comment/Reference: CAP 747 Appx 1 GR No. 22 para 5.11

4) is installed with seat electronic unit and multiplexing


techniques
Comment/Reference: PTV = Personal Television

5) 45°C
Comment/Reference: CAP 747 Appx 1 GR No. 22 para 5.2

6) you can only fit seats as stated in the Declaration of


Design and Performance (DPP)
Comment/Reference: CAP 747 Appx 1 GR No. 2 para 2.2

7) the button is pushed again

8) side cart guides and fore and aft latches

9) a controlled item
Comment/Reference: CAP 747 Appx 1 GR No. 22 para 4.1

10) 3 inches
Comment/Reference: CAP 747 GR No.2 Appx 1 para 4.3
Questions 1221 – 1230

1221.On pulling the fire handle, the aural warning

 goes off, the light goes off, hydraulics, bleed air


and generator goes off
 goes off, the light remains on, the generator goes
off line
 stays on, the light extinguishes, the generator goes
off line

1222.After a fire detection system test, the red light


remains ON. This indicates

 a short circuit
 high resistance
 a fire

1223.First digit number of halogenated hydrocarbon


extinguishers represents the number of

 fluorine atoms
 chlorine atoms
 carbon atoms

1224.What type of gas is used as a propellant for a high


rate of discharge extinguishing system?

 Hydrogen
 Nitrogen
 Carbon dioxide

1225.Which areas of a civil transport aircraft would have


smoke detectors fitted?

 Passenger cabins and integral fuel tanks


 Main wheel bays and engine bays
 Freight holds, baggage compartments and equipment
bays
1226.Engine wing firebottles should be charged with
nitrogen at 15°C to

 1250 psi
 600 psi
 1500 psi

1227.A fire that can be put out by a crew member in a cargo


bay is in a

 class B compartment
 class A compartment
 class D compartment

1228.In a capacitive fire detection system, if

 there is a break, the system will not work


 there is a break, the system will work but not test
satisfactory
 there is a short to earth, the system will work but
not test satisfactory

1229.A fire detection zone wiring can pass through another


fire zone when

 the system and the extinguishing system are the same


 the system is different but the extinguishant is the
same
 the system is the same but the extinguishant is
different

1230.When a fire test on a Systron Donor fire detector


system is carried out

 a signal passes through the unbroken centre


conductor to produce the warning
 it checks the integrity of the contacts
 the signal passes through the centre and back
through the outer to give warning
Answers for Questions 1221 – 1230

1) goes off, the light remains on, the generator goes off
line
Comment/Reference: B757 AMM

2) a short circuit

3) carbon atoms
Comment/Reference: Jeppesen Airframe Textbook Chapter
16 Page 16

4) Nitrogen
Comment/Reference: Jeppesen Airframe Textbook Chapter
16 page 20

5) Freight holds, baggage compartments and equipment bays


Comment/Reference: CS 25 search smoke detectors

6) 600 psi
Comment/Reference: Transport Category Aircraft Systems
Page 9-7. Pallett Aircraft Electrical Systems 3rd Ed
Page 166 & B737-3/4/500 AMM 26-22-11 pg 601

7) class B compartment
Comment/Reference: CS 25.857 & Transport Category
Aircraft Systems Page 9-3

8) there is a break, the system will work but not test


satisfactory
Comment/Reference: CAIPs AL/3-9 Para 4.2.1

9) the system and the extinguishing system are the same


Comment/Reference: CS 25.1203 (f)(2)

10) it checks the integrity of the contacts


Comment/Reference: Jeppesen a+p Technician Airframe
Textbook page 11-6 b describes a systron-donner system.
This is the same system the CAA call a sensor/responder
system in CAIPs EEL/1-7 para 2.2.4 in para d
Questions 1231 – 1240

1231.When inspecting lavatory installations

 the ashtray should not be located close to the


toilet paper holder
 'No Smoking' sign should be fitted next to the
ashtrays only
 the ashtray must be made of a thermoplastic material

1232.Lavatories must have, in the disposal container

 Freon extinguisher with black ball markings


indicating when discharged
 water/glycol extinguisher with white ball markings
indicating when discharged
 Freon extinguisher with white ball markings
indicating when discharged

1233.A fire detection loop must be capable of

 using master warning visual signals only


 using master warning visual and aural signals
 detecting fire/overheat condition within 5 seconds
and extinguishing such that after 30 seconds the
condition is no longer present

1234.A Systron-Donner fire detection system uses

 helium gas
 nitrogen gas
 oxygen gas

1235.A toilet is fitted with thermal protection

 to protect against continuous flushing


 to protect against the dangers of smoke and fire in
the event of an overheat
 to protect against freezing at altitude
1236.What external warning is there for an APU fire?

 Red light
 Aural
 Both aural and a red light

1237.Engine fire bottle pressure switches are operated by

 DC
 AC
 AC or DC

1238.Fire tests on aircraft internal furnishings must be


carried out

 only when equipment items fail random tests


 every five years
 every two years

1239.A cockpit check on the engine firewire system is a

 continuity check
 calibration check
 function check

1240.When the fire handle is operated it cuts off

 field current, hydraulics and fuel and operates fire


 extinguishers
 operates fire extinguisher only
 cuts off fuel and hydraulics. Generator remains on
line to provide electrical power
Answers for Questions 1231 – 1240

1) the ashtray should not be located close to the toilet


paper holder
Comment/Reference: CAP 747 Appx 1 GR No. 20 para 5.4

2) Freon extinguisher with black ball markings indicating


when discharged
Comment/Reference: The temperature sensitive strip
changes color from white to black at the indicated
temp. B737 MM 26-24-00 pg1. CS 25.854 a, b

3) using master warning visual and aural signals


Comment/Reference: Aircraft Electrical Systems 3rd Ed
Pallett Page 163

4) helium gas
Comment/Reference: Transport Category Aircraft Systems.
Jeppesen Page 9-22

5) to protect against the dangers of smoke and fire in the


event of an overheat
Comment/Reference: AWN 57 Para.2.3 (This AWN has been
deleted on 23OCT03) see CAP 455

6) Both aural and a red light


Comment/Reference: B737-3/4/500 AMM 26-15-00 pg 1 para
1.E.

7) DC
Comment/Reference: B737-3/4/500 AMM 26-21-00 pg 1 para
1.C.

8) only when equipment items fail random tests

9) continuity check
Comment/Reference: B737-400 AMM 26-11-00 pg 3 para 4.C.

10) cuts off field current, hydraulics and fuel and


operates fire extinguishers
Comment/Reference: B737-3/4/500 AMM 26-21-00 para 4.A.
on pg 5 & www.b737.org.uk/fireprotection.htm#Engines
Questions 1241 – 1250

1241.What is the contamination monitor for, in a Systron


Donner fire detection system?

 Dirty contacts
 Volts drop
 Integrity comparator

1242.Fireproofing of cabin material should be done

 every 2 years
 every 5 years
 when a sample fails

1243.What test is carried out to ensure serviceability of a


squib?

 Bottles are fired


 A continuity check
 Safety ohmmeter is used to check voltage

1244.In a gas firewire system, what causes the firewire


caption to come on when heat is applied?

 Pressure
 Continuity
 Capacitance increase

1245.Toilet fire detection issues can be found in

 AWN 80
 AWN 83
 CAAIPs
1246.The outside casing of a firewire has

 28VDC potential above ground


 115VDC potential above ground
 same potential as ground

1247.A cargo compartment which will contain a fire itself


and ventilate is a class

 D
 C
 A

1248.Smoke detectors consist of a

 lamp detecting 2% smoke


 photoelectric cell detecting 2% smoke
 photoelectric cell detecting 10% smoke

1249.In a toilet smoke condition, what would be the


indication?

 Audible warning in the cabin


 Either a light or an audible warning in the cabin
 Light in the cabin

1250.What do you check when carrying out an insulation


check on a continuous firewire loop?

 The outside of the firewire to earth


 The firewire
 The firewire, sensor & controller
Answers for Questions 1241 – 1250

1) Dirty contacts
Comment/Reference: EEL/1-7 2.2.4 (d)

2) when a sample fails


Comment/Reference:

3) A continuity check
Comment/Reference: A continuity check is carried out -
with a safety ohmmeter. But a safety ohmmeter does not
check voltage. B737-3/4/500 AMM 26-21-00 pg 1 para 3.D.
(1)

4) Pressure
Comment/Reference: Jeppesen A&P Airframe Technician
Textbook Page 16-8 Lindberg System

5) AWN 83
Comment/Reference: CAP 747 Appx 1 GR No. 20 or the old
AWN 83

6) same potential as ground


Comment/Reference:

7) D
Comment/Reference: CS 25.857

8) photoelectric cell detecting 10% smoke


Comment/Reference: Transport Category Aircraft Systems
Page 9.2

9) Either a light or an audible warning in the cabin


Comment/Reference: CS 25.854

10) The firewire


Comment/Reference: Pallett Aircraft Electrical System
3rd Ed pg 163 fig 10.17
Questions 1251 – 1260

1251.When the fire-handle switch is pulled, a

 red warning light stays on. Generator, hydraulics, &


fuel is cut off
 red warning light goes off. Generator goes offline
 red warning light stays on. Generator, hydraulics &
fuel stays on

1252.An indicator pin is protruding from a portable fire


extinguisher. What does it indicate?

 Bottle is empty
 Bottle is full
 The bottle is still pressurized

1253.What are the minimum number of fire bottles in a cabin


with 201-300 people?

 4 Methyl Bromide
 3 BCF
 4 Fire Extinguishers

1254.How often do hand-held extinguishers have to be


weighed?

 Every 5 years
 Bi-annually
 Annually

1255.The weight of an extinguisher is stamped

 on head fitting
 on base of body
 on bracket
1256.How can fire extinguisher be identified as being used?

 Check the tell-tale wire


 Weighed
 Check the pressure

1257.Portable fire extinguishers are operated by

 turning upside down and squeeze button/trigger


 breaking the tell tale/tamper-proof seal then
operated by the handle
 one single operation until fire extinguisher is
empty

1258.How many fire extinguishers are required on an


aircraft with 501 passengers?

 7
 5
 6

1259.How many fire extinguishers are required on an


aircraft having 401 - 500 passengers?

 6
 5
 8

1260.CO2 fire extinguishers are used on

 all fires in fuselage


 baggage holds only
 engines only
Answers for Questions 1251 – 1260

1) red warning light stays on. Generator, hydraulics, &


fuel is cut off
Comment/Reference: B737 MM. Warning light is an
indication of fire. Light goes off only when fire is
extinguished

2) Bottle is empty

3) 4 Fire Extinguishers
Comment/Reference: CS 25.851

4) Annually
Comment/Reference: CAIPs AL/3-10 para 6.3 and 7.1

5) on head fitting
Comment/Reference: CAIPs AL/3-10, para 6.3

6) Weighed
Comment/Reference: AL/3-10 6.3

7) breaking the tell tale/tamper-proof seal then operated


by the handle
Comment/Reference:

8) 7
Comment/Reference: CS 25.851
501-600 passengers 

9) 6
Comment/Reference: CS 25.851

10) all fires in fuselage


Comment/Reference: AC65-9A pg 501-502 & AC65-15A pg 415
& AL/3-10 3.3
Questions 1261 – 1270
1261.In an aircraft flying control system employing servo-
tabs, the installation of external ground locks to the
main control surface

 would also prevent movement of the control column


 is unnecessary since the system is irreversible and
therefore the control surface cannot be displaced by
the wind
 would not prevent movement of the control column

1262.If aerodynamic balancing is increased, what is the


effect on the centre of pressure of the control
surface?

 It is not effected
 It moves back
 It moves forward

1263.A control system in the parallel mode,

 the control pedals moves only in the approach


 the pedals deflect with the rudder deflection
 when the rudder moves the control pedals stay in
position

1264.A full time yaw damper system detects

 band pass frequencies


 all frequencies
 only low frequencies

1265.A static friction check is carried out during control


surface rigging using

 a cable tensiometer to ensure cable tensions are


correct
 a spring balance to ensure there is a minimum amount
of feel when the controls are moved
 a spring balance to measure control 'break-out
force'
1266.If flaps try to move asymmetrically on selection, they

 drive to the up position and lock


 stop moving and stay in the same position
 drive to the down position and lock

1267.A stick-shaker comes into operation when the aircraft

 is approaching a stall
 is approaching the 'critical mach number'
 goes supersonic

1268.With respect to flight spoilers, they

 can operate both on the ground and in flight


 only operate in flight
 only operate on the ground

1269.Considering an aileron to rudder crossfeed, in a left


wing down turn, the

 rudder moves left


 rudder remains in the centre
 rudder moves right

1270.Dutch roll is eliminated by

 the Dutch Roll damper


 the yaw damper
 differential ailerons
Answers for Questions 1261 – 1270

1) would not prevent movement of the control column


Comment/Reference: Aircraft Flight Barnard and Philpot,
Second Edition Page 253 (first line)

2) It moves forward

3) the pedals deflect with the rudder deflection

4) band pass frequencies


Comment/Reference: Pallett Automatic Flight Control 4th
ed Page 222 on.

5) a spring balance to measure control 'break-out force'


Comment/Reference:

6) stop moving and stay in the same position


Comment/Reference:

7) is approaching a stall
Comment/Reference:

8) can operate both on the ground and in flight


Comment/Reference: Flight spoilers are used in flight
and on the ground. However Ground spoilers can ONLY be
used on the ground. Jeppesen Transport Category
Aircraft Systems Page 5-6. Pallett Automatic Flight
Control 2nd Edition Page 51

9) rudder moves left


Comment/Reference:

10) the yaw damper


Comment/Reference: Automatic Flight Control 4th ed
Pallett pg 222
1271.The bearing used in a fibre pulley is

 a plain bearing
 a ball bearing
 a roller bearing

1272.When the primary stops fitted to a control run have


been contacted

 the secondary stops will just be in contact


 the secondary stops have been over-ridden
 a clearance exists at the secondary stops

1273.Excessive wear on both sides of the control cable


pulley groove is evidence of

 pulley misalignment
 excessive cable tension
 cable misalignment

1274.A universal protractor used to measure degrees of


aileron travel should be zeroed

 with the aileron in the neutral position


 when the aircraft is in level flight attitude
 with the aileron in the down position

1275.A flying control static friction check

 will demonstrate that a high force is always


necessary to operate non-powered controls.
 can only be carried out during flight
 demonstrates the force the pilot requires to
overcome system inertia and move the controls
1276.What is an aileron balance cable for?

 To allow the cable to be tensioned


 To allow aircraft to fly hands off
 To correct for wing heaviness

1277.With the elevator trim wheel adjusted fully aft (nose


up), range of movement of the elevator is restricted

 for 'up' travel movement


 neither up nor down, as movement of the trim tab
would make no difference to the movement
 for 'down' travel movement

1278.On the ground, spoilers/speedbrakes deploy to

 30°
 60°
 15°

1279.A horn balance on a control will

 increase degree of movement at high speed


 decrease degree of movement at low speed
 decrease degree of movement at high speed

1280.In normal flight, if the control column is pulled


back, a servo tab fitted to the elevator will

 move up
 remain in line with the elevator
 move down
Answers for Questions 1271 – 1280

1) a ball bearing
Comment/Reference: Ball bearings are used in fibre
pulleys

2) a clearance exists at the secondary stops


Comment/Reference: The clearance is on the secondary
stops

3) pulley misalignment
Comment/Reference: Jeppesen A&P Technician Airframe
Textbook Page 1-45

4) with the aileron in the neutral position


Comment/Reference: Jeppesen A&P technician Airframe
Textbook Page 1-40

5) demonstrates the force the pilot requires to overcome


system inertia and move the controls
Comment/Reference:

6) To allow the cable to be tensioned


Comment/Reference: Jeppesen A&P technician Airframe
Textbook Page 1-27

7) for 'up' travel movement


Comment/Reference: Use of trim reduces the associated
control movement direction range

8) 60°
Comment/Reference: Transport Category Aircraft Systems
Jeppesen Page 5-6. In the air - 30 degrees and a
further 15 degrees with ailerons.

9) increase degree of movement at high speed


Comment/Reference: For a given pilot effort, the horn
balance increases movement.

10) move down


Comment/Reference:
Questions 1281 – 1290

1281.In normal flight, if the control wheel is moved to the


left, then the aileron on the right wing will

 increase the angle of attack of the right hand wing


 decrease the angle of attack of the right hand wing
 keep the angle of attack of the right wing constant

1282.On an aircraft fitted with elevons in normal flight,


if a column is moved forward, the elevons will

 both move up
 both move down
 remain stationary

1283. To avoid moments bending the root of the wing, the


aircraft will be fitted with

 integrated fuel monitoring system


 symmetrical flight control trim
 automatic trim control for controlled turns and
rudder movements

1284.A yaw damper gain could be fitted with

 increased sensitivity for increased speed


 decreases sensitivity for increased speed
 decreases sensitivity for decreased speed

1285.In an aircraft system employing balance tabs, the


installation of external ground locks between the
control surface and the balance tabs

 would prevent the movement of the control column


 would not prevent the movement of the control column
 is unnecessary since the system is irreversible
1286.The two switches for pitch trim are located

 on the control column with the captain having


priority over the First Officer.
 on the control column acting like an 'AND' switch
 on the control column acting like an 'OR' switch

1287.If the flaps are sluggish or erratic on final


approach, the possible cause may be

 flap motor internal leakage


 flap tracks out of rigging
 bearing seized

1288.In fully Fly By Wire aircraft, wing root bending


moments are reduced by

 operation of the elevator


 symmetrical application of ailerons
 transfer of fuel from one wing tank to another

1289.Yaw damping rate changes with

 altitude
 airspeed
 flap position

1290.What is the main purpose of a friese aileron?

 Decrease drag on the up going wing


 Increase drag on the up going wing
 Help pilot overcome aerodynamic loads
Answers for Questions 1281 – 1290

1) increase the angle of attack of the right hand wing


Comment/Reference: The downgoing aileron (the right
one) has an increase in AoA locally, which makes the
wing lift, thus reducing the AoA of the wing as a
whole. Choose your interpretation.

2) both move down

3) symmetrical flight control trim


Comment/Reference: Probably referring to Load
Alleviation Function. Jeppesen Transport Category
Aircraft Systems Page 5-29.

4) increased sensitivity for increased speed


Comment/Reference: Automatic Flight Control, Pallett
Page 223

5) would prevent the movement of the control column


Comment/Reference: A+P Technician Airframe Textbook
Page 1-29

6) on the control column acting like an 'AND' switch


Comment/Reference: The 'split' trim switch has one half
closing a relay to 'enable' power to the trim motor,
and to engage the clutch. The other switch half is to
supply the power to the motor. Both must be operated
together, to operate the trim.

7) flap motor internal leakage


Comment/Reference: Jeppesen Transport Category Aircraft
Systems Page 5-19 & B737-3/4/500 AMM 27-51-00 pg 109
‘slow flap operation’

8) symmetrical application of ailerons


Comment/Reference: CAIPs EEL/1-2

9) airspeed
Comment/Reference: Avionic Fundamentals page 292

10) Decrease drag on the up going wing


Comment/Reference: The Friese aileron increases drag on
the up-going aileron, which is on the down going wing.
And Vise versa on the other aileron
Questions 1291 – 1300

1291.What happens to a hydraulically operated flight


control surface when pressure is removed?

 Go to neutral
 Remain in last position
 Droop

1292.Oscillation and hunting of flight control surface are


prevented by

 feedback from the servo motor


 feedback from control surface
 fitting a tachogenerator

1293.Stick shaker activates at a speed which is above the


stalling speed by

 7%
 10.321%
 4%

1294.A wing mounted stall warning vane

 gives a visual indication of impending stall


 moves down at impending stall
 moves up at impending stall

1295.During an autopilot controlled turn

 ailerons and rudder will move


 ailerons, rudder and THS will move
 ailerons, rudder and elevators will move
1296.On a full fly-by-wire system, a speedbrake screwjack
servomotor is controlled by the

 flight augmentation computer


 flight management and guidance computer
 flight control computer

1297.In a fly-by-wire aircraft, what controls the roll


spoilers?

 Flight management computer


 Flight control computer
 Flight augmentation computer

1298.Mass balance weights are used to

 balance the tabs


 counteract flutter on control surfaces
 balance the trailing edge of flying control surfaces

1299.Active load control involves

 varying lift force to control vertical movement of


the aircraft
 limiting the deflection of control surface with
airspeed
 intervention & monitoring the human pilot

1300.Active load control uses

 aileron and spoiler


 elevator and aileron
 elevator and stab
Answers for Questions 1291 – 1300

1) Droop
Comment/Reference: The drop in pressure releases a
valve which lets the fluid pass easily from one side of
actuator piston to the other. This is to prevent a
hydraulic lock and permit manual reversion. Surfaces
behave like manually controlled surfaces.

2) fitting a tachogenerator
Comment/Reference: Automatic Flight Control Pallett 4th
Ed pg 158 (feedback)

3) 7%
Comment/Reference: Aircraft Electricity & Electronics,
Eismin 5th Edition Page-370

4) Moves up at impending stall


Comment/Reference:

5) Ailerons, rudder and elevators will move


Comment/Reference:

6) Flight control computer


Comment/Reference: Transport Category Aircraft Systems
Page 5-26

7) Flight control computer


Comment/Reference: Transport Category Aircraft Systems
Page 5-26

8) counteract flutter on control surfaces


Comment/Reference: CAP 562 Leaflet 11-22 Appx 27-1 pg
39

9) varying lift force to control vertical movement of the


aircraft
Comment/Reference: Automatic Flight Control 4th Ed
Pallett and Coyle Page 292

10) aileron and spoiler


Comment/Reference: Transport Category Aircraft Systems
5-29 (Load Alleviation Function)
Questions 1301 – 1310

1301.The purpose of the autopilot servo-motor torque


setting is to

 prevent control surface runaway


 protect the servo motor
 damp the system oscillation

1302.In a fully Fly By Wire Aircraft, ground spoilers are


deployed automatically when the aircraft is on ground
and

 weight on ground switch is activated


 thrust reversers are deployed
 brakes are deployed

1303.In a fully Fly By Wire aircraft, rudder trim is nulled


by the

 Flight Guidance and Management Computer


 electric flight control unit
 Flight Augmentation Computers

1304.Aileron input is fed into the yaw damper system to

 prevent adverse yaw in a turn


 prevent nose pitching down
 prevent nose pitching up

1305.Pitch trimming in autopilot is initiated by

 C of G movement
 continued pitch input
 pitch of aircraft in cruise
1306.Differential aileron control will

 cause a nose up moment


 cause a nose down moment
 prevent yawing in conjunction with rudder input

1307.On a fly-by-wire aircraft, what controls stabilizer


trim?

 ELAC
 ELAC and SEC
 SEC

1308.In an automatic flight control system, when may the


yaw damper be applied?

 During manual control only


 During automatic control only
 During either manual or automatic control

1309.Flutter can be prevented by

 balance panels
 trim tabs
 mass balance

1310.In a fully fly by wire system, if the elevator loses


all electrical power

 servos move to neutral and lock


 servos remain stationary and provide damping
 servos lock at last position
Answers for Questions 1301 – 1310

1) prevent control surface runaway


Comment/Reference: Automatic Flight Control Pallett and
Coyle

2) thrust reversers are deployed


Comment/Reference: A320 Ground Spoiler Logic diagram
also, www.b737.org.uk/flightcontorls.htm

3) Flight Augmentation Computers


Comment/Reference: Transport Category Aircraft Systems
Page 5-26

4) prevent adverse yaw in a turn

5) continued pitch input


Comment/Reference: If the elevator is deflected over a
long time, the AFCS trims the horizontal stabilizer to
eliminate the elevator deflection load

6) prevent yawing in conjunction with rudder input


Comment/Reference:

7) ELAC and SEC


Comment/Reference: Transport Category Aircraft Systems
5-23

8) During either manual or automatic control


Comment/Reference: Automatic Flight Control Pallett
Page 222 - 226

9) mass balance
Comment/Reference: CAP 562 Leaflet 11-22 Appx 27-1 pg
39

10) servos remain stationary and provide damping


Comment/Reference: A319/A320/A321 Flight Crew Operating
Manual
Questions 1311 – 1320

1311.In an auto trim system, for the trim system to operate

 autopilot need not be engaged


 operation of the trim controls is required
 autopilot must be engaged

1312.In an autopilot coordinated turn, when the turn angle


is reached

 the ailerons are faired


 both ailerons are down
 one is up one is down

1313.How is automatic angle of attack protection provided?

 Reduce flap deployment


 Autothrottle applying more power
 Fast/Slow indication

1314.A single failure of fly by wire

 has no effect on the aircraft's operation


 will limit the flight profile
 will reduce the operational height and speed

1315.Fly-by-wire load alleviation function in turbulent


weather conditions will result in

 spoiler moving symmetrically upward


 ailerons moving symmetrically upward
 ailerons and spoiler moving symmetrically upward
1316.Autotrim will switch to 'slow' when

 landing gear up and locked


 flaps are retracted
 flaps are extended

1317.How is the stabiliser automatically controlled in


normal manual operation?

 Pitch Trim
 Electric Trim
 Mach/Speed Trim

1318.Spoiler position feedback is provided by a

 a microswitch
 an R.V.D.T.
 a S.C.M.

1319.Rudder 'Q' limiting

 increases feel as airspeed increases


 restricts rudder movement with increase in airspeed
 increases rudder movement with increase in airspeed

1320.Elevons combine the functions of both

 elevator and aileron


 rudder and elevator
 rudder and aileron
Answers for Questions 1311 – 1320

1) autopilot need not be engaged


Comment/Reference: Automatic Flight Control, Pallett
Page 213 - 218

2) the ailerons are faired


Comment/Reference: Due to the aircraft's neutral
lateral stability, the ailerons are neutralised
(faired) and the aircraft will hold the turn

3) Autothrottle applying more power


Comment/Reference:

4) has no effect on the aircraft's operation


Comment/Reference:

5) ailerons and spoiler moving symmetrically upward


Comment/Reference: However, Automatic Flight Control
4th ed Pallett pg 292 under Gust Load Alleviation it
says just ailerons (the answer came from the original)

6) flaps are retracted


Comment/Reference:

7) Mach/Speed Trim
Comment/Reference:

8) an R.V.D.T.
Comment/Reference:

9) restricts rudder movement with increase in airspeed


Comment/Reference: Automatic Flight Control 4th Ed
Pallett pg 33 (control gearing)

10) elevator and aileron


Comment/Reference:
Jeppesen A&P Technician Airframe Textbook Page 1-16 Fig
1-36 & AC65-15A pg 18 5th para lh side
Questions 1321 – 1330

1321.A differential aileron system is designed to

 prevent adverse yaw


 minimise flutter
 compensate for aileron reversal

1322.An artificial feel system is required

 for power assisted control systems


 for proportional control systems
 for power operated control systems

1323.A tandem P.F.C.U.

 has the actuator rams co-axial


 has the actuator rams parallel
 has two control surfaces under its control

1324.What is the purpose of a differential Frise aileron?

 Has no effect on rate of yaw and turn


 To increase the drag to increase the rate of yaw and
turn
 To decrease the drag and decrease the rate of yaw
and turn

1325.Active load control uses

 elevator and stab


 ailerons and spoilers
 elevator and ailerons
1326.An artificial feel system is necessary in a powered
flying control system to

 prevent overloading of the power control units


 increasing the sensitivity of the control system
 provide the pilot with simulated stick forces

1327.In the event of hydraulic failure in a power control


system, a requirement of the manual reversion is that
it must be

 operated by the standby hydraulic system


 possible, but not recommended
 automatic and instantaneous

1328.The purpose of control cable regulators is to

 maintain preset cable tensions during flight


 compensate for low temperature only
 compensate for high temperature only

1329.Range of movements of power operated flying control


surfaces are limited by

 travel of the jack body


 mechanical stops in the control system
 travel of the jack ram

1330.The aircraft is controlled about the lateral axis by


the

 ailerons
 elevator
 rudder
Answers for Questions 1321 – 1330

1) prevent adverse yaw


Comment/Reference:

2) for power operated control systems


Comment/Reference:

3) has the actuator rams co-axial


Comment/Reference:
www.tpub.com/content/engine/14105/css/14105_157.htm

4) To increase the drag to increase the rate of yaw and


turn
Comment/Reference:

5) ailerons and spoilers


Comment/Reference: Automatic Flight Control 4th ed pg
292. gust alleviation says spoilers only.

6) provide the pilot with simulated stick forces


Comment/Reference:

7) automatic and instantaneous


Comment/Reference:

8) maintain preset cable tensions during flight


Comment/Reference:

9) mechanical stops in the control system


Comment/Reference:

10) elevator
Comment/Reference: AC65-15A pg 44
Questions 1331 – 1340

1331.The aircraft is controlled about the normal axis by


the

 rudder
 elevator
 ailerons

1332.The purpose of the Servo Valve in a power operated


control is

 to provide pressure to operate the control


 to direct hydraulic fluid to the jack in response to
the pilot's control in cockpit
 to revert the system to manual operation

1333.A stick shaker is a device which

 gives a short period of extra lift to assist take


off
 vibrates the control column near stalling speed
 helps extricate an aircraft from soft ground

1334.The aircraft is controlled about the longitudinal axis


by the

 rudder
 elevator
 ailerons

1335.Ruddervators when moved, will move

 either opposite each other or together, depending on


the selection
 together only
 opposite to each other only
1336.As a consequence of the C of G being close to its aft
limit

 the stick forces will be high in fore and aft pitch,


due to the high longitudinal stability
 the stick forces to manoeuvre longitudinally will be
low due to the low stability
 the stick forces when pitching the nose down will be
very high

1337.An anti-balance tab is moved

 hydraulically
 when the C.G. changes
 via a fixed linkage

1338.A servo tab is operated

 by a trim wheel and moves in the opposite direction


to the main control surfaces when moved
 directly by the pilot to produce forces which in
turn move the main control surfaces
 automatically, and moves in the same direction as
the main control surfaces

1339.When checking full range of control surface movement,


they must be positioned by

 moving them by hand directly until against the


primary stops
 moving them by hand directly until against the
secondary stops
 operating the flightdeck/cockpit controls until the
system is against the primary stops

1340.The higher speed of the high wing in a banked turn


causes it to have more drag than the low wing, this may
be compensated for by

 use of the rudder control


 increasing the 'nose up' pitch by use of the
elevators
 operating the ailerons into the opposite sense once
the correct angle of bank has been reached
Answers for Questions 1331 – 1340

1) Rudder
Comment/Reference: AC65-15A pg 44.
vertical axis = normal axis

2) to direct hydraulic fluid to the jack in response to


the pilot's control in cockpit
Comment/Reference: Jeppesen A & P Technician Textbook
page 12-38

3) vibrates the control column near stalling speed


Comment/Reference: Aircraft Electricity & Electronics
5th ed Eisman pg 370 (Stall Warning)

4) ailerons
Comment/Reference: AC65-15A pg 41

5) either opposite each other or together, depending on


the selection
Comment/Reference: Jeppesen A&P Technician Airframe
Textbook Page 16 & AC65-15A pg 45

6) the stick forces when pitching the nose down will be


very high
Comment/Reference:

7) via a fixed linkage


Comment/Reference:

8) directly by the pilot to produce forces which in turn


move the main control surfaces
Comment/Reference: Jeppesen A&P Technician Airframe
Textbook Page 23

9) operating the flightdeck/cockpit controls until the


system is against the primary stops
Comment/Reference:

10) use of the rudder control


Comment/Reference:
Questions 1341 – 1350

1341.The type of flap which extends rearwards when lowered


is called a

 Fowler flap
 split flap
 plain flap

1342.Which of the following trailing edge flaps give an


increase in wing area?

 Split flap
 Slotted flap
 Fowler flap

1343.Which of the following is not a primary flying


control?

 Tailplane
 Rudder
 Elevator

1344.A tab which assists the pilot to move a flying control


by moving automatically in the opposite direction to
the control surface is called a

 geared balance tab


 servo tab
 trim tab

1345.What is fitted on the aircraft to enable the pilot to


reduce his speed rapidly in event of severe turbulence,
or speed tending to rise above the 'Never Exceed
Limit'?

 Lift dumpers
 Air brakes
 Wheel brakes
1346.When spoilers are used asymmetrically, they combine
with

 ailerons
 rudder
 elevators

1347.What is used to correct any tendency of the aircraft


to move towards an undesirable flight attitude?

 Trim tabs
 Balance tabs
 Spring tabs

1348.With hydraulic ram operated flying controls, when the


pressure is reduced to zero, the flying control
surfaces

 stay at neutral
 remain in the position they were when power was
removed
 droop

1349.Powered flying control actuators

 provide feedback to the pilot


 transmit pilot input to the control surfaces
 are operated by the autopilot only

1350.A stick shaker operates

 at the onset of the stall


 just before the stall
 after the stall
Answers for Questions 1341 – 1350

1) Fowler flap
Comment/Reference: AC65-15A pg 48

2) Fowler flap
Comment/Reference: AC65-15A pg 48

3) Tailplane
Comment/Reference: AC65-15A pg 40

4) geared balance tab


Comment/Reference: AC65-15A pg 46

5) Air brakes
Comment/Reference: AC65-15A pg 22-23

6) ailerons
Comment/Reference:

7) Trim tabs
Comment/Reference: AC65-15A 46

8) droop
Comment/Reference:

9) transmit pilot input to the control surfaces


Comment/Reference:

10) just before the stall


Comment/Reference: Flight Instruments and Automatic
Flight Control Systems Page 168
Questions 1351 – 1360

1351.In a power assisted flight control system, the feel


required is normally supplied by

 'Q' feel
 stretching springs
 air loads on the control surfaces

1352.Irreversibility of a fully powered control surface is


provided by

 nulling of the servo valve


 air loads on the control surface
 the jack ram being attached to structure

1353.'Q' feel units use

 pitot pressure only


 static pressure only
 pitot and static pressures

1354.Spoiler panels are positioned so that when deployed

 pitch trim is not affected


 roll will not occur
 no yaw takes place

1355.Powered flight control systems are used on transport


aircraft

 because the pilot has too many tasks to perform


 because at low level air density is high
 due to the increase in speed and size of aircraft
1356.A differential aileron control system results in

 aileron drag being reduced on the outer wing in a


turn
 aileron drag being compensated by small rudder
movements
 aileron drag being reduced on the inner wing in a
turn

1357.Spoiler panels are controlled by

 a speed brake lever only


 the aileron control wheel only
 the aileron control wheel and speed brake lever

1358.A lift transducer is normally fitted

 on the elevator control system


 to the nose area of the aircraft
 on the leading edge of the wing

1359.If electrical power is removed from a fly by wire


spoiler system, the spoiler panels

 remain where they are


 go to full up
 go to down if any are not already down

1360.In a fly-by-wire control system, the follow-up


transducer is fitted to

 the hydraulic actuator


 the control system computer
 the pilot's input
Answers for Questions 1351 – 1360

1) air loads on the control surfaces


Comment/Reference: AL/3-7 8.1 and 8.2.1 Power
'assisted' controls have aerodynamic feedback. Power
'operated' controls use a Q feel system

2) nulling of the servo valve


Comment/Reference: Nulling the servo valve produces a
hydraulic lock.

3) pitot and static pressures


Comment/Reference: Avionics Fundamentals pg 257 fig 17-
31

4) pitch trim is not affected


Comment/Reference:

5) due to the increase in speed and size of aircraft


Comment/Reference:

6) aileron drag being reduced on the outer wing in a turn


Comment/Reference:

7) the aileron control wheel and speed brake lever


Comment/Reference:

8) on the leading edge of the wing


Comment/Reference: Lift transducer, also known as a
stall sensor

9) go to down if any are not already down


Comment/Reference:

10) the hydraulic actuator


Comment/Reference:
Questions 1361 – 1370

1361.The down position of the speed brake lever is detented


to

 allow the ground speed brakes to be used on


touchdown
 lock the speed brakes down during flight
 prevent inadvertent operation

1362.Asymmetric flap operation is prevented to keep to a


minimum the tendency of

 yawing
 pitch changes
 rolling

1363.High thrust stall warning is activated when the thrust


of opposite engine reads

 greater than 50%


 greater than 90%
 greater than 75%

1364.A differential aileron control is one which gives

 a larger aileron up travel than down


 the down-going aileron more travel than the up-going
one
 equal aileron travel in each direction, but variable
for stick movement

1365.When the control column is moved back and the hand-


wheel turned to the left

 the left aileron moves up and the elevators move up


 the elevators move down and the left aileron moves
down
 the elevators move up and the left aileron moves
down
1366.A series type yaw damper

 moves the rudder and the ailerons to damp out Dutch


roll
 moves the rudder pedals and the rudder
 moves the rudder only

1367.A stick shaker

 vibrates the elevators to cause buffeting of the


airframe
 is an eccentric flywheel driven by an electric motor
and mounted on the control column
 is a linear actuator attached to the base of the
control column

1368.If the control column is moved forward and to the left

 left aileron moves up, right aileron down, elevator


down
 left aileron moves up, right aileron moves down,
elevator up
 left aileron moves down, right aileron up, elevator
down

1369.On an aircraft fitted with spoilers for lateral


control, roll to the right is caused by

 left spoiler extending, right spoiler remaining


retracted
 both spoilers extending
 right spoiler extending, left spoiler remaining
retracted

1370.When an aileron trim control in the cockpit is moved


to counteract a tendency to fly left wing low, an
aileron trim tab fitted to the port aileron will

 move up and cause the left aileron to move down but


the right aileron will remain neutral
 move up and cause the left aileron to move down and
the right aileron to move down to a lesser degree
 move up and this will cause the left aileron to move
down and the right aileron to move up
Answers for Questions 1361 – 1370

1) prevent inadvertent operation


Comment/Reference:

2) rolling
Comment/Reference:

3) greater than 75%


Comment/Reference: B737-3/4/500 AMM 27-32-00 pg 6 para
4.F.(3)

4) a larger aileron up travel than down


Comment/Reference:

5) the left aileron moves up and the elevators move up


Comment/Reference:

6) moves the rudder only


Comment/Reference:

7) is an eccentric flywheel driven by an electric motor


and mounted on the control column
Comment/Reference: AL/3-21 page 22 para 13.3.2 and
Transport Category Aircraft Systems Page 211

8) left aileron moves up, right aileron down, elevator


down
Comment/Reference:

9) right spoiler extending, left spoiler remaining


retracted
Comment/Reference:

10) move up and this will cause the left aileron to move
down and the right aileron to move up
Comment/Reference:
Questions 1371 – 1380

1371.You have adjusted the elevator trim tab to correct for


nose heavy. What was the direction of travel of the
trim tab?

 The elevator trim tab has moved up


 The port elevator tab has moved up and starboard
moved down
 The elevator trim tab has moved down

1372.Pulleys in a flight control system are made from

 stainless steel
 brass
 Tufnol or aluminium alloy

1373.A tension regulator in the flight control cable system


of a large all metal aircraft is used primarily to

 decrease cable tension in cold weather


 retain a set tension
 increase cable tension in cold weather

1374.An interrupter arm incorporated in a flying control


locking mechanism

 locks the ailerons and elevators


 restricts movement of the throttles
 dampens the rudder movement

1375.The yaw damper must be designed to allow the rudder to


control in response to the aircraft's

 roll frequency
 dutch roll frequency
 pitch Frequency
1376.The alpha vane signal can be fed to __________ when
the aircraft is close to stall

 flap position
 throttle lever
 fast/slow switch

1377.Moving the elevator trim wheel fully aft, then the

 aircraft tends to go nose down


 aircraft tends to roll to the right
 aircraft tends to go nose up

1378.Ground spoilers are used to

 increase wing loading on take-off


 prevent FOD ingestion into the engines on landing
and taxiing
 decrease wing loading on landing

1379.The outboard ailerons on some aircraft

 are isolated at slow speeds


 are isolated to improve sensitivity
 are isolated to prevent aileron reversal

1380.The effect of the rudder on an aircraft can be


increased by

 increasing the altitude of the aircraft


 increasing the distance of the control surfaces from
the aircraft's centre of gravity
 decreasing the velocity of the aircraft
Answers for Questions 1371 – 1380

1) The elevator trim tab has moved down


Comment/Reference:

2) Tufnol or aluminium alloy


Comment/Reference:

3) retain a set tension


Comment/Reference:

4) restricts movement of the throttles


Comment/Reference:

5) Dutch roll frequency


Comment/Reference:

6) throttle lever
Comment/Reference:

7) aircraft tends to go nose up


Comment/Reference:

8) decrease wing loading on landing


Comment/Reference:

9) are isolated to prevent aileron reversal


Comment/Reference:

10) increasing the distance of the control surfaces from


the aircraft's centre of gravity
Comment/Reference:
Questions 1381 – 1390

1381.A fuel crossfeed valve is normally powered by

 DC
 Battery
 AC

1382.A fuel crossfeed valve is usually in

 the open position


 the closed position
 its last position

1383.Sliding couplings in fuel systems are used on

 air or vapour systems


 low pressure systems
 high pressure systems

1384.When carrying out an insulation check on a fuel probe,


what is the Maximum voltage to use

 30 V
 250 V
 500 V

1385.A fuel cross-feed valve is lubricated

 by the fuel
 by internal cooling
 by air
1386.Fuel cross feed valves are operated by the

 115 V AC
 battery
 28 V DC bus

1387.Fuel systems that are interconnected must have

 air spaces interconnected


 separate venting for each tank
 no airspace

1388.As fuel level increases, system capacitance

 decreases
 no change
 increases

1389.When refueling

 it is important not to fill the expansion space in


the tank
 the fuel nozzle will automatically cut off the
supply when the tank is full
 it is important to fill the expansion space in the
tank

1390.When using Biopor microbial fuel growth treatment

 it is applied dry and left


 it is applied wet and flushed out
 it is added to the fuel and burnt
Answers for Questions 1381 – 1390

1) DC
Comment/Reference: 737 AMM Chapter 28-22-00 (Download
from the Tutorial Support section)

2) the closed position


Comment/Reference: Fuel crossfeed valves must be closed
for flight

3) air or vapour systems


Comment/Reference: CAIPs AL/3-17 5.8.3

4) 250 V
Comment/Reference: The fuel probe excitation is 115V
AC. As a rule of thumb, use an insulation tester 2x the
normal operating voltage.

5) by the fuel
Comment/Reference:

6) 28 V DC bus
Comment/Reference: 737 MM 28-22-00 Page 1, A320 FCOM
28.30 pg1

7) air spaces interconnected


Comment/Reference: CAIPs AL/3-17

8) increases
Comment/Reference: AC65-15A pg 486 & The dielectric
constant of kerosene is more than that of air. B757 MM
Chapter 28-41-00

9) it is important not to fill the expansion space in the


tank
Comment/Reference: Jeppesen A&P Technician Airframe
Textbook Page 15-18

10) it is added to the fuel and burnt


Comment/Reference: Jeppesen Aircraft Gas Turbine
Powerplants 7-3
Questions 1391 – 1400

1391.Microbial growth is

 brown black slime


 green sludge
 red dots

1392.Bladder type fuel tanks are secured by

 contact adhesive
 press type rivets
 bolts

1393.A zero fuel test on a fuel quantity indicating system


is carried out with

 tanks completely full


 tanks completely empty
 tanks empty apart from unusable fuel

1394.After a tank bag has been replaced, a fuel flow test


should be carried out at

 tank shut-off valve point


 tank outlet
 engine inlet

1395.In biologically contaminated fuel, corrosion takes


place

 the top of the tank


 the bottom 2 inches of the tank
 below the fungal colonies
1396.If an aircraft has ability to transfer fuel in flight,
provision must be made to

 shut off fuel before structural damage


 stop engine from using tank during transfer
 provide overflow facility to dump fuel

1397.In a fuel system with interconnected vents

 the expansion space must be 10% of the tank capacity


 the expansion space must be 2% of the tank capacity
 an expansion space is not required

1398.Fuel tanks interconnected must have

 protection against overfill for tanks and fuel


system
 vents to allow overfueling
 tank where fuel is fed from both tanks to engine
inlet

1399.When a FADEC system loses air data input it goes to

 hard reversion
 mechanical reversion
 soft reversion

1400.Pressure refuelling is carried out at

 40 PSI
 15 PSI
 20 PSI
Answers for Questions 1391 – 1400

1) brown black slime


Comment/Reference: AC65-9A pg 79 & Jeppesen A&P
Technician Airframe Textbook Page 15-38

2) press type rivets


Comment/Reference: AL/3-15 4.1.3 says press studs or
cords

3) tanks empty apart from unusable fuel


Comment/Reference: AL/10-3 11.14.1.e

4) engine inlet
Comment/Reference:

5) below the fungal colonies


Comment/Reference: AL/3-15 para 8.3

6) shut off fuel before structural damage


Comment/Reference: JAR 25.957

7) the expansion space must be 2% of the tank capacity


Comment/Reference: CS 25.969

8) protection against overfill for tanks and fuel system


Comment/Reference: CS 25.969

9) soft reversion
Comment/Reference: TTS FADEC Booklet Page 13

10) 40 PSI
Comment/Reference: AL/3-17 5.4 says 'up to 50 psi'
Questions 1401 – 1410

1401.Cross feed valves permit fuel transfer from

 left tank to right tank


 any tank to any engine
 tank to tank

1402.What is fitted to the fuel jettison pipe to prevent


vaporisation?

 A nozzle
 A corona device
 An anti-corona device

1403.What happens to the capacitance in a fuel contents


gauge as fuel is used?

 Decreases
 Depends on altitude
 Increases

1404.When an aircraft is put into storage for medium / long


periods, the fuel tanks should

 be full
 have some fuel left in to prevent sealant from
deteriorating/drying out
 be empty

1405.Integral fuel booster pumps

 require ram air cooling


 lets the fuel do cooling
 require no cooling
1406.The purpose of a scavenge system within a fuel system
is

 to ensure that all available fuel can be used


 to ensure that all unused fuel is returned to the
tanks efficiently
 to ensure that any water in the fuel is constantly
dispersed thus reducing microbiological
contamination

1407.In a light fixed wing aircraft, the fuel warning light


indicates

 ice blockage, filter popped or pump fail


 the tank does not pressurise
 pump failure

1408.How is fuel supplied to a turbine engine?

 By a fuel boost pump


 By suction from the engine driven fuel pump
 By a gravity feed pump

1409.A fuel tank suction valve will open when

 both the engine driven and booster pump fail


 the booster pump fails
 the engine driven pump fails

1410.To prevent water condensation in a fuel tank

 the tank should be filled with the minimum fuel


possible
 the tank should be filled as full as possible
 the filler cap should be left open
Answers for Questions 1401 – 1410

1) any tank to any engine


Comment/Reference: B737 MM 28-22-00 Page 1

2) An anti-corona device
Comment/Reference:

3) Decreases
Comment/Reference: AC65-15A pg 486 & Aircraft
Electricity & Electronics Eismin Page 353

4) have some fuel left in to prevent sealant from


deteriorating/drying out
Comment/Reference: AL/3-15 9.3

5) lets the fuel do cooling


Comment/Reference:

6) to ensure that any water in the fuel is constantly


dispersed thus reducing microbiological contamination
Comment/Reference: Transport Category Aircraft Systems
Page 6.17

7) ice blockage, filter popped or pump fail


Comment/Reference:

8) By a fuel boost pump


Comment/Reference:

9) the booster pump fails


Comment/Reference:

10) the tank should be filled as full as possible


Comment/Reference:
Questions 1411 – 1420

1411.At the lowest point of each vent line you would


normally find a

 float valve
 NACA duct
 self draining non-return valve

1412.To protect the centre tank fuselage area from a vapour


build up

 a shroud vent and overboard drain is fitted


 cabin air is bled around the tank
 ram air is circulated around the tank

1413.When a fuel jettison valve is in transit, the magnetic


indicator would show

 amber and cross line


 amber and in line
 green and in line

1414.When considering the basic categories of fuel tank


construction, which of the following is most prone to
leaks?

 Rigid
 Integral
 Flexible

1415.On removal of a fuel tank booster pump, fuel is


observed running out of the boost pump canister, this
could be caused by

 fuel leaking past the inlet screens


 fuel draining out of the pressure switch bleed
 the pressure outlet check valve is not correctly
seated
1416.Crossfeed valves are normally fitted with

 two 28V DC motors


 two single phase AC motors
 two three-phase AC motors

1417.To replace a water drain valve seal

 the seal is not replaceable, the whole drain


assembly must be replaced
 the tank must be de-fuelled
 the seal of the valve is replaceable without
draining the tanks

1418.A shroud vent and overload drain would normally be


found on

 the APU fuel feed


 fuel tank vent pipes
 fuel jettison system

1419.To shut-off the LP fuel supply to the engine for the


purpose of engine removal you would

 close the HP fuel lock


 pull the fire handle
 close the cross bleed valve

1420.A leak from a fuel tank which manifests as fuel


dripping from the tank is classified as a

 stain
 run
 seep
Answers for Questions 1411 – 1420

1) self draining non-return valve


Comment/Reference:

2) ram air is circulated around the tank


Comment/Reference: B737-3/4/500 AMM 28-13-00 pg 1

3) amber and cross line


Comment/Reference:

4) Integral
Comment/Reference:

5) the pressure outlet check valve is not correctly seated


Comment/Reference:

6) two 28V DC motors


Comment/Reference:

7) the seal of the valve is replaceable without draining


the tanks
Comment/Reference:

8) fuel tank vent pipes


Comment/Reference:

9) pull the fire handle


Comment/Reference: After first pulling the firebottle
CBs

10) run
Comment/Reference: AC65-9A pg 97 &
AL/3-15 page 8 para 5.4.1
Questions 1421 – 1430
1421.Calcium Chromate tablets can be fitted in integral
tanks. Before installation the linen bag should be
wetted with

 Methylated Spirit
 Kerosene
 Water

1422.A check valve is fitted to a fuel jettison system to

 prevent the centre from being defuelled


 automatically stop the fuel jettison operation after
a period of time
 prevent the dumping of the outer tanks

1423.The advantages of integral fuel tanks are that they

 are easier to design and construct and use the


maximum volume of fuel
 are cheaper to manufacture, more durable and easier
to maintain
 use the maximum volume for fuel and have the minimum
amount of weight

1424.On Large transport aircraft fuel is delivered to each


engine using

 the same system for each engine


 a parallel system
 a separate system for each engine

1425.A purpose of a 'cut-out' in a hydraulic system

 is to relieve the pump of load when the operation of


services is complete and the accumulator charged
with fluid
 is to limit loss of fluid in the event of pipe
fracture
 is to prevent creep in jack operated services which
have several selected positions
1426.Hydraulic pressure can be restored by

 the use of a pressure/heat exchanger


 compressing the air charge in an accumulator
 compressing the fluid in a reservoir

1427.An hydraulic accumulator has low air/nitrogen


pressure. What will be the indications?

 Zero pressure indicated


 The cut-out valve chatters
 Full pressure indicated

1428.What type of valve prevents pressure from building up


until a demand is placed in an open-center hydraulic
system?

 A selector valve
 A bypass valve
 An unloading valve

1429.In an open-centre hydraulic system, system pressure is


provided

 only when the selector valve is set to ON


 only when the selector valve is set to OFF
 continuously

1430.In an open-centre hydraulic system, selector valves


are positioned

 either in series or in parallel depending on the


system design
 in series
 in parallel
Answers for Questions 1421 – 1430

1) Water
Comment/Reference:

2) prevent the dumping of the outer tanks


Comment/Reference:

3) use the maximum volume for fuel and have the minimum
amount of weight
Comment/Reference:

4) a separate system for each engine


Comment/Reference: CS 25.953

5) is to relieve the pump of load when the operation of


services is complete and the accumulator charged with
fluid
Comment/Reference: CAIPs AL/3-21 5.2

6) compressing the air charge in an accumulator


Comment/Reference: CAIPs AL/3-21 Para 7

7) The cut-out valve chatters


Comment/Reference:

8) A selector valve
Comment/Reference: Jeppesen Airframe Textbook Chapter 8
page 15

9) only when the selector valve is set to ON


Comment/Reference: Avotek Aircraft System Maintenance
1-17 & Jeppesen A+P Technician Airframe Textbook 8-15

10) in series
Comment/Reference: Avotek Aircraft System Maintenance
1-17 & Jeppesen A+P Technician Airframe Textbook 8-15
Questions 1431 – 1440

1431.What is the colour of the band on a hydraulic seal?

 Green
 White
 Yellow

1432.A maintainer valve in a hydraulic system will

 maintain a high pressure to all systems


 allow continued press to essential systems during
times of reduced supply
 dampen pressure inputs

1433.Where is the temperature overheat switch located in a


hydraulic pump?

 In the case drain


 In the pressure line
 In the suction line

1434.What happens if a component has an internal hydraulic


leak?

 Increase in fluid pressure


 Fluid loss
 Increase in fluid temperature

1435.What happens to hydraulic fluid when it is overheated?

 Increase acidity
 Increase viscosity
 Increase alkalinity
1436.How is the hydraulic pump depressurising solenoid
actuated?

 Energised Closed for fail-safe


 Energised Open
 Energised both open and closed depending on cockpit
selection

1437.How do you prevent hydraulic fluid frothing?

 By pressurising
 Vent Reservoir to atmosphere
 Pass over a tray

1438.Accumulators as fitted to aircraft hydraulic systems

 are only ever used in an emergency


 provide additional fluid if leaks occur
 store fluid under pressure

1439.A Skydrol hydraulic fluid seal would be made of

 natural rubber
 butyl rubber, ethylene propylene or Teflon
 synthetic rubber

1440.Synthetic hydraulic fluid subjected to overheating is


high in

 viscosity
 acidity
 alkalinity
Answers for Questions 1431 1440

1) Green
Comment/Reference: AC65-9A pg 167 & Jeppesen A&P
Technician Airframe Textbook Page 8-44

2) allow continued press to essential systems during times


of reduced supply
Comment/Reference: Jeppesen A&P Technician Airframe
Textbook Page 8-31 Also known as priority valves CAIPs
AL/3-21 5.3

3) In the case drain


Comment/Reference: Jeppesen Transport Category Aircraft
Systems 7-10 and 7-8

4) Increase in fluid temperature


Comment/Reference: Increase in temperature is an
indication of internal leaks

5) Increase acidity
Comment/Reference: Hi acidity is a common problem
revealed after an oil system analysis

6) Energised Closed for fail-safe


Comment/Reference: The depressurising valve (sometimes
called an 'offloading valve') is energised closed, to
fail-safe open

7) By pressurising
Comment/Reference: Jeppesen A&P Technician Airframe
Textbook 8-16

8) store fluid under pressure


Comment/Reference: CAIPs AL/3-21 Para 7

9) butyl rubber, ethylene propylene or Teflon


Comment/Reference: CAIPs AL/3-21 Para 3.3.1

10) acidity
Comment/Reference: Overheating causes acidity
Questions 1441 – 1450

1441.A normal hydraulic system will

 show pressure and source of hydraulics


 show fluid temperature and quantity
 illuminate a light in the cockpit when the system is
ready

1442.A constant volume hydraulic system uses what to


relieve pressure in system when no services are being
used?

 ACOV (Automatic Cut Out Valve)


 Pressure relief valve
 Return line back to pump

1443.The control of hydraulically powered emergency


electrical generator frequency is by

 angle of swashplate
 restriction valve
 IDG

1444.Hydraulic systems normally operate at

 300 PSI
 1800 PSI
 3000 PSI

1445.When flushing hydraulic systems

 flush with any hydraulic oil


 flush with methylated spirit
 flush with same hydraulic oil
1446.When replenishing a hydraulic system

 use the same/correct hydraulic fluid


 use any hydraulic fluid
 use any hydraulic fluid made by the same
manufacturer

1447.When replacing a flexible pipe

 tighten only hand tight


 do a bonding check
 use two spanners to stop pipe from twisting

1448.How do you remove air from hydraulic fluid?

 Pass through a restriction


 Vent to atmosphere
 Pass over a tray

1449.A pressure control relay in a hydraulic pump is

 energised to the run position


 used to prevent anything happening until there is
hydraulic pressure
 de-energised to start the pump

1450.A constant delivery pump has its output varied with

 servo pressure
 regulators
 gears
Answers for Questions 1441 – 1450

1) show pressure and source of hydraulics


Comment/Reference: Some systems show temperature, but
they are not 'normal'.

2) ACOV
Comment/Reference: CAIPs AL/3-21 12.2

3) angle of swashplate
Comment/Reference:

4) 3000 PSI
Comment/Reference: Transport Category Aircraft Systems
Jeppesen Pages 7-5 & 7-7

5) flush with same hydraulic oil


Comment/Reference:

6) use the same/correct hydraulic fluid


Comment/Reference: Transport Category Aircraft Systems
Jeppesen Page 7-2

7) use two spanners to stop pipe from twisting


Comment/Reference: CAP 562 Leaflet 5-5 para 6.8.1

8) Pass over a tray


Comment/Reference:

9) energised to the run position


Comment/Reference:

10) regulators
Comment/Reference: Constant delivery is constant flow
rate.
Questions 1451 – 1460

1451.A variable angled pump starts at

 half way position


 minimum stroke
 maximum stroke

1452.A check flow valve in a hydraulic system prevents

 reverse flow
 overpressure
 pump cavitation

1453.A duplex seal consist of

 a hard rubber square section ring with a soft rubber


square section inner ring
 a rubber 'T' section ring sandwiched between two
Teflon rings
 a square section rubber ring within a steel washer

1454.When is a hydraulic de-pressurising valve energised?

 When there is sufficient hydraulic pressure


available
 When selected on
 When selected off

1455.A fixed volume axial pump contains the following


rotating parts:

 Drive Shaft, variable Swash Plate & Cylinder Block


 Drive Shaft, fixed Swash Plate & Valve Block
 variable Swash Plate & Valve Block
1456.An hydraulic closed system is one where

 pressure is maintained at the selector valves at all


times
 most of the major components of the system are
included in a self contained unit
 there is no pressure at the selector valves when no
functions are required

1457.What constitutes a hydraulic power pack?

 Hydraulic oil accumulator


 Reservoir, pump, selector valve, filter
 Variable swashplate pump

1458.If an accumulator loses air pressure there will be

 pump cavitation
 rapid fluctuations of instruments
 rapid depressurisation of system

1459.A marker number on a pipe indicates

 the specification of the pipe throughout the pipe


run
 the thickness of the wall on the pipe
 the type of fluid used to pressure test the pipe

1460.One reason for fitting an accumulator in a hydraulic


system is to

 relieve excess pressure


 minimize the possibility of pump cavitation
 absorb pressure surges
Answers for Questions 1451 – 1460

1) maximum stroke
Comment/Reference: CAIPs AL/3-23

2) reverse flow
Comment/Reference:

3) a hard rubber square section ring with a soft rubber


square section inner ring
Comment/Reference:

4) When selected off


Comment/Reference: AL/3-21 4.2

5) Drive Shaft, fixed Swash Plate & Valve Block


Comment/Reference: AL/3-21 Figure 2

6) pressure is maintained at the selector valves at all


times
Comment/Reference: AL/3-21 12.1 - 12.3

7) Reservoir, pump, selector valve, filter


Comment/Reference: Jeppeson A & P Technician Airframe
P8-15

8) rapid fluctuations of instruments


Comment/Reference:

9) the specification of the pipe throughout the pipe run


Comment/Reference:

10) absorb pressure surges


Comment/Reference: AC65-15Apg 324
Questions 1461 – 1470

1461.A thermal relief valve is fitted to a hydraulic system


to

 prevent a leak back of pressure


 prevent excess temperature
 relieve excess pressure

1462.A component in a hydraulic system which ensures


instantaneous action when a service is selected is

 selector
 engine driven pump
 accumulator

1463.A shuttle valve is used for

 change over from main to auxiliary system in the


case of failure
 maintaining fluid press when the emergency system
fails
 preventing fluid loss from a leaking jack

1464.Restrictor valves in a hydraulic system are used to

 limit the maximum pressure


 restrict the rate of pressure build up
 control the rate of system operation

1465.The purpose of a mechanical sequence valve is to


ensure the operation of

 brake anti-skid units


 landing gear and doors
 safety switches
1466.The purpose of a non-return valve is to

 restrict the movement of components


 isolate one component from another
 direct fluid to the hydraulic actuator

1467.The end fittings on a NRV are normally different to

 allow a full flow from the valve


 facilitate bleeding of the system
 prevent incorrect installation

1468.A hydraulic accumulator is charged with initial air


pressure of 1000 PSI . When the hydraulic system
pressure of 3000 PSI is reached, the air pressure is

 3000 PSI
 1000 PSI
 4000 PSI

1469.Automatic cut-out valve hammering is caused by

 moisture in the fluid


 low accumulator pressure
 relief valve set too high

1470.Hand pumps are normally single cylinder double acting


because of the

 piston ram displacement


 relief valve
 two NRVS fitted
Answers for Questions 1461 – 1470

1) relieve excess pressure


Comment/Reference: Thermal relief valves relieve the
pressure build-up due to excessive temperature

2) accumulator
Comment/Reference:

3) change over from main to auxiliary system in the case


of failure
Comment/Reference: AL/3-21 6.4

4) control the rate of system operation


Comment/Reference: AL/3-21 6.2

5) landing gear and doors


Comment/Reference:

6) direct fluid to the hydraulic actuator


Comment/Reference: AL/3-21 Page 10

7) prevent incorrect installation


Comment/Reference: AL/3-21 Page 10

8) 3000 PSI
Comment/Reference:

9) low accumulator pressure


Comment/Reference:

10) two NRVS fitted


Comment/Reference: AC65-15A pg 318
CAIPs AL/3-21 Page 6
Questions 1471 – 1480

1471.During a leak test of a hydraulic system, system


pressure

 must be at maximum
 is not important
 must be minimum working

1472.Constant delivery hydraulic pumps

 are self idling


 require an automatic cut-out valve and accumulator
in the system
 require an automatic cut-out valve in the system

1473.Mechanical sequence valves are operated by

 the operation of a NRV


 the pilot
 a striker pin

1474.Pressure relay valves are fitted

 before a pressure sequence valve


 before a pressure relief valve
 before a pressure gauge

1475.Before checking the initial pressure in the hydraulic


accumulator

 the reservoir must be checked for correct level


 all air must be bled from the system
 the fluid pressure must be released
1476.Hydraulic reservoirs are pressurized to

 maintain a constant fluid level


 minimize the possibility of pump cavitation
 provide a reserve of stored energy

1477.In aircraft hydraulic systems, it is permissible to


use

 any hydraulic fluid available


 only the specified fluid
 any fluid of the same specific gravity

1478.The essential components of a hydraulic system of the


constant delivery type are an

 accumulator and relay valve


 accumulator and automatic cut-out valve
 accumulator and shuttle valve

1479.An automatic cut-out valve will

 limit pump wear


 raise fluid boiling point
 prevent the hydraulic lock forming

1480.A maintaining valve is fitted to a hydraulic system

 to ensure a supply of fluid to essential services in


the event of system failure
 to maintain a back pressure to the accumulator
 relieve excessive pressure to return
Answers for Questions 1471 – 1480

1) must be at maximum
Comment/Reference:

2) require an automatic cut-out valve and accumulator in


the system
Comment/Reference:

3) a striker pin
Comment/Reference:

4) before a pressure gauge


Comment/Reference: AL/3-21 13.3.2

5) the fluid pressure must be released


Comment/Reference:

6) minimize the possibility of pump cavitation


Comment/Reference: CAIPs AL/3-21 Para 15 Page 7

7) only the specified fluid


Comment/Reference:

8) accumulator and automatic cut-out valve


Comment/Reference:

9) prevent the hydraulic lock forming


Comment/Reference:

10) to ensure a supply of fluid to essential services in


the event of system failure
Comment/Reference:
Questions 1481 – 1490

1481.A thermal relief valve

 senses fluid temperature change


 provides cooling for the fluid
 senses fluid pressure

1482.Hydraulic systems are normally flushed with

 the same type of fluid as used in the system


 methylated spirits
 any clean hydraulic fluid

1483.Phosphate Ester hydraulic fluid requires which kind of


seals?

 Synthetic Rubber
 Natural Rubber
 Butyl Rubber

1484.A RAT hydraulic pump is for

 flying controls if hydraulic power lost


 landing gear and auto-brake systems
 nose wheel steering

1485.Hydraulic hand-pump fluid supply is taken from

 a stack pipe higher than the normal level


 the bottom of the reservoir
 a tap into a convenient return line
1486.Hand pumps in hydraulic systems are normally

 double acting
 single acting
 low pressure

1487.To enable a hydraulic system to operate two components


requiring different pressures

 a pressure reducing valve is used


 a pressure relief valve is used
 a pressure regulating valve is used

1488.The force exerted by a hydraulic cylinder is equal to

 area x volume
 area x pressure
 pressure x stroke

1489.A tandem PFCU

 has the actuator rams co-axial


 has the rams parallel
 has two control surfaces under its control

1490.The reason why slack must be left in a flexible hose


during installation, is a flexible hose under pressure

 contracts in length and diameter


 expands in length and diameter
 contracts in length and expands in diameter
Answers for Questions 1481 – 1490

1) senses fluid pressure


Comment/Reference: A&P Technician Airframe Textbook 8-
14

2) the same type of fluid as used in the system


Comment/Reference:

3) Butyl Rubber
Comment/Reference: AL/3-21 3.3

4) flying controls if hydraulic power lost


Comment/Reference:

5) the bottom of the reservoir


Comment/Reference: AL3-21 8.1

6) double acting
Comment/Reference: Jeppeson A & P technician Airframe
P8-23.

7) a pressure reducing valve is used


Comment/Reference:

8) area x pressure
Comment/Reference:)

9) has the actuator rams co-axial


Comment/Reference:
www.tpub.com/content/engine/14105/css/14105_157.htm

10) contracts in length and expands in diameter


Comment/Reference: A&P Mechanics General Handbook page
115 & CAP 562 Leaflet 5-5 para 6
Questions 1491 – 1500

1491.An automatic cut-out valve is necessary in a hydraulic


system with

 a fixed volume pump


 a variable volume pump
 pressures above 3000 PSI

1492.A pressure operated ice detector would

 be completely covered in ice before causing an alarm


to sound on the flight deck
 have a build up of ice that causes a torque switch
to illuminate a flight deck annunciator
 have a build up of ice on the leading edge that
causes a warning light to illuminate on the flight
deck

1493.Ice formed on the object or surface will change its


shape, this changes is more detrimental and vulnerable
to a

 large object
 small object
 fast moving object

1494.In which ice detection method, the activation of


warning system is through accumulation of ice on a
probe projected into the airstream?

 Impingement method
 Inferential method
 Ice accretion method

1495.The inflatable tube of the de-icer boots is made of

 synthetic rubber
 rubberised fabric
 natural rubber
1496.When testing windshield wipers, you should

 operate them on a dry windshield


 lift the wipers away from the windshield
 use a continuous flow of water on the windshield

1497.What is the correct way of measuring the windshield


wiper force prior to adjustment?

 By measuring the maximum total angle of sweep


 With a torque screwdriver and light thumb pressure
 With a spring tension scale

1498.How is a serrated rod ice detector bench tested?

 By a motor load test


 Motor test and go/no-go gap measurement
 Screwdriver torque test

1499.Windshield heating provides

 increases strength to resist cabin pressure


 impact resistance enhancement
 thermal expansion for a tighter fit

1500.At what pressure and temperature is anti ice fluid


applied?

 70°F at 10 PSI
 7°F at 100 PSI
 70°C at 100 PSI
Answers for Questions 1491 – 1500

1) a fixed volume pump


Comment/Reference:

2) have a build up of ice on the leading edge that causes


a warning light to illuminate on the flight deck
Comment/Reference:

3) small object
Comment/Reference: AL/11-6 page 1

4) Ice accretion method


Comment/Reference: CAIP AL/11-6 page 1

5) rubberised fabric
Comment/Reference: AC65-15A pg 287 & AL/11-1 page 4

6) use a continuous flow of water on the windshield


Comment/Reference:

7) With a spring tension scale


Comment/Reference: B737 AMM 30-42-11

8) Screwdriver torque test


Comment/Reference: 'Retard rotation [of the detector]
with slight thumb pressure using a torque screwdriver.'
Ref: CAIPs AL/11-6

9) impact resistance enhancement


Comment/Reference: Jeppesen A&P Technician Airframe
Textbook Page 13-6 (last paragraph)

10) 70°C at 100 PSI


Comment/Reference: CAIPs AL/11-3 5.2.2
Questions 1501 – 1510

1501.The advantage of leading edge fluid de-icing is

 more efficient
 aerodynamic shape is not disturbed
 less of a fire hazard

1502.On large transport aircraft, the windshield wiper


system is

 one system for both sides but with the same power
source
 independent on each side but with the same power
source
 independent on each side with different power
sources

1503.Prior to securing a leading edge de-icing boot you


must

 remove all paint


 shave rivet to prevent further abrasion
 paint the surface

1504.On a 'hot rod' type of ice detector, it is switched on

 when selected by the crew


 when in the air
 all the time

1505.Ice formation on wings is due to

 suspended ice crystals melting on contact with the


wing and instantly re-freezing
 ice crystals forming layers on contact with the wing
 supercooled water changing state on contact with the
wing
1506.When a vibrating rod ice detector has de-iced, the
warning lamp on the flight deck

 goes out after one more cycle


 goes out immediately
 goes out after a set period of time

1507.De-misting of passenger windows is provided by

 sealed window
 air from the cabin
 an electrical heating element

1508.A deicer boot is completely bonded to the leading edge


to

 provide more efficient deicer cycles


 prevent electrical static build up
 provide smoother airflow over leading edge

1509.In a cockpit window heater system, the autotransformer

 supplies AC Power for heating


 supplies DC power for heating
 steps up output for severe weather conditions

1510.Windshield rain repellent is applied

 when rain is on windows and spread by wipers


 when in heavy rain so vision is unobscured
 before rain and spread on window surface by wipers
Answers for Questions 1501 – 1510

1) aerodynamic shape is not disturbed


Comment/Reference: The advantage of leading edge fluid
de-icing is aerodynamic shape is not disturbed

2) independent on each side with different power sources


Comment/Reference: Aircraft Electrical Systems,
Pallett, Page 178

3) remove all paint


Comment/Reference: CAIPs AL/11-1 4.2.2

4) when selected by the crew


Comment/Reference: AL/11-6 3.2.1

5) supercooled water changing state on contact with the


wing
Comment/Reference: AL/11-6 2

6) goes out after a set period of time


Comment/Reference: CAIPs AL/11-6 3.4.1

7) air from the cabin


Comment/Reference: CAIPs AL/3-24 fig 1

8) prevent electrical static build up


Comment/Reference: CAIPs AL/11-1 3.4 (unless they mean
'bonded' as in cemented. In which case the answer is b)

9) steps up output for severe weather conditions


Comment/Reference: Aircraft Electrical Systems Pallett
Page 62/63 (Note: a transformer cannot 'supply' power)

10) when rain is on windows and spread by wipers


Comment/Reference: www.b737.org.uk/iceandrain.htm
Jeppesen, A&P Airframe Textbook Page 13-1
Questions 1511 – 1520

1511.Pneumatic rain removal systems

 use a pneumatic motor to drive windscreen wipers


 use engine bleed air at high velocity to remove
water droplets from windscreen
 are not permitted on large transport aircraft

1512.Windscreen wiper torque tests are carried out at

 the shaft end


 centre point of the blade
 blade attachment end

1513.Windscreen autotransformers

 are used to supply extra current under difficult


conditions
 step up voltage
 step down voltage

1514.An ice deposit formed when liquid water flows over the
airframe before freezing, and which is dense, tough and
sticks closely to the surface is called

 hoar Frost
 glaze Ice
 rime Ice

1515.When testing pitot head heaters

 they can only be checked by noting the temperature


rise of the probe
 they must only be switched on for the minimum time
required to check serviceability
 they should be switched on for five minutes to allow
to stabilise before taking ammeter readings
1516.A rotary knife edge ice detector provides warning of
ice by

 decreased torque caused by ice formation slowing the


rotating wheel and illuminating a warning light in
the cockpit
 ice formation stopping the rotation of a rotary
knife edge and illuminating a warning light in the
cockpit
 increased torque caused by ice formation slowing the
rotating wheel and illuminating a warning light in
the cockpit

1517.Windscreen heating is supplied from

 frequency wild generator, via a rectifier


 DC generator, via a transformer
 frequency wild generator, direct to the windscreen

1518.A wing thermal anti-ice annunciator is illuminated


permanently in flight deck without selection being
made. The most probable cause would be

 normal
 a short circuit
 an open circuit

1519.Air for anti-icing of the wings is obtained from

 a combustion heater
 engine compressors
 air conditioning ducting

1520.The usual material for pipelines in a fluid deice


system

 stainless steel
 nylon
 Monel
Answers for Questions 1511 – 1520

1) use engine bleed air at high velocity to remove water


droplets from windscreen
Comment/Reference: Jeppesen A & P Technician Airframe
Textbook page 13-18

2) the shaft end


Comment/Reference:

3) step up voltage
Comment/Reference: AL/11-4 4.2.2

4) glaze Ice
Comment/Reference: www.tpub.com/weather2/5-8.htm
FAA AC 20-147 & AC65-15A pg 285

5) they must only be switched on for the minimum time


required to check serviceability
Comment/Reference:

6) increased torque caused by ice formation slowing the


rotating wheel and illuminating a warning light in the
cockpit
Comment/Reference: Aircraft Electrical Systems,
Pallett, page 174

7) frequency wild generator, direct to the windscreen


Comment/Reference: Aircraft Electrical Systems,
Pallett, page173, fig.10-29

8) a short circuit
Comment/Reference:

9) engine compressors
Comment/Reference:

10) nylon
Comment/Reference:
Questions 1521 – 1530

1521.When operating a windscreen wiper on the ground, make


sure to

 place soft cloth between blade and window


 use slow wiper only
 use water as lubricant when operating

1522.What is run-back ice?

 Glime ice
 Rime ice
 Glaze ice

1523.Spongy brakes are usually a result of

 air in the system


 internal leakage
 external leakage

1524.Over inflated tyres may cause damage to the

 wheel hub
 wheel flange
 brake drum

1525.A tyre valve cap is used to

 prevent a leak
 secure the valve by screwing up to the shoulder of
the wheel
 secure the valve by screwing up to the valve base
1526.How is a wheel and tyre assembly stored?

 Three quarters of full pressure


 Inflated to 20-30 PSI
 With full pressure

1527.The operational hydraulic pressure for the nose wheel


steering system of large aircraft normally comes from

 the landing gear down line


 an independent hydraulic system
 the landing gear up line

1528.When removing the wheel with a multiple disks brake,


the wheel brake should be

 set OFF (released)


 disconnected first
 set ON (applied)

1529.Twin contact tread tyres are fitted to the nose wheel


to assist in

 deflecting water away from rear mounted engines


 preventing aquaplaning
 preventing nose wheel shimmy

1530.A brake debooster valve is provided for

 decreasing the pressure and slowly releasing the


brakes
 increasing the pressure and applying the brakes
rapidly
 applying brake pressure slowly and releasing the the
brakes quickly
Answers for Questions 1521 – 1530

1) use water as lubricant when operating


Comment/Reference: CAAIPs Leaflet 6-8 para 5.1.2

2) Glaze ice
Comment/Reference: FAA AC20-147

3) air in the system


Comment/Reference: CAAIPs 5-8 Jeppesen A&P Technician
Airframe Textbook Page 9-31

4) wheel flange
Comment/Reference: Jeppesen A&P Technician Airframe
Textbook Page 1-32

5) prevent a leak
Comment/Reference:

6) Inflated to 20-30 PSI


Comment/Reference: CAP 562 Leaflet 5-7 para 15.4

7) the landing gear down line


Comment/Reference: B737-3/4/500 AMM 32-51-00 Figure 2
(Sheet 1) pg 4 & pg 11 para 6.A. & AL/3-6 page 15

8) set ON (applied)
Comment/Reference: AL/3-19 para 3.2

9) preventing nose wheel shimmy


Comment/Reference: AL/3-18 page 3

10) applying brake pressure slowly and releasing the brakes


quickly
Comment/Reference:
www.tpub.com/content/aviation/14018/css/14018_474.htm
Questions 1531 – 1540

1531.Anti-skid systems become deactivated at

 15-20 mph
 10-15 mph
 5-10 mph

1532.The landing gear is in the down and locked position


but the red light is still on. The most probable fault
is

 the landing gear up microswitch needs adjusting


 there is a short circuit in the microswitch
 the landing gear down microswitch needs adjusting

1533.Tubeless tyres are stored

 vertically
 horizontally maximum of four high with smallest
diameter tyre on top
 at 15 to 20 P.S.I.

1534.On a wheel bogie unit, positive camber is when the

 bottom of the wheels are closer together


 front of the wheels are closer together
 top of the wheels are closer together

1535.A badly corroded and pitted brake disk will

 produce a loss of fluid


 be serviceable, and provide better grip
 wear brake pads too quickly, and should be replaced
1536.How much oxygen is in a tyre (as a maximum)?

 15% by volume
 5% by pressure
 5% by volume

1537.A tyre specification 32 x 10.75 - 14. What does the


10.75 refer to?

 Section Width
 Bead diameter
 Overall diameter

1538.When servicing an oleo, fluid comes out of the air


valve when the leg is depressurised?

 The leg is serviceable - the separator has bottomed


 Separator plate stuck open
 Leaking air/oil seals

1539.The type of fluid in an oleo strut will depend upon

 type of fluid most readily available


 the type of seal material
 heat generated in operation of the system

1540.A restrictor valve

 restrict the extent of travel of the undercarriage


 may be used to slow down undercarriage extension
 speed up the flow in one direction
Answers for Questions 1531 – 1540

1) 15-20 mph
Comment/Reference: Avotek Aircraft System Maintenance
3-69

2) the landing gear down microswitch needs adjusting


Comment/Reference: Aircraft Electrical Systems, Pallett
Page 176 & B737 AMM 32-45-00 Page 9 and Figure 6

3) vertically
Comment/Reference: Jeppesen A&P Technician Airframe
Textbook Page 9-47 & CAP 562 Leaflet 5-7 para 15.2.1

4) bottom of the wheels are closer together


Comment/Reference: Jeppesen A&P Technician Airframe
Textbook Page 9-13

5) wear brake pads too quickly, and should be replaced


Comment/Reference: Badly corroded brakes should be
replaced

6) 5% by volume
Comment/Reference: CAP 747 Appx 1 GR No. 16 para 3.1 &
CS 25.733

7) Section Width
Comment/Reference: CAP 562 Leaflet 5-7 para 3.2

8) The leg is serviceable - the separator has bottomed


Comment/Reference: CAIPs AL/3-6

9) heat generated in operation of the system


Comment/Reference: The type of fluid in an oleo strut
will depend upon heat generated in operation of the
system

10) may be used to slow down undercarriage extension


Comment/Reference: CAIPs AL/3-6 Page 7&8
Questions 1541 – 1550

1541.The function of the oil in a shock absorber is to

 damp the rebound


 absorb the landing shock
 ensure the separator does not bottom

1542.The purpose of the flutter plate in a shock absorber


is to

 permit free flow of oil during compression and


restriction of oil during extension
 separate the oil from the air
 restrict the compression of the air

1543.Made up wheels should be stored

 horizontal no more than 4 high


 vertical at 20/30 PSI charge
 vertical at working pressure

1544.After MLG down and securely locked, a red light is


illuminated in the flight deck. The possible causes are

 shorted sensor
 wiring problem
 out of adjustment sensor

1545.When checking for alignment of a MLG, check

 symmetry, twist, tracking


 symmetry, tracking, camber
 symmetry, splay, tracking
1546.These markings are found on a tyre
32 x 10.45 R 14.
What does the number 32 mean?

 Inner diameter
 Outer diameter
 Width

1547.A green/grey dot marking on aircraft tyre casing


represents

 military reference
 breather points
 the light part of the tyre

1548.In detachable flange type wheels, the flange is


secured by

 a lock ring
 bolts
 a retainer plate

1549.Fuse plugs in aircraft tyres are

 for overpressure protection


 to deflate the tyre before removal
 for over temperature protection

1550.A breaker strip in an aircraft tyre is

 to provide a wear indication


 to indicate the position for tyre levers
 to provide strength
Answers for Questions 1541 – 1550

1) damp the rebound


Comment/Reference: Jeppesen A&P technician Airframe
Textbook Page 9-5

2) permit free flow of oil during compression and


restriction of oil during extension
Comment/Reference:

3) vertical at 20/30 PSI charge


Comment/Reference: CAP 562 Leaflet 5-7 para’s 15.2.1 &
15.4

4) out of adjustment sensor


Comment/Reference: Aircraft Electrical Systems Pallett
Page 176

5) symmetry, tracking, camber


Comment/Reference: A+P Technician Airframe Textbook
Page 9-13

6) Outer diameter
Comment/Reference: CAP 562 Leaflet 5-7 para 3.2

7) breather points
Comment/Reference: CAIPs AL/3-18 3.6, CAAIPs Leaflet 5-
7 2.4

8) a lock ring
Comment/Reference: CAP 562 Leaflet 5-7 para 4.8.1 &
AC65-15A pg 375

9) for overtemperature protection


Comment/Reference: B737-3/4/500 AMM 32-45-00 para 2.B.
(6) pg 2 & AL/3-19 3.1.2 Fuse plugs protect the tyre
from explosion due to overpressure, albeit the
overpressure as a result of temperature rise

10) to provide strength


Comment/Reference:
www.dunlopaircrafttyres.com/tyrecare/dm1172/dm1172.pdf
Questions 1551 – 1560

1551.If fluid leaks along with air whilst oleo charging,


this is

 normal
 caused by a leaky seal
 due to excessive charging pressure

1552.A red dot / triangle mark on aircraft tyres means

 the light part of the tyre


 military reference
 breather points

1553.On selection of MLG down, the gear extends then begins


to retract again. The cause is

 a faulty actuator
 a faulty selector valve
 gear lowered at too high an airspeed

1554.When fitting a tyre, the red dot should be positioned

 on the other side of the wheel opposite to the


charging valve
 adjacent to the charging valve
 opposite the charging valve

1555.Composite brake units

 weigh the same as normal brake units and fade away


at high temperatures
 have less weight than normal brake units but fade
away at high temperatures
 have less weight than normal brake units and have
increased efficiency at high temperatures
1556.A restrictor valve can be used

 to reduce the speed of the undercarriage extension


 to increase the speed of undercarriage retraction
 to increase the speed of the undercarriage extension

1557.When an undercarriage is lowered, it tends to creep


back up. The fault could be

 a leaky selector valve


 a stuck relief valve
 the emergency system

1558.Vents holes are found on

 tubeless tyres
 tubed tyres
 tubed and tubeless tyres

1559.At what temperature does a yellow fuse plug melt at?

 250°C
 150°C
 200°C

1560.The cam plate in a nose undercarriage is

 to align for nose wheel on nose undercarriage


retraction
 part of the shimmy damper
 for alignment of the nose wheel steering on nose
undercarriage extension
Answers for Questions 1551 – 1560

1) caused by a leaky seal


Comment/Reference: AL/3-6 3.3.3 iii

2) the light part of the tyre


Comment/Reference: CAIPs AL/3-18 3.6, CAAIPs Leaflet 5-
7 3.7

3) a faulty selector valve


Comment/Reference:

4) adjacent to the charging valve


Comment/Reference:
www.dunlopaircrfttyres.com/tyrecare/dm1172/dm1172.pdf
Dunlop Aircraft Tyres General Servicing Instructions
Para 3.2.1.10

5) have less weight than normal brake units and have


increased efficiency at high temperatures
Comment/Reference: A&P Technician Airframe Textbook
Page 9-20 'Carbon Brakes'

6) to reduce the speed of the undercarriage extension


Comment/Reference: CAIPs AL/3-6 4.2

7) a leaky selector valve


Comment/Reference:

8) tubed and tubeless tyres


Comment/Reference: AL/3-18 2.4. A&P Technician Airframe
Textbook Page 9-42 says both tubed and tubeless. &
Dunlop General Servicing Instructions

9) 200°C
Comment/Reference: Boeing 757 Carbon Brake wheel,
Description and Operation 32-42-82 Part AHA1648 -
Yellow Plug Temperature 390°F

10) to align for nose wheel on nose undercarriage


retraction
Comment/Reference:
Questions 1561 – 1570

1561.What is tyre creep related to?

 Tyre moving around the wheel


 Horizontal movement of the tyre
 Vertical movement of the tyre

1562.'Aquaplaning' can be reduced by

 an anti-skid device
 lowering slats
 increased flaring

1563.Why is a hydraulic damper fitted to a nose wheel


steering system?

 To centralise the nose leg assembly during an up


selection
 To centralise the nose wheel during an up selection
 To reduce vibration and shimmy

1564.What decides the type of oil used in an undercarriage


leg?

 The material of the leg


 Neither, any oil can be used
 The types of seals the leg uses

1565.In an anti-skid system

 brakes are modulated to give most efficient braking


 brakes release on rising torque
 brakes release on falling torque
1566.If an undercarriage oleo has insufficient deflection,
the likely cause is

 oil level too high


 air pressure too high
 loss of pressure/leakage

1567.A Maxaret is used in what system?

 Self centring landing gear


 Tyre inflation
 Skid control

1568.The pilot receives an audible warning on the flight


deck as the aircraft is descending to land. The most
likely reason for this warning is

 the airspeed is too high


 the landing gear is not locked down
 the rate of descent is too high

1569.A restrictor in the landing gear hydraulic retraction


and lowering system is positioned in the

 landing gear down line


 return line from the landing gear selector
 landing gear up line

1570.Wheel speed information is used in auto speed brake


systems

 to ensure the aircraft is still rolling


 to ensure the aircraft has touched down and to give
a slight time lapse before spoiler deployment
 to stow the spoilers automatically when the aircraft
has nearly stopped
Answers for Questions 1561 – 1570

1) Tyre moving around the wheel


Comment/Reference: CAP 562 Leaflet 5-7 Para 9

2) an anti-skid device
Comment/Reference:

3) To reduce vibration and shimmy


Comment/Reference:

4) The types of seals the leg uses


Comment/Reference:

5) brakes are modulated to give most efficient braking


Comment/Reference: Jeppesen A&P Technician Airframe
Textbook Page 10-37

6) air pressure too high


Comment/Reference:

7) Skid control
Comment/Reference:

8) the landing gear is not locked down


Comment/Reference: Jeppesen A & P Airframe Technician
Textbook page 10-12

9) landing gear up line


Comment/Reference: The restrictor is placed in the
retraction line to slow the rate of exit of fluid from
the jack on extension, to slow the rate of extension.
CAIPs AL/3-6 fig 4 shows them in the down line - this
is a well known error in CAIPs

10) to ensure the aircraft has touched down and to give a


slight time lapse before spoiler deployment
Comment/Reference:
Questions 1571 – 1580

1571.A fusible plug fitted to an aircraft wheel prevents

 tyre bursts due to high temperatures created by


excessive braking
 over inflation of tyres during servicing
 brakes seizure, if brake stators and rotors weld
together under excessive temperatures

1572.If the shuttle valve was stuck in the normal position

 normal braking will not be available


 neither system will be available
 emergency braking will not be available

1573.The breaker strip of a tyre is fitted

 between the bead and the casing piles


 between the tread rubber and the casing piles
 between the apex strip and the casing piles

1574.Nose wheel steering in a modern aircraft is by

 a separate pilot operated control


 the control column
 rudder pedals

1575.The fusible plugs installed in some aircraft wheel


will

 eliminate the need to check air pressure


 facilitate servicing of the wheel assembly
 melt at a specified elevated temperature
1576.Excessive wear in the shoulder area of an aircraft
tyre is an indication of

 excessive toe-in
 under-inflation
 over-inflation

1577.Ribbed tyres with marker tie bars may be worn to

 the base of the tie bar


 the top of the tie bar
 1 mm from the rib base

1578.On large aircraft, bogie type undercarriages are used


to

 prevent skidding
 absorb increased landing shock
 spread the weight over a large area

1579.To position the bogie beam at a suitable angle for


retraction and landing, a

 castoring damper is used


 snubber is used
 hop damper is used

1580.On large aircraft, braked wheel assemblies are


normally inflated with

 nitrogen
 nitrogen and not more than 5% of air
 an argon and CO2 mixture
Answers for Questions 1571 – 1580

1) tyre bursts due to high temperatures created by


excessive braking
Comment/Reference: CAAIPs Leaflet 5-8 para 3.1.3 and
A&P Technician Airframe Textbook 9-6

2) emergency braking will not be available


Comment/Reference:

3) between the tread rubber and the casing piles


Comment/Reference: AC65-15A pg 378 fig 9-49 &
CAAIP Leaflet 5-7 fig 1

4) a separate pilot operated control


Comment/Reference:

5) melt at a specified elevated temperature


Comment/Reference: CAP 562 Leaflet 5-8 para 3.1.3

6) under-inflation
Comment/Reference:

7) the top of the tie bar


Comment/Reference: CAP 562 Leaflet 5-7 para 10.4.5 b)

8) spread the weight over a large area


Comment/Reference:

9) hop damper is used


Comment/Reference:

10) nitrogen and not more than 5% of air


Comment/Reference: CS 25.733 (e)
Questions 1581 – 1590

1581.Brake deboosters

 assist the operation of the anti-skid unit


 increase the hydraulic pressure entering the brakes
 reduce the hydraulic pressure entering the brakes

1582.A modulator is fitted in conjunction with

 anti-skid units
 non-return valve
 brake control valves

1583.In the case of pressurized aircraft, the nose-wheel


bay

 is subject to cabin pressure


 is pressurized to a value higher than ambient but
less than cabin pressure
 is not pressurized

1584.An undercarriage that has the axle mounted directly


onto the shock absorber is of which type

 Articulated
 Direct Acting
 Hydro Mechanical

1585.The minimum aquaplaning ground speed is

 8.6 times the tyre pressure in bar


 8.6 times the root square of the tyre pressure in
psi
 9.6 times the root square of the tyre pressure in
psi
1586.Tubed tyres are stored

 horizontally, up to 4 in total staggering them to


prevent distortion of beads
 horizontally, up to 4 in total with the smallest at
the top
 horizontally, up to 4 in total in a rack with
supporting tubes so each tyre is supported at 2
points

1587.On a main landing gear, what is positive camber?

 Top of wheels closer to fuselage


 Front of wheels closer to fuselage
 Bottom of wheels closer to fuselage

1588.When the landing gear is locked up, the cockpit


indicator shows

 no indication
 red light
 green light

1589.When checking the alignment of a main landing gear,


check

 symmetry, tracking, twist


 symmetry, tracking, camber
 symmetry, tracking, splay

1590.Tyre creep could be caused by

 overpressure
 excessive wear
 under pressure
Answers for Questions 1581 – 1590

1) reduce the hydraulic pressure entering the brakes


Comment/Reference:
www.tpub.com/content/aviation/14018/css/14018_474.htm

2) anti-skid units
Comment/Reference: AL/3-21 6.6

3) is not pressurized
Comment/Reference:

4) Direct Acting
Comment/Reference:

5) 8.6 times the root square of the tyre pressure in psi


Comment/Reference:

6) horizontally, up to 4 in total with the smallest at the


top
Comment/Reference: CAAIPs Leaflet 5-7 15.2.1 and 15.2.2

7) Bottom of wheels closer to fuselage


Comment/Reference:

8) no indication
Comment/Reference: Jeppesen A & P Airframe Technician
Textbook page 10-12

9) symmetry, tracking, camber


Comment/Reference:

10) under pressure


Comment/Reference:
Questions 1591 – 1600

1591.When inflating a tyre and you notice ice in the valve


stem

 wait for ice to melt before carrying on with


inflation
 heat the valve stem gently with a blow torch
 carry on inflating as ice in the valve stem is no
problem

1592.A method of helping to prevent aquaplaning is by


fitting tyres which have

 a plain tread
 twin contact
 water dispersing treads

1593.Un-mounted tubeless tyres must be stored

 in the manufactures boxes


 horizontally
 vertically

1594.With a single oleo leg with a stub axle used, the


torque links will

 allow the wheel to shimmy


 assist the wheel to castor
 keep the wheel in track

1595.When fitting a tyre to a hub, the red spot on the tyre


should be in line with

 opposite side to the valve assembly


 maker's serial number
 the valve assembly
1596.A red or yellow line on an inner tube would indicate

 light spot
 heavy spot
 balance indicator

1597.On a multi-brake unit with automatic adjusters, brake


wear is checked by

 application of the brakes and checking indicator pin


protrusion
 using a go-no go gauge to measure the gap between
the cylinder and the thrust plate
 measuring the protrusion of the indicator pins with
the brakes released

1598.With aircraft lights - which of the following is true?

 Starboard light red, port light green, tail light


White
 Starboard light red, port light green, tail light
Red
 Starboard light green, port light red, tail light
white

1599.Emergency floor lighting system is inoperative, then

 the aircraft is allowed to fly in daylight


conditions only
 the aircraft is allowed to fly empty to a main base
 the aircraft is not allowed to fly until repaired

1600.A white navigation light, as viewed in the horizontal


plane, will be at least

 20 candela
 240 candela
 50 candela
Answers for Questions 1591 – 1600

1) wait for ice to melt before carrying on with inflation


Comment/Reference:

2) water dispersing treads


Comment/Reference:

3) vertically
Comment/Reference: CAP 562 Leaflet 5-7 para 15.2.1

4) keep the wheel in track


Comment/Reference: Jeppesen A & P Airframe Technician
textbook page 9-12

5) the valve assembly


Comment/Reference: AC65-15A pg 386 & Jeppesen A & P
Airframe Technician Textbook page 9-49 &
www.dunlopaircrafttyres.com/tyrecare/dm1172/DM1172.pdf

6) heavy spot
Comment/Reference:
www.dunlopaircrafttyres.com/tyrecare/dm1172/dm1172.pdf
Installation and removal of tyres para 2. B.

7) application of the brakes and checking indicator pin


protrusion
Comment/Reference: Dunlop component maintenance manual
Ch. 32-42-98

8) Starboard light green, port light red, tail light white


Comment/Reference: AC65-15A pg 459-460 & Pallett
Aircraft Electrical Systems 3rd Ed Pg 145

9) the aircraft is allowed to fly empty to a main base


Comment/Reference: Can fly without passengers in
accordance with the MEL. CAP 562 Leaflet 5-11 para 3.3

10) 20 candela
Comment/Reference: CS 25.1391 Table
Questions 1601 – 1610

1601.A fluorescent tube contains

 phosphor coatings, rare gases and mercury vapour


 orange coatings, rare gases and mercury vapour
 iodine coatings and rare gases

1602.A white steady light is required

 of at least 3 candelas, at the rear of the aircraft


110 degrees either side of dead astern
 of at least 3 lumens, at the rear of the aircraft
110 degrees either side of dead astern
 of at least 3 candelas, at the rear of the aircraft
70 degrees either side of dead astern

1603.How many floor path lights can you fly with


unserviceable?

 15%
 25%
 20%

1604.What is the arc of a landing light?

 This is not covered in European legilsation


 20°
 15°

1605.The visible angle of a white tail navigation light is

 11°
 110°
 140°
1606.Wing navigation lights must be visible through which
angle?

 125°
 110°
 180°

1607.Cockpit dome lighting is provided by the

 battery bus and ground services bus


 battery bus
 ground services bus

1608.Upper and lower strobe lights are coloured

 red
 white
 green

1609.Cargo bay lights on a modern aircraft are supplied by

 AC handling bus
 DC handling bus
 the battery bus

1610.What will happen if the Master Dim and test switch is


switched to the on position?

 Rectangular indicator lights will illuminate


 All lights will illuminate
 Dome lights will illuminate
Answers for Questions 1601 – 1610

1) phosphor coatings, rare gases and mercury vapour


Comment/Reference:

2) of at least 3 candelas, at the rear of the aircraft 70


degrees either side of dead astern
Comment/Reference: CS 25.1391 & CAP 393 Rules of the
Air Rule 11 (2) (a) (iii)

3) 25%
Comment/Reference: CAP 562 Leaflet 5-11 para 2.11
25 Percent rule

4) This is not covered in the current European legislation


Comment/Reference: CS 25.1383

5) 140°
Comment/Reference: Pallett Aircraft Electrical Systems
3rd Ed Page 146, & CS 25.1387

6) 110°
Comment/Reference: Aircraft Electrical Systems Pallett
Page 146 & CS 25.1387 para (b)(c)

7) battery bus and ground services bus


Comment/Reference: Aircraft Electricity & Electronics,
Eismin 5th Ed page 256

8) red
Comment/Reference: Transport Category Aircraft Systems
Jeppesen Page 7-2 & CS 25.1401

9) AC handling bus
Comment/Reference: BAe 146 AMM (AC ground service
busbar) although other aircraft (A340, B747) use 28VDC
ground bus

10) Rectangular indicator lights will illuminate


Comment/Reference:
Questions 1611 – 1620

1611.Escape route lighting must not have more than

 20% obscured
 15% obscured
 10% obscured

1612.How many emergency lights are allowed to be


inoperative?

 None
 25%
 10%

1613.The angle of a runway turnoff light is

 60°
 40°
 50°

1614.Cabin fluorescent lighting circuits are supplied with

 28 V AC
 115 V AC
 28 V DC

1615.How are passenger reading lights normally tested?

 By using a READ LIGHT TEST switch on the cabin


attendant panel
 Each one is switched on individually at passenger
panel
 By using a READ LIGHT TEST switch at any passenger
panel
1616.In what position should the selector switch be for a
standby and emergency lighting system during flight?

 Armed
 ON
 OFF

1617.The emergency lighting system must be designed so that


after any single transverse vertical separation of the
fuselage during crash landing the maximum amount of
emergency lighting that fails is

 25%
 10%
 15%

1618.A rotating beacon must have a minimum light rating of

 50 candelas
 100 candelas
 20 candelas

1619.Service lights include

 avionics bay lights, engine scanning lights, baggage


compartment lights
 baggage compartment lights, avionics bay lights,
refuelling lights
 refuelling lights, engine scanning lights, logo
lights

1620.If the 'blow-back' device on a landing lamp operates,


how is it reset?

 The linkage must be repositioned and latched when


the aircraft is on the ground.
 It cannot. The unit must be replaced because of the
high stress loads experienced
 Select full 'RETRACT'
Answers for Questions 1611 – 1620

1) 20% obscured
Comment/Reference:

2) 25%
Comment/Reference: CS 25.812 (l) (1), & CAP 562 Leaflet
5-11 para 2.11.1

3) 50°
Comment/Reference: Aircraft Electrical Systems Pallett
Page 146

4) 115 V AC
Comment/Reference: Jeppesen A&P Technician Airframe
Textbook Page 7-76 (figure 7-115)

5) By using a READ LIGHT TEST switch on the cabin


attendant panel
Comment/Reference:

6) Armed
Comment/Reference: Transport Category Aircraft Systems
11-23 & CS 25.812 para (f)(3)

7) 25%
Comment/Reference: CAP 562 Leaflet 5-11 para 2.11.1
CS 25.812

8) 20 candelas
Comment/Reference: CAP 393 Section 2 Rules of the Air
Para. 49 5 (d) (ii) & CS 25-1401 (f)

9) baggage compartment lights, avionics bay lights,


refuelling lights
Comment/Reference: EEL/1-10 3.5

10) Select full 'RETRACT'


Comment/Reference: EEL/1-10 3.3.1
Questions 1621 – 1630
1621.Before touching or disconnecting a strobe light head,
a time period must elapse to avoid electrical shock or
burning. That time period is at least

 5 minutes
 1 minute
 2 minutes

1622.The rear light of an aircraft must be white and, in


addition, must show through an inclusive angle of

 140 degrees and be 3 candelas minimum


 110 degrees and be 5 candelas minimum
 110 degrees and be 40 candelas minimum

1623.Self-illuminating signs

 require a period of daylight, or intense artificial


light to operate
 are self powered and contain phosphor and helium gas
 are instantly seen in dark areas by persons who are
not dark adapted

1624.When an annunciator light is selected to 'DIM', the


resistor is in

 series with the light and the transistor is


conducting
 parallel with the light and the transistor is not
conducting
 series with the light and the transistor is not
conducting

1625.Storm lights are usually fluorescent lights that are


switched on

 to reduce the effect of lightning on the pilot's


sight
 because lightning has less effect on fluorescent
lamp units
 and the glare-shield lights are automatically dimmed
1626.Captain and First Officer's 'Dome' lights can be
dimmed

 individually
 together
 First Officer's only in emergency mode

1627.In the CWS system, the caution light is coloured

 amber
 red
 green

1628.On a CWS, which has the highest priority?

 Hydraulic pump failure


 Duct overheat
 Fire warning

1629.What inert gas is used in a typical strobe light?

 Xenon
 Freon
 Halon

1630.Floor proximity lighting is a mandatory requirement


and each light should be spaced

 at 60 inch intervals
 at 40 inch intervals
 at 70 inch intervals
Answers for Questions 1621 – 1630

1) 2 minutes
Comment/Reference: EEL/1-10 5.5

2) 140° and be 3 candelas minimum


Comment/Reference: CAP 393 Section 2 Rules of the Air
Para 2 a iii and CAIPs EEL/1-10 3.1 c)

3) are instantly seen in dark areas by persons who are not


dark adapted
Comment/Reference: EEL/1-10 4.8

4) series with the light and the transistor is conducting


Comment/Reference: The transistor is conducting all the
time the light is switched on. Pallett - Aircraft
Electrical Systems. Page 153

5) to reduce the effect of lightning on the pilot's sight


Comment/Reference:

6) together
Comment/Reference: B737 AMM 33-11-00 Page 5

7) amber
Comment/Reference: CAIPs EEL/1-10 4.4.1

8) Fire warning
Comment/Reference:

9) Xenon
Comment/Reference:

10) at 40 inch intervals


Comment/Reference: CAAIPs Leaflet 5-11 para 2.7.1
Questions 1631 – 1640

1631.Tritium Gas is used in a

 self illuminating lights


 strobe light
 landing Light

1632.A strobe light is a light unit that takes form of


glass tube filled with which gas and its light colour
is what?

 Helium gas and white


 Xenon Gas and blue-white
 Neon gas and blue

1633.The aircraft has a partial failure of its emergency


lighting system

 ground the aircraft


 continue with reduced passenger load
 ferry flight to main base for rectification

1634.Navigation lights are supplied by the following


circuit:

 Individual circuits
 Dual circuit
 Single circuit

1635.Automatic no smoking sign will illuminate

 when landing gear is down


 below 10,000ft and descending
 unless switched off by the pilot
1636.To purge an unpleasant odour/moisture from an oxygen
system, you must

 completely empty and fill the system with air at


least 3 times
 completely empty and fill the system with nitrogen
at least 3 times
 completely empty and fill the system with oxygen at
least 3 times

1637.When charging an oxygen cylinder, the pressure goes to


zero. Where is the likely fault?

 In the temperature compensator


 In the pressure gauge
 In the mask

1638.Anoxia is due to

 lack of oxygen
 release of nitrogen bubbles in the blood
 low air pressure on the body

1639.The dilutor demand regulator functions

 all the time


 only when the supply valve is opened by the user
 when the user breathes in

1640.To measure moisture in an oxygen system use

 litmus paper
 a hygrometer using the dew point method
 a glass plate
Answers for Questions 1631 – 1640

1) self illuminating lights


Comment/Reference: EEL/1-10 4.8.1

2) Xenon Gas and blue-white


Comment/Reference: EEL/1-10 3.2.3

3) continue with reduced passenger load


Comment/Reference: A club66 user who got this, queried
it with the CAA. He was told (by the CAA assessor) that
this is the answer. However, no reference has been
found.

4) Single circuit
Comment/Reference: Aircraft Electricity & Electronics
5th Ed Eismin Page 250

5) unless switched off by the pilot


Comment/Reference: 737 AMM 33-24-00

6) completely empty and fill the system with oxygen at


least 3 times
Comment/Reference: AC65-15A pg 600

7) In the mask
Comment/Reference:

8) lack of oxygen
Comment/Reference: CAP 562 Leaflet 5-9 Para 2.1

9) when the user breathes in


Comment/Reference: CAP 562 Leaflet 5-9 para 3.5 &
Jeppesen A&P Technician Airframe Textbook Page 14-10

10) a hygrometer using the dew point method


Comment/Reference: CAP 562 Leaflet 5-9 para 6.4.1 or
CAIPs AL/3-25 Para 6.4
Questions 1641 – 1650

1641.Oxygen cylinder test dates

 are variable depending on discharge


 may be stencilled on the neck of the cylinder
 are painted in white on the cylinder

1642.O2 system pressure has leaked away after charging.


What is the most probable cause?

 Supernumerator regular
 Temperature compensator
 Breathing regulator

1643.If an O2 system is leaking, where would you most likely


find the cause?

 Thermal Relief Valve


 Loose connection
 Breathing mask

1644.As the cabin altitude reaches 3042 metres

 the 02 masks deploy automatically and the crew must


activate the system from the cockpit
 the 02 masks deploy automatically and the crew
activates the system from the emergency panel
 the 02 masks deploy automatically and hang half way
extended

1645.Oxygen cylinders are normally fitted with

 temperature gauge
 pressure and quantity gauges
 oxygen purity gauge
1646.A demand regulator in a portable oxygen bottle opens

 all the time


 when breathing
 when 100% selected

1647.Portable oxygen bottles are fitted with

 a demand regulator
 an overpressure indicator
 an over temperature indicator

1648.American made crew oxygen cylinders are

 black in colour with a RH thread


 green in colour with a RH thread
 green in colour with a LH thread

1649.Which connector has a left hand thread?

 O2
 N2
 Freon

1650.What is the principle of an O2 generator?

 Sodium chlorate and iron using a mechanical


detonator producing O2 when mixed with air
 Sodium chloride and iron mixed by an electronic
detonator
 Sodium chlorate and iron mixed by an electronic
detonator
Answers for Questions 1641 – 1650

1) are painted in white on the cylinder


Comment/Reference: CAP 562 Leaflet 5-9 para 4.2.3
CAIPs AL/3-25 para 4.2.2.

2) Breathing regulator
Comment/Reference:

3) Breathing mask
Comment/Reference: Breathing masks are the most common
cause of leaks. O2 systems being turned on during pilot
pre flight checks. Refer to any fim/tsm

4) the O2 masks deploy automatically and hang half way


extended
Comment/Reference:

5) pressure and quantity gauges


Comment/Reference: CAP 562 Leaflet 5-9 para 4.9

6) when breathing
Comment/Reference: CAP 562 Leaflet 5-9 para 3.5 & AC65-
15A pg 594 & A&P Technician Airframe Textbook 14-9

7) a demand regulator
Comment/Reference:

8) green in colour with a RH thread


Comment/Reference: CAP 562 Leaflet 5-9 para 4.2.3
states green for cylinders for American origin. (l/h
threads are for charging points)

9) O2
Comment/Reference: CAAIP Leaflet 5-9 para 5.13.3(e)

10) Sodium chlorate and iron mixed by an electronic


detonator
Comment/Reference: CAP 562 Leaflet 5-9 para 3.6.2 &
AC65-15A pg 589
Questions 1651 – 1660

1651.What pressure are oxygen cylinders usually pressurised


to?

 3000 PSI
 300 PSI
 1800 PSI

1652.Oxygen storage cylinders, once charged should be

 turned on and safety wire-locked


 turned off
 turned on by crew

1653.A thermal compensator is used in

 a fuel system
 an oxygen system
 an hydraulic system

1654.When charging an oxygen bottle with gaseous oxygen,


the oxygen is

 passed into the bottle slowly to keep the


temperature at approximately ambient
 passed into the bottle as quickly as possible to
keep the temperature down
 passed into the bottle at any rate as this does not
affect temperature

1655.Where are the left handed threads found in an oxygen


system?

 Charging connection
 Shut-off valve
 Line valve
1656.Portable O2 diluter demand valve works when

 100% is selected
 the user breathes in
 less than 500 psi is in bottle

1657.A double headed oxygen pressure regulator is used for

 extra supply in case of emergency


 additional flightcrew
 changes in altitude

1658.If an aircraft suddenly drops pressure at 14,000 ft

 passenger must get mask from overhead stowage


 attendant must drop masks
 masks drop automatically

1659.How do oxygen cylinders show over pressure

 Green rupturing disc


 Red rupturing disc
 Audible warning

1660.The life of an aircraft oxygen cylinder is

 4 years
 2 years
 10 years
Answers for Questions 1651 – 1660

1) 1800 PSI
Comment/Reference: AL/3-25 4.2, & CAP 562 Leaflet 5-9
3.3.1 and A&P Technician Airframe Textbook 14-8

2) turned on and safety wire-locked


Comment/Reference: CAP 562 Leaflet 5-9 para 5.6.2 h) &
AC65-15A pg 593

3) an oxygen system
Comment/Reference: CAP 562 Leaflet 5-9 para 4.12 or
CAIPs AL/3-25 4.12

4) passed into the bottle slowly to keep the temperature


at approximately ambient
Comment/Reference: CAP 562 Leaflet 5-9 para 5.13.3 f) &
A&P Mechanic Airframe Handbook Page 598

5) Charging connection
Comment/Reference: CAAIP Leaflet 5-9 para 5.13.3(e)

6) the user breathes in


Comment/Reference: AC65-15A pg 594 & A&P Technician
Airframe Textbook 14-9

7) additional flightcrew
Comment/Reference:

8) masks drop automatically


Comment/Reference: CAP 562 Leaflet 5-9 para 3.6.2 &
Transport Category Aircraft Systems 8-8

9) Green rupturing disc


Comment/Reference: CAP 562 leaflet 5-9 para 4.13 or
CAIPs AL/3-25 4.13

10) 4 years
Comment/Reference: Jeppesen A & P Technician Airframe
Textbook page 14-8 or CAP 562 Leaflet 5-9 para 5.9 or
CAIPs AL/3-25 5.9. By 'life' they must mean 'time
between overhaul'
Questions 1661 – 1670

1661.A discharged chemical generator is indicated by

 a broken tell-tale wire


 a change in colour
 a protruding pin

1662.A chemical oxygen generator has a supply duration of


at least

 5 minutes
 15 minutes
 30 minutes

1663.The oxygen line service valve must be

 wire locked open


 only be wire locked with telltale wire
 wire locked closed

1664.Oxygen equipment must be carried by aircraft capable


of flying above

 4,000 ft
 8,000ft
 10,000ft

1665.The critical altitude for oxygen requirement is


approximately

 4,000 ft
 38,000 ft
 25,000 ft
1666.If oxygen is not being supplied in normal flight
conditions, the cabin altitude must be

 at a maximum of 12,000 ft.


 below 10,000 ft.
 at or below 8,000 ft.

1667.Pressure breathing systems must be used at altitudes

 below 40,000 ft.


 above 30,000 ft.
 above 40,000 ft.

1668.In a diluter demand oxygen system

 each member of the crew has a continuous oxygen


supply
 oxygen is supplied with a continuous pressure flow
 each member of the crew has a regulator

1669.In a continuous flow oxygen system, oxygen is supplied

 only when the supply has been regulated by the pilot


 when the mask is plugged into the socket
 on passenger inhaling into the mask

1670.In the diluter demand oxygen system, selection of


emergency on this regulator will result in

 100% oxygen at positive pressure


 100% oxygen continuous flow at positive pressure
 100% oxygen supply as the user inhales
Answers for Questions 1661 – 1670

1) a change in colour
Comment/Reference: CAP 562 Leaflet 5-9 para 3.6.4 &
CAIPs AL/3-25

2) 15 minutes
Comment/Reference: A&P Technician Airframe Textbook
14-8 & CAP 562 Leaflet 5-9 is not in the SEP07 version
CS 25.1439 (b)(5)

3) wire locked open


Comment/Reference: CAP 562 Leaflet 5-9 para 5.6.2 h) &
CAIPs AL/3-25

4) 10,000ft
Comment/Reference: CAP 562 leaflet 5-9 para 3.2 & CAIPs
AL/3-25 3.1

5) 25,000 ft
Comment/Reference: CAP 562 Leaflet 5-9 pg 2 table 1 &
CAIPs AL/3-25 Table 1

6) at or below 8,000 ft.


Comment/Reference: CAP 562 leaflet 5-9 para 3.1 & CAIPs
AL/3-23

7) above 40,000 ft.


Comment/Reference: CAP 562 leaflet 5-9 pg 2 table 1 &
CAIPs AL/3-25 table 1

8) each member of the crew has a regulator


Comment/Reference: CAP 562 leaflet 5-9 para 3.5 & fig 3
& CAIPs AL/3-25 fig 3

9) when the mask is plugged into the socket


Comment/Reference: CAP 562 leaflet 5-9 para 3.4 & AC65-
15A pg 587

10) 100% oxygen continuous flow at positive pressure


Comment/Reference: CAP 562 Leaflet 5-9 para 4.7(c)
Questions 1670 – 1680

1671.The cylinder of a portable oxygen set is made of

 alloy steel
 aluminium
 stainless steel

1672.Portable oxygen cylinders are normally charged to

 2,000 p.s.i.
 1,800 p.s.i.
 1,200 p.s.i.

1673.Rate of flow of oxygen is given in

 litres/minute
 pounds/minute
 litres/pounds

1674.Oxygen cylinders are removed for overhaul and tested

 every 3 years
 every 6 years
 every 4 years

1675.Oxygen bottle capacity varies between

 100 - 3200 litres


 80 - 2250 litres
 200 - 2250 litres
1676.Oxygen bottle test and working pressures can be found

 stamped on a metal label


 stamped on the neck ring
 stencilled on the bottle

1677.After installation, the oxygen bottle on/off valve is

 left in the 'on' position


 left in the 'off' position until required
 wire locked in the 'on'

1678.Dangerous pressure rise in oxygen cylinders

 is relieved by under pressurising the bottle


 is relieved by a thermostat
 is relieved by a bursting disc

1679.To avoid condensation forming in an oxygen cylinder

 cylinder pressure is left at 200/300 p.s.i.


 cylinder pressure is left at 100/200 p.s.i.
 cylinder pressure is left at 300/400 p.s.i.

1680.What determines the material used for oxygen pipe


lines?

 The whims of the aircraft designer


 The length of the pipe runs
 The pressure used in the system
Answers for Questions 1671 – 1680

1) alloy steel
Comment/Reference: CAP 562 leaflet 5-9 is obsolete
AC65-15A pg 587 & CAIPs AL/3-25 3.6

2) 1,800 p.s.i.
Comment/Reference: CAP 562 leaflet 5-9 para 3.7.1 &
AC65-15A pg 587-588 & CAIPs AL/3-25 3.6

3) litres/minute
Comment/Reference: CAP 562 leaflet 5-9 para 3.7.2 &
AC65-15A pg 588

4) every 4 years
Comment/Reference: CAP 562 leaflet 5-9 para 5.9 & CAIPs
AL/3-25 5.8

5) 80 - 2250 litres
Comment/Reference: CAP 562 Leaflet 5-9 para 4.2.1 &
CAIPs AL/3-25 4.2

6) stamped on the neck ring


Comment/Reference: CAP 562 leaflet 5-9 para 5.9 & CAIPs
AL/3-25 4.2.2

7) wire locked in the 'on'


Comment/Reference: CAP 562 Leaflet 5-9 para 5.6.2(h) &
AC65-15A pg 589

8) is relieved by a bursting disc


Comment/Reference: CAP 562 leaflet 5-9 para 4.12 &
CAIPs AL/3-25 4.13

9) cylinder pressure is left at 200/300 p.s.i.


Comment/Reference: CAP 562 leaflet 5-9 para 5.6.2 b) &
CAIPs AL/3-25 5.5.1 ii)

10) The pressure used in the system


Comment/Reference: CAP 562 leaflet 5-9 para 4.3 & CAIPs
AL/3-25 4.3
Questions 1681 – 1690

1681.The direction of flow in an oxygen non return valve is


indicated

 by colour coding
 by an arrow
 by visual inspection

1682.Oxygen filters are made of

 sintered bronze
 carbon fibres
 steel wool

1683.Satisfactory operation of the oxygen systems is


indicated by

 lack of Anoxia
 flow indicators
 pressure indicators

1684.If a binding thread needs attention on an oxygen


system

 no lubricant may be used whatsoever


 Teflon tape may be used
 distilled water may be used sparingly

1685.The international marking for a breathing oxygen


pipeline is a series of

 diamonds
 rectangles
 dots
1686.Cylinders containing purified oxygen for breathing are
painted

 black with a white collar


 white with a black collar
 black

1687.A suitable leak detecting solution for use on oxygen


systems is

 grease free medical soap


 soapy water
 leak testing solution to Spec. MIL-L-25567B

1688.When charging an oxygen bottle with gaseous oxygen,


the oxygen is

 passed into the bottle as quickly as possible to


keep the temperature down
 passed into the bottle at any rate as this does not
affect temperature
 passed into the bottle slowly to keep temperature at
approximately ambient

1689.In areas closely associated with oxygen systems,


particular care must be taken to avoid leaving

 acrylic based plastic materials


 traces of oils or greases
 magnesium particles

1690.When working in the vicinity of an oxygen system

 no electrical power must be used


 oil must not come into contact with the system
 the area must be well ventilated
Answers for Questions 1681 – 1690

1) by an arrow
Comment/Reference: CAP 562 leaflet 5-9 para 4.4 & CAIPs
AL/3-25 Page 8

2) sintered bronze
Comment/Reference: CAP 562 leaflet 5-9 para 4.5 & CAIPs
AL/3-25 Page 8

3) flow indicators
Comment/Reference: CAP 562 Leaflet 5-9 para 4.11

4) Teflon tape may be used


Comment/Reference: CAP 562 leaflet 5-9 para 5.7 & CAIPs
AL/3-25, 5.6

5) rectangles
Comment/Reference:

6) black with a white collar


Comment/Reference: CAP 562 leaflet 5-9 para 4.2.3 &
CAIPs AL/3-25 4.2.2

7) leak testing solution to Spec. MIL-L-25567B


Comment/Reference: CAP 562 leaflet 5-9 para 5.8.2 &
CAIPs AL/3-25 5.7.1, AL/3-21 3.

8) passed into the bottle slowly to keep temperature at


approximately ambient
Comment/Reference: CAP 562 leaflet 5-9 para 5.13.3 f) &
AC65-15A pg 598 & CAIPs AL/3-25 5.12.2 (vi)

9) traces of oils or greases


Comment/Reference: CAP 562 leaflet 5-9 para 5.5.1 f)
A&P Technician Airframe Textbook Chapter 14-19 rh
column 2nd para. & AC65-15A pg 598 rh side.

10) oil must not come into contact with the system
Comment/Reference: CAP 562 leaflet 5-9 para 5.5.1 f)
Questions 1691 – 1700

1691.A chemical oxygen generator operates at

 45°C at 10 psi for 15 minutes


 45°F at 10 psi for 15 minutes
 45°K at 10 psi for 15 minutes

1692.When charging an oxygen bottle in situ

 charge in accordance with the temperature/pressure


graph
 slacken off the retaining straps first
 charge slowly through a water trap

1693.The gauge fitted to an oxygen bottle indicates

 temperature
 purity
 pressure

1694.The application of thread lubrication tape in an


oxygen system should be

 applied to all except the first two threads and not


more than one complete wrap of tape
 applied to all the threads and not more than one
complete wrap of tape
 applied to all except the first two threads and not
more than 3 complete wraps of tape.

1695.High pressure lines in oxygen systems are made of

 titanium
 stainless steel
 aluminium alloy
1696.Lubricate oxygen connector threads using

 WD40
 Teflon tape
 Hellerene

1697.How is an expended chemical oxygen generator


indicated?

 A pressure seal would be broken


 The indicator pin would be protruding
 By a change in colour of heat sensitive paint

1698.Oxygen purging is a process of

 measuring the flow rate from the regulator


 pressure testing the system
 removing moisture from the system

1699.To check an oxygen system for moisture

 a sniff test is used


 a hygrometer is used
 pass 72 litres/min through a filter paper in a clean
glass

1700.When the moisture separator is purged in a pneumatic


system, it dumps

 just the moisture trap


 the system between compressor and regulator valve
 the whole system
Answers for Questions 1691 – 1700

1) 45°C at 10 psi for 15 minutes


Comment/Reference: CAIPs AL/3-25 3.5.2

2) charge in accordance with the temperature/pressure


graph
Comment/Reference: CAP 562 Leaflet 5-9 para 5.13.3f) &
CAIPs AL/3-25 5.12.2 (vi) (a)

3) pressure
Comment/Reference: CAP 562 leaflet 5-9 all through it.
& CAIPs AL/3-25 4.2

4) applied to all except the first two threads and not


more than 3 complete wraps of tape.
Comment/Reference: CAP 562 Leaflet 5-9 Para 5.7 & CAIPs
AL/3-25 para 5.6

5) stainless steel
Comment/Reference: CAP 562 Leaflet 5-9 p8 para 4.3.1

6) Teflon tape
Comment/Reference: CAP 562 Leaflet 5-9 para 5.7 & CAIPs
AL 3-25 para 5.6

7) By a change in colour of heat sensitive paint


Comment/Reference: CAP 562 leaflet 5-9 para 3.6.4

8) removing moisture from the system


Comment/Reference: CAP 562 Leaflet 5-9 para 5.13.3 g)

9) a hygrometer is used
Comment/Reference: CAP 562 leaflet 5-9 para 6.4.1

10) just the moisture trap


Comment/Reference: Jeppesen A&P Technician Airframe
Textbook Page 8-54. CAIPs AL/3-22
Questions 1701 – 1710

1701.The pneumatic system pump is a

 piston type
 rotor vane type
 centrifugal type

1702.In a pneumatic system, the maintainer valve will be


fitted in supply lines to

 essential equipment
 all equipment
 non-essential equipment

1703.High pressure pneumatic source is a

 butterfly pump
 centrifugal Compressor
 reciprocating pump

1704.High pressure pneumatic pump is a

 butterfly pump
 spur gear
 reciprocating pump

1705.If the pneumatic water drain trap is left open for a


long time it will drain

 just the moisture trap


 all the system
 between the compressor and the PRV
1706.On a high pressure pneumatic system, if the drain plug
for oil and water is left open for long periods of
time, the system would

 lose all pressure


 lose pressure from the compressor side only
 lose pneumatic pressure partially

1707.Two compressors driven by separate engines use

 PRVs
 NRVs to prevent compressors driving each other
 Interconnected to share loads

1708.What is important about the air entering a dry air


pump?

 It must be filtered
 It must be temperature controlled
 It must be pressure controlled

1709.What type of compressor is used on a pneumatic system?

 Positive displacement type


 Spur gear type
 Rotary vane type

1710.Toilet flush motor is protected from overheat by

 water cooling
 thermal protection
 cooling fan on timer switch
Answers for Questions 1701 – 1710

1) piston type
Comment/Reference: AC65-15A pg 331 & Jeppesen A&P
Technician Airframe Textbook Page 8-56, Although
smaller systems may have a vane type.

2) non-essential equipment
Comment/Reference: CAIPs AL/3-22. Maintainer valves
work in same way as those in hydraulic systems (hence
the reference). Although they protect the flow to the
essential systems, they do so by being situated in the
non-essential lines, and closing off if the pressure
drops.

3) reciprocating pump
Comment/Reference: AC65-15A pg 331 & CAIPs AL/3-23 4.1

4) reciprocating pump
Comment/Reference: AC65-15A pg 331 & CAIPs AL/3-23 4.1

5) between the compressor and the PRV


Comment/Reference: CAIPs AL/3-22 Fig 1

6) lose pressure from the compressor side only


Comment/Reference: AL/3-22 figure 1

7) NRVs to prevent compressors driving each other


Comment/Reference:

8) It must be filtered
Comment/Reference:

9) Positive displacement type


Comment/Reference:

10) thermal protection


Comment/Reference: AWN 57 (deleted 2003)
Questions 1711 – 1720

1711.What is the purpose of thermal protection on electric


toilet motor?

 Stop motor running beyond 10 seconds if timer become


unserviceable
 Prevent toilet freezing
 Prevent motor overheating and become fire hazard

1712.Waste water drain masts

 are heated using low amperage with the aircraft on


ground only
 are heated using low amperage in the Air and on
ground
 are not heated

1713.Drinking water pipes are prevented from freezing by

 wrapping the pipes with heated tapes or ribbons


 installation of neoprene foam insulation
 placing the pipes adjacent to hot water piping

1714.Toilets are the subject of

 AWN 79
 AWN 80
 AWN 83

1715.Toilet waste valves are

 spring loaded closed


 not spring loaded
 spring loaded open
1716.The heater used on a drain mast would be a

 ribbon heater
 hot air blower
 induction heater

1717.A toilet recirculation fan is unserviceable in flight.

 It can be overcome by thermal compensating device


 You can use the toilet as the toxic chemical toilet
 Close toilet until landing

1718.The real time on a CMC is when

 existing faults page is selected on the CDU


 ground test page is selected on the CDU
 fault history page is selected on the CDU

1719.One of the inputs in a CMC is autopilot controls. The


data will be stored in

 erased only after end of sector


 volatile memory
 non-volatile memory

1720.In an autoland aircraft fitted with a CMC

 all faults are recorded in Volatile memory


 all faults are recorded in Non-Volatile memory
 only Primary Faults are recorded
Answers for Questions 1711 – 1720

1) Prevent motor overheating and become fire hazard


Comment/Reference: AWN 57 Para.2.3 (deleted 2003)

2) are heated using low amperage with the aircraft on


ground only
Comment/Reference: 737-3/4/500 AMM 30-71-00 para 1.B.
pg 1, 28VAC on ground and 115VAC in air. &
AC65-15A pg 303

3) wrapping the pipes with heated tapes or ribbons


Comment/Reference: AC65-15A pg 303

4) AWN 83
Comment/Reference: All the AWNs listed have now been
removed. Toilets are now covered by CAP 747 GR No. 20

5) spring loaded closed


Comment/Reference:

6) ribbon heater
Comment/Reference:

7) Close toilet until landing


Comment/Reference: IAW B737-400 mmel/ddg item 38-2, the
associate lavatory system may be inop. with lav door
secured closed and placard 'INOPERATIVE- DO NOT ENTER'.
Toilet recirc fan is part of the lav system.

8) existing faults page is selected on the CDU


Comment/Reference: Aircraft Electricity & Electronics
5th Ed Eismin Page 271

9) non-volatile memory
Comment/Reference:
Aircraft Electricity and Avionics 5th ed Eismin Page
273

10) all faults are recorded in Non-Volatile memory


Comment/Reference: Aircraft Electricity and Avionics
5th ed Eismin Page 271
Questions 1721 – 1730

1721.In a CMC system, where would you find a 'real time'


fault?

 In Fault History
 In Fault
 In Test

1722.A modern aircraft CMC uses

 a CRT screen
 an LED display
 a magnetic fault indicator

1723.A Flight Data Recorder is activated when

 the helicopter's engines are started


 the helicopter takes off
 when power is applied to the helicopter

1724.In a CMC, warning signals are generated by

 Warning computer
 FMC
 CMC

1725.An aircraft condition monitoring system

 sends information to the central maintenance cell


 detects the source of a fault
 stores information for long term error analysis
1726.With a ACMS quick access recorder

 the tape must not have been used before


 the tape can have been used previously if it is
first bulk erased
 you must use digital tape

1727.When should rain repellent be applied to a windscreen?

 When windscreen is wet


 When windscreen is dry
 When windscreen is either wet or dry

1728.Component P affects

 coefficient C
 coefficient A
 coefficient B

1729.The polythene sleeving over a HF antenna is to stop

 precipitation static
 lightning strike
 moisture ingress and corrosion

1730.On an aircraft communication system, HIRF interference


is most prominent

 across the whole frequency spectrum


 during transmissions only
 at specific frequencies
Answers for Questions 1721 – 1730

1) In Fault
Comment/Reference: Aircraft Electricity & Electronics
5th Ed Eismin, page 270-273. 'Real time' means
reporting of a fault as it occurs, not in Fault History
or Test.

2) a CRT screen
Comment/Reference:

3) the helicopter's engines are started


Comment/Reference: CAP 393 Section 1 part 5 para 62 (3)
JAR Ops-3

4) Warning computer
Comment/Reference:
Transport Category Aircraft Systems 11-5

5) stores information for long term error analysis


Comment/Reference:

6) the tape can have been used previously if it is first


bulk erased
Comment/Reference:

7) When windscreen is wet


Comment/Reference: If applied when windscreen is dry,
it smears, dries, and causes optical illusions.
www.b737.org.uk/iceandrain.htm

8) coefficient B
Comment/Reference: Read about Components P, Q and R,
then read about Coefficients A, B and C.

9) moisture ingress and corrosion


Comment/Reference:

10) at specific frequencies


Comment/Reference: Whilst HIRF affects the aircraft
across the whole frequency spectrum, individual systems
are susceptible to certain frequencies only
Questions 1731 – 1740

1731.What colour is the strobe light on the PORT wing?

 Red
 Green
 White

1732.Two fundamental properties of a gyro are

 wander and rigidity


 precession and wander
 rigidity and precession

1733.Calculate the battery terminal voltage of a NiCad


battery with 15 cells

 18 V
 30 V
 15 V

1734.What are passenger windows normally made from?

 Acrylic
 Acrylic/glass laminate
 Glass

1735.What is the unit which transforms hydraulic pressure


into linear motion?

 A hydraulic pump
 An actuating cylinder
 An accumulator
1736.A plain flap

 when extended rotates on a simple hinge as if to


bend the trailing edge of the wing down
 when retracted forms the lower surface of the wing
 when extended provides a slot between the wing and
the flap

1737.What is the unit which transforms hydraulic pressure


into rotary motion?

 A hydraulic motor
 A hydraulic pump
 An actuating cylinder

1738.An IRS cannot calculate

 local longitude
 local latitude or longitude
 local latitude

1739.Which device allows the free flow of fluid in one


direction and no fluid flow in the other direction?

 Metering piston
 Shutoff valve
 Check valve

1740.How would you identify an American oxygen bottle?

 It is green
 It is white with green wording
 It is painted the same colour as a British oxygen
bottle (black)
Answers for Questions 1731 – 1740

1) White
Comment/Reference: says STROBE light. RTFQ

2) rigidity and precession


Comment/Reference:

3) 18 V
Comment/Reference: 15 x 1.2V = 18V

4) Acrylic
Comment/Reference:

5) An actuating cylinder
Comment/Reference:

6) when extended rotates on a simple hinge as if to bend


the trailing edge of the wing down
Comment/Reference: AC65-15A pg 48

7) A hydraulic motor
Comment/Reference:

8) local longitude
Comment/Reference:

9) Check valve
Comment/Reference: AC65-15A pg 326

10) It is green
Comment/Reference: CAP 562 leaflet 5-9 para 4.2.3
Questions 1741 – 1750

1741.The fuel surge tank

 acts as a separate fuel supply tank


 acts as a vent in the main tank
 reduces wing bending moment

1742.The drag strut in the main landing gear is fitted to


absorb

 braking and take-off acceleration loads


 cross-wind loads on landing
 turning loads while taxiing

1743.After passing the Zone of Confusion of a VOR

 the phase difference is 90 degrees out


 the phase difference reverses
 the phase difference stays the same

1744.A positive coefficient A adjustment of a compass will


give a heading

 decrease on all headings


 increase on East/West headings
 increase on all headings

1745.A fire detection system should be

 designed to detect fires only


 positioned in a location determined by the
manufacture during the design
 designed to withstand vibration & mechanical loads
sustained during normal operation
1746.Fuel surge is prevented by

 having separate compartments within the tank


 having baffles within the tank and supports on the
outside to resist fuel surge
 incorporation of an additional surge tank

1747.Slats are used to

 ensure the boundary layer does not separate from the


wing surface too soon
 change the wing camber
 decrease the stalling angle

1748.A wing has a span of 50 feet and an area of 200 square


feet. Its mean chord would be

 7.5 feet
 4 feet
 10 feet

1749.What type of brakes are employed on a galley cart?

 Two colour-coded pedals


 A brake on one wheel only
 A single pedal bar

1750.Under what condition is indicated altitude the same as


true altitude?

 When at 18,000 feet MSL with the altimeter set at


29.92
 At sea level in standard conditions (ISA)
 Never
Answers for Questions 1741 – 1746

1) acts as a vent in the main tank


Comment/Reference:

2) braking and take-off acceleration loads


Comment/Reference:

3) the phase difference reverses


Comment/Reference: Automatic Flight Control 4th ed
Pallett pg 188 fig 6.9

4) increase on all headings


Comment/Reference: CAP 562 leaflet 8-2 & CAIPs AL/10-5
9.2.7

5) positioned in a location determined by the manufacture


during the design
Comment/Reference:

6) having baffles within the tank and supports on the


outside to resist fuel surge
Comment/Reference:

7) change the wing camber


Comment/Reference: AC65-15A pg 49 half way down left
column.

8) 4 feet
Comment/Reference:

9) Two colour-coded pedals


Comment/Reference:

10) At sea level in standard conditions (ISA)


Comment/Reference:
Questions 1751 – 1760

1751.How is rain repellent stored on an aircraft?

 Reservoir, pressurised by the pneumatic system


 Remote container
 Self pressurised container

1752.A tyre blows a fusible plug. You would

 just change the one with the blown plug


 change the damaged one and the adjacent one
 change all tyres on that truck

1753.Whilst taxiing at low speed, a wheel skids. What is


the likely cause?

 Faulty wheel speed transducer


 Anti-skid system is unserviceable
 There is no fault. Anti-skid system is disabled at
low speed

1754.When turbulence mode is selected, which other mode


will not be available?

 Heading Hold
 ALT Hold
 VOR

1755.What could cause fluctuations in instruments?

 Pitot blocked
 Static blocked
 Water in static line
1756.Priority messages in an aircraft centralised warning
system, go in order of (lowest to highest):

 TCAS, GPWS, windshear


 GPWS, windshear, TCAS
 TCAS, windshear, GPWS

1757.If seat belt stitching is worn, where are the


replacement stitches placed?

 Diagonally
 Alongside the original
 Over the originals

1758.Contact between titanium and phosphate ester fluid


should be avoided because

 it makes the titanium brittle


 it makes the titanium soft
 there is no effect

1759.The optimum atmospheric conditions for painting


aircraft is

 between 60°F and 77°F with less than 75% relative


humidity
 between 65°F and 75°F with less than 75% relative
humidity
 between 65°C and 75°C with more than 75% relative
humidity

1760.TCAS antenna is a

 directional antenna
 omnidirectional blade type L-band
 phased-array L-band
Answers for Questions 1751 – 1760

1) Self pressurised container


Comment/Reference: AC65-15A pg 307

2) change the damaged one and the adjacent one


Comment/Reference:

3) There is no fault. Anti-skid system is disabled at low


speed
Comment/Reference:

4) ALT Hold
Comment/Reference:

5) Water in static line


Comment/Reference:

6) TCAS, GPWS, windshear


Comment/Reference: See Aural Warning Priority Logic
table @
www.b737.org.uk/warningsystems.htm#Aural_Warnings_

7) Over the originals


Comment/Reference:

8) it makes the titanium brittle


Comment/Reference:

9) between 60°F and 77°F with less than 75% relative


humidity
Comment/Reference: CAIPs BL/6-20 para 5
Leaflet 2-7 para 5

10) directional antenna


Comment/Reference: B737 AMM 34-45-00 pg 1 para 1 b) &
Avionics Training, Systems Installation and
Troubleshooting pg 151
737 AMM 34-45-00 pg 7
Questions 1761 – 1770

1761.MLS frequency is

 6000-6100 MHz
 5000-6000 MHz
 5050-6050 MHz

1762.Automatic trim systems on some aircraft installations


are actuated with two speeds of operation. Operation at
high speed is used during

 mach trim
 flaps down
 aircraft at altitude and straight and level

1763.The vane of a stall warning system with a flapper


switch is activated by the change of the

 point of lowest pressure


 stagnation point
 centre of pressure

1764.What is true regarding deployment of Slats / Krueger


flaps?

 Krueger flaps increase the critical angle of attack,


Slats do not
 Slats form a slot, Krueger flaps do not
 Slats increase the critical angle of attack, Krueger
flaps do not

1765.On a swept back wing, in which of the following


locations would Krueger Flaps be fitted?

 The trailing edge


 Outboard leading edge
 Inboard leading edge
1766.An aeroplane performs a straight and level horizontal
flight at the same angle of attack at two different
altitudes: (all other factors of importance being
constant, assume ISA conditions and no compressibility
effects)

 the TAS at the higher altitude is higher


 the TAS at the higher altitude is lower
 the TAS at both altitudes is the same

1767.Extending the flaps while maintaining a constant angle


of attack (all other factors constant)

 the aircraft will yaw


 the aircraft will climb
 the aircraft will sink suddenly

1768.What pitching moment will be generated when Fowler


flaps are deployed on an aircraft with a high mounted
(T-tail) tailplane?

 An aircraft nose up pitching moment


 An aircraft nose down pitching moment
 The nose up pitching moment will be balanced by the
nose down pitching moment

1769.An increase in aspect ratio will

 cause VMD to be increased


 cause induced drag to increase
 cause VMD to be reduced

1770.Geometric washout means that

 the tip of the wing has less angle of attack than


the root
 the tip of the wing has more angle of attack than
the root
 there is an airflow along the wing that keeps it
clean
Answers for Questions 1761 – 1770

1) 5000-6000 MHz
Comment/Reference:
http://en.wikipedia.org/wiki/Microwave_Landing_System#R
ange_guidance

2) flaps down
Comment/Reference: Automatic Flight Control Pallett
page 219

3) stagnation point
Comment/Reference:

4) Slats form a slot, Krueger flaps do not


Comment/Reference:

5) Inboard leading edge


Comment/Reference:

6) the TAS at the higher altitude is higher


Comment/Reference:

7) the aircraft will climb


Comment/Reference:

8) An aircraft nose down pitching moment


Comment/Reference: the downwash off the flaps does not
effect the tail. And the flaps act like elevators

9) cause VMD to be reduced


Comment/Reference: Vmd = minimum drag

10) the tip of the wing has less angle of attack than the
root
Comment/Reference:
www.auf.asn.au/groundschool/umodule4.html
Questions 1771 – 1780

1771.How do vortex generators work?

 Take energy from free stream and introduce it into


the boundary layer
 Re-direct slantwise flow
 Reduce the adverse pressure gradient

1772.Winglets

 decrease the induced drag


 create an elliptical lift distribution
 decrease the static lateral stability

1773.A function of vortex generators in the transonic


regime is to

 reduce wing root compression effects


 prevent the rearward shift of CP on swept wing
stalls
 reduce boundary layer separation drag when
shockwaves form

1774.Total Drag at high Mach numbers is a combination of

 wave drag, interference drag, form drag, and induced


drag
 induced drag, wave drag, form drag, skin friction
drag and interference drag
 profile drag, form drag, induced drag and wave drag

1775.During acceleration in level flight

 the transition point on the wing moves forward


 the lift coefficient reduces
 the wing stagnation point moves aft
1776.As altitude increases, the equivalent airspeed at
which an aeroplane stalls in a particular configuration
will

 remain equal to the calibrated airspeed


 decrease as the true airspeed decreases
 remain the same regardless of altitude

1777.When an aeroplane is flying at an airspeed which is


1.3 times its basic stalling speed, the coefficient of
lift as a percentage of the maximum lift coefficient
(CLmax) would be

 59%
 77%
 130%

1778.To maintain level flight, if the angle of attack is


increased the speed must be

 increased in the same ratio as the lift/drag ratio


decreases
 reduced
 increased

1779.Flap selection at constant IAS in straight and level


flight will increase the

 lift and the drag


 maximum lift coefficient (CLmax) and the drag
 lift coefficient and the drag

1780.In straight and level flight, if airspeed ... the


angle of attack ... and the incidence angle...

 increases; decreases; decreases


 decreases; increases; remains constant
 increases; increases; decreases
Answers for Questions 1771 – 1780

1) Take energy from free stream and introduce it into the


boundary layer
Comment/Reference:

2) decrease the induced drag


Comment/Reference:

3) reduce boundary layer separation drag when shockwaves


form
Comment/Reference:

4) induced drag, wave drag, form drag, skin friction drag


and interference drag
Comment/Reference:

5) the lift coefficient reduces


Comment/Reference:

6) remain the same regardless of altitude


Comment/Reference:

7) 59%
Comment/Reference:

8) increased
Comment/Reference:

9) maximum lift coefficient (CLmax) and the drag


Comment/Reference:

10) decreases; increases; remains constant


Comment/Reference:
Questions 1781 – 1790

1781. What must happen to the CL when flaps are deployed


while maintaining a constant IAS in straight and level
flight?

 Decrease
 Remain constant
 Increase

1782.If VS is 100 kt in straight and level flight, during a


45 bank turn VS will be

 119 kts
 100 kts
 140 kts

1783.What is the effect on TAS if altitude is increased to


35,000 ft while flying at a constant Mach number?

 Decrease
 Remain the same
 Increase then remain constant

1784.If IAS is doubled, by which of the following factors


should the original CL be multiplied to maintain level
flight?

 0.25
 4.0
 0.5

1785.Stick shaker stall warnings should be activated at

 VS
 1.05 VS
 1.2 VS
1786.When an aircraft is in a steady climb, how will be the
wing stalling angle be affected?

 The stalling angle will increase with increasing


altitude
 The stalling angle will remain the same, regardless
of altitude
 The stalling angle will reduce with increasing
altitude

1787.The centre of pressure is in its most forward position

 when the angle of attack is equal to the stalling


angle of attack
 when the angle of attack is smaller than the
stalling angle of attack
 when the angle of attack exceeds the stalling angle
of attack

1788.The angle of attack at which an aircraft stalls

 increases with an increase in gross weight


 remains constant regardless of gross weight
 varies with gross weight and density altitude

1789.The following factors increase stall speed:

 a higher weight, selecting a higher flap setting, a


forward c .g. shift
 an increase in load factor, a forward c.g. shift,
decrease in thrust
 a lower weight, decreasing bank angle, a smaller
flap setting

1790.Stall speed in a turn is proportional to

 the square root of the load factor


 weight
 lift
Answers for Questions 1781 – 1790

1) Decrease
Comment/Reference:

2) 119 kts
Comment/Reference:

3) Decrease
Comment/Reference:

4) 0.25
Comment/Reference:

5) 1.05 VS
Comment/Reference:

6) The stalling angle will remain the same, regardless of


altitude
Comment/Reference:

7) when the angle of attack is equal to the stalling angle


of attack
Comment/Reference:

8) remains constant regardless of gross weight


Comment/Reference:

9) an increase in load factor, a forward c.g. shift,


decrease in thrust
Comment/Reference:

10) the square root of the load factor


Comment/Reference:
Questions 1791 – 1800

1791.An aeroplane has a stalling speed of 100 kt in a


steady level flight. When the aeroplane is flying a
level turn with a load factor of 1.5, the stalling
speed is

 122 kt
 141 kt
 82 kt

1792.A wing stalling angle is

 unaffected by a turn
 decreased in a turn
 increased in a turn

1793.What is a high speed stall?

 Separation of the airflow due to shockwave formation


 A stall caused by increasing the load factor (g)
during a manoeuvre
 A stall due to exceeding the critical angle of
attack at high speed during a manoeuvre

1794.What effect does an increased load have on an


aircraft?

 The aircraft will stall at a higher speed


 The aircraft will have a tendency to roll and yaw
 The aircraft will suffer immediate structural
failure

1795.Which stall has the greatest angle of attack?

 High speed stall (shock stall)


 Low speed stall
 Deep stall
1796.A jet aeroplane cruises buffet free at high constant
altitude in significant turbulence. Which type of stall
can occur if this aeroplane decelerates?

 Accelerated stall
 Low speed stall
 Shock stall

1797.On a swept wing aeroplane at low airspeed, the pitch


up phenomenon

 never occurs, since a swept wing is a remedy to


pitch up
 is caused by wingtip stall
 is caused by extension of trailing edge lift
augmentation devices

1798.Which of the following statements about stall speed is


correct?

 Increasing the angle of sweep of the wing will


decrease the stall speed
 Decreasing the angle of sweep of the wing will
decrease the stall speed
 Use of a T-tail will decrease the stall speed

1799.How are the wing stalling angle and the lift


coefficient affected when altitude increases and EAS is
held constant?

 The lift coefficient and the wing stalling angle


both remain constant
 The lift coefficient remains constant and the wing
stalling angle will reduce
 The lift coefficient and the wing stalling angle
both decrease

1800.The stall speed

 increases with an increased weight


 does not depend on weight
 decreases with an increased weight
Answers for Questions 1791 – 1800

1) 122 kt
Comment/Reference:

2) unaffected by a turn
Comment/Reference:

3) Separation of the airflow due to shockwave formation


Comment/Reference:

4) The aircraft will stall at a higher speed


Comment/Reference:

5) Deep stall
Comment/Reference:

6) Accelerated stall
Comment/Reference: An aeroplane that stalls during a
maneuver, will stall at a higher speed than if it
stalled in level flight. This is called an “accelerated
stall”.
An additional factor by which it stalls is √n
n=load factor
Straight and level flight n = 1,
but in a 60° turn, n = 2, and √2 = 1.41.

7) is caused by wingtip stall


Comment/Reference:

8) Decreasing the angle of sweep of the wing will decrease


the stall speed
Comment/Reference:

9) The lift coefficient and the wing stalling angle both


remain constant
Comment/Reference:

10) increases with an increased weight


Comment/Reference:
Questions 1801 – 1810

1801.Increase of wing loading will

 increase the stall speed


 decrease the minimum gliding angle
 decrease take off speeds

1802.The most common stall sensing devices are normally


located

 at or near the wing leading edge


 on the upper surface of the wing
 on the lower surface of the wing

1803.If angle of attack is increased beyond the critical


angle of attack, the lift coefficient ... and the
stagnation point moves...

 increases; rearward
 decreases; rearward
 decreases; forward

1804.In a turn, the wing stalling angle

 decreases
 increases
 remains unchanged

1805.As the angle of attack of a wing is increased, the


centre of pressure will reach its most forward position
on the wing

 after stall recovery


 just before the wing stalls
 when the aircraft is stalling
1806.Which kind of boundary layer has the strongest change
in velocity close to the surface?

 Turbulent boundary layer


 Laminar boundary layer
 No difference

1807.One disadvantage of the swept back wing is it is


stalling characteristics. At the stall

 wing root stall will occur first, which produces a


rolling moment
 tip stall will occur first, which produces a nose-
down moment
 tip stall will occur first, which produces a pitch-
up moment

1808.Which of the following aircraft designs would be most


prone to super stall?

 Swept forward wing


 Swept back wing
 T-tail

1809.The stall speed in a 60° banked turn increases by the


following factor

 1
 1.41
 2.00

1810.The speed range between high and low speed buffet

 increases during a descent at a constant IAS


 increases during climb
 decreases during a descent at a constant Mach number
Answers for Questions 1801 – 1810

1) increase the stall speed


Comment/Reference:

2) at or near the wing leading edge


Comment/Reference:

3) decreases; rearward
Comment/Reference:

4) remains unchanged
Comment/Reference:

5) just before the wing stalls


Comment/Reference:

6) Turbulent boundary layer


Comment/Reference:

7) tip stall will occur first, which produces a pitch-up


moment
Comment/Reference:

8) T-tail
Comment/Reference: Aircraft Instruments & Integrated
Systems. Pallett pg 76

9) 1.41
Comment/Reference:

10) increases during a descent at a constant IAS


Comment/Reference:
Questions 1811 – 1820

1811.The boundary layer of a wing is caused by

 a layer on the wing in which the stream velocity is


lower than the free stream velocity, due to friction
 suction at the upper wing side
 a turbulent stream pattern around the wing

1812.The most important problem of ice accretion on an


aeroplane during flight is

 increase in drag
 increase in weight
 reduction in CLmax

1813.The stalling speed in IAS will change according to the


following factors?

 May increase with altitude, especially high


altitude, will increase during icing conditions and
will increase when the c.g. moves forward
 Will increase with increased load factor, more flaps
and increased bank angle in a turn
 Will increase in a turn, higher temperature and will
increase when the c.g. moves aft.

1814.The pitch up effect of an aeroplane with swept wing in


a stall is due to the

 wing tip stalling first


 wing root stalling first
 forward movement of the centre of gravity

1815.When an aircraft with a typical aerofoil is in level


flight at low speed and high angle of attack, the
normal axis is

 nearly vertical
 horizontal from side to side
 vertical
1816.An aeroplane has a stall speed of 78 KCAS at its gross
weight of 6.850 lbs. What is the stall speed when the
weight is 5.000 lbs?

 91 KCAS
 78 KCAS
 67 KCAS

1817.What causes a swept wing aircraft to pitch-up at the


stall

 Separated airflow at the root


 Rearward movement of the CP
 Spanwise flow

1818.Load factors has the following meaning


 the ratio of a specified load to the weight of the
aircraft, the former being expressed in terms of
aerodynamic forces, inertia forces and ground
reactions
 the ratio of a specified load to the mass of the
aircraft, the former being expressed in terms of
aerodynamic forces, inertia forces and ground
reactions
 the ratio of a specified load to the mass or the
aircraft the former being expressed in terms of
aerodynamic and inertia forces

1819.When all other factors of importance are constant, the


stall speed increases when

 weight decreases
 spoilers are retracted
 pulling out of a dive

1820.Which of the following is the correct order of


configuration to give an increasing critical angle of
attack?

 Slats extended, clean wing, trailing edge flaps


extended
 Clean wing, trailing edge flaps extended, slats
extended
 Trailing edge flaps extended, clean wing, slats
extended
Answers for Questions 1811 – 1820

1) a layer on the wing in which the stream velocity is


lower than the free stream velocity, due to friction
Comment/Reference:

2) reduction in CLmax
Comment/Reference:

3) May increase with altitude, especially high altitude,


will increase during icing conditions and will increase
when the c.g. moves forward
Comment/Reference:

4) wing tip stalling first


Comment/Reference:

5) nearly vertical
Comment/Reference:

6) 67 KCAS
Comment/Reference:

7) Spanwise flow
Comment/Reference:

8) the ratio of a specified load to the weight of the


aircraft, the former being expressed in terms of
aerodynamic forces, inertia forces and ground reactions
Comment/Reference:

9) pulling out of a dive


Comment/Reference:

10) Trailing edge flaps extended, clean wing, slats


extended
Comment/Reference:
Questions 1821 – 1830

1821.What is the percentage increase in stall speed in a 45


bank turn

 10%
 19%
 45%

1822.When an aircraft wing stalls

 a swept back wing will stall from the tip and the CP
will move forward
 a wing which is not swept back will stall from the
root and the CP will move forwards
 a swept back wing will stall from the root and the
CP will move aft

1823.At the same weight, with the CG at its forward limit

 VS is higher, the stalling angle is unchanged


 VS is lower, the stalling angle is unchanged
 VS is higher, the stalling angle is greater

1824.Stick pusher is installed in aircraft when

 the aircraft has no yaw damper installed


 the aircraft has failed to meet the stalling
requirements by normal category
 the aircraft is directional unstable

1825.Two identical aeroplanes A and B are flying horizontal


steady turns. Further data is:
A: W=1500 kg Bank= 20° TAS= 130 kt
B: W= 1500 kg Bank= 20° TAS= 200 kt
Which of the following statements is correct?

 The turn radius A is larger than the turn radius B


 The rate of turn A is larger than the rate of turn B
 The load factor A is larger than the load factor B
1826.Which of the following situations leads to a
decreasing stall speed (IAS)?

 Increasing load factor


 Increasing air density
 Decreasing weight

1827.The purpose of a fixed spoiler on the leading edge of


a wing at the root is to

 prevent the wing from stalling at the root


 reduce the landing distance required
 ensure that the root of the wing stalls before the
tip does

1828.In a level turn with 60° lateral bank, the load factor
is 2.0 and the stall speed increases by

 10 %
 40 %
 50 %

1829.The critical angle of attack

 changes with an increase in gross weight


 increases if the CG is moved forward
 remains unchanged regardless of gross weight

1830.An aeroplane has been loaded in such a manner that the


centre of gravity is located behind the aft centre of
gravity limit. One undesirable flight characteristic a
pilot might experience with this aeroplane would be

 a longer takeoff run


 difficulty in recovering from a stalled condition
 stalling at higher-than-normal airspeed
Answers for Questions 1821 – 1830

1) 19%
Comment/Reference:

2) a swept back wing will stall from the tip and the CP
will move forward
Comment/Reference:

3) VS is higher, the stalling angle is unchanged


Comment/Reference:

4) the aircraft has failed to meet the stalling


requirements by normal category
Comment/Reference:

5) The rate of turn A is larger than the rate of turn B


Comment/Reference:

6) Decreasing weight
Comment/Reference:

7) ensure that the root of the wing stalls before the tip
does
Comment/Reference:

8) 40%
Comment/Reference:

9) remains unchanged regardless of gross weight


Comment/Reference:

10) difficulty in recovering from a stalled condition


Comment/Reference:
Questions 1831 – 1840
1831.Vso is defined as the
 stalling speed or minimum takeoff safety speed
 stalling speed or minimum steady flight speed in a
specified configuration
 stalling speed or minimum steady flight speed in the
landing configuration

1832.The stalling speed in IAS will change according to the


following factors

 decrease in a forward c.g. location, higher altitude


and due to the slip stream from a propeller on an
engine located forward of the wing
 increase during turn, increased mass and forward
c.g. location
 increase with increased load factor, icing
conditions and an aft c.g. location

1833.With the centre of gravity on the forward limit, the


stalling speed would be

 lower than with the centre of gravity on the aft


limit
 higher than with the centre of gravity on the aft
limit
 the same as with the centre of gravity on the aft
limit

1834.The following take place at the transition point on a


wing

 the airflow separates completely from the wing


surface
 the total dynamic and static pressure comes to a
standstill
 the boundary layer makes the transition from laminar
flow to the turbulent boundary layer

1835.A low wing loading (aircraft weight has been reduced)

 increases stalling speed


 decreases stalling speed and landing speed
 increases take-off run, stalling speed and landing
speed
1836.Stalling speed in a 15° bank level turn is 60kt. The
stalling speed in a 45° bank level turn will be

 50 kts
 70 kts
 60 kts

1837.Which of the following is the speed in level flight


that would activate the stall warning?

 1.2VS1G
 1.05VS1G
 VS1G + 15kts

1838.How does stalling speed (IAS) vary with altitude?

 It remains constant
 It remains constant at lower altitudes but decreases
at higher altitudes due to compressibility effects
 It increases with increasing altitude, because the
density decreases

1839.As the centre of gravity is changed, recovery from a


stall becomes progressively

 becomes progressively more difficult as the centre


of gravity moves aft.
 becomes progressively more difficult as the centre
of gravity moves forward
 is unaffected by centre of gravity position

1840.Which of the following statements about boundary


layers is correct?

 The turbulent boundary layer is thinner than the


laminar boundary layer
 The turbulent boundary layer has more kinetic energy
than the laminar boundary layer
 The turbulent boundary layer gives a lower skin
friction than the laminar boundary layer
Answers for Questions 1831 – 1840

1) stalling speed or minimum steady flight speed in the


landing configuration
Comment/Reference: V speeds at Wikipedia.

2) increase during turn, increased mass and forward c.g.


location
Comment/Reference:

3) higher than with the centre of gravity on the aft limit


Comment/Reference:

4) the boundary layer makes the transition from laminar


flow to the turbulent boundary layer
Comment/Reference:

5) decreases stalling speed and landing speed


Comment/Reference:

6) 70 kts
Comment/Reference:

7) 1.05VS1G
Comment/Reference: Vs1g = one g stall.
http://en.wikipedia.org/wiki/V_speeds

8) It increases with increasing altitude, because the


density decreases
Comment/Reference:

9) becomes progressively more difficult as the centre of


gravity moves aft.
Comment/Reference:

10) The turbulent boundary layer has more kinetic energy


than the laminar boundary layer
Comment/Reference:
Questions 1841 – 1850

1841.The angle of attack at which a wing stalls will

 increase if the centre of gravity is moved forward


 increase if the centre of gravity is moved aft
 remain the same regardless of centre of gravity
position

1842.Entering the stall the centre of pressure of a


straight (1) wing and of a strongly swept back wing (2)
will

 not move (2) move forward


 move aft, (2) move forward
 move aft, (2) not move

1843.Compared with stalling airspeed (VS) in a given


configuration, the airspeed at which stick shaker will
be triggered is

 greater than VS
 VS
 less than VS

1844.A boundary layer fence on a swept wing will

 improve the low speed characteristics


 increase the critical Mach Number
 improve the high speed characteristics

1845.If the straight and level stall speed is 100 kt, what
will be the stall speed in a 1.5g turn?

 81 kts
 122 kts
 150 kts
1846.The function of the stick pusher is

 to activate and push the stick forward prior to


stick shaker
 to vibrate the controls
 to activate and push the stick forward at or beyond
a certain value of angle of attack

1847.How does stalling speed vary with load factor?

 It decreases inversely with the square root of the


load factor
 It increases proportionally with the square root of
the load factor
 It decreases inversely with the load factor

1848.When entering a stall, the CG of a straight wing will


(i) and of a strongly swept wing will (ii)

 (i) move aft, (ii) move aft


 (i) not move, (ii) not move
 (i) move aft, (ii) not move

1849.After the transition point between the laminar and


turbulent boundary layer

 the mean speed and friction drag increases


 the mean speed increases and the friction drag
decreases
 the boundary layer gets thicker and the speed
decreases

1850.The wing of an aeroplane will never stall at low


subsonic speeds as long as

 the angle of attack is smaller than the value at


which the stall occurs
 the IAS exceeds the power-on stall speed
 there is a nose-down attitude
Answers for Questions 1841 – 1850

1) remain the same regardless of centre of gravity


position
Comment/Reference:

2) (1) move aft, (2) move forward


Comment/Reference:)

3) greater than VS
Comment/Reference:

4) improve the low speed characteristics


Comment/Reference:

5) 122 kts
Comment/Reference:

6) to activate and push the stick forward at or beyond a


certain value of angle of attack
Comment/Reference:

7) It increases proportionally with the square root of the


load factor
Comment/Reference:)

8) (i) not move, (ii) not move


Comment/Reference:

9) the mean speed and friction drag increases


Comment/Reference:

10) the angle of attack is smaller than the value at which


the stall occurs
Comment/Reference:
Questions 1851 – 1860

1851.The load factor is

 the ratio of thrust to weight


 the ratio of lift to drag
 the ratio of lift to weight

1852.The input to a stick shaker comes from

 the angle of attack only


 angle of attack, and sometimes the rate of change in
angle of attack
 the airspeed, and sometimes the rate of change in
airspeed

1853.Stick pushers must be installed in aeroplanes with


dangerous stall characteristics. Dangerous stall
characteristics include:

 pitch down and increase in speed


 pitch down and minor wing drop
 excessive wing drop and deep stall

1854.The sensor of a stall warning system can be activated


by a change in the location of the

 transition region
 stagnation point
 centre of lift

1855.Which type of stall has the largest associated angle


of attack?

 Deep stall
 Shock stall
 Low speed stall
1856.Which statement is correct about the laminar and
turbulent boundary layer?

 Separation point will occur earlier in the turbulent


layer
 Friction drag is lower in the turbulent layer
 Friction drag is lower in the laminar layer

1857.A slotted flap will increase the CLmax by

 increasing the camber of the aerofoil and improving


the boundary layer
 increasing only the camber of the aerofoil
 increasing the critical angle of attack

1858.When a trailing edge flap is lowered fully

 the C of P moves to the rear and lift/drag ratio is


increased
 the C of P moves to the rear and lift/drag ratio is
unaffected
 the C of P moves to the rear and lift/drag ratio is
decreased

1859.Which statement is correct?

 Spoiler extension decreases the stall speed and the


minimum rate of descent, but increases the minimum
descent angle
 Extension of flaps causes a reduction of the stall
speed, the maximum glide distance also reduces
 Extension of flaps will increase (CL/CD)max, causing
the minimum rate of descent to decrease

1860.When the angle of attack of an aircraft is


progressively increased, the wing centre of pressure
will reach its most forward position

 when the aircraft has stalled


 at the maximum lift coefficient (Clmax)
 at the optimum angle of attack of the wing
Answers for Questions 1851 – 1860

1) the ratio of lift to weight


Comment/Reference:

2) angle of attack, and sometimes the rate of change in


angle of attack
Comment/Reference:

3) excessive wing drop and deep stall


Comment/Reference:

4) stagnation point
Comment/Reference:

5) Deep stall
Comment/Reference:

6) Friction drag is lower in the laminar layer


Comment/Reference:

7) increasing the camber of the aerofoil and improving


the boundary layer
Comment/Reference:

8) the C of P moves to the rear and lift/drag ratio is


decreased
Comment/Reference:

9) Extension of flaps causes a reduction of the stall


speed, the maximum glide distance also reduces
Comment/Reference:

10) at the maximum lift coefficient (Clmax)


Comment/Reference:
Questions 1861 – 1870
1861.What is the most effective flap system?

 Fowler flap
 Single slotted flap
 Split flap

1862.An aeroplane with swept back wings is equipped with


slats and/or leading edge (L.E.) flaps. One possible
efficient way to arrange the leading edge devices on
the wings is

 Wing roots: L.E. flaps Wing tips: slats


 Wing roots: L.E. flaps Wing tips: no devices
 Wing roots: slats Wing tips: L.E. flaps

1863.Which of the following statements about the difference


between Krueger flaps and slats is correct?

 Deploying a slat will increase critical angle of


attack, deploying a Krueger flap does not
 Deploying a slat will form a slot deploying a
Krueger flap does not
 Deploying a Krueger flap will increase critical
angle of attack, deploying a slat does not

1864.Vortex generators

 change the turbulent boundary layer into a laminar


boundary layer
 transfer energy from the free airflow into the
boundary layer
 reduce the span wise flow on swept wing

1865.When vortex generators are fitted they will normally


be found

 towards the wing root to act as a stall inducer


 towards the wing trailing edge
 near the wing leading edge in front of control
surfaces
1866.During the extension of the flaps at a constant angle
of attack the aeroplane starts to: (all other factors
of importance being constant)

 sink suddenly.
 bank
 climb

1867.Deploying a Fowler flap, the flap will

 just move aft


 move aft, then turn down
 turn down, then move aft

1868.An aeroplane has the following flap settings: 0°, 15°,


30° and 45°. Slats can be selected too. Which of the
above selections will produce the greatest negative
influence on the CL/CD ratio?

 Flaps from 15° to 30°


 Flaps from 0° to 15°
 Flaps from 30° to 45°

1869.The effect of Winglets is

 elliptical pressure distribution increases


 decrease in stall speed
 reduction in induced drag

1870.When flaps are extended in a straight and level flight


at constant IAS, the lift coefficient will eventually

 remain the same


 decrease
 increase
Answers for Questions 1861 – 1870

1) Fowler flap
Comment/Reference:

2) Wing roots: L.E. flaps Wing tips: slats


Comment/Reference:

3) Deploying a slat will form a slot deploying a Krueger


flap does not
Comment/Reference:

4) transfer energy from the free airflow into the


boundary layer
Comment/Reference: Stall (flight) at wikipedia

5) near the wing leading edge in front of control


surfaces
Comment/Reference: Stall (flight)at wikipedia at
bottom of page

6) climb
Comment/Reference:

7) move aft, then turn down


Comment/Reference:

8) Flaps from 30° to 45°


Comment/Reference:

9) reduction in induced drag


Comment/Reference:

10) remain the same


Comment/Reference:
Questions 1871 – 1880
1871.When spoilers are used as speed brakes

 they do not affect wheel braking action during


landing
 at same angle of attack, CD is increased and CL is
decreased
 at same angle of attack, CL remains unaffected

1872.When a trailing edge flap is lowered during flight


from take-off position to fully down position, one
will experience

 a small increase in lift and a large increase in


drag
 a large increase in lift and a small increase in
drag
 a large increase in lift and a large increase in
drag

1873.What is the purpose of an auto-slat system?

 Provide automatically slat IN selection after take-


off
 Ensures that the slats are always extended when the
ground/flight system is in the ground position
 Extend automatically when a certain value of angle
of attack is exceeded

1874.Which of the following series of configurations has an


increasing critical angle of attack?

 flaps only extended, clean wing, slats only extended


 clean wing, flaps only extended, slats only extended
 slats only extended, flaps only extended, clean wing

1875.Where on the surface of a typical aerofoil will flow


separation normally start at high angles of attack?

 upper side trailing edge


 upper side leading edge
 lower side leading edge
1876.If the flaps are lowered but the airspeed is kept
constant, to maintain level flight

 the nose must be pitched down


 the altitude must be held constant
 the nose must be pitched up

1877.The effects of leading edge slats

 increase boundary layer energy, move suction peak on


to slat and increase CLmax angle of attack
 increase camber, increase suction peak on main wing,
increase effective angle of attack and move CLmax to
higher angle of attack
 increase boundary layer energy, increase suction
peak on main wing section, move CLmax to a higher
angle of attack

1878.Flaps are used in order to

 increase max lift coefficient by increasing max


angle of attack
 decrease stalling speed and reduce max angle of
attack thereby achieving a more nose down attitude
near and at stalling speed
 increase max L/D

1879.Deflection of leading edge flaps will

 decrease CLmax
 increase critical angle of attack
 decrease drag

1880.During flap down selection in a continuous straight


and level flight at constant IAS and weight

 the lift coefficient and the drag coefficient


increase
 the stall speed increases
 the centre of pressure moves aft
Answers for Questions 1871 – 1880

1) at same angle of attack, CD is increased and CL is


decreased
Comment/Reference:

2) a small increase in lift and a large increase in drag


Comment/Reference:

3) Extend automatically when a certain value of angle of


attack is exceeded
Comment/Reference:

4) flaps only extended, clean wing, slats only extended


Comment/Reference:

5) upper side trailing edge


Comment/Reference:

6) the nose must be pitched down


Comment/Reference:

7) increase boundary layer energy, increase suction peak


on main wing section, move CLmax to a higher angle of
attack
Comment/Reference:

8) decrease stalling speed and reduce max angle of attack


thereby achieving a more nose down attitude near and
at stalling speed
Comment/Reference:

9) increase critical angle of attack


Comment/Reference:

10) the centre of pressure moves aft


Comment/Reference:
Questions 1881 – 1890

1881.CLmax may be increased by the used of

 flaps
 slats
 both flaps and slats

1882.Compared with the flap up configuration the maximum


angle of attack for the flaps down configuration is

 smaller
 larger
 unchanged

1883.Compared with the flap down configuration the maximum


angle of attack for the flaps up configuration is

 larger
 smaller
 unchanged

1884.How is the pitching moment affected if flaps are


deployed in straight and level flight?

 Depends on wing position


 Pitch up
 Pitch down

1885.What is the effect of lowering leading edge and


trailing edge flaps in flight?

 Cl increases, Cd remains the same and the stalling


angle of attack increases.
 Cl increases, Cd increases and the stalling angle of
attack increases
 Cl decreases, Cd increases and the stalling angle of
attack reduces
1886.Lowering the inboard flaps causes the wing Centre of
Pressure

 to move forward
 to move inboard towards the wing root
 to move outboard towards the wing tips

1887.Slats

 re-energise the boundary layer thereby decreasing


the stalling angle of attack
 re-energise the boundary layer thereby increasing
the stalling angle of attack
 de-energise the boundary layer, thereby decreasing
the stalling angle of attack

1888.When deploying the flaps the effective angle of attack

 decreases
 remains the same
 increases

1889.Vortex generators mounted on the upper wing surface


will

 increase the interference drag of the trailing edge


flaps
 decrease the stalling speed by increase of the
tangential velocity of the swept wing
 decrease the shock wave induced separation

1890.Vortex generators mounted on the upper wing surface


will

 decrease the stalling speed by increase of the


tangential velocity of the swept wing
 increase the shock wave induced separation
 decrease the interference drag of the trailing edge
flaps
Answers for Questions 1881 – 1890

1) both flaps and slats


Comment/Reference:

2) smaller
Comment/Reference:

3) larger
Comment/Reference:

4) Depends on wing position


Comment/Reference:

5) Cl decreases, Cd increases and the stalling angle of


attack reduces
Comment/Reference:

6) to move inboard towards the wing root


Comment/Reference:

7) re-energise the boundary layer thereby increasing the


stalling angle of attack
Comment/Reference:

8) increases
Comment/Reference:

9) decrease the shock wave induced separation


Comment/Reference:

10) decrease the interference drag of the trailing edge


flaps
Comment/Reference:
Questions 1891 – 1900

1891.It is possible to reduce the span wise airflow over


swept wings, due to adverse pressure gradients, by

 wing fences
 trailing edge vortex generators
 increased anhedral

1892.After take-off the slats (when installed) are always


retracted later than the flaps. Why?

 Because SLATS EXTENDED provides a better view from


the cockpit than FLAPS EXTENDED
 Because FLAPS EXTENDED gives a large decrease in
stall speed with relatively less drag
 Because SLATS EXTENDED gives a large decrease in
stall speed with relatively less drag

1893.In order to maintain straight and level flight at a


constant airspeed, whilst the flaps are being
retracted, the angle of attack will

 remain constant
 increase
 decrease

1894.If flaps are deployed at constant IAS in straight and


level flight, the magnitude of tip vortices will
eventually: (flap span less than wing span)

 decrease
 remain the same
 increase

1895.The trailing edge flaps, when extended

 worsen the best angle of glide


 significantly increase the angle of attack for
maximum lift
 improve the best angle of glide
1896.One of the main purposes of using flaps during
approach and landing is to

 decrease the angle of descent without increasing the


airspeed
 increase the angle of descent without increasing the
airspeed
 shift the centre of gravity aft

1897.The use of a slot in the leading edge of the wing


enables the aeroplane to fly at a slower speed because

 it changes the camber of the wing


 it delays the stall to a higher angle of attack
 the laminar part of the boundary layer gets thicker

1898.What is the effect of deploying leading edge flaps?

 Not affect the critical angle of attack


 Increase the critical angle of attack
 Decrease the critical angle of attack

1899.The lift coefficient CL of a wing at a given angle of


attack

 is constant and not affected by high lift devices


 is increased by the use of high lift devices
 is dependent on the surface area of the wing

1900.The optimum angle of attack for a typical aerofoil is


about..., and the actual angle of attack will be close
to this optimum angle during...

 16°, a stall
 4°, cruise
 4°, a stall
Answers for Questions 1891 – 1900

1) wing fences
Comment/Reference:

2) Because SLATS EXTENDED gives a large decrease in stall


speed with relatively less drag
Comment/Reference:

3) increase
Comment/Reference:

4) decrease
Comment/Reference:

5) worsen the best angle of glide


Comment/Reference:

6) increase the angle of descent without increasing the


airspeed
Comment/Reference:

7) it delays the stall to a higher angle of attack


Comment/Reference:

8) Increase the critical angle of attack


Comment/Reference:

9) is increased by the use of high lift devices


Comment/Reference:

10) 4°, cruise


Comment/Reference:
Questions 1901 – 1910

1901.Upon extension of a spoiler on a wing

 only CL is decreased (CD remains unaffected)


 CD is increased and CL is decreased
 CL is increased, while CD remains unaffected

1902.If the weight of an aircraft is increased, the maximum


lift-to-drag ratio will

 not be affected
 increase
 decrease

1903.Speed brakes are a device used on large transport


category aircraft

 to prevent aquaplaning
 to increase drag in order to maintain a steeper
gradient of descent
 for speed reduction after landing

1904.Why are vortex generators often fitted on aircraft


with straight wings?

 To delay boundary layer separation


 To reduce span wise airflow towards the wing tips on
the extrados
 To reduce span wise airflow towards the wing root on
the intrados

1905.The boundary layer of a body in a moving airstream is

 a layer of air over the surface where the airspeed


is changing from free stream velocity to zero
velocity
 a layer of air which is moving at free stream speed
 a thin layer of air over the surface where the air
is stationary
1906.The transition point located on the wing is the point
where

 the boundary layer changes from laminar flow to


turbulent flow
 the airflow changes from subsonic to supersonic flow
 airflow starts separating from the wing

1907.The boundary layer is considered to be turbulent

 between the transition and separation points


 just in front of the transition point
 just aft of the separation point

1908.The advantage of a turbulent boundary layer over a


laminar boundary layer is

 decreases energy
 less tendency to separate
 thinner

1909.A laminar boundary layer is... and has... drag than a


turbulent layer.

 thicker; more
 thinner; less
 thicker; less

1910.When pulling out of a dive (e.g. looping) the angle of


attack

 decreases
 remains the same
 increases
Answers for Questions 1901 – 1910

1) CD is increased and CL is decreased


Comment/Reference:

2) not be affected
Comment/Reference:

3) to increase drag in order to maintain a steeper


gradient of descent
Comment/Reference: AC65-15A pg 40 bottom rh para.

4) To delay boundary layer separation


Comment/Reference:

5) a layer of air over the surface where the airspeed is


changing from free stream velocity to zero velocity
Comment/Reference:

6) the boundary layer changes from laminar flow to


turbulent flow
Comment/Reference:

7) between the transition and separation points


Comment/Reference:

8) less tendency to separate


Comment/Reference:

9) thinner; less
Comment/Reference:

10) increases
Comment/Reference:
Questions 1911 – 1920

1911.In which phase of the take-off is the aerodynamic


effect of ice located on the wing leading edge most
critical?

 During climb with all engines operating


 All phases of the take-off are equally critical
 The last part of the rotation

1912.Which tab’s movement is directly from the control


column?

 Anti-Balance tab
 Balance tab
 Servo tab

1913.A pilot's safety harness buckle and adjustment strap


Is

 not allowed to slip under any forces


 allowed to slip if side loads exceed 2.5 G
 allowed to slip if vertical acceleration exceeds 5G

1914.A vacuum mercury gauge reads 5 in. is this pressure

 above ambient pressure


 above zero pressure
 below ambient pressure

1915.Wing skin on a large transport aircraft is made from

 7118
 7075
 2024
1916.Chemical fire extinguishers extinguish fires by

 cools the material on fire below ignition point


 releases oxygen to extinguish the fire
 reacting with the oxygen in the air or the flame,
rendering it inert

1917.Oxygen bottles should not be left with less that 500


psi pressure because

 it can damage the diluter demand regulator and would


require maintenance
 the oxygen becomes tainted causing anoxia
 it can cause corrosion within the bottle

1918.A fire test is carried out and the test button is


released, but the light remains on. This is because

 there is a fire
 of a high resistance fire wire
 of a broken fire wire

1919.Mach trim operation is enabled by

 an electric Motor
 a mach trim actuator
 an autopilot trim motor

1920.The CO2 in a life raft is released

 Electrically
 on contact with water
 manually
Answers for 1911 – 1920

1) The last part of the rotation


Comment/Reference:

2) Servo Tab
Comment/Reference: AC65-15A Chap 2 pg 46

3) not allowed to slip under any forces


Comment/Reference:

4) below ambient pressure


Comment/Reference:

5) 7075
Comment/Reference:

6) reacting with the oxygen in the air or the flame,


rendering it inert
Comment/Reference:

7) it can cause corrosion within the bottle


Comment/Reference:

8) of a broken fire wire


Comment/Reference: B737-3/4/500 AMM 26-11-00 pg 6 para
5.b.

9) an autopilot trim motor


Comment/Reference:

10) electrically
Comment/Reference:
Questions 1921 – 1930

1921.A fire detection system can pass through one zone to


another providing

 the system is duplicated


 it is separated by steel
 the system is protected

1922.Deploying a secondary flight control surface will

 not affect the angle of attack


 decrease the angle of attack of the aerofoil
 increase the angle of attack of the aerofoil

1923.To check if a landing gear leg requires servicing

 The aircraft must have an exact load onboard and air


charge to give an exact leg extension
 The leg extension can vary with aircraft load. The
leg pressure and extension should be checked against
a chart in the AMM
 The leg must be serviced to give an exact extension
regardless of aircraft load

1924.The 'fasten seatbelt' light switch should be in reach


of the

 captain
 first officer
 both the captain and the first officer

1925.A shock wave is a very thin region in which there is a


sudden decrease of

 temperature
 velocity
 pressure
1926.For a wing of low thickness-chord ratio the critical
mach number will be

 higher than wing of high thickness-chord ratio


 lower than wing of high thickness-chord ratio
 the same as a wing of high thickness-chord ratio

1927.Sweepback on the wing will

 reduce tendency to tip stall


 increase tendency to tip stall
 cause stall to occur at lower angles of attack

1928.High speed buffet is caused by

 airflow being detached by shock wave and flow


striking the tail
 high speed airflow striking the leading edge
 the shock wave striking the tail

1929.A structure which if it failed would cause aircraft to


crash is

 essential
 integral
 primary

1930.The method used on a work card to indicate the


approximate location of a component is

 datum
 stations
 zones
Answers for questions 1921 – 1930

1) the system is protected


Comment/Reference:

2) decrease the angle of attack of the aerofoil


Comment/Reference:

3) The leg extension can vary with aircraft load. The leg
pressure and extension should be checked against a
chart in the AMM
Comment/Reference:

4) both the captain and the first officer


Comment/Reference:

5) velocity
Comment/Reference:

6) higher than wing of high thickness-chord ratio


Comment/Reference:

7) increase tendency to tip stall


Comment/Reference:

8) airflow being detached by shock wave and flow striking


the tail
Comment/Reference:

9) primary
Comment/Reference:

10) zones
Comment/Reference:
Questions 1931 – 1940

1931.A loss of strength resulting from cyclic loading is


known as

 metal fatigue
 age hardening
 fail safe

1932.To enable a hydraulic system to operate two components


with different pressures, a

 pressure reducing valve is used


 pressure regulating valve is used
 pressure relief valve is used

1933.Restrictor valves are fitted in hydraulic systems to

 restrict the rate of pressure build-up


 limit the maximum pressure
 control rate of system operation

1934.Fly by wire systems rely on inputs from

 the pilot via control rods, and electrical signals


 electrical signals
 air-data and 'Q' feel

1935.What placard must be visible above a waste paper bin


in a toilet?

 No Cigarettes
 No Naked Flame
 No Smoking
1936.ATA zone 500 is the

 Right wing
 Left wing
 Engine

1937.ATA zone 330 is the

 Right horizontal stabilizer


 Vertical stabilizer
 Left horizontal stabilizer

1938.When applying a doubler plate, what else could you use


for extra strength?

 Thicker material
 A tripler plate
 A waffle plate

1939.Stringers

 run the full length on all aircraft


 are attached to the frame
 do not pass through bulkheads

1940.Kevlar should be stored

 vertically
 in moisture proof bags
 horizontally
Answers for Questions 1931 – 1940

1) metal fatigue
Comment/Reference:

2) pressure reducing valve is used


Comment/Reference:

3) control rate of system operation


Comment/Reference:

4) the pilot via control rods, and electrical signals


Comment/Reference:

5) No Smoking
Comment/Reference: CAP 747 GR.20 4.2.3

6) Left wing
Comment/Reference: ATA Chapter 06

7) left horizontal stabiliser


Comment/Reference: ATA chapter 06-20 (A319)

8) A waffle plate
Comment/Reference:

9) do not pass through bulkheads


Comment/Reference:

10) in moisture proof bags


Comment/Reference:
Questions 1941 – 1950

1941.A firewall bulkhead is usually made from

 steel
 tungsten
 titanium

1942.Aileron reversal is caused by a structure that has a


low stiffness in

 shear
 torsion
 bending

1943.The first component that the charged air enters from


the IP compressor is

 the check valve


 the bleed valve
 the pre-cooler

1944.On the ramp, the pre-cooler obtains its cooling action


by

 ram air
 gas expansion
 fan inducement

1945.The electrical spill-valve operates on signals from

 the venturi metering duct


 the pressure controller
 the latching solenoid
1946.A standby ADI uses

 a vertical gyro
 a space gyro
 an earth gyro

1947.An artificial horizon uses

 a directional gyro
 a rate gyro
 a vertical gyro

1948.Following replacement of a separately mounted compass


corrector box, a compass calibration swing will be
required

 only if initial variation is greater than 5 degrees


 only if the correctors are not matched to the ones
being removed
 without exception

1949.During climb the pressure in the VSI capsule will be

 equal to the pressure in the case


 greater than the pressure in the case
 lower than the pressure in the case

1950.Corrections to a mechanical ASI are for

 square law
 square law error and temperature
 square law and temperature
Answers for 1941 – 1950

1) I did not get the answer or the ref

2) torsion
Comment/Reference:

3) I did not get the answer or the ref

4) fan inducement
Comment/Reference:

5) the venturi metering duct


Comment/Reference:

6) vertical gyro

7) a vertical gyro
Comment/Reference:

8) without exception
Comment/Reference:

9) lower than the pressure in the case


Comment/Reference:

10) square law error and temperature


Comment/Reference:
Questions 1951 – 1960

1951.Vertical gyros are limited to 85 degrees movement in


pitch to prevent

 gimbal lock
 pitch overshoot
 gyro topple

1952.The magnets in a compass corrector micro-adjuster are

 adjusted for coefficient A


 adjusted to obtain the neutral position
 always set to the neutral position

1953.The term HIRF means

 High Intensity Radiated Fields


 High Interference Radiated Fields
 High Intensity Radio Fields

1954.The power supply for aircraft is

 28VDC, 220VAC, 400Hz


 28VDC, 115VAC, 400Hz
 28VDC, 115VAC, 200Hz

1955.Reactive electrical loads are adjusted by

 adjustment of the regulator output


 varying the frequency circuit
 adjustment of generator speed
1956.Inspection proof testing of seat-belts is carried out
To

 0.5g
 4.5g
 9g

1957.Fuel tanks are numbered

right to left
fwd to aft
aft to fwd

1958.The flow rate of anti-icing fluid is determined

by speed of pump
by flow control valve
by flow control tubes

1959.The ice protection system that is manufactured on the


aircraft skin is

heater mat
overshoe
spraymat

1960.A silica gel desiccant is required to be replaced

when half pink and half blue


when all pink
when all blue
Answers for Questions 1951 – 1960

1) gimbal lock
Comment/Reference: Aircraft Instruments and Integrated
Systems. EHJ Pallett Page

2) adjusted to obtain the neutral position


Comment/Reference:

3) High Intensity Radiated Fields


Comment/Reference: CAP 756 appx 2 abbreviations

4) 28VDC, 115VAC, 400Hz


Comment/Reference:

5) adjustment of the regulator output


Comment/Reference: Aircraft Electrical Systems 3rd Ed
Pallett Page 50/51

6) 9g
Comment/Reference: CS 25.561

7) fwd to aft
Comment/Reference:

8) by flow control tubes


Comment/Reference:

9) spraymat
Comment/Reference:

10) when all pink


Comment/Reference:
Questions 1961 – 1970

1961.A MCDU is

to store fault data


to transmit data to ground
to dialog with the central maintenance computer

1) to dialog with the central maintenance computer


Comment/Reference:

Vous aimerez peut-être aussi